Text
                    N
\
>
\
4
\
\
i
¦/
v
ьп н


МОСКОВСКИЙ ЦЕНТР НЕПРЕРЫВНОГО MATEMAT.ОБРАЗ. Б.ВЛАСЬЕВСКИЙ.П Ш1-05-00
ББК 22.1 Г17 Рецензенты: доктор физико-математических наук, профессор МГУ В. М. Тихомиров, кандидат педагогических наук, зав. кабинетом математики МГИУУ С. М. Саакян. Гальперин Г. А., Толпыго А. К. Г17 Московские математические олимпиады: Кн. для уча- учащихся/Под ред. А. Н. Колмогорова.— М.: Просвещение, 1986.—303 с, ил. Книга содержит задачи всех Московских математических олимпиад за 50 лет их про- проведения. К большинству задач даны ответы, указания, решения. В книге много интерес- интересных задач, связанных с современными научными проблемами. Книга предназначена для учащихся VII—X классов средней школы, интересующихся математикой, а также может быть использована учителями во внеклассной работе. © Издательство «Просвещение», 1986
ПРЕДИСЛОВИЕ РЕДАКТОРА Нашей стране необходимо иметь много математиков-иссле- математиков-исследователей, способных делать открытия в самой математике и применять ее нестандартным образом, требующим большой изобретательности. Обычно серьезных успехов достигают те на- научные работники, которые начали тренироваться в такого рода деятельности еще в школьные годы. В возрасте 17—19 лет мно- многие из них уже начинают делать настоящие открытия. Откла- Откладывая вовлечение молодых людей в напряженную научную работу, мы безвозвратно теряем многих из тех, кто мог бы сделаться творчески активным ученым. Обращаясь к самим школьникам, всерьез собравшимся стать настоящими математиками, скажу следующее. Как и в спорте, тренировка юного математика требует затраты большого вре- времени. Будет очень хорошо, если вы возьметесь самостоятельно просматривать предлагаемый сборник задач, выберете из их числа какую-нибудь задачу, которая покажется вам наиболее ин- интересной по формулировке, и приметесь, не заглядывая в реше- решения, размышлять над ней, не боясь потратить на нее многие, многие часы. Напомню по этому поводу высказывание одного из самых замечательных советских математиков — Бориса Николаевича Делоне, по мнению которого большое на- научное открытие отличается от хорошей олимпиадной задачи только тем, что для решения олимпиадной задачи требуется 5 ча- часов, а получение крупного научного результата требует затраты 5000 часов. Борис Николаевич любил несколько преувеличенные формулировки, не понимайте его «5000 часов» слишком букваль- буквально. Но типичным для математика, который атакует трудную проблему, является способность напряженного размышления над ней целыми днями. Если задача упорно не выходит, то разум- разумно взяться за другую. Но хорошо также после некоторого пере- перерыва вернуться к первоначальной. Зрелым математикам тоже иногда бывает полезно на некоторое время отложить занятие какой-либо неподдающейся проблемой. Нередко после некото- некоторого перерыва решение неожиданно выплывает из подсознания. Своим успехам на олимпиаде естественно радоваться и даже гордиться ими. Неудачи же на олимпиаде не должны чрезмер- чрезмерно огорчать и приводить к разочарованию в своих способностях к математике. Для успеха на олимпиаде необходимы некото- некоторые специальные типы одаренности, которые вовсе не обяза- обязательны для успешной исследовательской работы. Уже само на- наличие назначенного очень ограниченного срока для решения задач многих делает совершенно беспомощными. Но существуют 3
и такие математические проблемы, которые могут быть решены лишь в результате очень длительного и спокойного размышле- размышления и формирования новых понятий. Много такого рода проб- проблем было решено замечательным советским топологом П. С. Алек- Александровым. Не случайно Павел Сергеевич Александ- Александров говорил, что если бы во времена его юности были ма- математические олимпиады, то, возможно, он вообще не сделался бы математиком: его главные достижения в математике явились не плодом быстро работающей изобретательности, а итогом дли- длительного и углубленного созерцания. Я надеюсь, что наш сборник окажется неоценимым пособием для всех руководителей школьных кружков и местных олимпиад. Для них я хочу высказать два замечания. Вначале Московские математические олимпиады были рас- рассчитаны на учащихся IX—X классов. Начиная же с 1940 г. к участию в олимпиадах приглашались также семиклассники и восьмиклассники. Такой выбор начального возраста представ- представляется мне обоснованным. Это тот возраст, когда склонности и способности к математике уже начинают проявляться достаточно явственно. Можно, конечно, устраивать олимпиады и для млад- младшеклассников, но при этом следует иметь в виду, что из числа мальчиков и девочек, выделившихся в V—VI классах в состяза- состязании по решению задач, большинство в старших классах эти свои особые способности, а часто и сам интерес к математике поте- потеряют При организации олимпиад для того или иного контингента участников чрезвычайно существенно, чтобы уровень трудности задач был надлежащим образом заранее правильно оценен. Сле- Следует планировать его так, чтобы наиболее сильные участники могли решить большую часть задач, а с другой стороны, чтобы не было чрезмерного преобладания участников, не решивших ни одной задачи. Некоторые сведения о фактически обнару- обнаружившейся трудности задач можно найти в отчетах об олим- олимпиадах, печатающихся в журналах «Математика в школе» и «Квант». К сожалению, в Московских математических олимпиа- олимпиадах уровень трудности не всегда выбирался правильно. Но содер- содержание задач было обычно на очень высоком уровне. В предисловии составителей подробно рассказывается об огромном опыте Московских математических олимпиад, о том, как процесс создания олимпиадных задач шел в неразрывной связи с работой математических кружков при МГУ. Коллектив руководителей университетских математических кружков про- проделал огромную, уникальную работу, итоги которой сейчас перед вами. Замечательна и заслуживает большой благодарности работа составителей — Г. А. Гальперина и А. К. Толпыго. Академик А. Н. Колмогоров
ПРЕДИСЛОВИЕ АВТОРОВ Из истории Московских олимпиад В этой книге собраны задачи всех Московских математи- математических олимпиад, начиная с первой, состоявшейся в 1935 г., к большинству из них даны ответы, указания и решения. В сво- своей совокупности эти задачи представляют собой плод многолет- многолетней коллективной работы студентов, аспирантов и преподава- преподавателей механико-математического факультета МГУ ряда поколе- поколений, а в последнее время также математиков МГПИ им. В. И. Ле- Ленина, МИИТа и факультета вычислительной математики и ки- кибернетики МГУ. Наша книга создавалась не на пустом месте. Двадцать лет назад задачи Московских олимпиад 30—50-х годов почти в пол- полном объеме были собраны и опубликованы в «Сборнике задач Московских математических олимпиад» (см. [21] в списке ли- литературы). При создании этого сборника была проделана огром- огромная работа как его составителем А. А. Леманом, так и всеми, кто способствовал его изданию. Однако сборник [21] давно уже стал библиографической редкостью и для большинства сегод- сегодняшних школьников практически недоступен. Кроме того, в нем содержатся задачи только первых 27 олимпиад (до 1964 г включительно). Критический пересмотр задач этих олимпиад позволил нам обнаружить ряд недочетов, которые мы по мере своих сил и воз- возможностей ликвидировали. Таким образом, ряд задач первых олимпиад, а также задачи последних 20 олимпиад в нашей кни- книге впервые предлагаются широкому кругу читателей. В ре- результате новое поколение школьников получает возможность по- познакомиться с тем огромным набором интересных и красивых идей, который заключен в более чем тысяче задач Московских олимпиад, а также и с историей этих олимпиад. Об истории пер- первых 27 олимпиад хорошо рассказано в статье, написанной В. Г. Болтянским и И. М. Ягломом для книги [21]; в ней они сумели ярко отразить дух Московских олимпиад, царивший в те годы. Мы приносим благодарность авторам за любезное разре- разрешение воспользоваться материалами их статьи. 5
—о— Московские математические олимпиады уже давно стали тра- традиционными. Какова их цель и как они проводятся? В 30-е годы была осознана необходимость участия ученых- математиков в работе со школьниками. Инициаторами такой работы выступили в Ленинграде член-корреспондент АН СССР Б. Н. Делоне и профессор В. А. Тартаковский, а в Москве —- член-корреспондент АН СССР Л. Г. Ш н и р е л ь- м а н и профессор (впоследствии член-корреспондент АН СССР) Л. А. Л ю с т е р н и к. Весной 1934 г. в Ленинграде была про- проведена первая в СССР школьная математическая олимпиада, а с осени 1934 г. в Москве, в Институте математики АН СССР, начали регулярно читаться лекции по математике для учащихся старших классов. Одновременно по инициативе Л. А. Люстер- ника начала выходить серия «Популярная библиотека по ма- математике», предназначенная специально для школьников. С целью привлечения к активным занятиям способных школь- школьников, интересующихся математикой, весной 1935 года правле- правление Московского математического общества, подхватив ини- инициативу ленинградцев, приняло решение о проведении I Москов- Московской математической олимпиады. К этому мероприятию москов- московские математики отнеслись с большим воодушевлением. В орг- оргкомитет олимпиады вошли профессора-математики МГУ, среди них А. Н. Колмогоров, Л. А. Люстерник, Л. Г. Шни- рельман, В. Ф. Каган, С. А. Яновская и др. Предсе- Председателем оргкомитета стал президент Московского математи- математического общества П. С. Александров. Олимпиада ставила своей целью выявить наиболее способных учащихся, привлечь внимание широких масс школьной молодежи к важнейшим проб- проблемам и методам современной математики и хотя бы частично показать, над чем работает отечественная математическая наука, каковы ее достижения и какие задачи стоят перед ней. В I олимпиаде приняло участие 314 школьников — большой успех в то время. Во втором (заключительном) туре приняло участие 120 человек, из которых трое получили первые премии, а пятеро школьников — вторые; кроме того, 44 школьника полу- получили почетные призы. Для многих школьников победа на олим- олимпиаде определила характер их будущей научной деятельности. Любопытен был набор задач второго тура первой олимпиады. Были предложены три серии задач: А, В и С. Как рассказал нам А. Н. Колмогоров, это было сделано по его инициативе, чтобы дать возможность проявить себя ученикам с разным скла- складом математического мышления: вычислительным (или «алго- «алгоритмическим»), геометрическим, комбинаторно-логическим (под- (подробно об этом можно прочесть в брошюре «О профессии мате- математика» [13]). В соответствии с этими типами мышления и под- подбирались серии задач на первую олимпиаду. 6
Успех I Московской олимпиады способствовал полной пере- перестройке всей работы со школьниками, в частности возник Школь- Школьный математический кружок при МГУ. Организаторами его яви- явились Л. А. Люстерник, Л. Г. Шнирельман и И. М. Гельфанд. Работа кружка проводилась в двух направ- направлениях: в чтении разнообразных по тематике лекций и в засе- заседаниях кружка. На лекции приходили сначала десятки, а затем и сотни школьников Москвы. Первоначально проводились лек- лекции для учащихся VIII—X классов, а затем, с 1940 г., были об- образованы две группы: для VII—VIII и IX—X классов. В своих выступлениях лекторы излагали в популярной форме серьезные математические результаты, включая научные достижения самых последних лет. В 1950 г. Гостехиздат (переименованный затем в Физматгиз и вошедший теперь в издательство «Наука») начал издавать спе- специальную серию книг «Популярные лекции по математике», большинство которых возникло при обработке лекций, прочи- прочитанных в математическом кружке при МГУ. Часть лекций была также опубликована в сборниках «Математическое просве- просвещение». Наряду с лекциями регулярно проводились и секционные за- заседания кружка. Ими руководили, как правило, студенты стар- старших курсов и аспиранты мехмата МГУ (за исключением двух секций 1936—1937 гг.: секции геометрических методов теории чисел, руководимой Л. Г. Шнирельманом, и секции ка- качественной геометрии, руководимой А. Н. Колмогоро- Колмогоровым). Сначала на секциях в основном делали доклады школь- школьники, но вскоре выяснилось, что такие доклады являются мало- малоэффективной формой работы кружка. Дело в том, что большин- большинство сообщений школьников на секциях оказывались скучными для всех участников кружка (кроме, быть может, самого до- докладчика). Ведь для того, чтобы сделать хороший доклад, не- недостаточно полностью понять все то, что сказано в отрывке ма- математического текста, указанного руководителем секции. До- Доклад должен заинтересовать слушателей и заставить их заду- задуматься над услышанным; в нем должны быть выпукло препод- преподнесены подлежащие рассмотрению задачи, должны быть отте- оттенены основные идеи решения, ярко обрисованы красивые, ори- оригинальные места доказательств и т. д. Кроме того, известно, что хорошую лекцию редко может прочесть человек, познания ко- которого в данном вопросе ограничиваются лишь конкретным ма- материалом, связанным лишь с данной темой. Поэтому доклад школьника обычно уступает по качеству рассказу опытного лектора. Решительная перестройка работы секций связана с именем талантливого математика и блестящего педагога Д. О. Шкля- р с к о г о, в то время студента МГУ, руководившего рабо- работой кружков в 1938—1941 гг. (В 1942 г. Д. О. Шклярский 7
d возрасте 23 лет погиб в партизанском отряде, сражавшемся с немецко-фашистскими захватчиками в тылу врага.) Доклады школьников на заседаниях его секции, за редким исключением, не заслушивались, а вместо этого руководитель каждый раз сам читал краткую лекцию по той или иной теме. В конце лекции школьники рассказывали о решенных ими задачах, предлагав- предлагавшихся на предыдущих заседаниях. Наличие задач разной труд- трудности позволяло Д. О. Шклярскому вовлекать в активную работу практически всех участников секции, а повторение рас- рассказанного школьником решения задачи достигало сразу двух целей: слушатели лучше усваивали это решение, а автор решения получал наглядный урок ясного и четкого изложения матема- математического доказательства. Такая система принесла замечательные плоды: в 1938 г. на IV олимпиаде участники секции Д. О. Шклярского за- завоевали половину всех премий A2 из 24), в том числе все 4 пер- первые премии! Итоги IV олимпиады произвели на руководителей других секций настолько сильное впечатление, что уже со сле- следующего года практически все они пересмотрели форму заня- занятий с учащимися. С тех пор форма работы кружка, найденная Д. О. Ш к л я р с к и м, стала господствующей. С самых первых лет работы кружка возникла традиция из- издания ежегодного небольшого сборника подготовительных задач к олимпиаде, который вручался участникам кружка и всем желающим принять участие в олимпиаде. Задачи первых четырнадцати Московских математических олимпиад и другие материалы секций кружка частично были опубликованы в трех книгах серии «Избранные задачи и тео- теоремы элементарной математики»; первая книга [25] посвящена арифметике и алгебре, вторая [26] и третья [27] — геометрии (планиметрии и стереометрии соответственно). Среди авторов этих трех книг первым указан Д. О. Ш к л я р с к и й, формально не участвовавший в подготовке издания, но всей своей деятель- деятельностью содействовавший появлению этих книг. Если кружок привлекал к систематической работе несколько сот московских школьников, то число участников Московской олимпиады всегда было значительно больше и достигало не- нескольких тысяч. Все аудитории во время проведения олимпиад в указанные годы были переполнены, и приходилось размещать часть школьников в лабораториях физического, химического и биологического факультетов МГУ. Форма проведения олимпиады практически не изменилась со времени I олимпиады 1935 г. Первые 36 олимпиад A935— 1973 гг.) проводились в два тура, по воскресеньям в конце мар- марта—начале апреля. 1-й тур являлся отборочным; на нем каж- каждому из участников предлагалось решить 4—6 сравнительно не- несложных (но с изюминкой!) задач, причем участники уведомля- уведомлялись о том, что для прохождения на 2-й тур достаточно решить 8
две задачи. Через неделю после 1-го тура проводился разбор предложенных задач с указанием различных решений и типичных ошибок и объявлялись результаты тура. Еще через неделю про- проходил 2-й тур, на который приглашались все успешно прошед- прошедшие 1-й тур C0—50% его участников); в некоторых случаях ко 2-му туру допускались школьники, не прошедшие 1-й тур (это происходило по рекомендации одного из членов оргкомитета, усмотревшего нетривиальность мышления школьника, а иногда по личной просьбе школьника), а также школьники, по каким-либо причинам не сумевшие прийти на 1-й тур (как правило, из-за бо- болезни). Задачи 2-го тура были уже существенно сложнее за- задач 1-го тура. На решение задач на каждом туре отводилось 5—6 часов. Наконец, через неделю после 2-го тура проводился оконча- окончательный разбор задач. Обычно для разбора задач, придуман- придуманных в основном студентами и аспирантами, привлекали крупных математиков. Делалось это для того, чтобы соединить рассказы о решениях задач, предложенных на олимпиаде, с указанием более широких перспектив в математике. Так, один из нас был свидетелем того, как А. Н. Колмогоров перед аудиторией, состоявшей из нескольких сот восьмиклассников, разбирая ре- решение двух задач XXXVIII олимпиады, перешел к обсуждению современных проблем дискретной математики (теории графов и теории информации). В заключение проходило награждение победителей олим- олимпиады. Им вручались призы — математические книги с дарствен- дарственными надписями. Победителям присуждалось в среднем около десяти первых премий (по разным классам), около двадцати вторых и около тридцати третьих; кроме того, выдавалось не- несколько десятков похвальных отзывов 1-й и 2-й степени. При под- подведении итогов олимпиады и награждении победителей резуль- результаты 1-го тура, как правило, не учитывались. Задачи первых пяти олимпиад предлагались всем школьни- школьникам без разделения их на классы. Начиная с VI олимпиады A940 г.) учащиеся разделялись на два потока: отдельно сорев- соревновались школьники VII—VIII классов и отдельно — старше- старшеклассники. Во время Великой Отечественной войны произошел трехлетний перерыв в проведении Московских олимпиад — с 1942 по 1944 г.; в эти годы московские математики провели олимпиады в Ашхабаде и Казани. К сожалению, никаких материалов тех олимпиад не сохранилось. Начиная с XV олимпиады A952 г.) соревнования проводи- проводились уже по каждому классу в отдельности, хотя некоторые наибо- наиболее интересные задачи предлагались параллельно в нескольких классах. С самого начала проведения олимпиад большую организа- организационную работу взяли на себя Московский городской отдел на- народного образования и Московский городской институт усовер- 9
шенствования учителей. Сотрудники института совместно с наи- наиболее опытными учителями и преподавателями МГУ с 1949 г. стали проводить районные математические олимпиады; задачи этих олимпиад приведены в книге [11]. Это позволило привлечь к занятиям математикой еще более широкий круг школьников, не только старшеклассников, но и учеников V—VII классов. Если школьный математический кружок на протяжении при- примерно четверти века был господствующей формой внеклассной математической работы со школьниками, а Московская олим- олимпиада была, так сказать, фокусом, в котором сходились все линии этой работы, то за последние 20 лет формы такой работы стали заметно разнообразнее. Возникли специализированные матема- математические школы; в 1963 г. была создана вечерняя математи- математическая школа (ВМШ), а через год возникла заочная математи- математическая школа (ЗМШ). Вслед за МГУ свои математические олим- олимпиады стали проводить другие вузы Москвы; наряду с город- городскими олимпиадами возникла система республиканских, Всесо- Всесоюзных и, наконец, Международных олимпиад. Тем не менее Московская олимпиада долгое время не переставала играть осо- особую роль: получить на ней награду считалось весьма почетным, а по уровню сложности задач она превосходила все остальные математические олимпиады. Иногда время проведения несколь- нескольких олимпиад совпадало, и некоторые победители Московской олимпиады часто не имели возможности прийти на ее закры- закрытие, поскольку в это же самое время они участвовали в другой олимпиаде. Тогда им высылалось специальное приглашение для получения положенной награды. Текст приглашения носил шутливый характер: «Награждение наградами награжденных, не награжденных наградами на награждении 17 марта, состо- состоится 1 апреля». В 1961 г. на XXIV Московскую олимпиаду были приглашены команды областей, краев и союзных республик — так было по- положено начало проведению единой многоэтапной математи- математической олимпиады по всей стране. Первая Всесоюзная математическая олимпиада состоялась 16 апреля 1967 г. в г. Тбилиси. Поскольку 2-й тур XXX Московской олим- олимпиады проходил в тот же день, команду от Москвы пришлось фор- формировать по результатам 1-го тура. В последующие годы сроки Московских олимпиад были перенесены с апреля на март, с тем чтобы успеть отобрать команду на проходящую в середине апре- апреля Всесоюзную олимпиаду. Сначала команда Москвы отбиралась непосредственно по результатам Московской олимпиады. Позже стали устраивать- устраиваться дополнительные «отборочные туры», на которые приглаша- приглашались 15—20 человек, получившие первые, вторые, иногда даже третьи премии. Задачи отборочного тура выбирались, как пра- правило, из первоначально предлагавшихся для олимпиады и не включенных в нее по разным причинам. Отборочный тур, одна- 10
ко, никогда не считался дополнительным (завершающим) туром олимпиады. Дополнительный тур был проведен только один раз — на XXXIII Москов- Московской олимпиаде для учащихся VII класса A970 г.); он получил название «День Пифагора». Этот тур был проведен с отклонениями от сложившихся тради- традиций. Сначала школьникам предложили решить три задачи. Через два часа все работы участников были собраны и объявлен получасовой перерыв, после окон- окончания которого были даны еще три задачи. Интересным было и другое новшество: одну из задач «Дня Пифагора» C3.42) оргкомитет дал, не предполагая определенного на нее ответа (точнее, этот ответ был неизвестен): школьникам объявили, что задача эта исследо- исследовательская и надо попытаться продвинуться в ее решении как можно дальше. Другая задача «Дня Пифагора» C3.45), как сейчас выяснилось, полу- получила неожиданное распространение за рубежом, и в недавно изданной книге «Математический цветник» [18] в заметке «Занимательное столоверчение» приведено ее обобщение на случай «многорукого Али-Бабы». В заметке ска- сказано, что сначала она побывала на страницах февральского номера журнала „Scientific American" за 1979 г., где ее опубликовал Мартин Гарднер — известный популяризатор математики, хорошо знакомый советскому читателю по целому ряду книг и статей ([5] — [8]); однако сам Гарднер призна- признается, что задачу ему сообщил «Роберт Таппей из Торонто, по мнению ко- которого задача пришла к нам из Советского Союза» [18]. Таким образом, за- задача совершила большой путь, прежде чем вернуться домой, впрочем, в обоб- обобщенном виде. Если до 60-х годов организацией и проведением Москов- Московских олимпиад занимались преподаватели и аспиранты мех- мехмата МГУ и сотрудники Института математики АН СССР им. В. А. Стеклова, то позже и в других вузах и научных учрежде- учреждениях Москвы появилось много молодых математиков, которые с увлечением стали заниматься подготовкой задач и проведением олимпиад. Некоторые вузы столицы стали проводить свои соб- собственные олимпиады; кроме того, намного поднялся уровень районных олимпиад. Поэтому было решено привлечь к прове- проведению Московских олимпиад, кроме МГУ, еще два вуза: М Г П И им. В. И. Ленина и Московский институт инженеров транспорта (М И И Т), что и было сделано в 1975 г., а также проводить Мо- Московскую олимпиаду для младших школьников только в один тур. Начиная с 1981 г. олимпиаду по VII классу проводил факуль- факультет вычислительной математики и кибернетики МГУ; в настоя- настоящее время он проводит олимпиаду и по IX классу. Заседания оргкомитета по обсуждению задач, подведению итогов олим- олимпиады и другим вопросам проводились совместные, как правило, в МГУ; там же проводились разбор задач олимпиады и награж- награждение победителей. Первый тур XXXVII олимпиады A974 г.) проводился толь- к о для учащихся IX—X классов, а на XXXVIII—XL олим- олимпиадах — только для десятиклассников. Результаты 1-го тура на этих олимпиадах учитывались при подведении итогов, а школьники, решившие все задачи 1-го тура, получали премию или отзыв на разряд выше, чем могли бы получить просто по результатам 2-го тура. Начиная с XLI олимпиады 1978 г. было и
решено проводить по всем классам только один, заклю- заключительный тур, к участию в котором допускались побе- победители районных математических олимпиад. Мы не будем здесь подробно останавливаться на вопросах, связанных с работой оргкомитета олимпиады. Скажем только, что проведение олимпиады, выпуск сборника подготовительных задач, составление большого списка новых задач и отбор вари- вариантов задач олимпиады, а также проверка работ школьников и обсуждение различных организационных вопросов — колоссаль- колоссальная работа, о которой большинство школьников и не подозре- подозревает и которая не могла бы осуществляться без энтузиазма сту- студентов и аспирантов, без помощи Московского математического общества и других организаций. В период проведения олимпи- олимпиады рабочий день членов оргкомитета не нормирован: нередко он длится с раннего утра до поздней ночи. Работы проверяются и перепроверяются по нескольку раз, и немало школьников обя- обязаны своими премиями внимательному отношению членов орг- оргкомитета, не поленившихся еще и еще раз вчитаться в их работы и найти в них рациональное зерно. Что касается самих задач олимпиады, то, помимо требова- требований красоты, яркости, идейной содержательности и новизны, предъявляемых к каждой отдельной задаче, весьма серьезные требования предъявляются ко всему ансамблю задач в целом. Задачи должны быть разнообразны как по форме, так и по ле- лежащим в основе их решения идеям, однако эти решения не долж- должны выходить за рамки существующей школьной программы. Как правило, среди пяти-шести задач две задачи более простые; алгебраические и текстовые задачи перемежаются с геометри- геометрическими, и сложность задач обычно возрастает с увеличением номера задачи в варианте. Естественно, что из наиболее удач- удачных задач, отобранных для олимпиады, более легкие предлага- предлагались на 1-й тур или в младшие классы, а более сложные остав- оставлялись для заключительного тура. Отметим, что все предлагающиеся на олимпиаде задачи не- нестандартны. Их новизна и привлекательность в значительной степени объясняются тем, что они одухотворены свежими идея- идеями современной математики и каждая из них — это маленькая научная работа, открывающая решающему задачу новые гори- горизонты. Многие задачи связаны с различными разделами мате- математики, физики, механики. Приведем несколько примеров таких задач. Задача 9.14 появилась из кристаллографии и связана с ростом кристаллов. Задача 9.15 относится к тео- теории конечных проективных плоскостей. За- Задача 13.06 взята из знаменитого труда создателя неевклидо- неевклидовой геометрии Н. И. Лобачевского «Воображаемая геометрия». Задача 15.32 связана с задачей Лагранжав небес- небесной механике. С понятием «притягивающей» и «отталкиваю- 12
щей» точек в методе итераций связана задача 20.37 (все уравнения этой задачи имеют один и тот же вид: они, как говорят, «итерируют» первое уравнение). В решениях задач 21.19 и 31.24 используется понятие мировой линии в пространстве-времени. 26.31 — задача о диа- диаграммах Юнга, применяющихся в теории представлений симметрических групп. 29.13 и 29.18 — типичные задачи теории информации. Задача 30.31 — «преобразова- «преобразование пекаря» из эргодической теории. 31.13 — пер- первая задача теории кодирования {«проверка на четность» Хэмминга). Вопрос, поставленный в задаче 47.19, относится к одному из способов плотной упаковки ин- информации в памяти ЭВМ, а задачи 45.01, 45.17 и 48.18 косвенно связаны с теорией алгоритмов и вы- вычислен и й. 3 а д а ч а 48.15 взята из записных книжек вы- выдающегося математика Леонарда Эйлера. Подобные примеры можно было бы продолжить (например, указать ряд задач по теории чисел); к сожалению, в большинстве случаев, оставаясь на «школьном уровне», невоз- невозможно объяснить суть дела более подробно. Подготавливая решения, мы, где это возможно, стремились в примечаниях к задачам осветить связь с современной мате- математикой. Мы старались также указать идейные взаимосвязи между задачами разных олимпиад или одной и той же олимпи- олимпиады (так, например, вы легко заметите, что задача 40.23 явля- является развитием задачи 40.01). Кроме того, за последние 50 лет школьные программы не раз менялись и новые веяния в програм- программах тут же отражались на содержании задач Московских олим- олимпиад: в соответствующие годы предлагались задачи на комп- комплексные числа, задачи с производной и т. п. Со- Составленные нами решения и указания ориентированы на нынеш- нынешнюю школьную программу, хотя стоит сказать, что в ряде слу- случаев от участников олимпиады предполагалось иное, зачастую более громоздкое решение. Так, в задаче 15.32 мы воспользо- воспользовались известными нынешним школьникам свойствами интегра- интеграла и привели решение, занимающее несколько строк (в отли- отличие от более длинного решения этой задачи в книге [25] ). При всей огромной помощи энтузиастов составление и отбор задач олимпиады представляет одну из самых трудных частей работы оргкомитета. Этому посвящается целый ряд многоча- многочасовых заседаний, на которых члены оргкомитета спорят до хри- хрипоты, отстаивая одни задачи и отвергая другие. Бывает, что за- задачи на глазах меняются до неузнаваемости: иногда несколь- несколько, казалось бы, совершенно разных идей формируются в одну задачу, иногда же, напротив, одна задача распадается на две- три другие, быть может, даже из разных разделов математики. При отборе задач на олимпиаду их приходилось, с одной стороны, «засекречивать», а с другой стороны, пытаться разны- 13
ми способами выяснять, не давали ли руководители кружков (никогда не являвшиеся членами оргкомитета олимпиады) та- такой же или похожей задачи своим питомцам. Выяснение этого обстоятельства всегда было тонким делом: неизвестная ранее задача могла случайно «рассекретиться», широко распростра- распространившись среди студентов. Поэтому отбор задач представлял всегда большие трудности. Список олимпиадных задач нередко оказывался утвержден- утвержденным за день-два до тура, а бывало, что этот список печатался в ночь перед открытием олимпиады. Составление окончательных вариантов буквально накануне соответствующих туров позволя- позволяло сохранить задачи в секрете, а также воспользоваться всеми счастливыми находками, сделанными зачастую в последний мо- момент. Трудность задач на олимпиадах в разные годы заметно ко- колебалась. Самые трудные задачи первых олимпиад, которые решались отдельными их участниками, ныне стоят на уровне рядовых задач. Внимательный читатель заметит, что сам стиль задач поздних олимпиад существенно изменился по сравнению со стилем задач первых олимпиад. Однако сложность олимпиады в каждый конкретный год всегда была очень высокой. Иногда даже при всем желании облегчить варианты задач это не уда- удавалось сделать. Бывали и совсем трудные олимпиады. Таковы XXVII A964 г.), XXIX A966 г.), XXXI A968 г.), XXXV A972 г.); на этих олимпиадах некоторые задачи не решил никто (в ка- качестве примеров можно привести задачи 29.15, 31.26, 31.27, 35.32), а некоторые — только один участник (например, зада- задачу 35.30). Юбилейную XLVIII олимпиаду A985 г.) нельзя от- отнести к разряду очень трудных, но и на ней были задачи, кото- которые не решил ни один школьник в своем классе (задачи 48.04, 48.10, 48.15). Но это скорее исключение, чем правило; на большинстве олимпиад каждая задача были решена хотя бы одним участником и были довольно сложные задачи, которые решались многими участниками олимпиады. Самое сильное влияние на дух и характер олимпиады, содер- содержание и трудность ее задач оказывали профессора мехмата МГУ, возглавлявшие в разные годы оргкомитет Московской олим- олимпиады и направлявшиеся на эту работу правлением Московского математического общества. Председателями оргкомитетов в раз- разные годы являлись: академики АН СССР П. С. Александ- Александров A935 г.), А. Н.'Кол могоров A937 г., 1963 г., 1975 г.), И.Г.Петровский A947 г.), Л..С. П о н т р я г и н A940 г.), И. М. Гельфанд A945 г.); члены-корреспонденты АН СССР Б. Н.-Делоне A951 г.), Л. А. Люстерник A939 г.), Д. Е. Меньшов A953 г.), А. О. Гельфонд A941 г.), И. Р. Шафаревич A960 г., 1964 г., 1971 г.), Н. В. Ефи- Ефимов A962 г., 1965 г.), Н. С. Б а хвал о в A968 г.), В. И. А р- нольд A974 г.), О. Б. Л у па нов A981 — 1986 гг.); дей- 14
ствительный член АПН СССР А. И. Маркушевич A949 г.); член-корреспондент АПН СССР В. Г. Болтянский A958 г.); профессора, доктора физико-математических наук Н. А. Гла- Глаголев A936 г.), А. Г. Кур о ш A938 г.), С. А. Г а л ьперн A946 г.), В. В. Немыцкий A948 г., 1967 г.), М. А. К р е й- нес A950 г.), П. К. Ра шевский A952 г.), С. В. Б а х в а- лов A954 г.), Г. Е. Шилов A955 г.), О. А. Олейник A957 г.), Е. М. Ландис A959 г.), В. А. Ефремович A961 г., 1969 г.), А. А. Кронрод A966 г.), В. М. Алексе- Алексеев A970 г.), Б. П. Демидович A972 г.), А. А. Кирил- Кириллов A973 г.), А. В. А р х а н г е л ь с к и й A976 г.), В. А. У с- пенский A977 г.), Ю. И. Манин A978 г.), В. М. Ти- Тихомиров A979 г.), А. С. Мищенко A980 г.). Предсе- Председатели оргкомитетов не только проводили большую организа- организационную работу, но и руководили многочисленными заседа- заседаниями оргкомитетов. В обязанности каждого председателя вме- вменялось также проведение разбора задач в одном из классов. Председатель имел основных помощников: заместите- заместителя (з а м п р е д а; до 60-х годов он назывался ответствен- ответственным секретарем) и старших по классам. У зам- зампреда была масса обязанностей: договориться о проведении лек- лекций для школьников; дать указания о расклейке афиш об олим- олимпиаде; организовать проведение самой олимпиады; напечатать подготовительный сборник задач и бланки грамот для награжде- награждения победителей; распределить аудитории, в которых будет про- проходить олимпиада, а позже участвовать в разборе задач и на- награждении победителей. Кроме того, зампред обязан принимать участие в обсуждении и составлении задач. Один из авторов этой книги, будучи зампредом, испытал на себе всю сложность его функций. Старшие по классам также очень сильно загружены: кроме помощи зампреду в организационных вопросах, они должны от- отбирать наиболее понравившиеся им задачи для своего класса, из которых оргкомитет составляет потом окончательный вариант задач по этому классу; во время олимпиады следить за ее про- проведением (по своему классу), обходить аудитории и отвечать на вопросы участников (в заранее предусмотренном оргкоми- оргкомитетом объеме); организовывать проверку работ участников сво- своего класса; выявлять лучшие работы, а все свои мнения выно- выносить на регулярно проводившиеся заседания оргкомитета. Во время проведения самой олимпиады ее обслуживают мно- многочисленные дежурные-студенты, которые помогают школьникам разойтись по аудиториям, сидят в аудиториях, следят за тем, чтобы коридоры и другие помещения не превращались в дискус- дискуссионные клубы участников олимпиады, а к концу тура собирают работы участников. Затем старшие по классам распределяют собранные работы между студентами для проверки. Проверка составляет самую ответственную часть работы; ка- 15
ждую сколько-нибудь интересную работу читают несколько чле- членов оргкомитета, а работы, выдвигаемые на премию, рассматри- рассматривают все члены оргкомитета. Разнообразие требований, предъ- предъявляемых к работам участников, делает оценку каждого отдель- отдельного члена оргкомитета довольно субъективной, и лишь коллек- коллективный характер процедуры присуждения премий обеспечивает правильность и объективность окончательного решения. Для оценки работы применяется не балльная система, а бо- более гибкая система плюсов и минусов. Основные оценки, кото- которые могут быть выставлены за задачу, таковы: (+) —задача полностью решена; ( + ) — задача решена, но решение содержит мелкие пропуски или огрехи; (± ) — задача решена не полностью; ( + ) —задача не решена, но подход к решению правилен; (—) —задача не решена, или решена неправильно; (-г-) или (е) — задача не решена, но в черновиках или чистовой работе обнаружены некоторые разумные соображения; (—?) —ре- —решение неверно и содержит очень грубые ошибки. Употребля- Употребляется еще знак (!) (например, ( + !) или ( + !)), который озна- означает, что в решении, полном или неполном, имеются яркие и нео- неожиданные (иногда непредвиденные оргкомитетом) идеи. Напри- Например, на IX олимпиаде первую премию получил ученик X класса Эрик Балаш. Все отведенное на олимпиаде время он затратил на решение задачи 9.17, которая предполагалась оргкомитетом сравнительно несложной. Однако вместо ответа на конкретный вопрос, поставленный в задаче, Эрик провел подробное иссле- исследование, указав закон повторяемости тех членов последователь- последовательности Фибоначчи, которые оканчиваются четырьмя нулями, и правильно определил номер первого такого члена (равный 7501). Решение было удостоено оценки (±!) (Эрик не успел на олим- олимпиаде довести свое исследование до конца), и, несмотря на то что к решению остальных задач он и не приступал, ему была при- присуждена первая премия. /\ Над основным материалом нашей книги — задачами — тру- трудилось несколько поколений замечательных математиков, и боль- большинство задач несет на себе отпечаток незаурядности и кра- красоты. Установление авторства большинства задач Московских олимпиад (а их около 1300) потребовало бы в данный момент со- совершенно непосильной исследовательской работы. Составить да- даже приблизительный список авторов задач невозможно хотя бы потому, что часть задач возникала уже «на месте», в оргкоми- оргкомитете, и автор у них коллективный. Однако наиболее красивые (по признанию оргкомитета) и оригинальные задачи олимпиады придуманы вполне конкретны- конкретными авторами. Такие задачи получают сначала в оргкомитете (а после окончания олимпиады и в других кругах) широкое рас- 16
пространение, превращаясь в математический фольклор; «зна- «знатоки» сразу вспоминают их по кратким специфическим назва- названиям. Приведем фамилии нескольких авторов подобных задач: Н. Н. Константинов — 23.27 («Улитка»); Н. Б. Васильев — 25.39 («Коробочка»), 26.35 («Джентльмены»); А. С. Шварц — 24.32 («Матрица Шварца»); Б. Д. Гинзбург — 23.31 («Обход конем»); А. Г. Кушниренко — 33.36 («Апельсин»); А. П. Савин — 33.11 («Крайние короли»); А. Л. Тоом — 35.12 («Лес»); Д. Б. Фукс — 24.07 («Игра в скалярное произведение»), 26.50 и 31.31 («Фук- совы дуги»); Г. А. Гальперин — 34.20, 38.23, 39.04 («Прожекто- («Прожекторы»), 33.25 («Придворные короля Людовика»), 38.10 («Двойки, тройки и четверки»), 39.20 («Пятаки»), 48.12 («Аэродромы»); А. К. Толпыго — 31.13 («Телефоны»), 35.37 («Игра в карточки»); О. В. Ляшко — 35.23 («Странствующий рыцарь»); А. В. Зеле- винский — 34.33 («Банкир и Игрок»); М. Р. Ковтун — 34.15 («Спички»); А. В. Климов — 37.24 («Полоски»), 37.27 («Асте- («Астероиды»); А. И. Грюнталь — 36.14 («Многогранник»); С. А. Ели- Елисеев — 38.19 («Невыпуклое разрезание»); С. В. Конягин — 42.09, 42.14 («Химики и алхимики»); С. Б. Гашков — 45.17 («Машин- («Машинка Тьюринга»), 48.04 («Волки и заяц»); И. Н. Сергеев — 48.09 («Дядька Черномор»). Этот список было бы легко продолжить, но почти невозмож- невозможно завершить, и мы приносим свои извинения всем неупомяну- неупомянутым авторам задач Московских олимпиад. Считаем также своим долгом особо отметить многолетнюю работу Н. Н. Константино- Константинова по проведению Московских олимпиад и других математиче- математических соревнований школьников, душой которых он неизменно являлся. Академик АН СССР А. Н. Колмогоров, активно участво- участвовавший в проведении Московских математических олимпиад на протяжении всего их существования, много сделал и для нашей книги в качестве ее редактора и научного консультанта; мы поль- пользуемся случаем выразить ему нашу самую горячую признатель- признательность. Мы признательны также всем тем, кто помогал нам в ра- работе над сборником и в подготовке его публикации, в первую очередь В. М. Тихомирову, Н. Б. Васильеву, А. М. Абрамову, а также А. П. Савину, С. М. Саакяну, А. Л. Тоому, Е. А. Моро- Морозовой, Р. С. Черкасову. Мы надеемся, что еще много лет в аудиториях МГУ будут шуметь звонкие голоса школьников и будут раздаваться во- вопросы: — Когда следующая олимпиада? Г. А. Гальперин А. К. Толпыго
Указания к работе с книгой Наша книга состоит из двух частей. В первой части «Зада- «Задачи Московских математических олимпиад» приведены тексты задач всех Московских олимпиад с 1935 по 1985 г. включитель- включительно в том виде, как они предлагались на олимпиадах. Условия почти всех задач первых 27 олимпиад взяты из книги [21]; задачи олимпиад начиная с 1965 г. были собраны авторами настоящей книги. Следует, однако, отметить ряд пропусков в списке задач, приводимом в [21]. Так, в результате работы с литературой и архивами нам удалось обнаружить отсутствовавшие в [21] за- задачи 2.10; 10.04; 10.05; 10.10; 10.11; 11.11; 11.12; 11.17; 12.17; 15.13; 15.18; 17.25. Кроме того, был найден полный вариант XIV олимпиады (см. [37]), мало совпадающий, как выяснилось, с опубликованным в [21]. XIV олимпиада полностью реконструи- реконструирована. Отыскались также задачи 1-го тура II олимпиады и комплект задач для VII класса, предлагавшийся на 2-м туре XIX олимпиады. Однако по-прежнему не обнаружен 1-й тур IV олимпиады. Вторая часть нашей книги содержит ответы, решения и ука- указания к большинству задач. Большинство задач снабжено указаниями или ответами, и лишь некоторые наиболее интересные задачи — подробными ре- решениями. Задачи во второй части книги имеют двойную нумера- нумерацию: первое число означает номер олимпиады, второе — поряд- порядковый номер задачи в этой олимпиаде. Например, 12.04 озна- означает: XII олимпиада, задача 4. Тех же обозначений мы при- придерживаемся и при ссылках на ту или иную задачу. В заключение приведем некоторые- обозначения, часто повто- повторяющиеся в условиях и решениях различных задач, и дадим к ним пояснения. 1) [х\ —«целая часть числа х»— наибольшее целое число, не превосходящее х (примеры: [5]= 5, [1,5]= 1, ~г =0, [л;]=3, [-1,5]=-2 и т. д.). L J 2) п\= 1 •2-3-4-...-(rt—1)-п (читается: «п — факториал»); 0!= 1! = 1; B«)!! = 2-4-6-... -B« — 2)-2« (произведение всех четных чисел от 2 до 2«); B«— 1)!! = 1 -3-5-...-B« — 3) B«— 1) (произве- (произведение всех нечетных чисел от 1 до 2п — 1). 3) Сп = -—~—— число сочетаний из п по /г (равном-у числу способов выбрать k элементов из /г). 4) ab...c — натуральное число, записанное цифрами а, Ь, ..., с. 5) Последовательность Фибоначчи — это последовательность (хп), в которой Хп — хп-\ -\-Xn-2- Чаще всего это последователь- последовательность 0,1,1,2,3,5,8,13,21,34, ... . 18
6) Во всех задачах о турнирах предполагается, если не ого- оговорено другое условие, что каждый играет с каждым по одному разу (турнир в один круг). В шахматном турнире победа дает 1 очко, ничья -——, поражение — 0; в футболе все очки вдвое больше. В баскетболе и теннисе ничьих не бывает. 7) Многоугольник называется вписанным в другой многоуголь- многоугольник, если его вершины лежат на сторонах (в том числе и в вер- вершинах) этого многоугольника. 8) Во всех задачах о луче света, бильярдном шаре и т. д. предполагается, что выполнен закон: угол падения равен углу отражения. 9) Все многоугольники, если нет оговорки, предполагаются несамопересекающимися и выпуклыми. 10) Длиной конечной последовательности называется число ее членов. —о— Предлагаемая книга может быть полезна педагогам — руко- руководителям факультативов и математических кружков; вероятно, она заинтересует студентов и научных работников. Но в первую очередь книга предназначается для школьников старших классов, любящих математику. Здесь есть задачи довольно простые (это, как правило, задачи первых туров), есть и трудные. Особенно трудные задачи отмечены звездочкой. Не спешите преждевременно заглядывать во вторую часть книги. Не жалейте потратить несколько часов или даже дней на одну задачу, и в случае, если упорные раздумья не при- приведут вас к цели, загляните, в ответ или решение. Если вы сами справились с решением задачи, просмотрите на всякий случай приводимое в книге решение или указание: вдруг в нем вы обна- обнаружите что-то ценное для себя. Итак, попытайтесь помериться силами с участниками про- прошедших Московских олимпиад! Желаем вам успехов! Авторы.
ЧАСТЬ I ЗАДАЧИ МОСКОВСКИХ МАТЕМАТИЧЕСКИХ ОЛИМПИАД I ОЛИМПИАДА A935 г.) 1-й тур 1. Поезд проходит мимо наблюдателя в течение t\ с, а мимо моста длиной / м в течение /г с. Считается, что поезд проходит мимо моста начиная с того момента, когда локомотив въезжает на мост, и кончая моментом, когда последний вагон покидает мост. Определить длину и скорость поезда. 2. Построить квадрат, три вершины которого лежат на трех данных параллельных прямых. 3. Найти объем правильной четырехугольной пирамиды со сторонами основания а и плоскими углами при вершине, рав- равными углам наклона боковых ребер к плоскости основания. 2-й тур. Серия А 4. Построить треугольник по точкам пересечения с описан- описанной окружностью продолжений медианы, биссектрисы и высоты, проведенных из одной вершины. 5. На поверхности куба найти точки, из которых диагональ видна под наименьшим углом. 2-й тур. Серия В 6. Сколько действительных решений имеет система xy-z2=\? 7. Решить систему f x3-y3 = 2b, I х2у— xy2 = b. 8. Вычислить сумму 13 + 334-53 + - + B«—IK. 2-й тур. Серия С 9. Сколькими способами можно раскрасить куб в шесть дан- данных цветов, если любые две различные грани должны быть раскрашены разными красками? (Различными считаются те рас- раскраски, которые не совмещаются при поворотах куба.) 20
10. Сколькими способами можно представить число п в ви- виде суммы трех целых положительных слагаемых? 11. Обозначим через М (а, Ь, с, ..., k) наименьшее общее кратное, а через D (а, Ь, с, ..., k) — наибольший общий дели- делитель чисел а, Ь, с, ..., k. Доказать: a) M(a,b)-D(a, b) = ab; б) II ОЛИМПИАДА A936 г.) 1-й тур 1. Найти четырехзначное число, являющееся полным квад- квадратом, первая цифра которого равна второй, а третья — чет- четвертой. 2. Построить такой равнобедренный треугольник, чтобы у любого вписанного в него прямоугольника, две вершины кото- которого лежат на основании треугольника, периметр был величи- величиной постоянной. 3. Представить произвольное натуральное число в виде вы- выражения, в запись которого входят только 3 двойки и произ- произвольные математические знаки. (Задача П. Дирака.) 4. Дан круг и точка Р вне его. С помощью циркуля и ли- линейки построить диаметр круга, который виден из точки Р под данным углом. 5. Найти 4 последовательных натуральных числа, произве- произведение которых равно 1680. 2-й т у р 6. Решить систему / = а, 7. Дан угол, меньший 180°, и точка М вне его. Провести через точку М прямую так, чтобы треугольник, одна из вер- вершин которого совпадает с вершиной данного угла, а две другие есть точки пересечения его сторон с проведенной прямой, имел данный периметр. 8. Доказать, что если стороны прямоугольника и его диа- диагональ — целые числа, то его площадь — целое число, крат- кратное 12. 9. Сколькими способами можно разложить миллион в про- произведение трех множителей? Разложения, отличающиеся поряд- порядком множителей, считаются различными. 10. В пространстве расположены три плоскости и шар Сколь- Сколькими различными способами можно поместить в пространстве 21
второй шар так, чтобы он касался трех данных плоскостей и первого шара? Ill ОЛИМПИАДА A937 г.) 1-й тур 1. Решить систему 2*. Даны прямая и две точки А и В по одну сторону от нее. Найти на прямой такую точку М, чтобы сумма МА-\-МВ равнялась заданному отрезку. 3. По двум скрещивающимся прямым скользят два отрезка. Доказать, что объем тетраэдра с вершинами в концах этих отрезков не зависит от положения последних. 2-й т у р 4. Даны три точки, не лежащие на одной прямой. Через каж- каждые две из них провести окружности так, чтобы три проведен- проведенные окружности имели в точках пересечения взаимно перпен- перпендикулярные касательные. 5*. В пространстве расположен правильный додекаэдр. Сколь- Сколькими способами можно провести плоскость так, чтобы она вы- высекла на додекаэдре правильный шестиугольник? 6. На сколько частей разделяют я-угольник его диагонали, если никакие три диагонали не пересекаются в одной точке? IV ОЛИМПИАДА A938 г.) 2-й т у р 1. В пространстве даны точки О\, О2, О3 и точка А. Точка А симметрично отражается относительно точки О\, полученная точка А\ — относительно Ог, полученная точка Л2 — относитель- относительно Оз. Получаем некоторую точку Аз, которую также после- последовательно отражаем относительно О\, О2, Оз. Доказать, что последняя полученная точка совпадает с А. 2. На сколько частей могут разделить пространство п пло- плоскостей? 3. Построить треугольник по основанию, высоте и разности углов при основании. 4. Сколько существует натуральных чисел, меньших тысячи, которые не делятся ни на 5, ни на 7? 22
V ОЛИМПИАДА A939 г.) 1-й тур 1. Решить систему уравнений 3xyz-x3-y3-z3 = b3, I 2. Доказать, что cos -^-+cos ^-— —^- О О Z 3. Даны три точки А, В, С. Через точку А провести прямую так, чтобы сумма расстояний от точек В и С до этой прямой была равна заданному отрезку. 4. Решить уравнение -\а— VG-\-x = x. 5. Доказать, что во всяком неравнобедренном треугольни- треугольнике биссектриса лежит между медианой и высотой, проведенными из той же вершины. 2-й т у р 6. Разложить на целые рациональные множители выражение а]0 + а5+\. 7. Даны два многочлена от переменной х с целыми коэффи- коэффициентами. Произведение их есть многочлен с четными коэффици- коэффициентами, не все из которых делятся на 4. Доказать, что в одном из многочленов все коэффициенты четные, а в другом — хотя бы один нечетный. 8. Даны две точки А и В и окружность. Найти на окруж- окружности точку X так, чтобы прямые АХ и ВХ отсекли на окружно- окружности хорду CD, параллельную данной прямой MN. 9. Найти остаток от деления на 7 числа 10. Дана правильная пирамида. Из произвольной точки Р ее основания восставлен перпендикуляр к плоскости основа- основания. Доказать, что сумма отрезков от точки Р до точек пере- пересечения перпендикуляра с плоскостями граней пирамиды не за- зависит от выбора точки Р на основании. 11. На какое самое большое число частей можно разбить пространство пятью сферами? VI ОЛИМПИАДА A940 г.) 1-й тур VII—VIII классы 1. Разложить на множители: (b — cK-\-(c — df-\-{a — bf. 2. Пароход от Горького до Астрахани идет 5 суток, а от 23
Астрахани до Горького — 7 суток. Сколько дней будут плыть по течению плоты от Горького до Астрахани? 3. Сколькими нулями оканчивается произведение всех целых чисел от 1 до 100 включительно? 4. Провести окружность данного радиуса, касающуюся дан- данной прямой и данной окружности. Сколько решений имеет за- задача? IX—X классы 5. Решить систему уравнений I х-\-у — Ъ. 6. Все целые числа, начиная с единицы, выписаны подряд. Таким образом, получается следующий ряд цифр: 123456789101112131415... . Определить, какая цифра стоит на 206788-м месте. 7. Построить окружность, равноудаленную от четырех точек плоскости. Сколько решений имеет задача? 8. В плоскости даны две прямые. Найти множество всех точек, разность расстояний которых от этих прямых равна за- заданному отрезку. 9. Найти все трехзначные числа, равные сумме факториалов своих цифр. 2-й т у р VII — VIII классы 10. Найти четырехзначное число, являющееся точным квад- квадратом, если первые две цифры, а также последние две цифры его одинаковы. 11. Точки А, В, С — вершины вписанного в окружность правильного треугольника. Точка D лежит на меньшей дуге АВ. Доказать, что AD + BD = DC. 12. Данным четырехугольником неправильной формы наст- настлать паркет, т. е. покрыть всю плоскость четырехугольниками, равными данному, без пропусков и перекрытий. 13. Сколько существует таких пар целых чисел х, у, за- заключенных между 1 и 1000, что х2-\-у2 делится на 49? IX — X классы 14. На бесконечном конусе, угол развертки которого равен а, взята точка. Из этой точки в обе стороны проводится ли- линия так, что после развертки она превращается в отрезки пря- прямых. Найти число ее самопересечений. 24
15. Что больше: 300! или 100300? 16. Центр О описанной около треугольника ABC окружно- окружности отражается симметрично относительно каждой из прямых, содержащих сторону дЛВС. По трем полученным точкам О\, Ог, Оз восстановить треугольник ABC, если все остальное стерто. 17. Доказать неравенство О2 Оз ал а„ а, (щ, п2, ..., ал—I, ап — положительные числа) 18. Сколько существует положительных целых чисел х, меньших 10 000, для которых 2* —jc делится на 7? VII ОЛИМПИАДА A941 г.) 1-й тур VII — VIII классы 1. Построить треугольник, если известны его высота и ме- медиана, выходящие из одной вершины, и радиус описанной около треугольника окружности. 2. Дописать к 523 ... три цифры так, чтобы полученное шести- шестизначное число делилось на 7, 8 и 9. 3. Дан четырехугольник; А, В, С, D — последовательные се- середины его сторон, Р, Q — середины диагоналей. Доказать, что треугольник ВСР равен треугольнику ADQ. 4. Через точку Р, лежащую вне окружности, проводятся все- всевозможные прямые, пересекающие эту окружность. Найти мно- множество середин хорд, отсекаемых данной окружностью на этих прямых. 5. Доказать, что произведение четырех последовательных целых чисел в сумме с единицей дает полный квадрат. IX — X классы 6. См. задачу 2. 7. На сторонах параллелограмма вне его построены квадра- квадраты. Доказать, что их центры являются вершинами некоторого квадрата. 8. Доказать, что многочлен с целыми коэффициентами аох" + а\хп~] +... + ап- \ принимающий при х = 0 и *= 1 нечетные значения, не имеет целых корней. 9. Построить треугольник ABC по точкам М и N — осно- основаниям высот AM и BN, и прямой, на которой лежит сторо- сторона АВ. 25
10. Решить уравнение \х + 11 - U| +3U- 11 -2U-2I =х 11. Сколько корней имеет уравнение sin je=-^-? 2-й т у р VII—VIII классы 12. Доказать, что из пяти различных по величине квадратов нельзя сложить прямоугольник. 13. Дан треугольник ЛВС. Требуется разрезать его на наи- наименьшее число частей так, чтобы, перевернув эти части на дру- другую сторону, из них можно было сложить тот же треуголь- треугольник ABC. 14. Дан треугольник ABC. Точка М, лежащая внутри него, движется параллельно стороне ВС до пересечения со стороной СА, затем параллельно стороне АВ до пересечения со стороной ВС, затем параллельно стороне СА и т. д. Доказать, что через некоторое число таких шагов точка вернется в исходное поло- положение, и найти это число. 15. Найти целое число а, при котором (х — а)(х—10)+1 раз- разлагается в произведение (х + Ь) (х -\- с) с целыми Ъ и с. 16. Доказать, что квадрат любого простого числа р>3 при делении на 12 дает в остатке 1. 17. Построить треугольник ABC по трем точкам Н\, #2, Н3, которые являются симметричными отражениями точки пересе- пересечения высот искомого треугольника относительно его сторон. IX—X классы 18. Доказать, что из шести попарно различных по величине квадратов нельзя сложить прямоугольник. 19. Некоторое количество точек расположено на плоскости так, что каждые 3 из них можно заключить в круг радиуса г= 1. Доказать, что тогда и все точки можно заключить в круг радиу- радиуса 1. 20. Найти такие отличные от нуля неравные между собой целые числа а, Ъ, с, чтобы выражение х {х — а) {х — Ь) (х — с)+ 1 представлялось в виде произведения двух многочленов с целы- целыми коэффициентами. 21. Решить в целых числах уравнение х-\-у = х2 — ху-\-у2. 22. В пространстве даны две скрещивающиеся перпендику- перпендикулярные прямые. Найти множество середин всех отрезков данной длины, концы которых лежат на этих прямых. 23. Построить прямоугольный треугольник по двум медиа- медианам, проведенным к катетам. 26
VIII ОЛИМПИАДА A945 г.) 1-й тур VII—VIII классы 1. См. задачу 6 для k = 7. 2. Доказать, что при любом целом положительном п спра- справедливо неравенство 7Г^Т+7Т^+-+^">Т • 3. Двузначное число в сумме с числом, записанным теми же цифрами, но в обратном порядке, дает полный квадрат. Найти все такие числа. 4. Доказать, что разносторонний треугольник нельзя разре- разрезать на два равных треугольника. 5. К двум окружностям, касающимся извне, проведены об- общие внешние касательные, и точки касания соединены между собой. Доказать, что в полученном четырехугольнике суммы противоположных сторон равны. IX—X классы 6. Разделить сР-Ь2" на (а + Ь) (a2 + b2) (G4 2"' ) 7. Найти трехзначное число, всякая натуральная степень которого оканчивается на три цифры, составляющие исходное число (в том же порядке). 8. Система уравнений второго порядка (х2-у2 = 0, \(х-аJ + у2=\ имеет, вообще говоря, четыре решения. При каких значениях а число решений системы уменьшается до трех или до двух? 9. Прямоугольный треугольник ABC с катетами а и Ъ дви- движется по плоскости так, что вершины А и В его острых углов скользят по сторонам данного прямого угла. Доказать, что мно- множеством точек С является отрезок, и найти его длину. 2-й т у р VII—VIII классы 10. Даны шесть цифр: 0, 1, 2, 3, 4, 5. Найти сумму всех четырехзначных четных чисел, которые можно написать этими цифрами (одна и та же цифра может повторяться). 11. Из картона вырезали два равных многоугольника, сов- совместили их и проткнули в некоторой точке булавкой. При пово- повороте одного из многоугольников около этой «оси» на 25°30' он снова совместился со вторым многоугольником. Каково наимень- наименьшее возможное число сторон у таких многоугольников? 12. Сторона AD параллелограмма ABCD разделена на п рав- равных частей. Первая точка деления Р соединена с вершиной В. 27
Доказать, что прямая ВР пересекает диагональ АС в точке Q такой, что /3Q= -AC. 13. Вершины А, В, С треугольника ABC соединены отрез- отрезками соответственно с точками А\, B]t C\, лежащими на про- противоположных сторонах треугольника. Доказать, что середины отрезков АА\, ВВ\, СС\ не лежат на одной прямой. IX—X классы 14. Решить в целых числах уравнение ху-\-Зх — Ъу=—3. 15. Некоторые из чисел щ, а2, .... а„ равны +1, остальные равны —1. Доказать, что 2 sin(a, В частности, при а{ =a2 = ... = an = 1 имеем: 16. Окружность радиуса, равного высоте некоторого правиль- правильного треугольника, катится по стороне этого треугольника. До- Доказать, что дуга, высекаемая сторонами треугольника на ок- окружности, все время равна 60°. IX ОЛИМПИАДА A946 г.) 1-й тур VII—VIII классы 1. Какое наибольшее число острых углов может встретиться в выпуклом многоугольнике? 2. На прямой даны три точки А, В, С. На отрезке А В построен равносторонний треугольник АВС\, на отрезке ВС по- построен равносторонний треугольник ВСА\. Точка М — середи- середина отрезка АА\, точка N — середина отрезка СС\. Доказать, что треугольник BMN равносторонний. (Точка В лежит между точками А и С; точки А\ и С\ расположены по одну сторону от прямой АВ.) 3. Найти четырехзначное число, которое при делении на 131 дает в остатке 112, а при делении на 132 дает в остатке 98. 4. Решить систему уравнений = —3, =— 9, = —6, = — 2, 28
5. Доказать, что в произведении после раскрытия скобок и приведения подобных членов не оста- останется членов, содержащих х в нечетной степени. IX—X классы 6. В пространстве даны две пересекающиеся плоскости а и р. На линии их пересечения дана точка А. Доказать, что из всех прямых, лежащих в плоскости а и проходящих через точку А, наибольший угол с плоскостью р образует та, которая перпен- перпендикулярна к линии пересечения плоскостей аир. 7. Через точку А, лежащую внутри угла, проведена пря- прямая, отсекающая от этого угла наименьший по площади тре- треугольник. Доказать, что отрезок этой прямой, заключенный между сторонами угла, делится в точке А пополам. 8. Доказать, что п2-\-Зп-\-5 ни при каком натуральном п не делится на 121. 9. Доказать, что для любого натурального п справедливо соотношение =jP=2"-Bn-l)!! 10. Доказать, что если а и C — острые углы и а<р, то 2-й т у р VII—VIII классы П. В шахматном турнире участвовали два ученика VII клас- класса и некоторое число учеников VIII класса. Два семиклассника набрали вместе восемь очков, а все восьмиклассники набрали по одинаковому числу очков. Сколько восьмиклассников участвова- участвовало в турнире? 12. Доказать, что выражение х5 + ЗА — 5*У — 15х2у3 + 4ху' + 12 </5 не равно 33 ни при каких целых значениях х и у. 13. На сторонах угла АОВ от вершины О отложены отрезки О А и ОВ, .причем ОА>ОВ. На отрезке О А взята точка М, на отрезке ОВ — точка N так, что AM = BN=x. Найти значение х, при котором отрезок MN имеет наименьшую длину. 14*. Из тридцати пунктов А\, А2, ..., Лзо, расположенных на прямой MN на равных расстояниях друг от друга, выходят тридцать прямых дорог. Эти дороги располагаются по одну сторону от прямой AfN и образуют с нею следующие углы: 29
№ № № 1 60° 11 125° 21 28° 2 30° 12 65° 22 158° 3 15° 13 85° 23 30° 4 20° 14 86° 24 25° 5 155° 15 80° 25 5° 6 45° 16 75° 26 15° 7 10° 17 78° 27 160° 8 35° 18 115° 28 170° 9 140° 19 95° 29 20° 10 50° 20 25° 30 158° Из всех тридцати пунктов выезжают одновременно трид- тридцать автомобилей, едущих, никуда не сворачивая, по этим доро- дорогам с одинаковой скоростью. На каждом из перекрестков уста- установлено по шлагбауму. Как только первая (по времени) машина проезжает перекресток, шлагбаум закрывается и преграждает путь всем следующим машинам, попадающим на этот перекре- перекресток. Какие из машин проедут все перекрестки на своем пути, а какие застрянут? 15. Автобусная сеть города устроена следующим образом: 1) с любой остановки на любую другую остановку можно попасть без пересадки; 2) для любой пары маршрутов найдется, и притом единственная, остановка, на которой можно пересесть с одного из этих маршрутов на другой; 3) на каждом маршруте ровно три остановки. Сколько автобусных маршрутов в городе? IX—X классы 16. В шахматном турнире участвовали ученики IX и X клас- классов. Десятиклассников было в десять раз больше, чем девятиклассников, и они набрали вместе в 4,5 раза больше очков, чем все девятиклассники. Сколько очков набрали девяти- девятиклассники? 17. Дана последовательность Фибоначчи 0, 1, 1, 2, 3, 5, 8, ... . Найдется ли среди 100 000 001 ее первых членов число, оканчи- оканчивающееся четырьмя нулями? 18. На сторонах PQ, QR, RP треугольника PQR отложены отрезки АВ, CD, EF. Внутри треугольника задана точка So. Найти множество точек S, для которых сумма площадей треуголь- треугольников SAB, SCD и SEF равна сумме площадей треугольников SoAB, SoCD, SoEF. Рассмотреть особо случай, когда ^-= _CD_EF QR ~ RP 19. В городе 57 автобусных маршрутов. Известно, что: 1) с любой остановки на любую другую остановку можно попасть без пересадки; 2)для любой пары маршрутов найдется, и притом 30
только одна, остановка, на которой можно пересесть с одного из маршрутов на другой; 3) на каждом маршруте не менее трех остановок. Сколько остановок имеет каждый из 57 марш- маршрутов? 20. См. задачу 14. X ОЛИМПИАДА A947 г.) 1-й тур VII—VIII классы 1. Найти остаток от деления многочлена л;243 + л;8|+л;27 + л;9 + +л:3 + л; на х— 1. 2. Доказать, что из 9 последовательных натуральных чисел всегда можно выбрать одно число, взаимно простое с остальными. 3. Найти коэффициенты при х17 и х после раскрытия скобок и приведения подобных членов в выражении (х7 + хъ + 1 J0. 4. Дан выпуклый пятиугольник ABCDE. Сторонами, противо- противоположными вершинам А, В, С, D, Е, назовем соответственно отрезки CD, DE, ЕА, АВ, ВС. Доказать, что если произволь- произвольную точку М, лежащую внутри пятиугольника, соединить прямыми со всеми его вершинами, то из этих прямых либо ровно одна, либо ровно три, либо ровно пять пересекут стороны пятиуголь- пятиугольника, противоположные вершинам, через которые они проходят. 5. Точка О является точкой пересечения высот остроуголь- остроугольного треугольника ABC. Доказать, что три окружности, проходя- проходящие: первая через точки О, А, В, вторая — через точки О, В, С и третья — через точки О, С, А, равны между собой. IX—X классы 6. Найти коэффициент при х2 после раскрытия скобок и приведения подобных членов в выражении ((...(((х—2J — 2J — -2J-...-2J-2J (k скобок). 7. См. задачу 2 для 16 последовательных чисел. 8. Сколько различных по величине или по расположению квадратов, состоящих из целого числа клеток, можно начертить на шахматной доске в 64 клетки? 9. В каком из выражений A + *2-*3)'000 или A -х2 + х3I000 после раскрытия скобок и приведения подобных членов будет стоять больший коэффициент при л:20? 10. Вычислить с точностью до 0,00001 произведение 11. Найти множество всех прямых в пространстве, проходящих через данную точку М на данном расстоянии d от данной прямой АВ. 31
2-й т у р VII—VIII классы 12. Некоторые из 20 металлических кубиков, одинаковых по размерам и внешнему виду, алюминиевые, остальные — дюралевые (более тяжелые). Как при помощи не более чем 11 взвешиваний на чашечных весах без гирь определить число дюралевых кубиков? (Предполагается, что все кубики могут быть алюминиевыми, но не могут быть все дюралевыми.) 13. Сколько цифр в десятичной записи числа 2100? 14. На плоскости даны пять точек, из которых никакие три не лежат на одной прямой. Доказать, что из этих точек можно выбрать четыре, являющиеся вершинами выпуклого четы- четырехугольника. 15. Доказать, что никакой выпуклый 13-угольник нельзя разре- разрезать на параллелограммы. 16. Из чисел 1, 2, ... , 200 выбрано 101 число. Доказать, что среди выбранных чисел найдется пара таких, что одно из них делится на другое. IX—X классы 17. п проволочных треугольников расположены в простран- пространстве так, что каждые два из них имеют общую вершину и в каждой вершине сходится одно и то же число k треугольников. Найдите все значения п и k, при которых такое расположение возможно. 18. В числовом треугольнике 1 111 12321 1367631 каждое число равно сумме чисел, расположенных в предыдущей строке над этим числом и над его соседями справа и слева (отсутствующие числа считаются равными нулю). Доказать, что в каждой строке, начиная с третьей, найдется четное число. 19. Внутри квадрата расположен выпуклый четырехугольник, внутри которого взята точка А. Известно, что девять точек— вершины квадрата, вершины четырехугольника и точка А — расположены так, что никакие три из них не лежат на одной прямой. Доказать, что из них можно выбрать пять точек, лежащих в вершинах выпуклого пятиугольника. 20. Из чисел 1, 2, ..., 200 выбрали одно число, меньшее 16, и еще 99 чисел. Доказать, что среди выбранных чисел найдутся два таких, одно из которых делится на другое. 21. Доказать, что если четыре грани тетраэдра равновелики, то они равны между собой. 32
XI ОЛИМПИАДА A948 г.) 1-й тур VII—VIII классы 1. Сумма обратных величин трех целых положительных чисел равна 1. Каковы эти числа? Найти все решения. 2. Найти все возможные расположения четырех точек на плоскости, при которых попарное расстояние между этими точка- точками принимают только два значения а и Ь. При каких значе- значениях отношения -j- такие расположения возможны? 3. На плоскости проведено п прямых. Доказать, что облас- области, на которые эти прямые разбивают плоскость, можно так закрасить двумя красками, что никакие две соседние области (т. е. области, соприкасающиеся по отрезку прямой) не будут закрашены одной и той же краской. IX—X классы 2" 2 22"~' 2 4. Если число целое, то число —^—— целое. Доказать. Fl ?• — 1 5. Доказать без помощи таблиц, что 1 :+-¦ 1 log2 П log5 П ' 6. Даны две треугольные пирамиды ABCD и A'BCD. Точка А' лежит внутри пирамиды ABCD. Доказать, что сумма плоских углов при вершине А' пирамиды A BCD больше суммы плоских углов при вершине А пирамиды ABCD. 7. Даны окружность и точка А вне ее. Из этой точки мы совершаем путь по некоторой замкнутой ломаной, состоящей из отрезков прямых, касательных к окружности, и заканчиваем путь в точке А (рис. 1). Участки пути, по которым мы приближаемся к центру окружности, берем со знаком «плюс», остальные — со знаком «минус». Доказать, что алгебраическая сумма длин участков пути с указанными знаками равна нулю. 2-й т у р VII—VIII классы 8. Решить в натуральных числах урав- уравнение ху = у* (х =?/= у). Найти все решения. 9. Доказать, что в любом треуголь- треугольнике имеет место неравенство R^2r (/? и г — радиусы описанного и вписан- вписанного кругов соответственно), причем ра- равенство R = 2r имеет место только дчя правильного треугольника. 2 Заказ 247 33
10. Может ли фигура иметь более одного, но конечное число центров симметрии? 11. Доказать, что если отрезок, соединяющий середины про- противоположных сторон выпуклого четырехугольника, равен полу- полусумме двух оставшихся сторон, то эти две стороны параллельны. 12. На плоскости расположены два прямолинейных зеркала, образующих угол а. Доказать, что, если сс = —, и только в этом случае, любой луч, несколько раз отразившись от зеркал, в конце концов уйдет в направлении, противоположном тому, по которому пришел. Найти число его отражений. IX—X классы 13. Найти все рациональные положительные решения уравне- уравнения ху = ух {хфу) (указать формулу, дающую все решения). 14*. Поместить в куб окружность наибольшего возможного радиуса. 15. Сколько различных целочисленных решений имеет нера- неравенство |х| + \у\ <Ю0? 16. Каково наибольшее возможное число лучей в простран- пространстве, выходящих из одной точки и образующих попарно тупые углы? 17. В пространстве расположены три плоских зеркала, обра- образующих октант (трехгранный угол с прямыми плоскими угла- углами). Доказать, что любой луч света, попавший в этот зеркаль- зеркальный октант, после нескольких отражений от его граней уйдет в направлении, противоположном тому, по которому пришел. Найти число его отражений. (Ср. с задачей 12.) XII ОЛИМПИАДА A949 г.) 1-й т у р VII—VIII классы 1. Показать, что 27 1958—10 8878+10 1528 делится на 26 460 без остатка. 2. Доказать, что если плоский многоугольник имеет несколько осей симметрии, то все они пересекаются в одной точке. 3. Доказать, что равенство х -\-y2-)-z2 = 2xyz для целых чисел х, у, z возможно только при х = у = 2=0. 4. Дана плоская замкнутая ломаная периметра 1. Доказать, что можно начертить круг радиуса —, покрывающий всю ломаную. 5. Доказать, что для любого треугольника отрезок, соединя- соединяющий центры вписанной и вневписанной окружностей, делится описанной окружностью пополам. 34
IX—X классы 6. Найти такие целые числа х, у, г, и, что 7. Как расположены плоскости симметрии ограниченного тела, если оно имеет две различные оси вращения? 8. Найти действительные корни уравнения -± (o<a<-L) . 9. Имеется набор из 4« положительных чисел. Из любых четырех попарно различных чисел этого набора можно составить геометрическую прогрессию. Доказать, что в таком случае в наборе имеются по крайней мере п одинаковых чисел 10. Доказать, что если у шестиугольника противоположные стороны параллельны и диагонали, соединяющие противополож- противоположные вершины, равны, то вокруг него можно описать окружность. 2-й т у р VII—VIII классы 11. Двенадцать полей расположены по окружности. На четы- четырех соседних полях стоят четыре разноцветные фишки: красная, желтая, зеленая и синяя. Одним ходом можно передвинуть любую фишку с поля, на котором она стоит, через четыре поля на пятое, если оно свободно, в любом из двух возможных направле- направлений. После нескольких ходов фишки стали опять на те же четыре поля. Как они могут при этом переставиться? 12. Даны два треугольника ABC и DEF и точка О. Берется любая точка X в треугольнике ABC и любая точка Y в тре- треугольнике DEF, после чего треугольник OXY достраивается до параллелограмма OXYZ. Доказать, что все полученные таким образом точки Z образуют многоугольник. Сколько сторон он мо- может иметь? Доказать, что его периметр равен сумме периметров исходных треугольников. 13. Имеется тринадцать гирь, масса каждой из которых равна целому числу граммов. Известно, что любые двенадцать из них можно так разложить на две чаши весов, по шесть на каждую, что наступит равновесие. Доказать, что все гири имеют одну и ту же массу. 14. В произвольном шестиугольнике середины сторон соедине- соединены через одну. Доказать, что точки пересечения медиан двух образовавшихся треугольников совпадают. 2* 35
15. Если имеется сто любых целых чи- чисел, то среди них всегда можно взять не- несколько (или может быть одно) так, что в сумме они дадут число, делящееся на 100. Доказать. IX—X классы 16. См. задачу 11 17. Сложить из одинаковых кирпичи- кирпичиков, один из которых изображен на рисун- рисунке 2, выпуклый многогранник. 18. В данный треугольник вписать цент- центрально-симметричный многоугольник мак- максимальной площади. 19*. Доказать, что числа вида 2" при различных натуральных п могут начинаться на любую заранее заданную комбинацию цифр. 20. Доказать, что к квадрату нельзя приложить более восьми не налегающих друг на друга квадратов такого же размера. Рис. 2 XIII ОЛИМПИАДА A950 г.) 1-й тур VII—VIII классы 1. Имеется шахматная доска с обычной раскраской (границы квадратов считаются окрашенными в черный цвет). Начертить на ней окружность наибольшего радиуса, целиком лежащую на черных полях, и доказать, что большей окружности того же рода начертить нельзя. 2. Имеется 555 гирь массой 1 г, 2 г, 3 г, 4 г, ..., 555 г. Разложить их на три равные по массе кучи. 3. См. задачу 10 для и = 3 окружностей. j 4. Пусть a, b и с — длины сторон треугольника; А, В и С — величины противоположных углов. Доказать, что Аа + ВЬ + Сс^-^-(АЬ+Ас + Ва + Вс + Са + СЬ). 5. Из пункта А в другие можно попасть двумя способами: выйти сразу и идти пешком или, вызвав машину и подождав ее определенное время, ехать на ней. В каждом случае используется способ передвижения, требующий меньшего времени. При этом оказывается, что если конечный пункт отстоит на 1 км, 2 км, 3 км, то на дорогу понадобится соответственно 10 мин, 15 мин, 17,5 мин. Скорости пешехода и машины, а также время ожида- ожидания машины принимаются неизменными. Сколько времени пона- понадобится для достижения пункта, отстоящего от А на 6 км? Э6
IX X к л а с с ы 6. Пусть А — произвольный угол, В и С — острые углы. Всегда ли существует такой угол А", что sin X — sin в sin с ., 1 — cos В cos С cos A 7. Из двух треугольных пирамид с общим основанием одна лежит внутри другой. Может ли сумма ребер внутренней пирамиды быть больше суммы ребер внешней? 8. Имеется 81 гиря массами I2 г, 22 г, З2 г, .... 812 г. Разло- Разложить их на три равные по массе кучи. 9. Решить уравнение 10. Даны п окружностей: О\, О2, О3, ..., О„, проходящих через одну точку О. Вторые точки пересечения О\ с О2, О2 с Оз, ..., О„ с О\ обозначим соответственно через At, A2, ., Ап. На О\ берем произвольную точку В\. Проводим через А\ и Bi прямую до второго пересечения с О2 в точке В2, затем про- проводим через А2 и В2 прямую до пересечения с О3 в точке Вз и т. д., пока не получим точку Вп на окружности О„. Прове- Проведем через точки Вп и Ап прямую до второго пересечения с О[ в точке Вп + 1. Доказать, что Вп + \ совпадает с В\. Замечание. В случае совпадения каких-либо точек Bk и Ак нужно через точку Аи проводить касательную к окружности Ok до пересечения с Ok+[ в точке 2-й т у р VII—VIII классы 11. В выпуклом 13-угольнике проведены все диагонали. Они разбивают его на многоугольники. Возьмем среди них многоугольник с наибольшим числом сторон Какое самое боль- большое число сторон он может иметь? 13 5 7 99 1 12. Доказать, что —.—.—.—.....^<—. 13. В треугольник вписана окружность, около которой описан квадрат так, что ни одна сторона квадрата не параллельна ни одной стороне треугольника. Доказать, что длина части границы квадрата, лежащей вне треугольника, меньше половины периметра квадрата. 14*. На окружности расположены 20 точек. Эти точки соеди- соединяются десятью хордами, не имеющими общих концов и не пересекающимися между собой. Сколькими способами это можно сделать? 37
IX—X классы 15. См. задачу 11 для 1950-угольника. 16. Числа 1, 2, 3, ..., 101 выписаны в ряд в каком-то порядке. Доказать, что из этого ряда всегда можно вычеркнуть 90 чисел так, что оставшиеся 11 будут расположены в порядке возраста- возрастания или убывания. 17. Около сферы описан пространственный четырехугольник. Доказать, что все точки касания лежат в одной плоскости. 18. Можно ли провести в городе 10 автобусных маршру- маршрутов и установить на них остановки так, что для любых 8 марш- маршрутов найдется остановка, не лежащая ни на одном из них, а любые 9 маршрутов проходят через все остановки? XIV ОЛИМПИАДА A951 г.) 1-й тур VII—VIII классы 1. Доказать, что многочлен хх2 — х9 + х*— л;+1 при всех зна- значениях х положителен. 2. Пусть ABCD и A'B'C'D' — два выпуклых четырехугольника с соответственно равными сторонами (АВ = А'В', ВС = В'С и т. д.). Доказать, что если /LA>/LA\ то /LB</-B', Z.C> Z.C, о IZ , 2,00 .. U4 2,00.. 02 3. Какое число больше:,, ^ „,ч, , п„„ „, или - 'A,00.. 04)-'+ 2,00 04 A,00...02J + 2,00.. 02" (Во всех указанных числах после запятой сначала идут 10 нулей.) 4. На плоскости даны равнобочная трапеция ABCD и точка Р. Доказать, что из отрезков PA, PB, PC, PD можно построить четырехугольник. 5. Имеется кусок цепи из 60 звеньев, каждое массой 1 г. Какое наименьшее число звеньев надо расковать, чтобы из об- образовавшихся частей можно было составить все массы в 1 г, 2 г, ..., 59 г, 60 г? (Раскованное звено также имеет массу 1 г.) IX—X классы 6. Найти первые три цифры после запятой в десятичной записи частного от деления числа 0,123456789101112...495051 на число 0,515049...121110987654321. 7. Из всех выпуклых многоугольников, у которых одна сторона равна а и сумма внешних углов при вершинах, не прилегающих к этой стороне, равна 120°, найти многоугольник наибольшей площади. 8. Имеются две концентрические окружности. Вокруг меньшей из них описан многоугольник, целиком находящийся внутри 38
большей окружности. Из общего центра на стороны многоуголь- многоугольника опущены перпендикуляры, которые продолжены до пересе- пересечения с большей окружностью; каждая из полученных точек пересечения соединена с концами соответствующей стороны много- многоугольника. При каком условии построенный так звездчатый много- многоугольник будет разверткой пирамиды? 9. См. задачу 5 для цепи из 150 звеньев. 10. Даны три параллельные прямые на равных расстояниях друг от друга. Как надо изображать точками соответствующих прямых величины сопротивления, напряжения и силы тока в проводнике, чтобы, прикладывая линейку к точкам, изображаю- изображающим значения сопротивления R и значения силы тока /, получить на шкале напряжения точку, изображающую величину напряже- напряжения U = /•/?? (Точка каждой шкалы изображает одно и только одно число.) 2-й тур VII—VIII классы 11. Доказать, что число 100...00500...001 не является кубом 46 нулей 99 н>лей никакого целого числа. 12*. На плоскости даны три точки А, В, С и три угла /LD, /LE, /LF, каждый из которых меньше 180°, и в сумме составляющие 360°. Построить с помощью линейки и транспорти- транспортира точку О плоскости такую, что /LAOB = Z.D, /LBOC= Z.E и /LCOA = /LF. (С помощью транспортира можно перенести любой данный угол так, чтобы одна из его сторон заняла произ- произвольное наперед заданное положение.) 13. Доказать, что сумма 13 + 23-т-... + п3 при всех п есть полный квадрат. 14. Какой фигурой может быть образ треугольника при цент- центральной симметрии, центр которой не лежит в плоскости треугольника? 15. На консультации было 20 школьников и разбиралось 20 задач. Оказалось, что каждый решил две задачи и каждую задачу решили двое. Доказать, что можно организовать разбор задач так, чтобы каждый школьник рассказал одну из решенных им задач и все задачи были разобраны. 16. При делении многочлена л:1951—1 на многочлен Р (х) = = хА-\-х3 + 2л;2 -+- х + 1 получается частное и остаток. Найти в част- частном коэффициент при х14. IX—X классы 17. В л-угольную пирамиду вписана сфера. Доказать, что если совместить все боковые грани пирамиды с плоскостью 39
основания, повернув их вокруг соответствующих ребер основания, то все точки касания этих граней со сферой сольются в одну точку Н, а вершины граней расположатся на окружности с центром Н. 18*. Имеется несколько чисел, каждое из которых меньше чем 1951. Наименьшее общее кратное любых двух из них больше чем 1951. Доказать, что сумма обратных величин этих чисел меньше 2. 19. Из всех ортогональных проекций правильного тетраэдра на различные плоскости найти проекцию наибольшей площади. 20. Окружность обладает тем свойством, что внутри ее можно двигать вписанный правильный треугольник так, чтобы каждая вершина треугольника двигалась по этой окружности и описывала ее целиком. Найти плоскую замкнутую несамопере- секающуюся кривую с тем же свойством, отличную от окруж- окружности. 21*. Автобусный маршрут содержит 14 остановок (считая две конечные). В автобусе одновременно могут ехать не более 25 пассажиров. Доказать, что во время поездки автобуса из одного конца в другой: а) найдутся 8 различных остановок А\, В\, А2, В2, Аз, В3, Л4, В*, таких, что для всех k = l, 2, 3, 4 ни один пассажир не едет от At до Вк', б) может оказаться, что не существует 10 различных остановок А\, Ви ¦¦-, Аь, Въ, которые обладали бы аналогичными свойствами. (Считается, что пассажир едет от остановки А до остановки В, если он вошел в автобус на остановке А, а вышел на остановке В.) XV ОЛИМПИАДА A952 г.) 1-й тур VII класс 1. В треугольник ABC вписана окружность, касающаяся его сторон в точках L, М, N. Доказать, что треугольник LMN всегда остроугольный. 2. Доказать тождество (ax + by + czf + (bx + cy + azf + {сх + + ау + bzf = (ex + by + azf + {bx + ау + czf + (ах + су + bzf. 3. Если все грани параллелепипеда — равные между собой параллелограммы, то они являются ромбами. Доказать. 4. См. задачу 6 и ответьте на вопрос: когда должен выйти из N пешеход С, чтобы А и В прибыли в N одновременно? VIII класс 5. Доказать, что если ортоцентр делит все высоты треугольника в одном и том же отношении, то треугольник равносторон- равносторонний. (Ортоцентром называется точка пересечения высот треуголь- треугольника.) 40
6. Два человека А и В должны попасть из пункта М в пункт N, расположенный в 15 км от М. Пешком они могут пе- передвигаться со скоростью 6 км/ч. Кроме того, в их распоря- распоряжении имеется велосипед, на котором можно ехать со скоростью 15 км/ч. А и В отправляются в путь одновременно, причем А идет пешком, а В едет на велосипеде до встречи с пешехо- пешеходом С, идущим из N в М. Дальше В передает С велосипед и продолжает путь пешком, а С едет на велосипеде до встречи с А, передает ему велосипед, на котором тот и приезжает в N. С идет пешком с той же скоростью, что А и В. Время, затраченное А и В на дорогу, считается от момента их отправления из М до момента прибытия последнего из них в N. Когда должен выйти из N пешеход С, чтобы это время было наимень- наименьшим? 7. Доказать тождество 8. См. задачу 3. IX класс 9. Дана геометрическая прогрессия, знаменатель которой — целое число, не равное 0 и —1. Доказать, что сумма двух или большего числа произвольно выбранных членов не может равняться никакому члену этой прогрессии. 10. Доказать, что если |дг|<1, |у|<1, то *~у I <1. ' 1 — ху I 11. Треугольник ABC разбит прямой BD на два треуголь- треугольника. Доказать, что сумма радиусов окружностей, вписанных в треугольники ABD и DBC, больше радиуса окруж- окружности, вписанной в треугольник ABC. 12. Дана последовательность целых чисел, построенная сле- следующим образом: щ — произвольное трехзначное число; аг— сумма квадратов его цифр; аз — сумма квадратов цифр числа а.2 и т. д. Доказать, что в последовательности а\, аг, аз, ... обязательно встретится либо 1, либо 4. 13. См. задачу 18. X класс 14. Найти соотношение между arcsincosarcsin x и arccossinarccos x. 15. Доказать, что при целом п^2 и |х|<;1 справедливо неравенство 41
16. В трехгранный угол с вершиной 5 вписана сфера с центром в точке О. Доказать, что плоскость, проходящая через три точки касания, перпендикулярна прямой OS. 17. Доказать, что если при любом положительном р оба корня уравнения ax2-\-bx + с + Р = 0 действительны и положитель- положительны, то а = 0. 18. Даны три попарно скрещивающиеся прямые. Доказать, что они являются общими перпендикулярами к своим общим перпендикулярам. 2-й т у р VII класс 19. См. задачу 27 при п— 15. 20. В выпуклом четырехугольнике ABCD выполнено соотноше- соотношение AB-\-CD = BC-\-AD. Доказать, что окружность, вписанная в ААВС, касается окружности, вписанной в AACD. 21. Доказать, что если квадрат числа начинается с 0,9...99 A00 девяток), то и само число начинается с 0,9...9 (не менее чем 100 девяток). (Ср. с задачей 23.) 22. Дан отрезок АВ. Найти множество вершин С остроуголь- остроугольных треугольников ABC. VIII класс 23. Найти первые шестьдесят десятичных знаков числа У0,999...99 F0 девяток). 24. Из точки С проведены касательные СА и СВ к окружности О. Из произвольной точки N окружности опущены перпен- перпендикуляры ND, NE, NF соответственно на прямые АВ, СА и СВ. Доказать, что ND есть среднее пропорциональное между NE и NF. 25. Имеется семь жетонов с цифрами 1, 2, 3, 4, 5, 6, 7. Доказать, что никакие 2 семизначных числа, составленных с помощью этих жетонов, не делятся одно на другое. 26. 99 прямых разбивают плоскость на п частей. Найти все возможные значения п, меньшие 199. IX класс 27. Решить систему уравнений 1 xn-ixn 0, 1 — хпх\ =0 и исследовать решение при различных п. 42
28. Поместить в полый куб с ребром а три цилиндра диаметра -у- и высоты а так, чтобы они не могли менять своего положения внутри куба. 29. См. задачу 25. 30*. В равнобедренном треугольнике ABC угол ABC равен 20° и ВС = АВ. На сторонах АВ и ВС взяты соответ- соответственно точки Q и Р такие, что Z.РАС=50°, /LQCA =60°. Доказать, что угол PQC равен 30°. 31. Двести учеников выстроены прямоугольником по 10 чело- человек в каждом поперечном ряду и по 20 человек в каждом продольном ряду. В каждом продольном ряду выбран самый высокий ученик (если таких несколько, выбирается любой), а затем из отобранных 10 человек выбран самый низкий. С другой стороны, в каждом поперечном ряду выбран самый низкий ученик, а затем среди отобранных 20 человек выбран самый высокий. Какой из этих двух отобранных учеников окажется выше? X класс 32. Доказать, что при фиксированных а\, аг, -•-, Оз1 и переменном х сумма cos 32x + o3i cos 31лг + ... + а2 cos 2х-\-а\ cos x принимает как положительные, так и отрицательные значения. 33. См. задачу 28. 34. Доказать, что ни при каком целом а ни один много- многочлен вида Зх2п + ахп + 2 не делится на многочлен 2х2+ ахт + 3. 35. См. задачу 30. 36. См. задачу 31. XVI ОЛИМПИАДА A953 г.) 1-й т у р VI/ класс 1. Доказать, что в трапеции сумма углов при большем ос- основании меньше, чем при меньшем. 2. Найти наименьшее число, записываемое одними единица- единицами, которое делилось бы на 33...33 A00 троек). 3. Разделить отрезок пополам с помощью угольника (с по- помощью угольника можно проводить прямые и восставлять перпендикуляры, опускать перпендикуляры нельзя). 4. При любом натуральном п сумма п2 + 8п+15 не делится на /г+4. Доказать. VIII класс 5. Три окружности попарно касаются друг друга. Через три точки касания проводим окружность. Доказать, что эта окруж- 43
ность перпендикулярна каждой из исходных. (Углом между двумя окружностями называется угол между касательными к ним в точке пересечения.) 6. Доказать неравенство 2-V2+V2+-+V2 >J (в числителе п радикалов, в знаменателе на один меньше). 7. См. задачу 2. 8. См. задачу 3. IX класс 9. Найти множество точек плоскости, координаты которых удовлетворяют соотношению sin (х+у) = 0. 10. АВ и А\В[ — два отрезка, лежащие на скрещивающихся прямых, О и О\ — их середины. Доказать, что отрезок ОО\ меньше полусуммы отрезков АА\ и ВВ\. 11. Доказать, что многочлен л;200 • у200 + 1 нельзя представить в виде произведения многочленов от одного только х и от одного только у, т. е. в виде f(x)-g{y). 12. А — вершина правильного звездчатого пятиугольника, угол при которой меньше развернутого. Ломаная AA\BB\CC\DD\EE\ является его внешним контуром. Прямые АВ и DE пересекаются в точке F. Доказать, что многоугольник ABB\CC\DED\A равновелик четырехугольнику AD\EF. 13. См. задачу 6. X класс 14. См. задачу 9. 15. Даны прямой круговой конус и точка А. Найти множество вершин конусов, равных данному, с осями, параллельными оси данного конуса, и содержащих внутри данную точку А. 16. См. задачу 11. 17. См. задачу 12. 18. См. задачу 6. 2-й т у р VII класс 19. Доказать, что наибольший общий делитель суммы двух чисел и их наименьшего общего кратного равен наибольшему общему делителю самих чисел. 20. Около окружности описан четырехугольник. Его диагонали пересекаются в центре этой окружности. Доказать, что этот четырехугольник — ромб. 44
21. См. задачу 27 при п = 11. 22. 1000 точек являются вершинами выпуклого тысячеуголь- ника, внутри которого расположено еще 500 точек так, что никакие три из этих 1500 точек не лежат на одной пря- прямой. Данный тысячеугольник разрезан на треугольники таким образом, что все указанные 1500 точек являются вершинами треугольников и эти треугольники не имеют никаких других вер- вершин. Сколько треугольников получится при таком разреза- разрезании? 23. См. задачу 28 при п = 5. VIII класс 24. а, Ь, с и d — длины последовательных сторон четырех- четырехугольника, 5 — его площадь. Доказать, что S^ — (о + с) F+ d) 25. 1953 цифры выписаны по окружности. Известно, что если читать эти цифры по часовой стрелке, начиная с некоторого определенного места, то полученное 1953-значное число делится на 27. Доказать, что если начать читать по часовой стрелке с любого другого места, то полученное число также будет делиться на 27. ' 26. На окружности дано множество точек А,, Л2, А3, .... А„. Построим все возможные выпуклые многоугольники, вершинами которых являются точки из данного множества. Разобьем эти многоугольники на 2 группы. В первую группу входят все многоугольники, у которых А\ является вершиной. Во вторую груп- группу входят все многоугольники, у которых А\ вершиной не является. В какой группе больше многоугольников? 27. В плоскости расположено п зубчатых колес таким образом, что первое колесо сцеплено своими зубцами со вторым, второе — с третьим и т. д. Наконец, последнее колесо сцеплено с первым. Могут ли вращаться колеса такой системы? 28. Решить систему уравнений при а; =100: *, +2*2+ 2*з+ 2*4+ 2*5+ ... +2*»=1, *1+ЗХ2 + 5Х3 + 6Х4+6Х5+ ... *i + Зх2 + 5*3 + 7*4 + 9х5+ .- IX класс 29. См. задачу 25. 30. В плоскости даны дЛИг/Ь и прямая / вне его, образующая с продолжениями сторон треугольника А\А2, А^Аз, А3А1 соответственно углы а3, cci, а2. Через точки А\, Аъ, А3 проводятся прямые, образующие с / соответственно углы 45
л —а,, я —а2, я —а3. Доказать, что эти прямые пересекаются в одной точке. (Все углы отсчитываются от прямой / в одном направлении.) 31. Даны уравнения 2 0, A) = 0. B) Доказать, что если Х\ и Х2 (соответственно) — какие- либо корни уравнений A) и B), то найдется такой корень хъ уравнения -^-х2 + Ьх + с — О, что либо Xi <хз<.К2, либо 32. Дан лист клетчатой бумаги размером 101X200 клеток. Начиная от какой-либо угловой клетки идем по диагонали и каждый раз, дойдя до границы, меняем направление движения по правилу отражения света. Попадем ли мы когда-нибудь в какую-либо угловую клетку? 33. Разрезать куб на три равные пирамиды. X класс 34. Найти корни уравнения 1 ГН Г2 ... + (-1)в— "„I* =0. 35. См. задачу 30. 36. Пусть л-о=Ю9, хп = -~^-. Доказать, что 0 <х36—л[2< 37. См. задачу 33. 38. На бесконечной шахматной доске стоит конь. Найти все клетки, куда он может попасть ровно за In ходов. XVII ОЛИМПИАДА A954 г.) 1-й тур VII класс 1. Правильный звездчатый шестиугольник разрезать на 4 ча- части так, чтобы из них можно было составить выпуклый много- многоугольник. 2. Даны два выпуклых многоугольника AiA2.-An и В\В2...Вп- Известно, что AlA2 = BiB2, А2А3 = В2В3, ..., АпА1=ВпВ[ и п — 3 уг- угла одного многоугольника равны соответственным углам другого. Будут ли эти многоугольники равны? 3. Определить четырехзначное число, если деление этого числа на два разных однозначных производится по таким схемам: 46
w w *I* *I* *I* 4* ** о 4. Существуют ли целые числа тип, удовлетворяющие уравнению m2+1954 = n2? 5. Определить наибольшее значение отношения трехзначно- трехзначного числа к сумме его цифр. VIII класс 6*. Из квадрата размером 3X3 вырезать одну фигуру, ко- которая представляет собой развертку полной поверхности куба, длина ребра которого равна 1. 7. Из произвольной внутренней точки О выпуклого я-уголь- ника опущены перпендикуляры на стороны (или"на их продол- продолжения). На каждом перпендикуляре от точки О в направле- направлении к соответствующей стороне отложен вектор, длина кото- которого равна половине длины этой стороны. Определить сумму по- полученных векторов. 8. См. задачу 3. 9. Найти все решения системы, состоящей из уравнений вида = 0, где n=l, 2 10. См. задачу 4. IX класс 11. Доказать, что если xl + оi*о + aixt + азхо + Щ = 0 и + За 1*0+ 2а2-*:о + аз = 0, то многочлен x4 + a|jc3- делится на (х — х0J. 12. Дано число 123456789101112131415...9899100. Вычерк- Вычеркнуть 100 цифр так, чтобы оставшееся число было наибольшим. 13. Дано 100 чисел at, a2, ..., а1Оо, удовлетворяющих условиям а\ — 3< Доказать, что все эти числа равны между собой. 14. Дан треугольник ABC. Пусть 5 — любая точка внутри ?\АВС, А\, В\, С\—точки пересечения прямых AS, BS, CS соот ветственно со сторонами ВС, АС, АВ треугольника ABC. До- 47
казать, что по крайней мере в одном из полученных четырех- четырехугольников AB\SC{, C\SA\B, AXSB\C оба угла при d и В\, или С, и А,, или А, и В, не острые. 15. Существуют ли в пространстве такие четыре точки А, В, С, D, что AB = CD = 8 см, AC = BD=\0 см и AD = BC=13 см? X класс 16. Найти все действительные решения уравнения x2-j-2xsin(xy)+l=0. 17. См. задачу 12. 18. Дано 100 чисел а\, а2, ..., аюо, удовлетворяющих усло- условиям а,- — 4a, + i +За,+2^0 для всех i=l, 2, 3, ..., 100. Известно, что О| = 1. Найти а2, а3, ¦•¦, оюо- 19. См. задачу 14. 20. См. задачу 15. 2-й т у р VII класс 21. Дан лист клетчатой бумаги, каждый узел сетки которого обозначен некоторой буквой. Известно, что на любом отрезке, соединяющем два узла, обозначенные одной буквой и лежащие на одной линии, найдется по крайней мере один узел, обозна- обозначенный другой буквой. Какое наименьшее число различных букв требуется для этого? 22*. Решить систему 10л: 1 + 3x2 + 4хз + *4 + х5=0, 11 лг2 + 2*з + 2х4 + Зх5 + х6=0, 15*з + 4х4-j-5х5-j-4х6+*7 = 0, 2л:, + х2 — 3*з + 12х4 — Зх5 + *б + х7 = 0, Зх, + 2лг2 — Зхз + 4х4 + *5 — 16х6 + 2х7 = 0, 23. Сколько осей симметрии может иметь семиугольник? 24. Дано число 7V = 2-3-5-7-11-13-17-19-23-29-31 (произве- (произведение последовательных простых чисел). Пусть 1, 2, 3, 5 W — все его делители, выписанные в порядке возрастания. Под рядом делителей выпишем ряд из +1 и — 1 по следующему правилу: под 1 пишем +1; под числом, разлагающимся на четное число простых сомножителей, пишем +1. под остальными чис- числами пишем —1. Доказать, что сумма чисел построенного ряда равна нулю. 25. План города представляет собой плоскость, разбитую на одинаковые правильные треугольники. Стороны треугольни- 48
ков — шоссейные дороги, а вершины треугольников — пере- перекрестки; таким образом, на каждом перекрестке сходятся шесть дорог. Из точек А и В, расположенных на одной дороге (на одной стороне треугольника), одновременно в одном направле- направлении с одинаковыми скоростями выезжают две машины. Доехав до любого перекрестка, каждая машина может или продолжить свое движение в первоначальном направлении, или же повернуть на 120° вправо или влево. Смогут ли эти машины встретиться? (Требуется либо доказать, что при любом движении машин они не встретятся, либо указать такое их движение, при котором они встречаются.) VIИ класс 26. Из клетчатой бумаги вырезан квадрат 17X17. В каждой клетке квадрата написано одно из чисел 1, 2, 3, .... 70. Дока- Доказать, что существуют четыре различные клетки, центры которых А, В, С, D являются вершинами параллелограмма (AB\\CD) и сумма чисел, стоящих в клетках с центрами А и С, равна сумме чисел, стоящих в клетках с центрами В и D. 27. Даны четыре прямые т\, т.ч., т3, т4, пересекающиеся в одной точке О, занумерованные относительно О по часовой стрелке. Через произвольную точку А\ прямой т.\ проводим пря- прямую, параллельную /л4, до пересечения с прямой т2 в точке Аъ. Через А2 проводим прямую, параллельную т.\, до пересечения с шз в точке Аз- Через А3 проводим прямую, параллельную т2, до пересечения с т4 в точке Л4. Через А* проводим прямую, па- параллельную Шз, до пересечения с mi в точке В. Доказать, что 28. См. задачу 22. 29. См. задачу 23. 30. См. задачу 24 при Лг = 2-3-5.7-11-13-17-19-23-29-31 -37. IX класс 31. На двух лучах 1\ и /г, проходящих через точку О, отло- отложены отрезки ОА\ и ОВ\ на луче 1\ и ОА2 и OBz на луче /г; известно, что тпг^-рг^-- Найти множество всех точек пересече- UA2 Ut>2 ния прямых AAi и ВВ\ при вращении луча h вокруг точки О (луч 1\ неподвижен). ОА 32. См. задачу 27; доказать, что OB^L——. 33*. Сто положительных чисел Х\, х2, .... хюо удовлетворяют условиям 49
Доказать, что среди них найдутся три числа, сумма которых больше 100. 34. Если дана последовательность 15 чисел аи а2, а3, .... ais, то можно построить новую последовательность чисел Ь\, Ъч, Ьз, .--, fc is, где bi равно количеству чисел первой последователь- последовательности, меньших а,, 1=1, 2, ..., 15. Существует ли последователь- последовательность (а*), для которой последовательность (bk) выглядит так: 1, 0, 3, 6, 9, 4, 7, 2, 5, 8, 8, 5, 10, 13, 13? 35. Из точки А выходит пять отрезков: АВи АВ2, АВ3, АВ4, АВь. Из каждой точки В, могут выходить еще пять отрезков или ни одного отрезка и т. д., причем концы никаких двух отрезков построенной системы не совпадают. Может ли число свободных концов построенных отрезков равняться 1001? (Под свободным концом мы понимаем точку, из которой не выходит ни одного отрезка.) X класс 36. Сколько плоскостей симметрии может иметь треуголь- треугольная пирамида? 37. См. задачу 32. 38. См. задачу 33. 39. Известно, что модули всех корней уравнений = 0 и x2 меньше 1. Доказать, что модули корней уравнения также меньше 1. 40. Рассматриваются всевозможные десятизначные числа, записываемые при помощи цифр 1 и 2. Разбить их на два класса так, чтобы при сложении любых двух чисел одного класса по- получалось число, в написании которого содержится не менее двух троек. XVIII ОЛИМПИАДА A955 г.) 1-й тур VII класс 1. См. задачу 11 при k = 7. 2. Дан прямоугольный треугольник ABC. Из вершины В пря- прямого угла проведена медиана BD. Пусть К — точка касания стороны AD треугольника ABD с окружностью, вписанной в этот треугольник. Найти углы треугольника ABC, если К делит AD пополам. 50
3. Дан равносторонний треугольник ABC. На сторонах АВ и ВС взяты точки D и Е так, что AE = CD. Найти множество то- точек пересечения отрезков АЕ и CD. 4. Существует ли такое целое п, что п2-\-п-\- 1 делится на 1955? 5. Найти все прямоугольники, которые можно разрезать на 13 равных квадратов. VIII класс 6. Дано: 2"=10a + fc. Доказать, что если п>3, то аЪ де- делится на 6. Здесь п, а, Ъ — целые положительные числа, Ь<С 10. 7. Дан четырехугольник ABCD. На стороне АВ взята точка К, на стороне ВС — точка L, на стороне CD — точка Ж и на стороне AD — точка N так, что KB = BL = a, MD = DN—b. Пусть KL \ MN. Найти множество всех точек пересечения прямых KL и MN при изменении с и Ь. 8. См. задачу 16 при и = 7. 9. Какие выпуклые плоские области могут содержать прямую? 10. На окружности даны четыре точки А, В, С, D. Через каждую пару соседних точек проведена окружность. Вторые точки пересечения соседних окружностей обозначим через А\, В\, С\, D\ (некоторые из них могут совпадать с прежними). Доказать, что точки А\, В\, С\, Di лежат на одной окружности. IX класс 11. Числа 1, 2, .... k2 расположены в квадратной таблице: 1, 2 k, k+l, k + 2 2k, (k-l)k + i\ (k-\)k + 2, '.'.'.', W. Произвольное число выписывается, после чего из таблицы вычер- вычеркиваются строка и столбец, содержащие это число. То же самое проделывается с оставшейся таблицей из (k—IJ чисел и т. д. k раз. Найти сумму выписанных чисел. 12. Найти множество середин всех отрезков с концами на двух различных непересекающихся окружностях, лежащих одна вне другой. 13. Найти все действительные решения системы 14. р простых чисел а\, аг, .... ар образуют возрастающую арифметическую прогрессию и а\>р. Доказать, что если р — простое число, то разность прогрессии делится на р. 15. Дан л^ВС и точка D внутри него, причем AC — DA> 1 и ВС — BD>1. Берется произвольная точка Е внутри отрезка АВ. Доказать, что EC—ED>\. 51
X класс 16. Квадратная таблица в п2 клеток заполнена числами от 1 до п так, что в каждой строке и в каждом столбце встре- встречаются все эти числа. Если п нечетно и таблица симметрична относительно диагонали, идущей из левого верхнего угла в пра- правый нижний, то на этой диагонали встретятся все числа 1, 2, 3, ..., п. Доказать. 17. См. задачу 13. 18. См. задачу 15. 19. Дан трехгранный угол с вершиной О. Можно ли найти такое плоское сечение ABC, чтобы углы ОАВ, ОВА, ОСВ, ОАС, ОСА, ОВС были острыми? 2-й т у р VII класс 20. Решить в целых числах уравнение х3 — 2у3 — 4z3 = 0. 21. Доказать, что если трехчлен ах2 -\-bx-\-c при всех целых х является точной четвертой степенью целого числа, то а = Ь = 0. 22. Дан A ABC. Центры вневписанных окружностей Ю\, О2 и 0з соединены прямыми. Доказать, что ДО|О2Оз остроуголь- остроугольный. 23. В турнире собираются принять участие 25 шахматистов. Все они играют в разную силу, и при встрече всегда побеждает сильнейший. Какое наименьшее число партий требуется, чтобы определить двух сильнейших игроков? 24. Разрезать прямоугольник на 18 прямоугольников так, чтобы никакие два соседних не образовывали прямоугольник. VIII класс 25*. Трехчлен ах2-\-Ьх-\-с при всех целых х является точ- точным квадратом. Доказать, что тогда ах2 + Ъх -\- с = (dx + ef. 26*. Две окружности касаются друг друга внешним образом и третьей изнутри. Проводятся внешняя и внутренняя общие ка- касательные к первым двум окружностям. Доказать, что внутрен- внутренняя касательная делит пополам дугу, отсекаемую внешней ка- касательной на третьей окружности. 27. Точка О лежит внутри выпуклого и-угольника А\А2 ... А„ и соединена отрезками с его вершинами. Стороны и-угольника нумеруются числами от 1 до п, причем разные стороны нуме- нумеруются разными числами. То же самое делается с отрезками ОЛ |, ОА2, ..., ОА„. а) Для я = 9 найти нумерацию, при которой сумма номеров сторон для всех треугольников А\ОА2, А2ОА3, .... А„ОА\ одинакова, б) Доказать, что при п = 10 такой нумерации осуществить нельзя. 62
28. Неравенство Aa (Bb + Cc) + Bb {Aa + Cc)+Cc {Aa + Bb)> >±-(ABc2 + BCa2 + CAb2) (где а>0, fc>0, с>0 — данные числа) выполняется при всех Л>0, В>0, С>0. Можно ли из отрезков длины а, Ь, с составить треугольник? 29. Найти все такие числа а, что все числа [а], [2а], .... [Na] различны между собой и все числа — , — , .... — также различны между собой (натуральное N фиксировано). IX класс 30. Дан треугольник ABC. На сторонах АВ, ВС, СА взяты соответственно точки Ci, At, Bt так, что АС, ВА, СВ, 1 А,С В,А п На сторонах AtBi, BtCi, С|Лi треугольника А\В\С\ взяты соот- соответственно точки С2, А2, В2 так, что А,С2 ^В,Лг _ С|Вг С2В, А2С, В2А, Доказать, что А2С2\\АС, С2В2\\СВ, В2А2\\ВА. 31. Расположить на прямой систему отрезков длины 1, не имеющих общих точек (в том числе и общих концов), так, чтобы любая бесконечная арифметическая прогрессия (с любой раз- разностью и любым первым членом) имела общую точку с некото- некоторым отрезком системы. 32. Дано уравнение х" — а\Хп~1 — а2х"~2 —... — ап-\х — а„ = 0, где ai^O, a2^0, аз^О, ..., а„^0. Доказать, что это уравнение не может иметь двух положительных корней. 33. См. задачу 26. 34. Пять человек играют несколько партий в домино (двое на двое) так, что каждый играющий имеет каждого один раз партнером и два раза противником. Найти количество сыгран- сыгранных партий и все способы распределения играющих. X класс 35. Доказать, что если — несократимая рациональная дробь, являющаяся корнем полинома f (x) = aoxn-\-aix"~l -\-... -\-а„ с целыми коэффициентами, то р — kq есть делитель числа f (k) при любом целом k. 36. См. задачу 31. 37. На плоскости Р стоит прямой круговой конус. Радиус основания его равен г, высота — А. На расстоянии Н от пло- плоскости и 1 от высоты конуса находится источник света. Ка- 53
кую часть окружности радиуса R, лежащей в плоскости Р и кон- концентрической с окружностью, лежащей в основании конуса, осветит этот источник? 38. Имеется 1955 точек. Какое максимальное число троек точек можно из них выбрать так, чтобы каждые две тройки имели одну общую точку? 39. Дан треугольник А0В0С0. На его сторонах АоВо, ВоСо, С0А0 взяты соответственно точки Си А\, Вь на сторонах Л|ВЬ BiCi, С|Л| треугольника AiBtCi взяты соответственно точки Сг, А2, в\ и т. д. Известно, что АрВ, _ В0С, _ С0А, _ .. А,В2 _ В,С2 _ С,А2 __ 1 В,С0 ~~ С,А0 ~ А,Во ' В2С, ~ и вообще Апв„+1 в„сп+1 с„Ап+, J k при четном п, Вя+1с. С„+1Л„ Л„+1В„ . ^_ при нечетном п. / к2" Доказать, что треугольник ЛВС, образованный прямыми АоА\, В0В\, CqCi, содержится в треугольнике Ап„ф при любом п. XIX ОЛИМПИАДА A956 г.] 1-й т у р VII класс 1. Докажите, что на плоскости не существует таких четырех точек Л, В, С, D, что все треугольники ABC, BCD, CDA, DAB остроугольные. 2. Найти все двузначные числа, сумма цифр которых не ме- меняется при умножении числа на 2, 3, 4, 5, 6, 7, 8, 9. 3. Имеется замкнутая самопересекающаяся ломаная. Извест- Известно, что она пересекает каждое свое звено ровно один раз. Дока- Доказать, что число звеньев четно. 4. Найти все числа, на которые может быть сократима дробь 5/ + 6 , 8/ при целом значении /. 5. Какое наименьшее число точек можно выбрать на окруж- окружности, имеющей длину 1956, так, чтобы для каждой из этих то- точек нашлась ровно одна выбранная точка на расстоянии 1 и ровно одна на расстоянии 2 (расстояния измеряются по окружности)? VIII класс 6. На сторонах ЛВ и СВ треугольника ABC откладываются равные отрезки AD и СЕ произвольной длины. Найти множество середин отрезков DE. 54
7. В десятичной записи положительного числа а отброше- отброшены все десятичные знаки, начиная с третьего знака после запя- запятой (т. е. взято приближение числа а по недостатку с точностью до 0,01). Полученное число делится на а, и частное снова округ- округляется по недостатку с той же точностью. Какие числа при этом могут получиться? (Указать все значения.) 8. На окружности длины 15 выбрано п точек так, что для каждой имеется ровно одна выбранная точка на расстоянии 1 и ровно одна на расстоянии 2 (расстояния измеряются по окружно- окружности). Доказать, что п делится на 10. (Ср. с задачей 5.) 9. Пусть а, Ь, с, d, I — целые числа. Докажите, что если дробь g.7~. сократима на число k, то ad— be делится на k. (Ср. с задачей 4.) 10. В каждой клетке бесконечного клетчатого листа бумаги стоит число, равное среднему арифметическому четырех чисел, стоящих в соседних клетках. Из этого листа вырезана прямо- прямоугольная таблица. Доказать, что наибольшее из всех чисел таб- таблицы стоит с края. IX класс 11. Дан выпуклый четырехугольник ABCD. Четырехугольник KLMN образован центрами тяжести треугольников ЛВС, BCD, DBA, CDA. Доказать, что прямые, соединяющие середины про- противоположных сторон четырехугольника ABCD, пересекаются в той же точке, что и прямые, соединяющие середины противопо- противоположных сторон четырехугольника KLMN. 12. См. задачу 7, причем округление производится с точ- точностью до 0,001. 13. См. задачу 10. 14. Даны положительные числа h, Si, s2 и расположенный в пространстве треугольник ЛВС. Сколькими способами можно выбрать точку D так, чтобы в тетраэдре ABCD высота, опу- опущенная из вершины D, была равна h, а площади граней ACD и BCD равнялись соответственно s\ и S2? (Исследовать все воз- возможные случаи.) 15. См. задачу 9. А' класс 16. В треугольник вписан квадрат так, что две его верши- вершины лежат на основании, а две другие вершины — на боковых сторонах треугольника. Доказать, что сторона квадрата меньше 2г, но больше г^/2, где г — радиус окружности, вписанной в треугольник. 17. См. задачу 7, причем округление производится с точно- точностью до 0,0001. 55
18. См. задачу 9. 19. Дана замкнутая пространственная ломаная А\А2А3...Ап. Некоторая плоскость пересекает все ее звенья: Л|Л2 в точке В\, А2А3 в точке В2 Л„Л| в точке В„. А\В\ А?В<2 Лзбз А„Вп . Доказать, что _._._....._=!. 20. Доказать, что система уравнений ( Xi —х2 = а. имеет хотя бы одно решение Xi>0, х2>0, х3>0, лг4>0 тогда и только тогда, когда |а| + |Ь|<1. 2-й т у р VII класс 21. Точка О — центр круга, описанного около треугольни- треугольника ABC. Точки Ль В|, С\ симметричны точке О относительно прямых, содержащих стороны треугольника ЛВС. Доказать, что все высоты треугольника Л1В1С1 проходят через точку О, а все высоты треугольника ЛВС проходят через центр круга, опи- описанного около треугольника Л1В1С1. 22. Точки Л|, Л2, Аз, Л4, Л5, Л6 делят окружность радиуса 1 на шесть равных частей. Из точки А\ проведен луч U в направле- направлении Л2, из Л2 — луч /2 в направлении А3, .... из Л6 — луч /6 в направлении А\. Из точки Вь взятой на луче 1\, опущен перпен- перпендикуляр на луч /б, из основания этого перпендикуляра опускается перпендикуляр на /5 и т. д. Основание шестого перпендикуляра совпало с В\. Найти длину отрезка Л|ВЬ (Ср. с задачей 40.) 23. См. задачу 29. 24. См. условие задачи 27, в которой нужно доказать, что сумма чисел в любом столбце меньше 1035. 25*. Доказать, что у любого дерева можно оборвать —- его 8 листьев, оставив при этом не менее — тени, которую давало дерево. (Число листьев считать кратным 15; тенью от ствола и веток пренебречь.) VIII класс 26*. Груз массой 13,5 т упакован в некоторое число неве- невесомых ящиков. Масса каждого ящика с грузом не превосходит 350 кг. Доказать, что этот груз можно перевезти на 11 полуто- полуторатонках. 56
27. 64 неотрицательных числа, сумма которых равна 1956, расположены в форме квадратной таблицы по 8 чисел в каждой строке и в каждом столбце. Сумма чисел, стоящих на двух диаго- диагоналях, равна 112. Числа, расположенные симметрично относи- относительно любой диагонали, равны. Доказать, что сумма чисел в любой строке меньше 518. 28. Все точки данного отрезка АВ ортогонально проектиру- проектируются на всевозможные прямые, проходящие через данную точку О. Найти множество всех этих проекций. 29. 100 чисел, среди которых есть положительные и отрица- отрицательные, выписаны в ряд. Подчеркнуто, во-первых, каждое поло- положительное число, во-вторых, каждое число, сумма которого со следующим положительна, и, в-третьих, каждое число, сумма ко- которого с двумя следующими положительна. Может ли сумма всех подчеркнутых чисел оказаться отрицательной? равной нулю? 30*. В прямоугольнике площадью 5 кв. ед. расположены девять прямоугольников, площадь каждого из которых равна единице. Доказать, что площадь общей части каких-нибудь двух прямо- прямоугольников больше или равна —. IX класс 31. См. задачу 26. 32. В кубе, ребро которого равно 13, выбрано 1956 точек. Можно ли в этот куб поместить кубик с ребром 1 так, чтобы внутри него не было ни одной выбранной точки? 33. Взяли три числа х, у, г. Абсолютные величины их попар- попарных разностей обозначили через Х\, у\, Z\. Тем же способом по числам Х\, у\, Z\ построили числа х2, уг, 2г и т. д. Оказалось, что при некотором п справедливы равенства х„ = х, уп — у, zn = z. Зная, что х=\, найти у и z. 34. Четырехугольник описан около окружности. Доказать, что прямые, соединяющие соседние точки касания, пересекаются на продолжении диагонали или параллельны ей. 35*. На клетчатой бумаге выбраны три точки А, В, С, нахо- находящиеся в вершинах клеток. Доказать, что если треугольник ABC остроугольный, то внутри или на сторонах его есть по крайней мере еще одна вершина клетки. X к л а с с 36. Подряд выписаны п чисел, среди которых есть положи- положительные и отрицательные. Подчеркивается каждое положитель- положительное число, а также каждое число, сумма которого с несколькими непосредственно следующими за ним числами положительна. Доказать, что сумма всех подчеркнутых чисел положительна. (Ср. с задачей 29.) 57
37. См. задачу 30 для девяти произвотьных многоугольников. 38. См. задачу 33. 39*. Доказать, что если в треугольной пирамиде трехгран- трехгранные углы при каждой вершине составлены из одних и тех же троек плоских углов, то все грани этой пирамиды попарно равны. 40. На продолжениях сторон А\А2, А2А3, ..., АпА\ правиль- правильного /2-угольника А\А2 ... Ап построить точки Ви В2, ..., В„ так, чтобы В\В2 было перпендикулярно АХА2, В2В3 — перпендику- перпендикулярно А2А3, -.., ВпВ\ — перпендикулярно АпА\. XX ОЛИМПИАДА A957 г.| 1-й тур VII класс 1. Равнобочная трапеция разбивается диагональю на два равнобедренных треугольника. Найти углы трапеции. 2. Известно, что ах3 -\- Ьх2 -\- сх + d (a, b, с, d — данные на- натуральные числа) при любом целом х делится на 5. Доказать, что каждое из чисел а, Ь, с, d делится на 5.' 3. Улитка ползет по столу с постоянной скоростью. Через каждые 15 мин она поворачивает на 90°, а в промежутках меж- между поворотами ползет по прямой. Доказать, что она может вер- вернуться в исходный пункт только через целое число часов. 4. См. задачу 9. 5. От Л до В 999 км. Вдоль дороги стоят километровые столбы, на которых написаны расстояния до Л и до В: 0 999 1 998 2 997 998 1 999 0 Сколько среди них таких, на которых оба расстояния записаны только двумя различными цифрами? VIII класс 6. Найти множество четвертых вершин всех таких прямоуголь- прямоугольников, три вершины которых лежат на двух данных концентриче- концентрических окружностях (две вершины лежат на одной из окружностей, а третья — на другой), а стороны параллельны двум данным прямым. 7. См. задачу 3. 8. Из всех параллелограммов данной площади найти тот, у которого большая диагональ минимальна. 9. В прямоугольной таблице произведение суммы чисел любого столбца на сумму чисел любой строки равно числу, стоя- стоящему на их пересечении. Доказать, что сумма всех чисел в табли- таблице равна единице или все числа равны нулю. 58
10. Известно, что ах* + Ьх3 + сх2 + dx + е (а, Ь, с, d, e — дан- данные натуральные числа) при всех целых х делится на 7. Дока- Доказать, что все числа а, Ь, с, d, e делятся на 7. (Ср. с задачей 2.) IX класс 11. См. задачу 9. 12. Решить уравнение х3 — [л:]=3. 13. В четырехугольнике ABCD точка М — середина диагонали АС, N—середина диагонали BD. Прямая, проходящая через точки М и N, пересекает стороны АВ и CD в точках М' и N'. До- Доказать, что если ММ' = NN', то AD\\BC. 14. Школьник едет на олимпиаду в метро, платит рубль и получает сдачу. Доказать, что если он обратно поедет трамваем, то он сможет уплатить за проезд без сдачи. Примечание. До денежной реформы 1961 г. проезд в метро стоил 50 к., в трамвае — 30 к. и в обращении находились монеты достоинством в 1, 2, 3, 5, 10, 15, 20 к. 15. См. задачу 20. А" класс 16. При каких целых п число 20"+ 16"—3"— 1 делится на 323? 17. В пространстве построена замкнутая ломаная так, что все звенья имеют одинаковую длину и каждые три последователь- последовательные звена попарно перпендикулярны. Доказать, что число звень- звеньев делится на 6. (Ср. с задачей 3.) 18. См. задачу 13. 19. Школьник едет на занятия кружка трамваем, платит рубль и получает сдачу. Доказать, что на обратном пути он смо- сможет уплатить за проезд в трамвае без сдачи (см. примечание к задаче 14). 20. Плоский многоугольник Л|Л2Л3 ... А„-\Ап составлен из п>4 твердых стержней, соединенных шарнирно. Можно ли его деформировать в треугольник? 2-й т у р VII класс 21. Прямые О А и ОВ перпендикулярны. Найти множество концов М всех ломаных ОМ длины /, пересекающихся с каждой прямой, параллельной ОА или ОВ, не более чем в одной точке 22. Радиолампа имеет 7 контактов, расположенных по кругу и включенных в штепсель, имеющий 7 отверстий. Можно ли так занумеровать контакты лампы и отверстия штепселя, чтобы при включении лампы хотя бы один контакт попал на свое место (т. е. в отверстие с тем же номером)? 59
23. В треугольнике известны две стороны а и Ъ. Какой должна быть третья сторона, чтобы наибольший угол треуголь- треугольника имел наименьшую величину? 24. В треугольник вписана окружность. Точки касания яв- являются вершинами второго треугольника. В него вписана окруж- окружность, причем точки касания являются вершинами третьего тре- треугольника, который имеет те же углы, что и первоначальный тре- треугольник. Найти эти углы. 25. В последовательности Фибоначчи 1, 2, 3, 5, 8, 13, 21, ... выбрано 8 идущих подряд чисел. Доказать, что их сумма не входит в последовательность. VIII класс 26. В треугольнике известны две стороны а и Ъ. Какой должна быть третья сторона, чтобы наименьший угол треугольника имел наибольшую величину? (Ср. с задачей 23.) 27. См. задачу 39 при k = 8. 28. Внутри равностороннего треугольника ABC находится точка О. Прямая OG, соединяющая О с центром тяжести G треугольника, пересекает стороны треугольника (или их продол- продолжения) в точках А', В', С". Доказать, что А'О , В'О | СО Q I * A'G ' B'G ' C'G 29. Решить систему уравнений 30. В неравносторонний треугольник вписана окружность. Точки касания являются вершинами нового треугольника. В него вписана окружность, точки касания которой — вершины третьего треугольника, в третий вписана третья окружность и т. д. Дока- Доказать, что в образовавшейся последовательности треугольников нет подобных. (Ср. с задачей 24.) IX класс 31. Два прямоугольника расположены на плоскости так, что их стороны имеют восемь точек пересечения. Эти точки соедине- соединены через одну. Доказать, что площадь полученного четырехуголь- четырехугольника не изменится при параллельном переносе одного из прямо- прямоугольников. 32. См. задачу 37 при п = 99. 60
33. См. задачу 22 для 20 контактов, изменив вопрос. Всегда ли можно так соединить разъем, чтобы ни один контакт не был замкнут, т. е. не попал бы на свое место? 34. Разбить 1957 на 12 целых положительных слагаемых ci, а-2, а3, ..., Я|2 так, чтобы произведение с^-сг.'-сз! •-•• -сяг' было минимально. 35*. Три равные окружности касаются друг друга. Из произ- произвольной точки четвертой окружности, касающейся внешним обра- образом всех данных окружностей, проведены касательные к ним. Доказать, что сумма длин двух касательных равна длине третьей. X класс 36. Дан четырехугольник ABCD. Вписать в него прямоуголь- прямоугольник с заданными направлениями сторон. 37*. Найти все действительные решения системы 38. Точка G — центр шара, вписанного в правильный тетраэдр ABCD. Прямая OG, соединяющая G с точкой О, лежащей внутри тетраэдра, пересекает плоскости граней в точках А', В', С. D'. Доказать, что А'О . В'О . СО . Р'О__, A'G~*~ B'G~*~ C'G ~*~ D'G~ ' (Ср. с задачей 28.) 39. Доказать, что число всех цифр в последовательности 1, 2, 3, ... ,10* равно числу всех нулей в последовательности 1, 2, 3, ... , 10*+|. 40. Дано п целых чисел: fli = I, a2, ... , ап, причем а,-^а,-+1 ^2о,- (/=1, 2, 3, ... , п—1) и сумма всех чисел четна. Можно ли эти числа разбить на две группы так, чтобы суммы чисел в этих группах были равны? XXI ОЛИМПИАДА A958 г.) 1-й тур VII класс 1. Имеется система уравнений f*X-\-*y-\- *2 = 0, *х -\- *у -\- *z = О, *х -\- *у + *z = 0. SI
Два человека поочередно вписывают вместо звездочек числа. Доказать, что начинающий всегда может добиться того, чтобы система имела ненулевое решение. 2. В круге проведены два диаметра АВ и CD. Доказать, что если М — произвольная точка окружности, а Р и Q — ее проекции на диаметры АВ и CD, то длина отрезка PQ не зависит от выбора точки М. 3. Сколько существует четырехзначных номеров (от 0000 до 9999), у которых сумма двух первых цифр равна сумме двух последних цифр? 4. На плоскости даны точки Л и В. Построить такой квадрат, чтобы точки А и В лежали на его границе и сумма расстояний от точки А до вершин квадрата была наименьшей. 5. Дана следующая треугольная таблица чисел: 0 1 2 1957 1958 1 3 3915 Каждое число (кроме чисел верхней строки) равно сумме двух ближайших чисел предыдущей строки. Доказать, что число, стоящее в самой нижней строке, делится на 1958. VIII класс 6. Внутри треугольника ABC взята точка О. На лучах ОА, ОВ, ОС построены векторы единичной длины. Доказать, что сумма этих векторов имеет длину, меньшую единицы. 7. Доказать, что если уравнения с целыми коэффициентами имеют общий не целый корень, то pi=p2, (j/\=Q2- 8. На круглой поляне радиуса R растут три круглые со- сосны одинакового диаметра. Центры их стволов находятся п на расстоянии ~- от центра поляны в вершинах равносторон- равностороннего треугольника. Два человека, выйдя одновременно из диамет- диаметрально противоположных точек поляны, обходят поляну по краю с одинаковой скоростью в одном направлении и все время не видят друг друга. Увидят ли друг друга три человека, если они будут так же обходить поляну, выйдя из точек, находящих- находящихся в вершинах вписанного в поляну правильного треугольника? 9. См. задачу 13 при л = 1958. 10*. Проекции многоугольника на ось ОХ, биссектрису первого и третьего координатных углов, ось OY и биссектрису второго и четвертого координатных углов равны соответственно 4, Зу2, 5, 4-\/2. Площадь многоугольника равна S. Доказать, что S<17,5. 62
IX класс 11. Бесконечная плоская ломаная А0А\ ... А„ ... , все углы кото- которой прямые, начинается в точке Ао с координатами * = 0, у=1 и обходит начало координат О по часовой стрелке. Первое звено ломаной имеет длину 2 и параллельно биссектрисе четвер- четвертого координатного угла. Каждое из следующих звеньев пересе- пересекает одну из координатных осей и имеет наименьшую возможную при этом целочисленную длину. Расстояние ОА„ обозначается через г„, сумма длин первых п звеньев ломаной — через Sn. Доказать, что найдется п, для которого —> 1958. 12. Какое наибольшее число осей симметрии может иметь пространственная фигура, состоящая из трех прямых, из которых никакие две не параллельны и не совпадают? 13. Решить в целых положительных числах уравнение 1 — 2 + - 3 + T-i х* + 14. Отрезок длины 3й разбивается на три равные части. Первая и третья из них называются отмеченными. Каждый из отмеченных отрезков разбивается на три равные части, из которых первая и третья снова называются отмеченными, и т. д. до тех пор, пока не получатся отрезки длины 1. Концы всех отме- отмеченных отрезков называются отмеченными точками. Доказать, что для любого целого k можно найти две отмеченные точки, расстояние между которыми равно fe(l^fe^3"). X класс 15*. См. условие задачи 10. Доказать, что S^ 10. 16. Доказать, что 1155|958 + 341958=?л2 ни для какого целого п. 17. См. задачу 12. 18. На листе бумаги нарисован правильный 100-угольник, в вершинах которого по порядку написаны 1, 2, ..., 100. Затем эти числа переписывают в порядке удаления от переднего края листа. Если вершины находятся на равном расстоянии от края, сначала выписывается левое число, затем правое. Выписаны всевозможные различные наборы чисел, соответству- соответствующие разным положениям правильного 100-угольника на листе. Вычислить сумму чисел, стоящих во всех этих наборах на три- тринадцатом месте слева. 63
19*. Из четырех прямых на плоскости никакие две не парал- параллельны, никакие три не пересекаются в одной точке. По каждой прямой с постоянной скоростью идет пешеход. Известно, что первый пешеход встречается со вторым, третьим и четвертым, а второй — с третьим и четвертым. Доказать, что третий пешеход встречается с четвертым. 2-й т у р VII класс 20. Доказать, что на плоскости нельзя расположить больше четырех выпуклых многоугольников так, чтобы каждые два из них имели общую сторону. 21. Имеются два набора из + 1 и —1. в каждом по 1958 чисел. Доказать, что за некоторое число шагов можно превратить первый набор во второй, если на каждом шаге разрешается одновременно изменить знак у любых 11 чисел пер- первого набора. (Два набора считаются одинаковыми, если у них на одинаковых местах стоят одинаковые числа.) 22. Каждая грань куба заклеивается двумя равными прямо- прямоугольными треугольниками с общей гипотенузой, один из которых белый, другой черный. Можно ли эти треугольники располо- расположить так, чтобы при каждой вершине куба сумма белых углов была равна сумме черных углов? 23. Доказать, что если целое л>2, то (л!J>л". 24. Сторона клетки клетчатой бумаги равна 1. По линиям сетки построен прямоугольник размером тХл. Можно ли в прямоуголь- прямоугольнике провести по линиям сетки ломаную, которая ровно один раз проходила бы через каждый узел сетки, расположен- расположенный внутри или на границе прямоугольника? Если можно, то како- какова ее длина? VIII класс 25. Из бумаги вырезан многоугольник. Через две точки его границы проводится прямая, по которой многоугольник перегиба- перегибается. Доказать, что периметр многоугольника, получающегося по- после перегибания, не больше периметра исходного многоугольника. 26. Доказать, что для любых чисел а\ и а2, таких, что Qi^O, ai+a2=l, можно найти такие числа Ь\ и Ь2, что 62>0, Ь, + Ь2=1 и A,25 — а,N,+3A,25 — а2)Ь2>1. 27. Внутри Z.AOB взята точка С. Из нее опущены перпен- перпендикуляры: CD— на сторону ОА, СЕ— на сторону ОВ. Из точек D и Е опущены перпендикуляры: DN — на сторону ОВ, ЕМ — на сторону ОА. Доказать, что OC.LMN. 28. Доказать, что если целое и>1, то 1'.22-33. ..../!• 64
29. Обозначим через а наибольшее число непересекающихся кругов диаметра 1, центры которых лежат внутри многоуголь- многоугольника М, через b — наименьшее число кругов радиуса 1, которыми можно покрыть весь многоугольник М. Какое число больше- а или Ь? IX класс 30. См. задачу 35. 31. Из точки О провести п лучей на плоскости так, чтобы сумма всех попарных углов между ними была наибольшей. 32. Игральная доска имеет форму ромба с углом 60°. Каждая сторона ромба разделена на 9 частей. Через точки деления про- проведены прямые, параллельные сторонам и малой диагонали ромба, разбивающие доску на треугольные клетки. Если на некоторой клетке поставлена фишка, проведем через центр этой клетки три прямые, параллельные сторонам и малой диагонали ромба. Клетки, которые они пересекут, будут считаться побитыми фишкой. Каким наименьшим числом фишек можно побить все клетки доски? 33. Обозначим через а наименьшее число кругов радиуса 1, которыми можно полностью покрыть заданный многоугольник М, через b — наибольшее число непересекающихся кругов радиуса 1 с центрами внутри многоугольника М. Что больше: а или Ь? 34. Между зажимами А и В включено несколько сопротивле- сопротивлений. Каждое сопротивление имеет входной и выходной зажимы. Какое наименьшее число сопротивлений необходимо иметь и како- какова должна быть схема их соединения, чтобы при порче любых 9 сопротивлений между зажимами А и В цепь осталась замкнутой, но не было короткого замыкания? (Порча сопротивле- сопротивления: короткое замыкание или обрыв.) А" класс 35. Решить в целых положительных числах уравнение 36. В многоугольнике существуют такие точки А и В, что любая соединяющая их ломаная, проходящая внутри или по границе многоугольника, имеет длину больше 1. Доказать, что периметр многоугольника больше 2. 37*. В школе изучают 2л предметов. Все ученики учатся на «4» и «5». Никакие два ученика не учатся одинаково, ни про каких двух нельзя сказать, что один из них учится лучше другого. Доказать, что число учеников в школе не больше чем Примечание. Мы считаем, что один ученик учится лучше другого, если у него по всем предметам не хуже, чем у второго ученика, а по некоторым предметам — лучше. 3 3ah« 2-17 65
38. Стороны параллелограмма равны а и Ь. Найти отношение объемов тел, полученных при вращении параллелограмма во- вокруг стороны а и вокруг стороны Ь. 39. На п карточках написаны с разных сторон числа: на первой 0 и 1, на второй 1 и 2, ..., на п-й числа п—\ и п. Один человек берет из стопки несколько карточек и пока- показывает одну сторону. Указать все случаи, в которых второй может определить число, написанное на обороте последней показанной ему карточки. XXII ОЛИМПИАДА A959 г.) 1-й тур VII класс 1. Пусть а и b — натуральные числа. Напишем число b справа от числа а. Если число а четное, то разделим его на 2, если оно нечетное, то сначала вычтем из него единицу, а потом разделим его на 2. Получившееся число й\ напишем под числом а. Справа от числа щ напишем число 2Ь. С числом а\ проделаем ту же операцию, что и с числом а, и, получив число а2, напишем его под числом at. Справа от числа а2 за- запишем число 4Ь. Получив аналогичным образом число а^, напи- напишем справа от него число 8Ь, и т. д. Этот процесс про- продолжаем до тех пор, пока не получим в левом столбце число 1. Доказать, что сумма тех чисел правого столбца, слева от которых стоят нечетные числа, равна произведению ab. 2. Доказать, что число 22 — 1 делится на 3. 3*. Можно ли расположить все трехзначные числа, не окан- оканчивающиеся нулями, в последовательность так, чтобы последняя цифра каждого числа была равна первой цифре следующего за ним? 4. Как должна двигаться ладья по шахматной доске, чтобы побывать на каждом поле по одному разу и сделать наимень- наименьшее число поворотов? 5. Дан квадрат со стороной 1. Найти множество всех точек, сумма расстояний от которых до сторон этого квадрата или их продолжений равна 4. VIII класс 6. Даны две бочки бесконечно большой емкости. Можно ли, пользуясь двумя ковшами емкостью B—У2) л и д'2 л, перелить из одной в другую ровно 1 л? 7. Заметим, что если перевернуть лист, на котором написаны цифры, то цифры 0, 1, 8 не изменятся, 6 и 9 поменяются местами, остальные потеряют смысл. Сколько существует девяти- С6
значных чисел, которые при переворачивании листа не изме- изменяются? 8. Дан выпуклый четырехугольник ABCD. Середины сторон АВ и CD обозначим соответственно через К и М, точку пересечения отрезков AM и DK — через О, точку пересече- пересечения отрезков ВМ и СК — через Р. Доказать, что площадь четырехугольника МОКР равна сумме площадей треугольников ВРС и AOD. 9. См. задачу 4. 10. Даны две непересекающиеся окружности с центрами в точках О[ и О2. Пусть а, и о2 — внутренние касательные к этим окружностям, а3 и аА — внешние касательные к ним. Пусть, далее, о5 и а6 — касательные к окружности с центром в О|, проведенные из точки Ог, ат, as — касательные к окружности с центром в точке О2, проведенные из точки О\. Обозначим через О точку пересечения прямых а\ и Ог- Доказать, что можно провести две окружности с центром в точке О так, чтобы первая касалась а3 и па, вторая касалась а5, ас, о?, fle. причем радиус второй в два раза меньше радиуса первой. IX класс 11. Имеется 1959 положительных чисел: ait a2, ..., Qi9sh, сум- сумма которых равна 1. Рассматриваются всевозможные комби- комбинации из 1000 чисел, причем комбинации считаются совпадающи- совпадающими, если они отличаются только порядком чисел. Для каждой комбинации рассматривается произведение входящих в нее чисел. Доказать, что сумма всех этих произведений меньше 1 12. См. задачу 7. 13. Построить окружность, проходящую через две данные точки и отсекающую от данной окружности хорду данной длины. 14. Рассмотрим лист клетчатой бумаги со стороной клетки, равной 1. Пусть pk — число всех ломаных линий длины k, начи- начинающихся в точке О — некотором фиксированном узле сетки, причем все ломаные составлены из звеньев сетки. Доказать, что для любого к справедливо неравенство /7/г<2-Зк. 15*. Доказать, что не существует тетраэдра, в котором каждое ребро являлось бы стороной плоского тупого угла. X класс 16. Доказать, что не существует таких целых чисел х, у, г, что xk-\-yk = zk при условии 2>0, 0<х<&, 0<t/<&, k — нату- натуральное число. 17. См. задачу 8. 18. Существует ли тетраэдр, каждое ребро которого являлось бы стороной плоского тупого утла? (Ср. с задачей 15.) 3* С7
19. В квадратную таблицу NXN записаны все целые числа от 1 до N2 по следующему закону: 1 стоит на любом месте; 2 стоит в строке с номером, равным номеру столбца, содержащего 1; 3 стоит в строке с номером, равным номеру столбца, содержащего 2, и т. д. На сколько сумма чисел в строке, содержащей 1, отличается от суммы чисел в столбце, содержащем N2? 20. Дана невозрастающая последовательность положительных чисел: Oi>fi2>fi3> - >а«^ ..-; fii=^r. a,-\-a2+ ...+ап + ...= = 1. Доказать, что найдутся k чисел, из которых самое малень- маленькое больше половины самого большого. 2-й тур VII класс 21. Имеется два набора чисел: at >а-2> ...>ап и Ь\>Ь2> >...>Ьп. Доказать, что a\b\-\-a2b2-{- ... + anbri>aibri + a2bn-[-\- ... + апЬ{. 22. Дан треугольник ABC. Найти такую точку, что ее образ при осевой симметрии относительно прямой, содержащей любую сторону треугольника, лежит на описанной окружности. 23. На какое целое число надо умножить 999 999 999, чтобы получить число, состоящее из одних единиц? 24. Доказать, что в любом шестизначном числе можно пере- переставить цифры так, чтобы сумма первых трех цифр нового числа отличалась от суммы вторых трех цифр меньше чем на 10. 25. Дано п чисел: Х\, х2, ..., хп, при этом хи = ± 1, k = 1,2, ... , п. Доказать, что если Х\Хч-\-Х2Хз-^...-\-хп-\Хп-\-хпх\ =0, то п де- делится на 4. VIII класс 26. См. задачу 25. 27. Даны 12 чисел а1у а2, аз, ... , Qi2, причем имеют место следующие неравенства: О,, (Ою- fin+fi|2)<0. Доказать, что среди этих чисел найдется по крайней мере 3 положительных и 3 отрицательных. 28. Дан треугольник ABC; О\, О2, Оз — его вневписанные окружности. Для каждой пары из этих окружностей построим вторую общую внешнюю касательную (одна такая касательная уже проведена — это продолжение стороны ААВС). Три по- 68
строенные прямые образуют треугольник. Найти его углы, если углы треугольника ABC известны. 29. Даны два пересекающихся отрезка АВ и CD длины 1. Доказать, что по крайней мере одна из сторон четырехуголь- четырехугольника ABCD не меньше —-. 30*. Доказать, что шахматную доску размером 4X4 нель- нельзя обойти ходом шахматного коня, побывав на каждом поле ровно один раз. IX класс 31. Даны такие сто чисел х{, х2, ..., х1Оо, сумма которых равна 1, что каждая из абсолютных величин разностей Xk+\—Xk меньше —. Доказать, что из этих 100 чисел можно выбрать 50 чисел так, чтобы их сумма отличалась от — не больше чем на ——. Z 1UU 32. п отрезков длины 1 пересекаются в одной точке. Доказать, что хотя бы одна сторона 2«-угольника с вершинами в концах этих отрезков не меньше стороны правильного 2«-угольника, вписанного в окружность диаметра 1. 33*. Доказать, что не более одной вершины тетраэдра об- обладает тем свойством, что сумма любых двух плоских углов при этой вершине больше 180°. 34. Доказать, что существует бесконечно много чисел, не представимых в виде суммы трех кубов. 35. В углах шахматной доски размером 3x3 стоят кони: в верхних углах белые, а в нижних ¦— черные. Одним ходом разрешается переставить любого коня на любое свободное поле в соответствии с правилами шахмат. Мы хотим поставить белых коней в нижние углы доски, а черных коней в верхние углы. Доказать, что для этого потребуется не меньше 16 ходов. А" класс 36. См. задачу 34. 37. Пусть ABCD — пространственный четырехугольник, точки К\ и Кг делят соответственно стороны АВ и CD в отношении а, точки Кз и Kt делят соответственно стороны ВС и AD в отношении р. Доказать, что отрезки К1К2 и КзК* пересекаются. 38*. На плоскости дано несколько пересекающихся кругов, объединение которых имеет площадь, равную 1. Доказать, что из них можно выбрать некоторое количество попарно неперекры- неперекрывающихся, чтобы сумма их площадей была не менее — 69
39*. Дано п комплексных чисел с\, с2, ... , с„ таких, что если их представить себе как точки плоскости, то они являются вершинами выпуклого л-угольника. Доказать, что если комплексное число z обладает тем свойством, что г — С\ г — сг г — с„ то точка плоскости, соответствующая г, лежит внутри этого л-угольника. 40. Два диска разного диаметра разделены на 2п равных сектора каждый, и полученные при этом секторы выкрашены в бе- белый и черный цвет таким образом, что на каждом диске оказалось л белых и п черных секторов. Если укрепить оба диска на одной оси, проходящей через их центры, то окажется, что окружность — край меньшего диска — окрашена дважды: изнутри и снаружи. При этом одни части окружности окрашены в разные цвета, а остальные — в один цвет с обеих сторон. Доказать, что можно так повернуть меньший диск, что части первого рода составят не меньше половины длины окружности. XXIII ОЛИМПИАДА A960 г.) 1-й тур VII класс 1. Указать все денежные суммы, выраженные целым числом рублей, которые могут быть представлены как четным, так и нечетным числом денежных билетов. Примечание. Считать, что в обращении имеются билеты достоинством в 1, 3, 5, 10, 25, 50 и 100 р. 2. Три равные окружности с центрами О\, О2, О3 пере- пересекаются в данной точке. А\, Аг, Аз — остальные точки пересе- пересечения. Доказать, что треугольники О\О2Оз и А\А2Аг равны. 3. В составлении 40 задач приняло участие 30 студентов со всех 5 курсов. Любые два однокурсника придумали одинаковое число задач. Любые два студента с разных курсов придумали разное число задач. Сколько человек придумало по одной задаче? 4. М и N — точки пересечения двух окружностей с центра- центрами Oi и О2. Прямая О\М пересекает первую окружность в точке А\, а вторую — в точке А2. Прямая О2М пересекает первую окружность в точке Ви а вторую окружность — в точ- точке В-,. Доказать, что прямые А\В\, А2В2, MN пересекаются в одной точке. 5. Доказать, что число делителей числа п не превосходит 2-V"- 70
VIII класс 6. Доказать, что число, в десятичной записи которого имеется триста единиц, а остальные цифры — нули, не является точным квадратом. 7. В турнире каждый шахматист половину всех очков набрал во встречах с участниками, занявшими три последних места. Сколько человек принимало участие в турнире? 8. Через данную вершину А выпуклого четырехугольника ABCD провести прямую, делящую его площадь пополам. 9. Даны отрезки АВ, CD и точка О, причем никакие три из точек А, В, С, D, О не лежат на одной прямой. Конец отрезка называется «отмеченным», если прямая, проходящая через него и точку О, не пересекает другого отрезка. Сколько может быть «отмеченных» концов? 10*. Доказать, что существует бесконечно много натуральных чисел, не представимых в виде p-\-nik ни при каких простых р и натуральных п и k. IX класс 11. Доказать, что любая правильная дробь может быть пред- представлена в виде конечной суммы обратных величин попарно различных целых чисел. 12. См. задачу 10. 13. Даны выпуклый многоугольник и точка О внутри его. Любая прямая, проходящая через точку О, делит площадь много- многоугольника пополам. Доказать, что многоугольник — центрально- симметричный и О — центр симметрии. 14. Найти множество четвертых вершин прямоугольников, у которых две вершины лежат на данной окружности, а третья — в данной точке внутри окружности. X класс 15. Два равных правильных треугольника расположены в про- пространстве в параллельных плоскостях Р\ и Р?, причем от- отрезок, соединяющий их центры, перпендикулярен этим плоскостям. Найти множество точек, являющихся серединами всех отрезков, соединяющих точки одного треугольника с точками другого треугольника. 16. Числа a, b, n натуральные. Доказать, что если an+fcn и ¦ , а"+Ьп а-\-Ъ одновременно делятся на п, то —-j-j- в случае, если оно целое, также делится на п. 17. См. задачу 14. 18. В десятичной записи целого числа А все цифры, кроме первой и последней,— нули, первая и последняя — не нули, число 71
цифр — не меньше трех. Доказать, что А не является точным квадратом. 19. Даны числа щ, а2, а3, ... , ак, причем для всех нечетных п имеет место равенство Доказать, что те из чисел ci, a2, ... , а*, которые не равны нулю, можно разбить на пары таким образом, чтобы два числа, входящих в одну и ту же пару, были бы равны по абсолютной величине, но противоположны по знаку. 2-й т у р VII класс 20. Даны четыре точки А, В, С, D. Найти такую точку О, что сумма расстояний от нее до данных точек минимальна. 21. Доказать, что стороны произвольного четырехугольника являются сторонами некоторой трапеции. 22. Доказать, что любой несамопересекающиися пятиугольник лежит по одну сторону хотя бы от одной из своих сторон. 23. В каком-то году некоторое число ни в одном месяце не было воскресеньем. Найти это число. VIII класс 24. Каково наибольшее п, при котором можно так располо- расположить п точек на плоскости, чтобы каждые три из них служили вершинами прямоугольного треугольника? 25. Имеется бесконечная шахматная доска. Обозначим через (а, Ь) поле, расположенное на пересечении горизонтали с номером а и вертикали с номером Ь. Фишка с поля (а, Ь) может сделать ход на любое из восьми полей: {а±т, Ь + п), {а±п, Ь±т), где т, п — фиксированные числа, а знаки ( + ) и (—) комбини- комбинируются произвольно. Сделав х ходов, фишка вернулась на исход- исходное поле Доказать, что х четно. 26. См. задачу 21. 27*. Улитка ползет вперед (не поворачивая назад) с не- непостоянной скоростью. Несколько человек наблюдают за ней по очереди в течение 6 мин. Каждый начинает наблюдать раньше, чем кончает предыдущий, и наблюдает ровно 1 мин, причем замечает, что за эту минуту улитка проползла ровно 1 м. Дока- Доказать, что за 6 мин улитка сможет проползти самое большее 10 м. 28, Дан пятиугольник ABCDE, в котором AB = BC = CD = DE, B= Z.D = 90°. Доказать, что пятиугольниками, равными данно- данному, можно покрыть плоскость (без щелей и перекрытий). 72
IX класс 29. Имеется т точек, некоторые из которых соединены от- отрезками так, что каждая соединена с / точками. Какие значе- значения может принимать /? 30. Дан произвольный центрально-симметричный шестиуголь- шестиугольник. На его сторонах как на основаниях построены во внешнюю сторону правильные треугольники. Доказать, что середины отрез- отрезков, соединяющих вершины соседних треугольников, являются вершинами правильного шестиугольника. 31. Доказать, что никакую прямоугольную шахматную доску шириной 4 клетки нельзя обойти ходом шахматного коня, побы- побывав на каждом поле по одному разу и последним ходом вернувшись на исходную клетку. 32. Найти множество центров симметрии всех прямоугольни- прямоугольников, описанных около данного остроугольного треугольника. 33*. В квадрате со стороной 100 расположено N кругов радиуса 1, причем всякий отрезок длины 10, целиком располо- расположенный внутри квадрата, пересекает хотя бы один круг. Доказать, что Л^400. X класс 34. Число А делится на 1, 2, 3, ... , 9. Доказать, что если 2А представлено в виде суммы натуральных чисел, меньших 10, т. е. 2А — а\-\-а2-\- ... -\-аь, то из чисел щ, а2, ... , ak можно выбрать несколько таких, что их сумма равна А. 35. бп-значное число делится на 7. Последнюю цифру пере- перенесли в начало. Доказать, что полученное число также делится на 7. 36. Собралось п человек. Некоторые из них знакомы между собой, причем каждые два незнакомых имеют ровно двух общих знакомых, а каждые два знакомых не имеют общих знакомых. Доказать, что каждый из присутствующих знаком с одинаковым числом человек. 37. См. задачу 32. 38. Улитка должна проползти вдоль линий клетчатой бумаги путь длины 2п, начав и кончив свой путь в данном узле. Доказать, что число различных ее маршрутов равно (С?,,J. XXIV ОЛИМПИАДА A961 г.) 1-й тур VII класс 1. См. задачу 13 при четном п. 2. Имеется трехзначное число abc; возьмем число сЪа и вычтем из большего меньшее. Получим число а.\Ъ\С\, сделаем 73
с ним то же самое и т. д. (случай а{=0 допускается). До- Доказать, что на каком-то шаге мы получим или число 495, или 0. 3. Дан остроугольный треугольник AqBqCq. Пусть точки А\, В\, С\ — центры квадратов, построенных на сторонах ВоС0, С0А0, А0Во. С полученным треугольником А\В\С\ делаем то же самое и получаем треугольник AiBiCi и т. д. Дрказать, что гра- границы треугольников АпВпСп и Ап+\Вп+\Сп+\ пересекаются ровно в шести точках. 4. Имеется 100 точек на плоскости, причем расстояние между любыми двумя из них не превосходит 1, и если А, В, С — любые три точки из данных, то треугольник ABC тупоугольный. Доказать, что можно провести такую окружность радиуса 0,5, что все данные точки окажутся внутри ее или на ней самой. 5. На шахматной доске выбраны две клетки одинакового цвета. Доказать, что ладья, начав с первой из этих клеток, может обойти все клетки по разу, а на второй выбранной клетке побывать два раза. VIII класс 6. Даны треугольник ABC и точка О. Обозначим через Ми М2, Мз центры тяжести треугольников ОАВ, ОВС, ОСА соответ- соответственно. Доказать, что площадь треугольника М1М2М3 равна — площади треугольника ABC. 7. Играют двое, один из них загадывает набор из целых однозначных чисел Х\, х2, ... , х„, как положительных, так и отрицательных. Второму разрешается спрашивать, чему равна сумма aiXi-\-a2X2-\- ... -\-а„хп, где щ, а2, ..., а„—любой набор чисел. Каково наименьшее число вопросов, за которое отгады- отгадывающий может узнать задуманный набор? 8. См. задачу 3. 9. Доказать, что ладья может обойти все клетки прямоуголь- прямоугольной шахматной доски, побывав на каждой клетке ровно один раз, и вернуться в начальную клетку только в случае, если число клеток на доске четно. (Ср. с задачей 5.) 10. Набор чисел а, а-\-1, а-\-2, ..., a-\-k называется отрезком натурального ряда. Два отрезка натурального ряда, каждый длины 1961, подписаны один под другим. Доказать, что можно так переставить числа в каждом из отрезков, что после сложения чисел, стоящих друг под другом, снова получится отре- отрезок натурального ряда. IX класс 11. См. задачу 6. 12. См. задачу 7. 13. Доказать, что можно так расположить числа от 1 до п2 74
в таблице пХ«, чтобы суммы чисел каждого столбца были равны. 14.'См. задачу 18 в предположении, что k делится на 4. 15. На плоскости расположены п точек таких, что если А, В, С — любые три из них, то внутри треугольника ABC нет ни одной точки из данных. Доказать, что эти точки можно зануме- занумеровать так, что многоугольник AiA2A3... Ап будет выпуклым. X к л а ее 16. Дана последовательность ии чисел Фибоначчи 1, 1, 2, 3, 5, ... . Доказать, что usk делится на 5 при всех k= 1, 2, 3, . . 17. На плоскости проведено несколько полос разной ширины. Никакие две из них не параллельны. Как нужно параллельно перенести каждую из этих полос, чтобы площадь их общей части была наибольшей? 18. k человек ехали в автобусе без кондуктора. Известно, что ни у кого из пассажиров не было медных денег и монет крупнее 20 коп. Известно далее, что каждый пассажир уплатил за проезд и получил сдачу. Доказать, что наименьшее число монет, которое могло для этого потребоваться, равно k-И —j— . 19. Окружность S и точка О лежат в одной плоскости, причем О находится вне окружности. Построим произвольную сферу, проходящую через окружность S, и опишем конус с вершиной в точке О, касающийся этой сферы. Найти множество центров всех окружностей, по которым конусы касаются сфер. 20. Известно, что Zi-\-z2-\- ... -\-zn = 0, где z,- — комплексные числа. Доказать, что среди этих чисел найдутся два таких числа, что разность их аргументов больше или равна 120°. 2-й тур VII класс 21. Стороны произвольного выпуклого многоугольника покра- покрашены снаружи. Проводим несколько диагоналей многоугольника. Каждая из этих диагоналей тоже покрашена с одной стороны, т. е. с одной стороны отрезка проведена узкая цветная полоска. Доказать, что хотя бы один из многоугольников, на которые разбит диагоналями исходный многоугольник, весь покрашен сна- снаружи. (Допускается, чтобы в вершине многоугольника окраска заходила внутрь.) 22. В квадрате ABCD на сторонах АВ, ВС, CD и AD взяты точки Р, Q, R, S соответственно; оказалось, что фигура PQRS — прямоугольник. Доказать, что тогда PQRS — либо квад- квадрат, либо обладает тем свойством, что его стороны параллельны диагоналям квадрата. 75
23. Доказать, что среди любых 39 по- последовательных натуральных чисел обя- обязательно найдется такое, у которого сумма цифр делится на 11. 24. Дана таблица размером 4X4. Рис- 3- Показать, что можно так расставить семь звездочек в клетках этой таблицы, чтобы при вычеркивании любых двух строк и любых двух столбцов этой таблицы в остав- оставшихся клетках всегда была хотя бы одна звездочка. Доказать, что если звездочек меньше семи, то всегда можно так вычерк- вычеркнуть две строки и два столбца, что все оставшиеся клетки будут пустыми. 25. Доказать, что не существует целых чисел а, Ь, с, d, удовлетворяющих равенствам abed — a=1961, abed — 6= 961, abed — с= 61, abcd — d= 1. VIII класс 26. Дана фигура, состоящая из 16 отрезков (рис. 3). Дока- Доказать, что нельзя провести ломаную, пересекающую каждый из отрезков ровно один раз. Ломаная может быть незамкнутой и самопересекающейся, но ее вершины не должны лежать на отрезках, а стороны не могут проходить через общие концы отрезков. 27*. Диагональ прямоугольника равна d. Из его вершин как из центров описаны четыре окружности радиусами Г\, г2, г3, г4, причем Г\-\-Г2 = гз-\-Гц<^с1. Проведены две пары внешних каса- касательных к окружностям с номерами 1, 3 и 2, 4. Доказать, что в четырехугольник, образованный этими четырьмя прямыми, мож- можно вписать окружность. 28. У каждого из целых чисел k и k-\-l сумма цифр делится на 11, и между ними нет числа, обладающего тем же свойством. Какое наибольшее значение может принимать /? 29. См. задачу 24. 30. Дана четверка чисел а, Ь, с, d. Из нее получается новая ab, be, cd, da умножением каждого числа на следующее, а четвертого — на первое. Из второй четверки по этому же принци- принципу получается третья и т. д. Доказать, что среди полученных четверок никогда не встретится вновь четверка а, Ь, с, d, кроме случая, когда a = b=c = d=\. IX к л а с с 31. Точки А и В движутся равномерно и с равными угловыми скоростями по окружностям О\ и О2 соответственно 76
(по часовой стрелке). Доказать, что вершина С правильного ?\АВС также движется равномерно по некоторой окружности. 32. В клетки таблицы тХ« вписаны некоторые числа. Разрешается одновременно менять знак у всех чисел некоторого столбца или некоторой строки. Доказать, что, применяя несколь- несколько раз эту операцию, можно превратить данную таблицу в такую, у которой суммы чисел, стоящих в любом столбце и в любой строке, неотрицательны. 33. п точек соединены отрезками так, что каждая точка соеди- соединена с любой другой некоторым путем и нет таких двух точек, которые соединялись бы двумя разными путями. Доказать, что общее число отрезков равно п — 1. 34*. a, b, р — любые натуральные числа. Доказать, что най- найдутся такие взаимно простые числа k, l, что ak-\-bl делится на р. 35. Коля и Петя делят 2п-\-\ орехов, п^2, причем каждый хочет получить возможно больше. Предлагается три способа деления. Каждый проходит в три этапа, причем 1-й и 2-й этапы общие для всех трех способов. 1-й этап: Петя делит все орехи на две части, в каждой не меньше двух орехов. 2-й этап: Коля делит каждую часть снова на две, в каждой не меньше одного ореха. 3-й этап: при первом способе Коля берет наибольшую и наименьшую части. При втором способе Коля берет себе обе средние части. При третьем способе Коля берет либо большую и меньшую части, либо средние части, но за право выбора отдает Пете один орех. Определить, какой способ самый выгодный для Коли и какой наименее выгоден для него. X класс 36. Доказать, что для любых трех бесконечных последова- последовательностей натуральных чисел щ, а2, ..., а„, ¦¦¦', Ь\, Ь2, ..., Ьп, ...; С\, с2, ..., сп, ... найдутся такие номера р и q, что p^q p^q 37. В прямоугольник со сторонами 20 и 25 бросают 120 квадратов со стороной 1. Доказать, что в прямоугольник можно поместить круг диаметра 1, не пересекающийся ни с одним из квадратов. 38. См. задачу 32. 39. Расстояния от фиксированной точки Р плоскости до двух вершин А, В равностороннего треугольника ABC равны АР = 2, ВР = ?>. Определить, какое максимальное значение может иметь длина отрезка PC. 40. Дан произвольный набор из +1 и — 1 длиной 2к. Из него получается новый набор по правилу: каждое число умно- умножается на следующее за ним; последнее, 2*-е число, умножается на первое. С новым набором из +1 и — 1 проделывается то же самое и т. д. Доказать, что в конце концов получится набор, состоящий из одних единиц. 77
XXV ОЛИМПИАДА A962 г.) 1-й тур VII класс 1. Дана прямая /, перпендикулярная отрезку АВ и пересе- пересекающая его Для любой точки М прямой / строится такая точка N, что ANAB = 2AMAB; ANBA=2^MBA. Доказать, что модуль разности AN — BN не зависит от выбора точки М на прямой /. 2. Правильный треугольник, одна сторона которого отмечена, отражается симметрично относительно одной из своих сторон. Полученный треугольник в свою очередь отражается аналогично и т. д., пока на некотором шаге треугольник не придет в первона- первоначальное положение. Доказать, что при этом отмеченная сторона также займет исходное положение. 3. Пусть а, Ь, с, d — стороны четырехугольника, не являющего- являющегося ромбом. Доказать, что из отрезков а, Ь, с, d можно сло- сложить самопересекающийся четырехугольник. 4. Сумму цифр числа а обозначим через s(a). Доказать, что если s(a) = sBa), то число а делится на 9. 5. Даны п карточек; на обеих сторонах каждой карточки написано по одному из чисел 1, 2, ..., п, причем так, что каждое число встречается на всех п карточках ровно два раза. Доказать, что карточки можно разложить на столе так, что сверху окажутся все числа: 1, 2, ..., п. VIII класс 6. На сторонах АВ, ВС, СА правильного треугольника ABC найти такие точки X, Y, Z (соответственно), чтобы площадь треугольника, образованного прямыми СХ, BZ, AY, была вчетверо меньше площади треугольника ABC и чтобы было выполнено 7. См. задачу 2. 8. Доказать, что для любого целого d найдутся такие . п — 2т+ 1 целые т, п, что с= г . 9. См. задачу 4. 10. См. задачу 5. IX класс 11. Даны два пересекающихся отрезка АВ и CD. На них выбираются точки М и N (соответственно) так, что AM = CN. Найти положение точек М и N, при котором длина отрезка MN минимальна. 78
12. Конем называется фигура, ход которой состоит в перемеще- перемещении на п клеток по горизонтали и на 1 по вертикали (или наоборот). Конь стоит на некотором поле бесконечной шахмат- шахматной доски. При каких п он может попасть на любое задан- заданное поле? При каких п это невозможно? 13. См. задачу 4. 14. Дана система уравнений Х\ з ... Х1Э61 — Какие значения может принимать Х25? 15. Доказать, что в прямоугольнике площади 1 можно рас- расположить непересекающиеся круги так, чтобы сумма их радиу- радиусов была равна 1962. X класс 16. В задаче 11 заменить отрезки пересекающимися лучами. 17. На сторонах квадрата как на основаниях построены во внешнюю сторону равные равнобедренные треугольники с острым углом при вершине. Доказать, что получившуюся фигуру нельзя разбить на параллелограммы. 18. Доказать, что любое натуральное число можно предста- представить в виде суммы нескольких различных членов последователь- последовательности Фибоначчи 1, 2, 3, 5, 8, 13, ... 19. См. задачу 14. 20. См. задачу 15. 2-й т у р VII класс 21. У края бильярда, имеющего форму правильного 2/г-уголь- ника, стоит шар. Как надо пустить шар от борта, чтобы он, отразившись от всех бортов, вернулся в ту же точку? Доказать, что при этом длина пути шара не зависит от выбора началь- начальной точки. 22*. Дан равнобедренный треугольник ABC, в котором АВ—ВС, ВП — высота, М — середина стороны АВ, К — точка пересечения ВН с окружностью, проходящей через В, М и С. Доказать, что BK=-z-R, где R — радиус описанной около треу- треугольника ABC окружности. 23. Уголком называется фигура, составленная из трех квадра- квадратов со стороной 1 в виде буквы «Г». Доказать, что прямоуголь- 79
ник размером 1961 X 1963 нельзя разбить на уголки, а прямоуголь- прямоугольник размером 1963X1965 — можно. 24. Дано число 100 ... 01; число нулей в нем равно 1961. Доказать, что это число составное. 25. На плоскости даны 25 точек; известно, что из любых трех точек можно выбрать две, расстояние между которыми меньше 1. Доказать, что среди данных точек найдутся 13, лежащих в круге радиуса 1. VIII класс 26. Проведем в выпуклом многоугольнике некоторые диаго- диагонали так, что никакие две из них не пересекаются (из одной вершины могут выходить несколько диагоналей). Доказать, что найдутся по крайней мере две вершины многоугольника, из кото- которых не проведено ни одной диагонали. 27. Как надо расположить числа 1, 2, ..., 1962 в после- последовательности а\, а2, ... , Oi962, чтобы сумма \а,— а2\ + \а2 — а3\ + ... была наибольшей? 28. В окружность вписан неправильный п-угольник, который при повороте окружности около центра на некоторый угол а=^2я совмещается сам с собой. Доказать, что п — число составное. 29*. Из чисел х\, х2, х3, х4, xs можно образовать десять попарных сумм; обозначим их через а.\, а-2, ... , йю. Доказать, что, зная числа щ, а2, ..., йю (но не зная, разумеется, суммой каких именно двух чисел является каждое из них), можно восстановить числа Х\, х2, ..., хь- 30. Две окружности О\ и О2 пересекаются в точках ТА и Р. Обозначим через МА хорду окружности Оь касающуюся окруж- окружности О-2 в точке М, а через MB — хорду окружности О2, касающуюся окружности О\ в точке М. На прямой МР отложен отрезок РИ=МР. Доказать, что около четырехугольника МАНВ можно описать окружность. IX класс 31. Школьник в течение учебного года должен решать ровно по 25 задач за каждые идущие подряд 7 дней. Время, необхо- необходимое на решение одной задачи (любой), не меняется в течение дня, но меняется в течение учебного года по известному школьнику закону и всегда меньше 45 мин. Школьник хочет затратить на решение задач в общей сложности наименьшее время. Доказать, что для этого он может выбрать некоторый день недели и в этот день (каждую неделю) решать по 25 задач. 32. См. задачу 27, где вместо чисел 1, 2, ..., 1962 взяты 25 произвольных различных чисел. 80
33. Стороны выпуклого многоугольника, периметр которого равен 12, параллельно переносятся на расстояние d=\ во внеш- внешнюю сторону и продолжаются до пересечения. Доказать, что пло- площадь многоугольника увеличится по крайней мере на 15. 34. См. задачу 29. 35. Даны 2" конечных последовательностей из нулей и единиц, причем ни одна из них не является началом никакой другой. Доказать, что сумма длин этих последовательностей не меньше л-2". X класс 36. На данной прямой /, проходящей через центр О дан- данной окружности, фиксирована точка С. Точки А и А' распо- расположены на окружности по одну сторону от / так, что углы, образованные прямыми АС и А'С с прямой /, равны. Обозначим через В точку пересечения АА' и Л Доказать, что положение точки В не зависит от точки А. 37. См. задачу 32. 38. См. задачу 33. 39. Как надо расположить в пространстве прямоугольный параллелепипед, чтобы площадь его проекции на горизонтальную плоскость была наибольшей? 40. В шахматном турнире каждый участник сыграл с каждым другим одну партию. Доказать, что участников можно так зануме- занумеровать, что окажется, что ни один участник не проиграл непо- непосредственно следующему за ним. XXVI ОЛИМПИАДА A963 г.) 1-й тур VII класс * 1. Из вершины В произвольного треугольника ABC проведе- проведены вне треугольника прямые ВМ и BN так, что ААВМ = ACBN. Точки А' и С" симметричны точкам Л и С относительно прямых ВМ и BN (соответственно). Доказать, что АС = А'С. 2. а, Ь, с — такие три числа, что а-\-Ь-\-с = 0. Доказать, что b 3. Имеется 200 карточек размером 1X2, на каждой из кото- которых написаны числа +1 и —1. Можно ли так заполнить этими карточками лист клетчатой бумаги размером 4Х 100, чтобы произведения чисел в каждом столбце и в каждой строке образовавшейся таблицы были положительны? 4. См. задачу 9. 5. Можно ли так провести прямую на листе клетчатой бумаги размером 20X30, чтобы она пересекала 50 клеток? (Ср. с задачей 18.) 81
VIII класс 6. Qi, a2, ... , а„ — такие числа, что а\-\-а-2-\- ...+а,, = 0. Доказать, что в этом случае справедливо соотношение в сумму S входят все возможные произведения а,<з,, «=/=/. 7. Даны выпуклый четырехугольник ABCD площади S и точка М внутри его. Точки Р, Q, R, S симметричны точке М отно- относительно середин сторон четырехугольника ABCD. Найти площадь четырехугольника PQRS. 8. Решить в целых числах уравнение ^~\~—-\-^-=з. 9. На плоскости даны семь прямых, никакие две из которых не параллельны. Доказать, что найдутся две из них такие, что угол между ними меньше 26°. 10. Лист клетчатой бумаги размером 5X/Z заполнен кар- карточками размером 1X2. На каждой карточке написаны числа + 1 и —1. Известно, что произведения чисел по строкам и столбцам образовавшейся таблицы положительны. При каких п это возможно? (Ср. с задачей 3.) IX класс 11. Первый член и разность арифметической прогрессии — целые числа. Доказать, что найдется такой член прогрессии, в записи которого встречается цифра 9. 12. См. задачу 10. 13. а, Ъ, с — любые положительные числа. Доказать, что Ь + с *а + с ^а+Ь ""Т" 14. Из любых четырех точек на плоскости, никакие три из которых не лежат на одной прямой, можно так выбрать три, что треугольник с вершинами в этих точках имеет хотя бы один угол, не больший 45°. Доказать. 15. Можно ли в прямоугольник с отношением сторон 9:16 вписать прямоугольник с отношением сторон 4:7 так, чтобы на каждой стороне первого прямоугольника лежала вершина второго? А' класс 16. См. задачу 11. 17. Из любых шести точек на плоскости (из которых ника- никакие три не лежат на одной прямой) можно так выбрать три, что треугольник с вершинами в этих точках имеет хотя бы один угол, не больший 30°. Доказать. (Ср. с задачей 14.) 82
18. Какое наибольшее число клеток может пересечь прямая, проведенная на листке клетчатой бумаги размером гаХя клеток? 19. а, Ь, с — такие три числа, что abOO и а-\-Ь-\~с~>0. Доказать, что а" -\-Ь" + с" > 0 при любом натуральном п. 20. Дан произвольный треугольник ABC. Найти множество всех таких точек М, что перпендикуляры к прямым AM, ВМ, СМ, проведенные из точек А, В, С соответственно, пересекаются в одной точке. XI класс 21. Положительные числа х, у, z обладают тем свойством, что arctg x-farctg y-farctg г<л. Доказать, что сумма этих чисел больше их произведения. 22. Дана система из 25 различных отрезков с общим началом в данной точке Лис концами на прямой /, не проходящей через эту точку. Доказать, что не существует замкнутой 25-звенной ломаной, для каждого звена которой нашелся бы отрезок системы, равный и параллельный этому звену. 23. См. задачу 20. 24. Из цифр 1, 2, 3, 4, 5, 6, 7 составляются всевозмож- всевозможные семизначные числа, в записи которых каждая из этих цифр встречается ровно один раз. Доказать, что сумма всех таких чисел делится на 9. 25. Каждое ребро правильного тетраэдра разделено на три равные части. Через каждую полученную точку деления проведены две плоскости, параллельные соответственно двум граням тетра- тетраэдра, не проходящим через эту точку. На сколько частей постро- построенные плоскости разбивают тетраэдр? 2-й т у р VII класс 26. Завод выпускает погремушки в виде кольца с надетыми на него 3 красными и 7 синими шариками. Сколько различных погремушек может быть выпущено? (Две погремушки считаются одинаковыми, если одна из них может быть получена из другой только передвижением шариков по кольцу и переворачиванием.) 27. См. задачу 37. 28. Дан произвольный треугольник ABC и проведена та- такая прямая, пересекающая треугольник, что расстояние до нее от точки А равно сумме расстояний до этой прямой от точек В и С Доказать, что все такие прямые проходят через одну точку. 29. Какое наибольшее количество чисел можно выбрать из набора 1, 2, ..., 1963, чтобы сумма двух любых выбранных чисел делилась на 26? 30. Система точек, соединенных отрезками, называется связ- связной, если из любой точки можно попасть в любую другую по этим 83
отрезкам. Можно ли соединить пять точек отрезками в связную систему так, чтобы при стирании любого отрезка образовались ровно две связные системы точек, не связанные друг с другом? (Мы считаем, что в местах пересечения отрезков переход с од- одного из них на другой невозможен.) VIII класс 31. щ, а.2, ..., ап — произвольные натуральные числа. Обозна- Обозначим через bk количество чисел из набора аи а2, .--, а,г, удовлетво- удовлетворяющих условию <з,:>/г. Доказать, что 32. В таблицу 8X8 вписаны все целые числа от 1 до 64. Доказать, что в ней найдутся два соседних числа, разность между которыми не меньше 5. (Соседними называются клетки, имеющие общую сторону.) 33. Найти множество центров тяжести всех остроугольных треугольников, вписанных в данную окружность. 34. Какое наибольшее количество чисел можно выбрать из набора 1, 2, ..., 1963, чтобы сумма никаких двух чисел не делилась на их разность? 35*. По аллее длиной 100 м идут три джентльмена со скоро- скоростями 1, 2, 3 км/ч. Дойдя до конца аллеи, каждый из них пово- поворачивает и идет назад с той же скоростью. Доказать, что найдется отрезок времени в 1 мин, когда все трое будут идти в одном направлении. IX класс 36*. Даны произвольный треугольник ABC и точка X вне его. AM, BN, CQ — медианы треугольника ABC. Доказать, что площадь одного из треугольников ХАМ, XBN, XCQ равна сумме площадей двух других. 37. Какое наибольшее число точек самопересечения может иметь замкнутая 14-звенная ломаная, проходящая по линиям клетчатой бумаги так, что ни на какой линии не лежит более одного звена ломаной? 38. В правильном 10-угольнике проведены все диагонали. Сколько попарно неподобных треугольников при этом образова- образовалось? 39. В таблицу 9X9 вписаны все целые числа от 1 до 81. Доказать, что найдутся два соседних числа, разность между ко- которыми не меньше 6. 40. См. задачу 35. 84
X класс 41. Доказать, что при нечетном п уравнение x"-\-yn = z" не может иметь решений в целых числах, если {х-\-у) — простое число. 42. На листе бумаги нанесена сетка из п горизонтальных и п вертикальных прямых. Сколько различных 2/г-звенных замкну- замкнутых ломаных линий можно провести по линиям сетки так, чтобы каждая ломаная проходила по всем горизонтальным и всем вер- вертикальным прямым? 43. Из центра правильного 25-угольника отложены векторы во все его вершины. Как надо выбрать несколько векторов из этих 25-ти, чтобы их сумма имела наибольшую длину? 44. А', В', С, D', Е' — середины сторон выпуклого пяти- пятиугольника ABCDE. Доказать, что площади пятиугольников ABCDE и A'B'C'D'E' связаны соотношением 45*. Последовательность чисел at, а-2, а,„ ... образуется 2 | о по закону <2|=<22=1; <зл=—— (п^З). Доказать, что все числа в последовательности целые. XI класс 46. Доказать, что не существует попарно различных нату- натуральных чисел х, у, z, t, для которых было бы справедливо соотно- соотношение Xх-\-уу = гг-\-t'. 47. Доказать, что из одиннадцати произвольных бесконечных десятичных дробей можно выбрать две дроби, разность которых имеет в десятичной записи либо бесконечное число нулей, либо бесконечное число девяток. 48. Найти все многочлены Р (а), для которых справедливо тождество хР (х — 1)=^(х — 26) Р (х). 49. См. задачу 44. 50*. Доказать, что на сфере нельзя так расположить три дуги в 300° каждая, чтобы никакие две из них не имели общих точек (в том числе общих концов). XXVII ОЛИМПИАДА A964 г.) 1-й тур VII класс 1. В треугольнике ABC высоты, опущенные на стороны АВ и ВС, не меньше этих сторон (соответственно) Найти углы треугольника. 85
2. На данной окружности выбраны диаметрально противопо- противоположные точки Л и В и третья точка С. Касательная, проведен- проведенная к окружности в точке В, и прямая АС пересекаются в точ- точке М. Доказать, что касательная, проведенная к окружности в точке С, делит пополам отрезок ВМ. 3. Доказать, что сумма цифр числа, являющегося точным квадратом, не может равняться пяти. 4. На листе бумаги проведено 11 горизонтальных и 11 верти- вертикальных прямых, точки пересечения которых называются узлами. Звеном мы будем называть отрезок, соединяющий два соседних узла одной прямой. Какое наименьшее число звеньев надо сте- стереть, чтобы после этого в каждом узле сходилось не более трех звеньев? 5. Последовательность ао, щ, аг, ¦¦¦, ап, ... образована по закону Qo = Ci = l; an+\ =ап-\ап-\-1, п = \, 2, ... Доказать, что число а 1964 не делится на 4. VIII класс 6. См. задачу 1. 7. Найти все такие натуральные числа п, что число (л—1)! не делится на п2. 8. Решить в целых числах уравнение с переменными х, у, z ~~\Jx-\-\x-\-----\-^Jx = z, (у радикалов). 9. См. задачу 14 а). 10. Рассмотрим суммы цифр всех чисел от 1 до 1000 000 включительно. У полученных чисел снова рассмотрим суммы цифр и т. д., пока не получим миллион однозначных чисел. Каких чисел больше среди них: единиц или двоек? IX класс 11. Решить в положительных числах систему {ху = z, гх=у'. 12. Доказать, что произведение двух последовательных нату- натуральных чисел не является степенью никакого целого числа. 13. Известно, что при любом целом ?=/=27 число а — k3 де- делится без остатка на 27 — k. Найти а. 14. Доказать, что: а) если все углы шестиугольника равны, то его стороны удовлетворяют соотношениям а\—а*=аь — <22 = =а3 — Об', б) если длины отрезков щ, ..., ае удовлетворяют при- приведенным соотношениям, то из этих отрезков можно построить шестиугольник с равными углами. 15. В четырехугольнике ABCD из вершин А и С опущены перпендикуляры на диагональ BD, а из вершин В и D опущены 86
перпендикуляры на диагональ АС. Доказать, что четырехуголь- четырехугольники ABCD и MNPQ подобны (М, N, Р, Q — основания пер- перпендикуляров) . X — XI классы 16. Натуральное число N является квадратом и не окан- оканчивается нулем. После зачеркивания у этого числа двух пос- последних цифр снова получился точный квадрат. Найти наибольшее число N с таким свойством. 17. См. задачу 8. 18. Известно, что при любом целом kФТ1 число о — /г1964 делится без остатка на 27 — k. Найти а. (Ср. с задачей 13.) 19. См. задачу 14. 20. На какое наименьшее число непересекающихся тетраэд- тетраэдров можно разбить куб? 2-й т у р VII класс 21. На отрезке АС выбрана произвольно точка В и на отрезках АВ, ВС и АС как на диаметрах построены окружности О\, О2 и О. Через точку В проводится произвольная прямая, пересекающая окружность О в точках Р и Q, а окружности О\ и Ог — в точках R и S (соответственно). Доказать, что PR = QS. 22. Собрались 2л человек, каждый из которых знаком не менее чем с п присутствующими. Доказать, что можно вы- выбрать из них четырех человек и рассадить их за круглым столом так, что при этом каждый будет сидеть рядом со своими знако- знакомыми. 23. В квадрате со стороной 1 выбраны 102 точки, из ко- которых никакие три не лежат на одной прямой. Доказать, что найдется треугольник с вершинами в этих точках, площадь ко- которого меньше чем 0,01. 24. Через противоположные вершины А и С четырехуголь- четырехугольника ABCD проведена окружность, пересекающая стороны АВ, ВС, CD и AD соответственно в точках М, N, Р и Q. Известно, что DP — DQ = BM = BN = R, где R — радиус данной окружности. Доказать, что в этом случае сумма углов ABC и ADC данного четырехугольника равна 120°. 25. При каких натуральных числах а существуют такие нату- натуральные числа х и у, что х2-\-у2 = ? VIII класс 26. В п стаканов достаточно большой вместимости налито поровну воды. Разрешается переливать из любого стакана в лю- 87
бой другой столько воды, сколько имеется в этом последнем. При каких п можно за конечное число операций слить всю воду в один стакан? 27. Даны три точки А, В, С, лежащие на одной прямой, и точка О вне этой прямой. Обозначим через О\, О2, О3 центры окружностей, описанных около треугольников ОАВ, ОВС, ОАС. Доказать, что точки О\, О2, Оз и О лежат на одной окружности. 28. На квадратном клетчатом поле 99 X 99 клеток играют двое. Первый игрок ставит крестик на центр поля, вслед за этим второй игрок может поставить нолик на любую из восьми кле- клеток, окружающих крестик первого игрока. После этого первый ставит крестик на любое из полей рядом с уже занятыми и т. д. Первый игрок выигрывает, если ему удается поставить крестик на любую угловую клетку. Доказать, что при любой игре второго игрока первый всегда может выиграть. 29. Внутри равностороннего (не обязательно правильного) семиугольника А\А2Аз.-А7 взята произвольно точка О. Обозна- Обозначим через Hi, Н2, .... Н7 основания перпендикуляров, опущенных из точки О на стороны А\А2, А2А3, ..., А7А\ соответственно. Известно, что точки Hi, //2, ..-, Н7 лежат на самих сторонах, а не на их продолжениях. Доказать, что AiH\ -\~А2Н2-\-...-\-А7Н7 = НА НА НА НА 30*. В квадрате со стороной 1 взята произвольно 101 точ- точка (не обязательно внутри квадрата), причем никакие три из них не лежат на одной прямой. Доказать, что существует треуголь- треугольник с вершинами в этих точках, площадь которого не больше 0,01. IX класс 31. См. задачу 29. 32. См. задачу 26. 33. Доказать, что любое четное число 2/г может быть единст- единственным образом представлено в виде 2п = (х-\-уJ-\-Зх-\-у, где А' и у — целые неотрицательные числа. 34. В треугольнике ABC сторона ВС равна полусумме двух других сторон. Доказать, что биссектриса угла ВАС перпенди- перпендикулярна отрезку, соединяющему центры вписанной и описан- описанной окружностей треугольника. 35*. На клетчатой бумаге проведена замкнутая ломаная с вершинами в узлах сетки, все звенья которой равны. Дока- Доказать, что число звеньев такой ломаной четно. X класс 36. В п мензурок налиты п разных жидкостей; кроме того, имеется одна пустая мензурка. Можно ли за конечное число операций составить равномерные смеси в каждой мензурке (т. е. сделать так, чтобы в каждой мензурке было ровно — от пер-
воначального количества каждой жидкости и при этом одна мензурка была бы пустой)? Мензурка имеет деления, позво- позволяющие измерять объем налитой жидкости. 37. Дана система из п точек на плоскости, причем известно, что для любых двух точек данной системы можно указать дви- движение плоскости, при котором первая точка перейдет во вторую, а система перейдет сама в себя. Доказать, что все точки такой системы лежат на одной окружности. 38. В треугольнике ABC сторона ВС равна полусумме двух других сторон. Доказать, что вершина А, середины сторон АВ и АС и центры вписанной и описанной окружностей лежат на одной окружности. 39. См. задачу 35. 40*. Имеется бесконечное число карточек, на каждой из ко- которых написаны какие-то натуральные числа. Известно, что для любого натурального числа п можно указать ровно п карто- карточек, на которых написаны делители этого числа. Доказать, что любое натуральное число встречается хотя бы на одной карточке. XI класс 41. Из точки О на плоскости отложено несколько векторов, сумма длин которых равна 4. Доказать, что можно выбрать не- несколько векторов (или, быть может, один вектор), длина суммы которых больше 1. 42. См. задачу 33. 43. В треугольнике ABC сторона ВС равна полусумме двух других сторон. Через точку А и середины сторон АВ и АС про- проведена окружность и к ней из центра тяжести треугольника про- проведены касательные. Доказать, что одна из точек касания яв- является центром окружности, вписанной в треугольник ABC. (Ср. с задачами 34 и 38.) 44*. Пирог имеет форму правильного п-угольника, вписан- вписанного в окружность радиуса 1. Из середины каждой стороны про- проведены прямолинейные надрезы длины 1. Доказать, что при этом от пирога всегда будет отрезан какой-нибудь кусок. 45*. При дворе короля Артура собрались 2/г рыцарей, при- причем каждый из них имеет среди присутствующих не более п — 1 врага. Доказать, что Мерлин, советник Артура, может так рас- рассадить рыцарей за Круглым Столом, что ни один из них не будет сидеть рядом со своим врагом. XXVIII ОЛИМПИАДА A965 г.) 1-й тур VIII класс 1. Даны окружность О, прямая а, пересекающая ее, и точка М. Через точку М провести секущую b так, чтобы ее часть, 89
заключенная внутри окружности О, делиаась пополам в точке ее пересечения с прямой а. 2. Докажите следующий признак делимости на 37. Для того чтобы узнать, делится ли число на 37, надо разбить его на грани справа налево, по три цифры в каждой грани. Если сумма полученных'трехзначных чисел делится на 37, то и данное число делится на 37. (Слово «трехзначное» употреблено условно: неко- некоторые из граней могут начинаться с нулей и быть на самом деле двузначными или однозначными числами; не трехзначным числом будет и самая левая грань, если количество цифр нашего числа не делится на 3.) 3. Даны прямая а и два непараллельных отрезка АВ и CD по одну сторону от нее. Найти на прямой а такую точку М, чтобы треугольники АВМ и CDM были равновелики. 4. 30 команд участвуют в розыгрыше первенства по футболу. Доказать, что в любой момент состязаний имеются две команды, сыгравшие к этому моменту одинаковое число матчей. IX класс 5. Шестизначное число делится на 37. Все его цифры ра.? личны. Доказать, что из тех же цифр можно составить друго- шестизначное число, также делящееся на 37. 6. Внутри данного треугольника ABC найти такую точку О, чтобы площади треугольников АОВ, ВОС, СОА относились как 1:2:3. 7. Дан треугольник ABC, в котором сторона АВ больше ВС. Проведены биссектрисы АК и СМ (К лежит на ВС, М лежит на АВ). Доказать, что АМ>МК>КС. 8. В стране Иллирии некоторые пары городов связаны пря- прямым воздушным сообщением. Доказать, что там есть два города, связанные с равным количеством других городов. (Ср. с зада- задачей 4.) 9. Вдоль коридора положено несколько кусков ковровой до- дорожки. Куски покрывают весь коридор из конца в конец без пропусков и даже накладываются друг на друга, так что над некоторыми местами пола они лежат в несколько слоев. Дока- Доказать, что можно убрать несколько кусков, возможно достав их из-под других и оставив остальные в точности на тех же местах, где они лежали прежде, так, что коридор по-прежнему будет полностью покрыт, а общая длина оставленных кусков будет меньше удвоенной длины коридора. X класс 10. Окружности О\ и 0-2 лежат внутри треугольника ABC и касаются друг друга извне, причем окружность О\ касается сторон АВ и ВС, а окружность О2 — сторон АВ и АС. Доказать, 90
что сумма радиусов этих окружностей больше радиуса окруж- окружности, вписанной в треугольник. 11. См. задачу 5. 12. Концы отрезка постоянной длины скользят по сторонам данного угла. Из середины этого отрезка к нему восставлен перпендикуляр. Доказать, что отрезок перпендикуляра от его основания до точки пересечения с биссектрисой угла имеет пос- постоянную длину. 13. х—число, большее двух. Некто пишет на карточках числа: 1, х, х2, х3. ..., хк (каждое число только на одной кар- карточке). Потом часть карточек он кладет себе в правый карман, часть в левый, остальные выбрасывает. Доказать, что сумма чисел в правом кармане не может быть равна сумме чисел в ле- левом. (Ср. с задачей 15.) 14. Бумажный квадрат был проколот в 1965 точках. Из то- точек-проколов и вершин квадрата никакие три не лежат на одной прямой. Потом сделали несколько прямолинейных, не пересе- пересекающихся между собой разрезов, каждый из которых начинался и кончался только в проколотых точках или вершинах квадрата. Оказалось, что квадрат разрезан на треугольники, внутри кото- которых проколов нет. Сколько было сделано разрезов и сколько получилось треугольников? XI класс 15. Все коэффициенты многочлена равны единице, нулю или минус единице. Доказать, что все действительные корни данного многочлена, если они существуют, заключены в отрезке [ — 2; 2]. 16. На плоскости даны три точки. Построить три окружности, касающиеся друг друга в этих точках. Разобрать все возмож- возможные случаи. 17. В квадратном уравнении х2 -f-px -f- q = 0 коэффициенты р, q независимо пробегают все значения от — 1 до +1 вклю- включительно. Найти множество значений, которые при этом могут принимать действительные корни данного уравнения. 18. Даны окружность О, точка Л, лежащая на ней, пер- перпендикуляр к плоскости окружности О, восставленный из точ- точки А, и точка В, лежащая на этом перпендикуляре. Найти геометрическое место (множество) оснований перпендикуляров, опущенных из точки А на прямые, проходящие через точку В и произвольную точку окружности О. 19*. Даны двадцать карточек. Каждая из цифр от нуля до девяти включительно написана на двух из этих карточек. Можно ли расположить эти карточки в ряд так, чтобы нули стояли рядом, между единицами лежала ровно одна карточка, между двойками — две и т. д. до девяток, между которыми должно лежать девять карточек? 91
2-й т у р VII/ класс 20*. Дана последовательность ..., а_„ а~\, а0, Qi, ..., а„, ..., бесконечная в обе стороны, причем каждый ее член равен — суммы двух соседних. Доказать, что если какие-то два ее члена (не обязательно соседние) равны, то в ней есть беско- бесконечное число пар равных между собой чисел. 21. Дан прямоугольный бильярд размером 26 X 1965. Из нижней левой лузы под углом 45° к бортам выпускается шар. Доказать, что после нескольких отражений от бортов он упадет в верхнюю левую лузу. 22. Два неравных картонных диска разделены на 1965 равных секторов. На каждом диске произвольно выбраны 200 секторов и раскрашены в красный цвет. Меньший диск наложен на боль- больший так, что их центры совпадают, а секторы целиком лежат один против другого. Меньший диск поворачивают на всевоз- всевозможные углы, кратные —?=¦ -й части окружности, оставляя боль- ший диск неподвижным. Доказать, что по крайней мере при 60 положениях на дисках совпадут не более 20 красных секторов. 23*. Посередине между двумя параллельными улицами стоят в один ряд одинаковые дома-квадраты со стороной, равной а (см. рис. 4). Расстояние между улицами — За, а расстояние меж- между двумя соседними домами — 2а. Одна улица патрулируется полицейскими, которые движутся на расстоянии 9а друг от друга со скоростью v. К тому времени, как первый полицейский про- проходит мимо середины некоторого дома, точно напротив него на другой улице появляется гангстер. С какой постоянной скоростью и в какую сторону должен двигаться по этой улице гангстер, чтобы ни один полицейский его не заметил? IX класс 24. См. задачу 29. 25. Дан бильярд прямоугольной формы В его углах имеют- имеются лузы, попадая в которые шарик останавливается. Шарик вы- выпускают из одного угла бильярда под углом 45е к стороне. В ка- какой-то момент он попал в сере- середину некоторой стороны. Дока- Доказать, что в середине противопо- противоположной стороны он побывать не мог. 26. См. задачу 20. у 27. См. задачу 30. Б Рис. 4. 28. Найти геометрическое мес- 92
то (множество) центров равносторонних треугольников, опи- описанных около данного произвольного треугольника. X к л а с с 29*. Имеются 11 мешков с монетами и весы с двумя чашка- чашками и стрелкой, которые показывают, на какой чашке груз тяже- тяжелее и на сколько именно. Известно, что в одном мешке все монеты фальшивые, а в остальных — все монеты настоящие. Все настоя- настоящие монеты имеют одинаковую массу, а все фальшивые — тоже одинаковую, но другую. За какое наименьшее число взвешива- взвешиваний можно определить, в каком мешке фальшивые монеты? 30. На лист клетчатой бумаги размером п X п клеток кла- кладутся черные и белые кубики, причем каждый кубик занимает ровно одну клетку. Первый слой кубиков положили произвольно, а затем вспомнили, что каждый черный кубик должен граничить с четным числом белых, а каждый белый — с нечетным числом черных. Кубики во второй слой положили так, чтобы для всех кубиков первого слоя выполнялось это условие. Если для всех кубиков второго слоя это условие уже выполняется, то больше кубиков не кладут, если же нет, то кладут третий слой так, чтобы для всех кубиков второго слоя выполнялось это условие, и т. д. Существует ли такое расположение ку- кубиков первого слоя, что этот процесс никогда не кончится? 31. В прямоугольном бильярде размером р X 2q, где р и q — целые нечетные числа, сделаны лузы в каждом углу и в се- середине каждой стороны длиной 2q. Из угла выпущен шарик под углом 45° к стороне. Доказать, что шарик обязательно по- попадет в одну из средних луз. (Ср. с задачей 25.) 32. Все целые числа от 1 до 2и выписаны в строчку. Затем к каждому числу прибавили номер того места, на котором оно стоит. Доказать, что среди полученных сумм найдутся хотя бы две, дающие при делении на In одинаковый остаток. 33*. В ящике лежат два ящика поменьше, в каждом из них еще по два ящика и т. д. п раз. В каждом из 2" маленьких ящиков лежит по монете, причем одни — вверх гербом, а осталь- остальные — вверх решкой. За один ход разрешается переворачивать один любой ящик вместе со всем, что в нем лежит. Доказать, что не больше чем за п ходов можно расположить ящики так, что число монет, лежащих вверх гербом, будет равно числу монет, лежащих вверх решкой. XI класс 34. Найти все простые числа вида рр+1 (р— натуральное), содержащие не более девятнадцати цифр. 35. Доказать, что последние цифры чисел п" (п — нату- натуральное) образуют периодическую последовательность. 93
36*. Даны плоскость Р и две точки А и В но разные стороны от нее. Построить сферу, проходящую через эти точки и высе- высекающую из Р наименьший круг. 37. Дан многоугольник на плоскости, невыпуклый и несамо- пересекающийся. D — множество точек, принадлежащих тем диагоналям многоугольника, которые не выходят за его пре- пределы (т. е. ни одна точка диагонали не лежит вне многоуголь- многоугольника). Доказать, что любые две точки из D можно соединить ломаной, целиком принадлежащей D. 38. В каждой клетке квадратной таблицы т X т клеток стоит либо натуральное число, либо пуль. При этом если на пересечении строки и столбца стоит нуль, то сумма чисел в этой строке и в этом столбце не меньше т. Доказать, что сумма всех чисел в таблице не меньше чем '^- XXIX ОЛИМПИАДА A966 г.) 1-й тур VIII класс 1. Найти геометрическое место (множество) центров вписан- вписанных в треугольник ABC прямоугольников (одна сторона пря- прямоугольника лежит на АВ). 2. Найти все двузначные числа такие, что при умножении на некоторое целое число получается число, предпоследняя цифра которого равна пяти. 3. См. задачу 6. 4. См. задачу 10. 5. Из 28 костей домино убрали все кости с шестерками. Мож- Можно ли остальные кости выложить в цепь? IX—А"/ классы 6. Решить в целых положительных числах систему + t=xy. 7. При каком значении k величина Ак= ~ ,— максимальна? 8. Внутри окружности расположен выпуклый пятиугольник (его вершины могут лежать как внутри, так и на окружности). Доказать, что хотя бы одна из его сторон не больше стороны правильного пятиугольника, вписанного в эту окружность. 9. Доказать, что те натуральные k, для которых число k''-\-\ делится на 30, образуют арифметическую прогрессию. Найти ее. 10. Ка"кое максимальное число дамок можно расставить на черных полях шахматной доски размером 8X8 так, чтобы каж- каждую дамку била хотя бы одна из остальных? 94
2-й т у р VIII класс 11. Разделить циркулем и линейкой отрезок на 6 равных частей, проведя не более 8 линий (прямых, окружностей). 12*. Дано: «1 = 1, ак=\л/си-\-а?-{-... +а^_|] при k>\. Най- Найти «ЮОП. 13*. Из 19 шаров 2 радиоактивны. В условиях задачи 18 обнаружить оба радиоактивных шара за 8 проверок. 14. Сеть метро имеет на каждой линии не менее четырех станций, из них не более трех пересадочных. Ни на какой пере- пересадочной станции не пересекается более двух линий. Какое наи- наибольшее число линий может иметь такая сеть, если с любой стан- станции на любую можно попасть, сделав не больше двух пересадок? 15*. Доказать, что существует такое k, что число k\ начи- начинается с цифр 1966. IX—XI классы 16. См. задачу 11. 17. См. задачу 12, где ai = 1966 и требуется найти а|96б- 18*. Из 11 шаров два радиоактивны. Про любой набор шаров за одну проверку можно узнать, имеется ли в нем хотя бы один радиоактивный шар (но нельзя узнать, сколько их). Доказать, что менее чем за 7 проверок нельзя гарантировать нахождение обоих радиоактивных шаров, а за 7 проверок их всегда можно обнаружить. 19*. Из набора гирь массой I, 2, ..., 26 выбрать 6 гирь так, чтобы из них нельзя было составить два набора равных масс. Доказать, что нельзя выбрать 7 гирь, обладающих тем же свойством. 20. На клетчатой доске 11X11 отмечено 22 клетки так, что па каждой вертикали и на каждой горизонтали отмечено ровно 2 клетки Два расположения отмеченных клеток эквива- эквивалентны, если, меняя любое число раз вертикали между собой и горизонтали между собой, мы из одного расположения можем получить другое. Сколько существует неэквивалентных распо- расположений отмеченных клеток? XXX ОЛИМПИАДА A967 г.) 1-й тур VIII класс » I. Существуют ли два последовательных натуральных числа таких, что сумма цифр каждого из них делится на 125? Найти наименьшую пару таких чисел или доказать, что их не сущест- существует 95
2. Дан треугольник ABC. Найти на прямой АВ точку М та- такую, чтобы сумма радиусов окружностей, описанных около тре- треугольников АСМ и ВСМ, была минимальна. 3. Для зашифровки телеграфных сообщений требуется раз- разбить всевозможные десятичные «слова» — наборы из десяти то- точек и тире — на две группы так, чтобы любые два слова одной группы отличались не менее чем в трех разрядах. Указать способ такого разбиения или доказать, что его не существует. 4. Дан треугольник ABC. Найти геометрическое место (множество) точек М таких, что треугольники АВМ и ВСМ — равнобедренные. 5. Остап Бендер организовал в городе Фуксе раздачу слонов населению. На раздачу явилось 28 членов шахматного клуба и 37 не членов, причем Остап раздавал слонов поровну всем членам клуба и поровну не членам. Оказалось, что существует лишь один способ такой раздачи (так, чтобы раздать всех слонов). Какое наибольшее число слонов могло быть у О. Бендера? IX класс 6. Имеется лабиринт, состоящий из п окружностей, касаю- касающихся прямой / в точке М. Все окружности расположены по одну сторону от прямой, а их длины составляют геометри- геометрическую прогрессию со знаменателем 2. Два пешехода в разное время начали ходить по этому лабиринту. Их скорости одинаковы, а направления движения различны. Каждый из них проходит все окружности по порядку и, пройдя наибольшую, снова идет в меньшую. Доказать, что пешеходы встретятся. 7. Можно ли разрезать квадратный пирог на 9 равновеликих частей таким образом:"выбрать внутри квадрата две точки и соединить каждую из них прямолинейными разрезами со всеми четырьмя вершинами квадрата? Если можно, то какие две точки следует выбрать? 8. См. задачу 2. 9. Чему равна максимальная разность между соседними чи- числами среди тех, сумма цифр которых делится на 7? 10. Имеется 120-значное число. Его первые 12 цифр перестав- переставляются всеми возможными способами. Из полученных таким образом 120-значных чисел наугад выбирают 120 чисел. Дока- Доказать, что их сумма делится на 120. X класс 11. В квадрате расположено k точек (&>2). На какое наименьшее число треугольников нужно разбить квадрат, чтобы в каждом треугольнике находилось не более одной точки? 12. Доказать, что в круге радиуса 1 нельзя найти более 5 точек, все попарные расстояния между которыми больше 1. 96
13. Доказать, что уравнение 19л—17yJ = 50 не имеет реше- решений в целых числах. 14. В бесконечно большой каравай, занимающий все прост- пространство, в точках с целыми координатами впечены изюминки диаметра 0,1. Каравай разрезали на части несколькими плоско- плоскостями. Доказать, что найдется неразрезанная изюминка. 15. Из первых k простых чисел 2, 3, 5, ..., pk(k>4) состав- составлены всевозможные произведения, в которые каждое из чисел входит не более одного раза (например, 3-5, 3-5-...-р*, 11-13, 7 и т. д.). Обозначим сумму таких чисел через 5. Доказать, что 5 + 1 разлагается в произведение более 2k простых сомножи- сомножителей. 2-й т у р VII класс 16. В треугольнике ABC проведены высоты АЕ, ВМ и СР. Известно, что ЕМ\\АВ, ЕР\\АС. Доказать, что МР\\ВС. 17. Над квадратным катком нужно повесить четыре лампы так, чтобы они его полностью освещали. На какой наименьшей высоте нужно повесить лампы, если каждая лампа освещает круг радиуса, равного высоте, на которой она висит? 18*. Доказать, что существует целое число q такое, что в десятичной записи числа ^•21000 нет ни одного нуля. 19. Число у получается из натурального х некоторой пере- перестановкой его цифр. Доказать, что, каково бы ни было х, х + уф999...99 A967 девяток). 20. В четырех заданных точках на плоскости расположены точечные прожекторы, каждый из которых может освещать пря- прямой угол. Стороны этих углов могут быть направлены на север, юг, запад или восток. Доказать, что эти прожекторы можно направить так, что они осветят всю плоскость. VIII класс 21. См. задачу 18 для числа <7-21967. 22. Обозначим через d (N) число делителей N. Найти все N такие, что -!—-=р — целое простое число (числа 1 и N также считаются делителями). 23. На каждой стороне прямоугольного треугольника пост- построено по квадрату («Пифагоровы штаны»), и около полученной фигуры описана окружность. Для каких прямоугольных треуголь- треугольников это можно сделать? 24. На шахматной доске размером 1000 X 1000 находятся черный король и 499 белых ладей. Черные и белые фигуры ходят по очереди. Доказать, что, как бы ни ходили ладьи, король всегда может за несколько ходов встать под бой одной из них. 4 Заказ 247 97
25. Семь школьников решили за воскресенье обойти семь ки- кинотеатров, в каждом из которых сеансы начинаются в 9.00, 10.40, 12.20, 14.00, 15.40, 17.20, 19.00 и 20.40 (8 сеансов). На каждый сеанс шестеро шли вместе, а кто-нибудь один (не обязательно один и тот же) шел в другой кинотеатр. К вечеру каждый побы- побывал в каждом кинотеатре. Доказать, что в каждом кинотеатре был сеанс, на котором не был ни один из этих школьников. IX класс 26. Число у получается из натурального числа х некоторой перестановкой его цифр. Известно, что х-\-у= 100...00 B00' ну- нулей). Доказать, что х делится на 50. (Ср. с задачей 19.) 27. Дана последовательность целых положительных чисел Х[, Х2, ..., хп, все элементы которой не превосходят некоторого числа М. Известно, что jc*. = | агл _ i—Хк--Л при всех k>2. Какой может быть максимальная длина этой последователь- последовательности? 28. На каждой стороне прямоугольного А ЛВС построено по квадрату во внешнюю сторону («Пифагоровы штаны»). Оказа- Оказалось, что внешние вершины квадратов лежат на одной окружно- окружности. Доказать, что треугольник ABC равнобедренный. (Ср. с задачей 23.) 29*. Задано натуральное число К такое, что для любого натурального N, делящегося на К, число N тоже делится на К (N — число, состоящее из тех же цифр, что и N, но записан- записанных в обратном порядке, например 1967=7691. 450 = 54). Доказать, что К является делителем числа 99. 30. Испанский король решил перевесить по-своему портреты своих предшественников в круглой башне замка. Однако он хо- хочет, чтобы за один раз меняли местами только два портрета, висящие рядом, причем это не должны быть портреты двух коро- королей, один из которых царствовал сразу после другого. Кроме того, ему важно лишь взаимное расположение портретов (два расположения, отличающиеся поворотом круга, он считает оди- одинаковыми). Доказать, что, как бы сначала ни висели портреты, король может по этим правилам добиться любого нового их расположения. .Y класс 31*. Дана таблица fiXfi клеток. Таблица заполняется сле- следующим образом: в первую строчку записываются числа 1, 2, ..., п, и если в некоторой строчке записаны числа а\, ..., а*, ак+\, ат, ат + \, .., а„{т и п — k взаимно просты), то в сле- 98
дующей строчке записываются те же числа, но в таком порядке: ат + [, ..., п„, с/я-i, ..., ат, Ci, .., о*. Доказать, что после заполне- заполнения таблицы в каждом столбце будут написаны все числа от 1 до п. 32. См. задачу 28. 33. Можно ли расставить на окружности числа 1, 2, ..., 12 так, чтобы разность между двумя рядом стоящими числами была 3, 4 или 5? 34. В восьми данных точках пространства установлено по точечному прожектору, каждый из которых может осветить в про- пространстве октант (трехгранный угол со взаимно перпендикуляр- перпендикулярными ребрами). Доказать, что можно повернуть прожекторы так, чтобы они осветили все пространство. (Ср. с задачей 20.) 35*. Рассматриваются всевозможные n-значные числа, со- составленные из цифр 1, 2 и 3. В конце каждого из этих чисел при- приписывается цифра 1, 2 или 3 так, что к двум числам, у которых во всех разрядах стоят разные цифры, приписываются разные цифры. Доказать, что найдется п-значное число, в записи ко- которого участвует лишь одна единица и к которому приписывается единица. XXXI ОЛИМПИАДА A968 г.) 1-й тур VII класс 1. Число 4 обладает тем свойством, что при делении его па 2 q2 получается остаток меньше — , каково бы ни было q. Пере- Перечислить все числа, обладающие этим свойством. 2. Расставить в таблице 4X4 шестнадцать чисел так, чтобы сумма чисел по любой вертикали, горизонтали и диагонали рав- равнялась нулю. Примечание. Таблица имеет 14 диагоналей, включая все малые, состоящие из трех, двух и одной клеток. 3. Доказать, что для любых трех чисел, меньших 1 000 000, найдется число, меньшее 100, взаимно простое с каждым из них. 4. Как соединить 50 городов наименьшим числом авиалиний так, чтобы из любого города можно было попасть в любой дру- другой, сделав не более двух пересадок? VIII класс 5. В шахматном турнире участвовало 12 человек. После окончания турнира каждый участник составил 12 списков. В пер- первый список входит только он сам, во второй — он и те, у кого он выиграл, втретий — все люди из второго списка и те, у кого они выиграли, и т. д. В двенадцатый список вхо- входят все люди из одиннадцатого списка и те, у кого они выиграли. 4* 99
Известно, что для любого участника турнира в его двенадцатый список попал человек, которого не было в его одиннадцатом списке. Сколько ничейных партий было сыграно в турнире? 6. Даны числа 4, 14, 24, ..., 94, 104. Доказать, что из них нельзя вычеркнуть сначала одно число, затем из оставшихся еще два, затем еще три и, наконец, еще четыре числа так, чтобы после каждого вычеркивания сумма оставшихся чисел делилась на 11. 7. Можно ли вписать в окружность выпуклый семиугольник Л1Л2Л3Л4Л5Л6Л7 с углами AAt = 140°, Z.A2 = l20°, ^Л3=130°, A ° Л Л 8. Выбрать 100 чисел, удовлетворяющих условиям <;2л:99, так, чтобы выражение Х\—х2-\-хз— х4 + ...+л:99 — xiOo было максимально. 9. Можно ли расположить на плоскости 1000 отрезков так, чтобы каждый отрезок своими концами упирался строго внутрь других отрезков? IX класс 10. Существует ли четырехугольник ABCD площади 1 такой, что для любой точки О внутри его площадь хотя бы одного из треугольников ОАВ, ОВС, OCD, OAD есть иррациональное число? 11. См. задачу 9 для 1968 отрезков. 12. В коридоре длиной 100 м постелено 20 ковровых дорожек общей длиной 1000 м. Каково может быть наибольшее число не- застеленных кусков? (Ширина дорожки равна ширине коридора.) 13. Можно ли выбрать 100 000 номеров телефонов из 6 цифр каждый так, чтобы при одновременном вычеркивании из всех этих номеров fc-й цифры (ft = l, 2, 3, 4, 5, 6) получились все пятизначные номера от 00 000 до 99 999? 14. Доказать, что если р и q — два простых числа, причем <7 = р + 2, то p4-\-qp делится на p-\-q. X класс 15*. Из пункта Л одновременно вылетают 100 самолетов (флагманский и 99 дополнительных). С полным баком горючего самолет может пролететь 1000 км. В полете самолеты могут пере- передавать друг другу горючее. Самолет, полностью отдавший го- горючее другим, совершает планирующую посадку. Каким обра- образом надо совершать перелет, чтобы флагман пролетел возможно дальше? 16. Двое играют в следующую игру: имеется две кучки кон- конфет Играющие делают ход по очереди. Ход состоит в том, что играющий съедает одну из кучек, а другую делит на две (равные 100
или неравные) части. Если он не может разделить кучку, так как в ней всего одна конфета, то он ее съедает и выигрывает. В на- начале в кучках было 33 и 35 конфет. Кто выигрывает, начи- начинающий или его партнер, и как для этого надо играть? 17. Можно ли разбить множество всех целых неотрицатель- неотрицательных чисел на 1968 классов так, чтобы в каждом классе было хотя бы одно число и выполнялось бы следующее условие: если число т получается из числа п вычеркиванием двух рядом стоящих одинаковых цифр или одинаковых групп цифр, то т и п принадлежат одному классу (например, числа 7, 9339337, 93223393447, 932239447 обязательно принадлежат одному классу)? 18. По заданной последовательности положительных чисел q\, <?2, -¦-, qn, -¦¦ строится последовательность многочленов следующим образом: fo{x)=l, fi(x) = x, fn + l (x) = (l +qn) xfn(x) — — qnfn-[(x) для n^l. Доказать, чти все действительные корни п-го многочлена заключены между —1 и -f-1. 19. В пространстве расположены 4 попарно скрещивающиеся прямые /|, /2, /з, /4, причем никакие три из них не параллельны одной плоскости. Провести плоскость Р так, чтобы точки А\, An, А3, A4 пересечения этих прямых с Р являлись верши- вершинами параллелограмма. Сколько решений имеет задача? 2-й т у р VII класс 20. На плоскости отмечено 1968 точек, являющихся вершина- вершинами правильного 1968-угольника. Двое играющих по очереди соединяют две вершины многоугольника отрезком, соблюдая следующие правила: нельзя соединять две точки, хотя бы одна из которых уже соединена с другой, и нельзя пересекать уже проведенные отрезки. Проигрывает тот, кто не может сделать очередного хода согласно этим правилам. Как нужно играть, чтобы выиграть? Кто выиграет при правильной игре? 21. На плоскости даны три точки. Из них выбираются любые две, строится серединный перпендикуляр к отрезку, их соединяю- соединяющему, и все точки отражаются симметрично относительно этой прямой; затем из всех точек (старых и новых) снова выбирают какие-то две точки и весь процесс повторяют. Так делается беско- бесконечно много раз. Доказать, что в плоскости найдется такая пря- прямая, что все полученные точки будут лежать по одну сторону от нее. 22. Два маляра красят забор, огораживающий 100 дачных участков. Они приходят через день и красят по одному участку в красный или зеленый цвет. Первый маляр дальтоник и путает цвета: он помнит, что и в какой цвет он сам покрасил, и видит что покрасил второй маляр, но не знает, в какой цвет. Первый маляр добивается того, чтобы в наибольшем числе мест зеленый 101
участок граничил с красным. Какого наибольшего числа перехо- переходов может добиться первый маляр (как бы ни действовал вто- второй маляр)? 23. Двухсотзначное число 89252525 ... 2525 умножено на число 444*18г/27 (х и у— неизвестные цифры). Оказалось, что 53-я цифра полученного числа (считая справа) есть 1, а 54-я — 0. Найти хну. 24*. Ковбой Джимми поспорил с друзьями, что сумеет од- одним выстрелом пробить все четыре лопасти вертилятора. (Верти- лятор устроен следующим образом: на оси, вращающейся со скоростью 50 об/с, расположены на равных расстояниях друг от друга четыре полудиска, повернутые друг' относительно друга под какими-то углами и перпендикулярные оси.) Джимми может стрелять в любой момент и добиваться произвольной скорости пуль. Доказать, что Джимми выиграет пари. VIII класс 25. Разобьем все натуральные числа на группы так, чтобы в первой группе было одно число, во второй — два, в третьей — три и т. д. Можно ли это сделать таким образом, чтобы из суммы чисел в каждой группе нацело извлекался корень седьмой сте- степени? 26*. Две прямые на плоскости пересекаются под углом а. На одной из них сидит кузнечик. Каждую секунду он прыгает с одной прямой на другую (точка пересечения считается при- принадлежащей обеим прямым). Известно, что длина каждого его прыжка равна 1 и что он никогда не возвращается на то место, где был секунду назад. Через некоторое время кузнечик вернулся в первоначальную точку. Доказать, что угол а измеряется ра- рациональным числом градусов. 27*. Круглый пирог режут таким образом. Вырезают сектор с углом а, переворачивают его на другую сторону, а затем весь пирог поворачивают на угол р. Дано, что р<а<180°. Дока- Доказать, что после некоторого конечного числа таких операций каждая точка пирога будет находиться на своем первоначаль- первоначальном месте. 28. На бумажной ленте напечатаны автобусные билеты с но- номерами от 000 000 до 999 999. Затем синей краской отмечены те билеты, у которых сумма цифр, стоящих на четных местах, равна сумме цифр, стоящих на нечетных местах. Какая наи- наибольшая разность может оказаться между номерами двух сосед- соседних синих билетов? 29. Страна фарра расположена на 1 000 000 000 островов. Между некоторыми островами каждый день курсируют пароходы. Маршруты пароходов устроены так, что с каждого острова можно попасть на любой другой (возможно, за несколько дней). Вор и полицейский могут совершать не более одного рейса в день 102
на пароходе и не имеют никакой другой возможности попасть с острова на остров. Вор не ездит на пароходе 13-го числа каж- каждого месяца, а полицейский не суеверен и всегда знает, где на- находится вор. Доказать, что полицейский сможет поймать вора (т. е. оказаться с ним на одном острове). IX класс 30. На плоскости нарисован правильный пятиугольник АхАъАгА^Аь. Можно ли выбрать на плоскости множество точек, обладающее следующим свойством: через любую точку, не ле- лежащую внутри пятиугольника, можно провести отрезок, концы которого являются точками нашего множества, а через точки, лежащие внутри пятиугольника, такого отрезка провести нельзя? 31. На окружности радиуса 1 отмечена точка О, и из нее циркулем делается на окружности засечка по часовой стрелке радиусом /. Из полученной точки О\ в том же направлении тем же радиусом делается вторая засечка, и так повторяется 1968 раз. После этого окружность разрезается во всех 1968 за- засечках и получается 1968 дуг. Сколько различных длин дуг мо- может при этом получиться? 32. Белые и черные играют в следующую игру. В углах шах- шахматной доски стоят два короля: белый на ol, черный на Ш. Играющие делают ходы по очереди (начинают белые). Играющий может ставить своего короля на любое соседнее поле (если толь- только оно свободно), соблюдая следующие правила: нельзя уве- увеличить расстояние между королями (расстоянием между двумя королями называется наименьшее число ходов короля, за которое он может пройти с одного поля на другое: так, в начале игры расстояние между королями — 7 ходов). Выигрывает тот, кто поставит своего короля на противопо- противоположную кромку доски (белого короля на вертикаль h или вось- восьмую горизонталь, черного — на вертикаль а или первую гори- горизонталь). Как нужно играть, чтобы выиграть? Кто выигрывает при правильной игре? 33*. Известно, что а" — Ь" делится на п (a, b, n — натураль- натуральные числа, аФЬ). Доказать, что ~ — делится на п. а — о 34*. Дано натуральное число N. С ним производится сле- следующая операция: каждая цифра этого числа заносится на отдельную карточку (при этом разрешается добавлять или выбра- выбрасывать любое число карточек, на которых написана цифра 0). Затем эти карточки разбивают на две группы. В каждой из них карточки располагаются в произвольном порядке и получен- полученные два числа складываются. С полученным числом N\ проде- лывается такая же операция и т. д. Докажите, что за 15 шагов из N можно получить однозначное число. юз
X класс 35. Внутри выпуклого многоугольника М помещена окруж- окружность максимально возможного радиуса R (это значит, что внут- внутри М нельзя поместить окружность большего радиуса). Известно, что, двигая единичный отрезок (как твердый стержень) внутри многоугольника М, его можно повернуть на любой угол. До- Доказать, что R^—. 36. В таблице А размером 10X10 написаны какие-то числа. Обозначим сумму всех чисел в первой строке через s\, во второй — Si и т. д. Аналогично сумму чисел в первом столбце обозначим через t\, во втором — B и т. д. Составлена новая таблица В размером 10X10, в нее вписаны числа следующим образом: в первой клетке первой строки пишется наименьшее из чисел s\ и t\, в третьей клетке пятой строки пишется наименьшее из чисел S5 и /з, аналогично заполнена вся таблица. Оказалось, что можно так занумеровать клетки таблицы В числами от 1 до 100, что в клетке с k-м номером будет стоять число, меньшее или равное k. Какое максимальное значение может принимать при этих усло- условиях сумма всех чисел таблицы А? 37. Дана система уравнений = 0, V-19 I vl9_l_ I V19 П х\ -\-х2 -\-...-\-Xk =U, v21 _1_ v21 L I v21 1 V. Х\ -\-Х2 -\-...-\-Xk =1 (все Xt — действительные числа). Доказать, что при некоторых k такая система имеет решение. 38*. Правильный треугольник ABC разбит на N выпуклых многоугольников так, что каждая прямая пересекает не более 40 из них (мы говорим, что прямая пересекает многоугольник, если они имеют общую точку, в том числе если прямая про- проходит через вершину многоугольника). Может ли N быть больше миллиона? 39. На поверхности кубика мелом отмечено 100 различных точек. Доказать, что можно двумя различными способами поста- поставить кубик на черный стол (причем в точности на одно и то же место) так, чтобы отпечатки от мела на столе при этих спо- способах были разными. (Если точка отмечена на ребре или вер- вершине, она тоже дает отпечаток.) 104
XXXII ОЛИМПИАДА A969 г.) 1-й тур VII класс 1. Белая ладья преследует черного слона на доске 3X1969 клеток (они ходят по очереди по обычным правилам). Как должна играть ладья, чтобы взять слона? Первый ход делают белые. 2. Старинный замок был обнесен треугольной стеной. Каждая сторона стены была поделена на три равные части, и в этих точ- точках, а также в вершинах были построены башни. Всего вдоль стены было 9 башен: А, Е, F, В, К, ?, С, М, N. Со временем все стены и башни, кроме башен Е, К, М, разрушились. Как по оставшимся башням определить, где находились башни А, В, С, если известно, что башни А, В, С стояли в вершинах? 3. В Чили в феврале проходил международный турнир по футболу. Первое место с 8-ью очками занял местный клуб «Ко- «Коло-Коло». На очко отстало московское «Динамо» и заняло второе место. 3-е место с 4-мя очками занял бразильский клуб «Корин- тианс». 4-е место занял югославский клуб «Црвёна Звезда», также набравший 4 очка. Доказать, что по этим данным можно точно определить, сколько еще команд участвовало в турнире и по сколько очков они набрали. 4. Доказать, что никакая степень числа 2 не оканчивается четырьмя одинаковыми цифрами. 5. Имеется 1000 деревянных правильных 100-угольников, при- прибитых к полу. Всю эту систему мы обтягиваем веревкой. Натя- Натянутая веревка будет ограничивать некоторый многоугольник. До- Доказать, что у него более 99 вершин. VIII класс 6. См. задачу 4. 7. См. задачу 5 с заменой чисел 1000, 100, 99 на числа 57, 57, 56 соответственно. 8. См. задачу 1, заменив слона конем. 9. Дан отрезок АВ. Найти геометрическое место (множест- (множество) точек С таких, что в треугольнике ABC медиана, проведенная из вершины В, равна высоте, проведенной из вершины А. 10. Можно ли записать в строку 20 чисел так, чтобы сумма любых трех последовательных чисел была положительна, а сумма всех 20 чисел была отрицательна? IX класс 11. Найти все натуральные числа х, обладающие следую- следующим свойством: из каждой цифры числа х можно вычесть одну и ту же цифру афО (т. е. все цифры его не меньше а) и при этом получится (х — аJ. 105
12. Остров Толпыго имеет форму многоугольника. На нем расположено несколько стран, каждая из которых имеет форму треугольника, причем каждые две граничащие страны имеют целую общую сторону (т. е. вершина одного треугольника не лежит на стороне другого). Доказать, что карту этого острова можно так раскрасить тремя красками, чтобы каждая страна была закрашена одним цветом и любые две граничащие страны были закрашены в разные цвета. 13. Можно ли записать в строку 50 чисел так, чтобы сумма любых 17 последовательных чисел была положительна, а сумма любых 10 последовательных чисел была отрицательна? (Ср. с за- задачей 10.) 14. См. задачу 4. 15. В государстве царя Додона расположено 500 городов, каждый из которых имеет форму правильной 37-угольной звезды, в вершинах которой находятся башни. Додон решил обнести их выпуклой стеной так, чтобы каждый отрезок стены соединял две башни. Доказать, что стена будет состоять не менее чем из 37 отрезков. (Если несколько отрезков лежат на одной пря- прямой, то они считаются за один. Ср. с задачей 5.) X класс 16. На выходе из бетатрона установлены два одинаковых тон- тонких обруча, расположенные во взаимно перпендикулярных пло- плоскостях, причем каждый обруч проходит через центр другого. Частицы, вылетающие из бетатрона, пролетают по прямой сквозь оба обруча. По какой прямой они должны двигаться, чтобы находиться на наибольшем расстоянии от обручей, т. е. чтобы наименьшее расстояние, на которое частицы приближаются к обручу, было как можно больше? 17. Дана бесконечная последовательность чисел О|, ...", о„ Она периодична с периодом 100, т. е. O|=O|0i, 02 = 0102, ... ¦ Известно, что Oi^O, Oi + o2^0, Oi+02 + Оз^О и, вообще, сум- сумма О|+о2 + ... + ол неотрицательна при нечетном п и неполо- неположительна при четном п. Доказать, что I099I ^ |оюо1- 18. Колода перфокарт четырех цветов разложена в один ряд. Если две перфокарты одного цвета лежат рядом или через одну, то можно выбрасывать ту из них, которая левее. Кроме того, можно подкладывать справа любое количество перфокарт из дру- других колод. Доказать, что можно подкладывать и выбрасывать перфокарты таким образом, чтобы в конце концов их осталось только четыре. 19. Существует ли такое число h, что ни для какого на- натурального п число [h- 1969я] не делится на [h- 1969"~'j? 20. Дан квадрат ABCD. Найти геометрическое место (мно- (множество) таких точек М, что выполняется равенство /LAMB — = Z. CMD. 106
2-й т у р VII класс 21. Даны два целых положительных числа тип. Выписы- Выписываются все различные делители числа т. — числа а, Ь,..., k — и все различные делители числа п — числа s, t, ..., z. (Само число и 1 тоже включаются в число делителей.) Оказалось, что Доказать, что т = п. 22. С числом 123456789101112...9989991000 производится сле- следующая операция: зачеркиваются две соседние цифры а и Ь (а стоит перед Ь) и на их место вставляется число а-\-2Ь (можно в качестве а взять нуль, «стоящий» перед числом, а в качестве Ь — первую цифру числа). С полученным числом про- производится такая же операция и т. д. (Например, из числа 118 307 можно на первом шаге получить числа 218 307, 38 307, 117 307, 111 407, 11 837, 118 314.) Доказать, что таким способом можно получить число 1. 23. Некий фермер приобрел квадратный участок земли, обнес его забором и получил у доверчивого арендатора документ, в котором сказано, что он имеет право несколько раз произвести следующую операцию: провести прямую через любые две точки забора, огораживающего его участок, снести участок забора меж- между этими двумя точками по одну сторону от прямой и достроить такой же кусок забора с другой стороны симметрично снесенной части относительно выбранной прямой. Сможет ли он такими операциями увеличить площадь своего участка? 24. Двое играют в следующую игру. Каждый игрок по оче- очереди вычеркивает 9 чисел (по своему выбору) из последователь- последовательности 1, 2, 3, ..., 100, 101. После одиннадцати таких вычерки- вычеркиваний останутся два числа. Затем второй игрок присуждает первому столько очков, какова разница между этими оставшимися числами. Доказать, что первый игрок всегда сможет набрать по крайней мере 55 очков, как бы ни играл второй. 25*. В круглый пудинг радиуса 10 см запечена жемчужина радиуса 3 мм. Мы хотим ее найти. Для этого разрешается разрезать пудинг острым ножом по прямой на две (одинаковые или разные) части. Если жемчужина не попадет под нож, можно одну из этих частей снова разрезать; если она снова не будет обнару- обнаружена, можно разрезать одну из трех получившихся частей и т. д. Доказать, что, как бы мы ни резали, может случиться, что после 32 разрезов жемчужина все еще не будет обнаружена. Доказать, что можно так сделать 33 разреза, что жемчужина обязательно будет обнаружена, где бы она ни находилась. 107
VIII класс 26. См. задачу 2. 27. На шахматной доске на поле а\ стоит белый конь. Двое по очереди замазывают по одной клетке шахматной доски бокси- бокситовым клеем. При этом они должны замазывать так, чтобы конь мог пройти в любую иезамазанную клетку, нигде не при- приклеившись (конь ходит по обычным шахматным правилам). Про- Проигрывает тот, кто не может сделать очередного хода. Кто выигры- выигрывает при правильной игре: сделавший первый ход или его партнер? 28*. Имеется два правильных пятиугольника с одной общей вершиной. Вершины каждого пятиугольника нумеруются по ча- часовой стрелке цифрами от 1 до 5, причем в общей вершине ставится цифра 1. Вершины с одинаковыми номерами соединены прямыми. Доказать, что полученные четыре прямые пересекаются в одной точке. 29. Из натуральных чисел составляются последовательности, в которых каждое последующее число больше квадрата предыду- предыдущего, а последнее число в последовательности равно 1969 (по- (последовательности могут иметь разную длину). Доказать, что различных последовательностей такого вида меньше чем 1969. 30*. В ряд поставлено 100 кубиков: 77 черных и 23 белых. Они расставлены приблизительно равномерно, т. е. если в лю- любом месте отметить некоторое количество кубиков подряд и потом в другом месте отметить такое же число кубиков подряд, то число черных кубиков в первом наборе отличается от числа черных кубиков во втором наборе не более чем на 1; при этом, если первый набор стоит на л е в о м краю, то число черных кубиков в нем не больше, чем во втором наборе, а если первый набор стоит на правом краю, то число черных кубиков в нем не меньше, чем во втором наборе. Доказать, что если расставить другой набор из 77 черных и 23 белых кубиков так, чтобы выпол- выполнялись те же условия, то белые кубики будут стоять на тех'же местах. IX класс 31. Двое играют в следующую игру. Каждый игрок по очере- очереди вычеркивает одно число из ряда 1, 2, 3, ..., 27 до тех пор, пока не останется два числа. Если сумма этих чисел делится на 5, то выигрывает первый игрок, если не делится — то второй. Кто выиграет при правильной игре: первый или второй? 32. Имеется пятак, с помощью которого можно чертить окруж- окружности на плоскости (обводить пятак). Разрешается с его помо^- щью проводить окружность через одну или две данные точки (расположенные достаточно близко друг от друга). На плоскости заданы три точки, которые можно закрыть одним пятаком и ко- 108
торые не лежат на одной прямой или на одной окружности, равной окружности пятака. Построить четвертую точку так, чтобы полученные четыре точки лежали в вершинах параллелограмма. Разрешается использовать только один пятак. 33*. Одна под другой выписаны 2"~[ различных последова- последовательностей из нулей и единиц длины п. Известно, что для любых трех из выписанных последовательностей найдется такой номер р, что в р-м разряде у них стоит 1 (у всех трех). Доказать, что в некотором разряде у всех выписанных последовательностей стоит 1 и такой разряд только один. 34*. В стране Анчурии, где правит президент Мирафлорес, приблизилось время новых президентских выборов. В стране ров- ровно 20 миллионов избирателей, из которых только один процент поддерживает Мирафлореса (регулярная армия Анчурии). Ми- Мирафлорес, естественно, хочет быть избранным, но, с другой сторо- стороны, он хочет, чтобы выборы были «демократическими». «Демокра- «Демократическим голосованием» Мирафлорес называет вот что: все изби- избиратели разбиваются на равные группы; каждая из этих групп вновь разбивается на некоторое количество равных групп, при- причем большие группы могут разбиваться на разное количество меньших групп, затем эти группы снова разбиваются и т. д. В са- самых мелких группах выбирают представителя группы — выбор- выборщика — для голосования в большей группе: выборщики в этой большей группе выбирают выборщика для голосования в еще большей группе и т. д. Наконец, представители самых больших групп выбирают президента. Мирафлорес делит избирателей на группы по своей воле и инструктирует своих сторонников, как им голосовать. Сможет ли он так организовать «демократические» выборы, чтобы его выбрали? (В каждой группе выборщики выбирают своего представителя простым большинством. При равенстве голосов побеждает оппозиция.) 35*. Правильный 1000-угольник разбит непересекающимися диагоналями на треугольники. Доказать, что среди этих диагона- диагоналей найдется не менее 8 диагоналей, длины которых попарно различны. X класс 36. Два мудреца играют в следующую игру. Выписаны числа 0, 1, 2, ..., 1024. Первый мудрец зачеркивает 512 чисел (по своему выбору), второй зачеркивает 256 из оставшихся, затем снова первый зачеркивает 128 чисел и т. д. На десятом шаге второй мудрец зачеркивает одно число; остаются два числа. После этого второй мудрец платит первому разницу между этими числами. Как выгоднее играть первому мудрецу? Как второму? Сколько уплатит второй мудрец первому, если оба будут играть наилучшим образом? (Ср. с задачами 24 и 31.) 109
37. Жесткая проволока выгнута в форме правильного тре- треугольника, и края ее запаяны. Разрешается перегибать кусок проволоки, заключенный между любыми двумя ее точками так, чтобы отогнутый кусок был симметричен прежнему относитель- относительно прямой, проходящей через эти точки (если эти точки совпа- совпадают, то прямую через них можно проводить произвольным образом). Так разрешается делать несколько раз. Можно ли не- несколькими такими операциями получить правильный шестиуголь- шестиугольник того же периметра? (Ср. с задачей 23.) 38. См. задачу 25, заменив круг на шар радиуса 20 см и количество разрезов 32 и 33 на 65 и 66 соответственно. 39. На клетках шахматной доски 8X8 написаны числа, сум- сумма которых равна нулю. Затем каждую клетку разбили на 4 ячейки (вертикальными и горизонтальными линиями). Можно ли так вписать числа в ячейки, чтобы одновременно выполнялись условия: а) в ячейках у края доски стояли нули; б) для каждой клетки суммы чисел, стоящих в четырех ячейках этой клетки, равнялись числу, которое раньше было в ней написано; в) для каждого узла шахматной доски (там, где встречаются 4 клетки) сумма чисел, стоящих в примыкающих к нему четырех ячейках, равнялась нулю? 40*. Требуется расставить 1969 кубиков в ряд так, чтобы часть из них — число между 0 и 1969 — были белыми, а осталь- остальные — черными, причем цвета должны распределяться прибли- приблизительно равномерно (см. условие задачи 30). Доказать, что существует по крайней мере 1970 различных способов расстанов- расстановки кубиков в ряд так, чтобы выполнялись эти требования. XXXIII ОЛИМПИАДА {1970 г.) 1-й тур VII класс 1. На бесконечной шахматной доске на двух соседних по диагонали черных полях стоят две черные шашки. Можно ли дополнительно поставить на эту доску некоторое число черных шашек и одну белую таким образом, чтобы белая о д н и м^ х о- д о м взяла все черные шашки, включая две первоначально стоявшие? 2. На 99 карточках пишутся числа 1, 2, 3, .... 99. Затем карточки перемешиваются, раскладываются чистыми сторонами • вверх и на чистых сторонах снова пишутся числа 1, 2, 3, 4, .... 99. Для каждой карточки числа, стоящие на ней, складываются и 99 полученных сумм перемножаются. Доказать, что в результате получится четное число. 3. Внутри правильного треугольника ABC лежит точка О. Известно, что /LAOB = 113°, /LBOC = 123°. Найти углы треуголь- треугольника, стороны которого равны отрезкам ОА, ОВ, ОС. ПО
4. В наборе имеется 100 гирь, каждые две из которых отли- отличаются по массе не более чем на 20 г. Доказать, что эти ги- гири можно положить на две чашки весов, по 50 штук на каж- каждую, так, чтобы одна чашка весов была легче другой не более чем на 20 г. 5. В городе X имеется 1000 коттеджей, в каждом из которых живет по одному человеку. В один прекрасный день каждый че- человек переезжает из своего дома в какой-либо другой (переезд осуществляется так, что после него в каждом доме живет один жилец). Доказать, что после переезда можно так покрасить все 1000 коттеджей синей, зеленой и красной красками, чтобы у каждого хозяина цвет его нового дома отличался от цвета ста- старого дома. VIII класс 6. См. задачу 2. 7. Около окружности описан пятиугольник ABCDE, стороны которого —-целые числа и АВ = CD = 1. Окружность каса- касается стороны ВС в точке К. Найти длину отрезка В/С. 8. На прямоугольном листе бумаги расставлены 16 черных точек. Для каждой пары точек проделывается следующая опе- операция: они соединяются отрезком, и затем прямоугольник, диаго- диагональю которого является этот отрезок, а стороны параллельны сторонам листа, закрашивается красным цветом (имеется в виду, что черные точки просвечивают на красном фоне). Сколько сторон может иметь закрашенная фигура? (Указать все ответы, которые могут получиться при различных расположениях точек.) 9. На каждую чашку весов положили k гирь, занумерован- занумерованных числами от 1 до k, причем левая чашка перевесила. Ока- Оказалось, что если поменять чашками любые две гири с одинаковыми номерами, то всегда либо правая чашка начинает перевешивать, либо чашки приходят в равновесие. При каких k это возможно? 10. 12 теннисистов участвовали в турнире. Известно, что каж- каждые два теннисиста сыграли между собой ровно один раз и не было ни одного теннисиста, проигравшего все встречи. Дока- Доказать, что найдутся теннисисты А, В, С такие, что А выиграл у В, В у С, С у А. (В теннисе ничьих не бывает.) IX класс 11. Вдоль прямой дороги располагались 113 дворцов, в каж- каждом из которых жил король. Ежедневно один из них устраи- устраивал прием, на который утром съезжались все остальные, а вече- вечером слуги развозили их по домам. Так они жили год и никуда больше не выезжали. Доказать, что самый большой путь за этот год проехал один из королей, живущих в крайних дворцах. 12. Какое максимальное количество черных шашек можно рас- расставить на шашечной доске 8X8 так, чтобы простая белая шаш- Ш
ка могла взять их все за один ход, не попадая при этом в дамки? Шашки стоят на черных полях. 13. Дано 999-значное число. Известно, что если взять из него любые 50 последовательных цифр и вычеркнуть все осталь- остальные, то полученное число будет делиться на 2 °. (Оно может на- начинаться с нулей или просто быть нулем.) Доказать, что исходное число делится на 2999. 14. Построить треугольник по радиусу описанной окружности и биссектрисе угла А, если известно, что А В—Z. С = 90°. 15. Мудрый таракан, который видит не дальше, чем на 1 см, решил отыскать Истину. Находится она в точке, расстояние до которой D см. Таракан может делать шаги, каждый длиной не более 1 см, и после каждого шага ему говорят, приблизился он к Истине или нет. Таракан может помнить все, в частности направ- направление своих шагов. Доказать, что он сможет отыскать Истину, сделав не более — D-\-7 шагов. X класс 16. Масса каждой из 19 гирь не больше 70 г и равна целому числу граммов. Доказать, что из этих гирь нельзя составить более 1230 различных по массе наборов. 17. В угол ABC вписаны две непересекающиеся окружности О{ и Ог. М — точка касания О\ с ВА, Р — точка касания Ог с ВС. Доказать, что окружности О\ и Ог высекают на прямой МР хорды равной длины. 18. У числа 21970 зачеркнули его первую цифру и прибавили ее к оставшемуся числу. С результатом проделали ту же опера- операцию и т. д., до тех пор пока не получили 10-значное число. Доказать, что в этом числе есть две одинаковые цифры. 19. На плоскости даны 200 точек, никакие три из которых не лежат на одной прямой. Можно ли занумеровать эти точки номера- номерами от 1 до 200 так, чтобы все сто прямых, проходящих через точки с номерами 1 и 101, 2 и 102, ... , 100 и 200, попарно пересекались? -> 20. В некоторых клетках таблицы 100 X Ю0 стоят крестики. Известно, что в каждой строке стоит хотя бы один крестик и в каждом столбце тоже. Доказать, что можно отметить 10 строк и 10 столбцов так, что если стереть все крестики в отмеченных строках и столбцах, то в каждой неотмеченной строке останется хотя бы один крестик и в каждом столбце тоже. 2-й т у р VII класс 21. Доказать, что если целое положительное число k делится на 999 999 999 (9 девяток), то в его десятичной записи более 8 цифр отличны от нуля. 112
22. На окружности радиуса 1 отмечено 100 точек. Доказать, что на этой окружности можно найти такую точку, чтобы сумма расстояний от нее до всех отмеченных точек была больше 100. 23. В парке шесть узких аллей одинаковой длины, четыре из которых идут по сторонам квадрата и две по его средним линиям. По этим аллеям мальчик Коля убегает от папы и мамы. Смогут ли папа и мама поймать Колю, если он бегает втрое быстрее их (все трое все время видят друг друга)? 24. Квадратный лист бумаги разрезают по прямой на две части. Одну из этих частей снова разрезают по прямой на две части. Один из полученных трех листов бумаги разрезают на две части и т. д. Какое наименьшее число разрезов нужно сделать, чтобы получить 73 тридцатиугольника? 25. Король Людовик не доверяет некоторым своим придвор- придворным. Он составил полный список придворных и приказал каждому из них следить за одним из остальных. При этом первый придворный следит за тем, кто следит за вторым, второй следит за тем, кто следит за третьим, и т. д., предпослед- предпоследний следит за тем, кто следит за последним, последний следит за тем, кто следит за первым. Доказать, что у Людовика нечетное число придворных. VIII класс 26. Внутри круга радиуса 1 м расположены п точек. Дока- Доказать, что в круге или на его границе существует точка, сумма расстояний от которой до всех точек не меньше п метров. (Ср. с задачей 22.) 27. В маленьком зоопарке из клетки убежала обезьяна. Ее ло- ловят два сторожа. И сторожа, и обезьяна бегают только по дорож- дорожкам. Всего в зоопарке 6 прямолинейных дорожек: 3 длинные образуют правильный треугольник, 3 короткие соединяют середи- середины его сторон. В каждый момент времени обезьяна и сторожа ви- видят друг друга. Смогут ли сторожа поймать обезьяну, если обезьяна бегает в три раза быстрее сторожей? (Внача- (Вначале оба сторожа находятся в одной вершине треугольника, а обезьяна в другой. Ср. с задачей 23.) 28. На участке земли квадратно-гнездовым способом посажено 10 000 деревьев: 100 рядов по 100 деревьев. Какое наибольшее число деревьев можно срубить, чтобы выполнялось следующее условие: если встать на любой пень, то за деревьями не будет видно ни одного другого пня? Деревья считать достаточно тонкими. 29. На ленте записаны подряд 80 ненулевых цифр. Разрезаем ленту на несколько кусков, длина каждого из которых больше, чем одна цифра. Числа, получившиеся на кусках, складываем. Доказать, что имеется по крайней мере два различных способа разрезания ленты, при которых получаются одинаковые суммы. из
30. См. задачу 32. Доказать, что если проволока имеет форму ломаной и расстояние между ее концами больше B-f-2y2) м, то по всему коридору ее протащить нельзя. IX класс 31. /V деталей полотна игрушечной железной дороги имеют форму четверти окружности радиуса 10 см. Доказать, что, после- последовательно присоединяя их концами так, чтобы они плавно переходили друг в друга, нельзя составить такой путь, чтобы его начало совпадало с концом, а первое и последнее звенья образовывали бы нулевой угол (рис. 5). 32. Плоский коридор шириной 1 м имеет форму буквы «Г» и бесконечен в обе стороны. Необходимо изготовить плоский ку- кусок несгибаемой проволоки — не обязательно прямой — такой, чтобы его можно было протащить по всему коридору. Каково максимальное возможное расстояние между концами проволоки? 33. Во всех клетках таблицы ЮОхЮО стоят плюсы. Раз- Разрешается одновременно менять знаки во всех клетках одной строки или же во всех клетках одного столбца. Можно ли, пользуясь только этими операциями, получить 1970 минусов? 34. См. задачу 24; требуется получить сто 20-угольников. 35. По ребрам прозрачного куба ползают три паука и муха. Наибольшая скорость мухи в три раза больше наибольшей скорости пауков. Вначале пауки находились в одной вершине куба, муха — в противоположной. Смогут ли пауки поймать муху? (Пауки и муха все время видят друг друга.) X класс 36*. Около сферы радиуса 10 описан некоторый 19-гранник. Доказать, что на его поверхности найдутся две точки, расстоя- расстояние между которыми больше 21. 37. Доказать, что если целое положительное число k делится на 10 101 010 101, то в его десятичной записи по крайней мере 6 цифр отличны от нуля. 38*. См. задачу 35, где муху ловят 2 паука, а максимальные скорости паука и мухи совпадают. 39. Имеется натуральное число м>1970. Возьмем остатки от деления числа 2" на 2, 3, 4, ..., п. Доказать, что .сумма этих остатков больше 2л. 40. У Мерлина есть две таблицы ""*Ч^ I ^^~% ЮОХЮО; одна из них пустая, а на дру- ^V f гой, волшебной, написаны какие-то числа. \/ Первая таблица прибита к скале у вхо- V да в пещеру, а вторая — к стене внутри ' пещеры. Вы можете обвести на первой таб- Рис 5 лице какой-нибудь квадрат (размером 114
1X1, 2X2, ... или 100X100), расположенный в любом месте таблицы, и за шиллинг узнать у Мерлина сумму чисел, стоящих в клетках соответствующего квадрата на волшебной таблице. Какое наименьшее количество денег потребуется, чтобы узнать сумму чисел на диагонали волшебной таблицы? Дополнительный тур. День Пифагора VII класс 41. Число 1234567 ... 1000 умножили на какое-то целое число от 1 до 9 и вычеркнули из произведения все единицы. Оставшееся после этого число опять умножили на однозначное число и вычеркнули все единицы и так проделали много раз. Какое наименьшее число удастся таким образом получить? 42. Имеется зал размером 13X13 м, разбитый на метровые квадраты. Разрешается класть прямоугольные ковры произволь- произвольных размеров так, чтобы их стороны шли по сторонам решетки (в частности, по границе зала). Можно класть ковры так, чтобы они частично перекрывались и даже чтобы некоторые из них полностью перекрывались остальными, но не допускается, чтобы один ковер полностью лежал на другом или под другим (даже если между ними имеются еще ковры). Какое наибольшее количе- количество ковров вы можете положить, чтобы эти условия выполня- выполнялись? 43. При обычной игре в домино кости выкладываются так, чтобы разность между числами на соседних костях равнялась 0. Можно ли выложить все 28 костей в замкнутую цепь так, чтобы все эти разности равнялись ±1? 44. Можно ли разбить числа 1, 2, 3, ..., 33 на 11 групп, по 3 числа в каждой, так, чтобы в каждой группе одно из чисел равнялось сумме двух других? 45. Али-Баба пытается проникнуть в пещеру. У входа в нее стоит барабан с четырьмя отверстиями по бокам. Около каждого отверстия внутри поставлен переключатель, имеющий 2 положе- положения: «верх», «низ». Разрешается засунуть руки в какие-либо 2 отверстия, пощупать, как стоят переключатели, и переключить их произвольным образом (в частности, можно не переключать). После этого барабан приходит в быстрое вращение, так что после его остановки уже нельзя установить, какие именно переключа- переключатели трогали в прошлый раз. Разрешается повторить эту опера- операцию до 10 раз. Дверь в пещеру открывается в тот момент, когда все переключатели стоят одинаково. Доказать, что Али- Баба сумеет попасть в пещеру. 46. Известно, чго в кадр фотоаппарата, расположенного в точке О, не могут попасть предметы А а В такие, что угол АОВ больше 179°. На плоскости поставлено 1000 таких фото- 115
аппаратов. Одновременно каждым фотоаппаратом делают по од- одному снимку. Доказать, что найдется снимок, на котором сфотографировано не больше 998 фотоаппаратов. XXXIV ОЛИМПИАДА A971 г.) 1-й т у р VIII класс 1. Город обнесен стеной, имеющей форму 1000-угольника (несамопересекающегося, но не обязательно выпуклого). На каж- каждом углу с внешней стороны стоит часовой. Доказать, что найдется часовой, который видит менее пятисот других часовых. 2. Окружность пересекается с выпуклым пятиугольником ABCDE в точках А\, А2, В\, В2, ... , Е\, Е2 (рис. 6). Известно, что ЛЛ,=А42, BBt—BB2, СС\ = СС2, DDt = DD2. Доказать, что ЕЕ{=ЕЕ2. 3. Во всесоюзном футбольном турнире участвовало 25 команд. После окончания турнира оказалось, что ни в одной встрече ни одна из команд не забила в ворота противника более четы- четырех мячей. Какое самое низкое место в турнирной таблице могла занять команда города Тбилиси, забившая мячей больше, чем любая другая команда, и пропустившая меньше любой другой команды? 4. На клетчатой бумаге нарисован квадрат ЮОХЮО клеток. Внутри каждой клетки поставили красную или синюю точку, так что в каждом столбце оказалось 50 синих и 50 красных точек и в каждой строке оказалось 50 синих и 50 красных точек. Соединим каждые две красные точки, расположенные в соседних квадратах (имеющих общую сторону), красным отрез- отрезком, а каждые две синие точки в соседних квадратах — синим отрезком. Доказать, что красных отрезков получится столь- столько же, сколько и синих. 5. Обозначим количество цифр в числе А череЪ к (А). До- Доказать, что число /гE1090701)- -fcB1090701) делится на 2. IX к л а с с 6. В вершинах правильного 25-угольника расположены чис- числа щ, С2, аз, ..., «25, причем 01=02=... = 013= 1. О14 = О|5 = = ... = 025= — 1- Над этими чис- числами произвели следующую операцию: к каждому числу прибавили ближайшее к нему 116 Ог
число по часовой стрелке. Например, к а7 прибавили а8, к а2ъ прибавили а.\. Полу- Полученные числа Ь\, Ь-2 Ь2ъ расставили в вершинах вместо чисел ai, а*, аз, ... , а25 а, g, в том же порядке и произвели над ними ' ту же операцию и т. д. Всего эту опера- Рис 7 цию проделали 100 раз. Доказать, что одно из полученных чисел будет больше 1020. 7. Пусть задан выпуклый /г-угольник Р(/г>6), периметр кото- которого равен 2. Построим новый выпуклый /г-угольник М, верши- вершины которого являются серединами сторон /г-угольника Р. Дока- Доказать, что периметр многоугольника М больше 1. 8. На плоскости проведено п прямых (л>2), причем никакие две из них не параллельны и никакие три не пересекаются в одной точке. Известно, что можно повернуть плоскость вокруг некоторой точки О на некоторый угол а<180° так, что каждая из проведенных прямых совместится с какой-нибудь другой про- проведенной прямой. Указать все числа п, для которых это воз- возможно. 9. Дано число 2\ где k — натуральное число, большее 3. Доказать, что никакое число, полученное из данного перестанов- перестановкой его цифр, не равно 2", где п — любое натуральное число, большее k. 10. Доказать, что среди чисел [2h-^J2\ бесконечно много составных. X класс 11. Дана замкнутая пространственная ломаная с вершинами А\, А2, ... , Ап, причем каждое звено пересекает фиксирован- фиксированную сферу в двух точках, а все вершины ломаной лежат вне сферы. Эти точки делят ломаную на Зл отрезков. Известно, что отрезки, прилегающие к вершине А\, равны между собой. То же самое верно и для вершин Лг, Аз, ..., Ап—\. Доказать, что отрезки, прилегающие к вершине Л„, также равны между собой. (Ср. с задачей 2.) 12. У Пети имеется набор «Юный паркетчик», который состоит из дощечек, уложенных в один слой в прямоугольную коробку так, что они покрывают всю ее площадь. Каждая дощечка имеет площадь 3 см2 и имеет форму либо прямоуголь- прямоугольника, либо уголка (рис. 7). Петя сказал, что он потерял дощечку в форме уголка, сделал вместо нее прямоугольную дощечку и уложил после этого все дощечки вместе с новой в один слой в коробку. Можно ли утверждать, что он лжет? 13. Про последовательность х\, х2, ¦¦¦ , хп, ... известно, что для любого п>\ выполнено равенство Зхп — хп-\=п. Кроме того, известно, что \х\ \ < 1971. Вычислить Х]д7] с точностью до 0,000001. 117
14. п точек расположены в вершинах выпуклого л-угольника. Внутри этого n-угольника отметили k точек. Оказалось, что любые три из n-\-k точек не лежат на одной прямой и являются вершинами равнобедренного треугольника. Чему может быть рав- равно число /г? 15. Лежит кучка в 10 миллионов спичек. Двое играют в следую- следующую игру. Ходят по очереди. За один ход играющий может взять из кучки спички в количестве рп, где р — простое число, л = 0, 1, 2, 3,... (например, первый берет 25 спичек, второй — 8, первый — 1, второй — 5, первый — 49 и т. д.). Выигрывает тот, кто берет последнюю спичку. Кто выигрывает при правиль- правильной игре? 2-й т у р VII класс 16. Существует ли число, квадрат которого начинается с цифр 123456789 и кончается цифрами 987654321? 17. Дан квадрат ABCD и точка О внутри. Из точек А, В, С, D опускаются перпендикуляры АН\, ВН2, СН3, DH4 соответственно на прямые ВО, СО, DO, АО. Доказать, что прямые, на которых лежат эти перпендикуляры, пересекаются в одной точке. 18. В колбе находится колония из п бактерий. В какой-то момент внутрь колбы попадает вирус. В первую минуту вирус уничтожает одну бактерию, и сразу же после этого и вирус, и оставшиеся бактерии делятся пополам. Во вторую минуту новые два вируса уничтожают две бактерии, а затем и вирусы, и остав- оставшиеся бактерии снова делятся пополам, и т. д. Наступит ли такой момент времени, когда не останется ни одной бактерии? 19. Имеется сетка, состоящая из квадратов размером 1X1. Каждый ее узел покрашен в один из четырех данных цветов так, что вершины любого квадрата 1X1 покрашены в разные цвета. Доказать, что найдется прямая, принадлежащая сетке, такая, что узлы, лежащие на ней, покрашены в два цвета. 20. На плоскости расположены 7 точечных прожекторов. Каждый прожектор освещает угол в 90°. Если в квадранте, освещенном каким-либо прожектором, окажется другой прожек- прожектор, то от последнего упадет тень — темный бесконечный луч. Доказать, что эти 7 прожекторов можно расположить так, что от каждого из них будет падать тень длиной 7 км. VIII класс 21. Дано 29-значное число Х = а\а2а3 ... ( 01=^0). Известно, что для всякого k цифра а* встречается в записи данного числа азо-ь раз (например, если аю = 7, то цифра g2o встречается 7 раз). Найти сумму цифр числа X. 118
22. Имеется картонный 1000-угольник (не обязательно выпук- выпуклый) . Этот многоугольник разрезали по прямой линии один раз. Он распался при этом на несколько новых многоугольников. Какое наибольшее число треугольников могло получиться среди этих многоугольников? 23. Доказать, что сумма цифр числа К, не более чем в 8 раз превосходит сумму цифр числа 8/(. 24. Разрешается взять любое число, состоящее из нулей и четверок, и проделать с ним следующие операции: поделить на 2, на 3 или на 5, если это деление возможно нацело, или же вставить 0 или 4 между цифрами этого числа, или приписать О или 4 к числу с любой стороны (справа или слева). С получен- полученным числом можно проделывать те же операции и т. д. Можно ли таким способом получить любое натуральное число? 25. См. задачу 17. IX к л а с с 26. Дан выпуклый 1971-угольник. Из каждой его вершины Ап каждая его сторона, не проходящая через А„, видна под одним и тем же углом ап. Доказать, что многоугольник правиль- правильный. 27. См. задачу 21. 28. Можно ли каждую сторону квадрата так разделить на 100 частей, чтобы из полученных 400 отрезков нельзя было бы составить контура никакого прямоугольника, отличного от исход- исходного квадрата? 29. Окружность разделена на п равных частей, и в точках деления написаны числа х\, хг, - , хп, равные -f-1 или —1. 360° Если повернуть окружность на угол k- , перемножить числа в совместившихся точках и сложить полученные п произведений, то для всякого /г = 0, 1, ... , п— 1 эта сумма будет равной 0. Дока- Доказать, что число п является точным квадратом целого числа. 30. Доказать, что можно расставить в вершинах правильного n-угольника действительные числа Х\, х2, ... , хп, все отличные от 0, так, чтобы для любого правильного /г-угольника, все вершины которого являются вершинами исходного «-угольника, сумма чи- чисел, стоящих в его вершинах, равнялась 0. X класс 31. См. задачу 29. Дополнительный вопрос: чему может равняться число л? 32. Даны два набора чисел: а\,...,ап и Ь\, ... , Ь„. Расположим числа о* в возрастающем порядке, а числа bk — в убывающем порядке. Получатся наборы Qi^ ... ^а„, Ь\~^ 119
Доказать, что max(af + 6i, a2-\-b2, ... , ап 33. Банкир и Игрок играют в следующую игру. Банкир называет 1000-значное число А\. Игрок, узнав это число, предла- предлагает Банкиру произвольное число В\. После этого Банкир по своему усмотрению вычитает из большего числа меньшее или складывает их, а результат сообщает Игроку — это число А2. Затем Игрок предлагает Банкиру следующее число В2. Банкир повторяет ту же операцию с числами А2 и В2 и т. д. Игра кончается, если у Банкира оказывается одно из следующих чисел: 1, 10, 100, 1000 Доказать, что Игрок всегда может закончить игру, предложив Банкиру не более 20 своих чисел. 34. В пространстве даны точка О и п попарно непарал- непараллельных прямых. Точка О ортогонально проектируется на все данные прямые. Каждая из получившихся точек снова проекти- проектируется на все данные прямые и т. д. Существует ли шар, содержащий все точки, которые могут быть получены таким образом? 35*. Доказать, что сумма цифр числа N превосходит сумму цифр числа 55-Л/ не более чем в 5 раз. (Ср. с задачей 23.) XXXV ОЛИМПИАДА A972 г.) 1-й т у р VII класс 1. Дано 17 натуральных чисел: а\, а2,...,а17. Известно, что а°2 = а2>=аз*= ... =а1% =а"\. Доказать, что at=a2 — = ... =ап. 2. На конгресс приехали 1000 делегатов из разных стран. Каждый делегат знает несколько языков. Известно, что любые трое могут разговаривать между собой без помощи остальных. (При этом, возможно, одному из них придется переводить разговор двух других.) Доказать, что всех делегатов можно расселить в 500 комнатах так, чтобы в каждой комнате располагались 2 делегата и при этом они могли бы поговорить между собой. 3. Каждая вершина правильного 13-угольника покрашена ли- либо в черный, либо в белый цвет. Доказать, что существуют три точки одного цвета, лежащие в вершинах равнобедренного треугольника. 4. См. задачу 9; доказать, что АВ — ВС. VIII класс 5. В некоторых клетках квадратной таблицы пХп стоят звездочки. Известно, что если вычеркнуть любой набор строк (только не все), то найдется столбец ровно с одной невычерк- 120
нутой звездочкой. (В частности, если строки совсем не вычерки- вычеркивать, то столбец ровно с одной звездочкой существует.) Доказать, что если вычеркнуть любой набор столбцов (только не все), то найдется строка ровно с одной невычеркнутой звездочкой. 6. На плоскости лежат две одинаковые фигуры, имеющие форму буквы «Г». Концы коротких палочек у букв «Г» обозна- обозначим через А и А'. Длинные палочки разделены на п равных частей точками щ, .... оп-ь а\, ... , а'п-\ (точки деления ну- нумеруются от концов длинных палочек). Проводятся прямые Ащ, Аа.2, ... , Ло„_ь А'а\, А'а'ч, ... , А'а'п-\. Точку пересечения прямых Аа\ и А'а\ обозначим через Xt, прямых Аа.2 и А'а'2 — через Лг и т. д. Доказать, что точки Х\, Х2,...,Хп-\ образуют выпуклый многоугольник. 7. Имеется 1000 монет, среди них 0, 1 или 2 фальшивые. Известно, что фальшивые монеты имеют одинаковую массу, отличную от массы нефальшивых монет. Можно ли за три взвешивания на чашечных весах без гирь определить, есть ли фальшивые монеты и легче они или тяжелее нормальных? (Коли- (Количество фальшивых монет определять не надо.) 8. Имеется набор натуральных чисел, причем сумма любых семи из них меньше 15, а сумма всех чисел из набора равна 100. Какое наименьшее количество чисел может быть в наборе? 9. В треугольнике ABC проведены медианы AD и BE. Углы CAD и СВЕ равны 30°. Доказать, что треугольник ABC правильный. IX класс 10. В треугольнике ABC угол С тупой. На стороне АВ отмечены точки ? и Я, на сторонах АС и ВС — точки К и М соответственно. Оказалось, что АН = АС, ЕВ = ВС, АЕ=АК, ВН = ВМ. Доказать, что точки Е, Н, К, М лежат на одной окружности. 11. В клетках шахматной доски размером «Х« расставлены числа: на пересечении /г-й строки и m-го столбца стоит число at,m- При любой расстановке на этой доске п ладей, при которой никакие две из них не бьют друг друга, сумма закрытых чисел равна 1972. Доказать, что существует два набора чисел х\, х2, ... , хп и у\, ... , у„, что при всех /гит выполняется равенство akm=xk-{-ym. 12. В некотором лесу расстояние между любыми двумя деревь- деревьями не превосходит разности их высот. Все деревья имеют высоту не более 100 м. Доказать, что лес можно обнести забором дли- длиной 200 м. 13*. Натуральные числа т и п взаимно просты яп<га, Какое число больше: 1 ¦— -н 2-— + ... + «•— или 1 •— + 121
14. Город X состоит из 10 бесконечных параллельных проспек- проспектов, пересекающих через равные интервалы поперечные улицы. Два полицейских, двигаясь вдоль проспектов и улиц, пытаются обнаружить гангстера, который может прятаться за домами. Если гангстер окажется на одном проспекте или на одной улице с каким-либо полицейским, он будет обнаружен. Скорость гангсте- гангстера не более чем в 10 раз превышает скорость полицейских, причем полицейским известно, что он в начальный момент времени находился от них на расстоянии не более 100 кварталов. Дока- Доказать, что полицейские смогут обнаружить гангстера. X класс 15. В городе «Многообразие» живут п жителей, любые два из которых либо дружат, либо враждуют между собой. Каждый день не более чем один житель может начать новую жизнь: перессориться со всеми своими друзьями и подружиться со всеми своими врагами. Доказать, что все жители могут подружиться. (Примечание. Если А — друг В, а В — друг С, то Л — также друг С.) 16. Дана бесконечная последовательность чисел а\, п2, ..., ап,.... причем каждое следующее число получено из предыдущего приписыванием справа любой цифры, кроме 9. Известно, что п\ — произвольное десятизначное число. Доказать, что в этой последовательности не менее двух составных чисел. 17. У тетраэдра ABCD все двугранные углы острые, а противоположные ребра попарно равны. Найти сумму косинусов всех двугранных углов тетраэдра. 18. Имеется несамопересекающийся невыпуклый гс-угольник Р. Рассмотрим множество Т его внутренних точек, из которых видны все вершины Р. Доказать, что Т — выпуклый много- многоугольник, число сторон которого не больше п. ' 19. См. задачу 14. 2-й т у р VII класс 20. Дан выпуклый четырехугольник ABCD. Точка пересечения диагоналей обозначена через О. Известно, что периметры треугольников ABO, BCO, CDO, ADO равны между собой. Дока- Доказать, что ABCD — ромб. 21. На плоскости проведены четыре прямые а, Ь, с, d. Ника- Никакие две из них не параллельны и никакие три не пересекаются в одной точке. Известно, что прямая а параллельна одной из медиан треугольника, образованного прямыми Ь, с, d. Доказать, что прямая Ъ параллельна некоторой медиане треугольника, образованного прямыми а, с и d. 122
22. Даны 12 последовательных натуральных чисел. Доказать, что хотя бы одно из них меньше суммы своих делителей. 23*. В стране Мара расположено несколько замков. Из каждо- каждого замка ведут три дороги. Из какого-то замка выехал рыцарь. Странствуя по дорогам, он из каждого замка, стоящего на его пути, поворачивает либо направо, либо налево по отношению к дороге, по которой приехал. Рыцарь никогда не сворачивает в ту сторону, в которую он свернул перед этим. Доказать, что когда- нибудь он вернется в исходный замок. 24. Прямая пересекает стороны АВ и ВС треугольника ABC в точках М и К. Известно, что площадь треугольника MB К равна площади четырехугольника АМКС Доказать, что МВ+ВК >±_ АМ + СА+КС ^ 3 VIII класс 25. См. задачу 20. 26. Числа а, Ь, с, d, е и / — натуральные. Известно, что -г->-т->-7-, af— Ье=\. Доказать, что d~^b-\-\. 27. В городе Никитовка двустороннее движение. В течение двух лет в городе проходил ремонт всех дорог. Вследствие этого в первый год на некоторых дорогах было введено односторон- одностороннее движение. На следующий год на этих дорогах было восста- восстановлено двустороннее движение, а на остальных дорогах введено одностороннее движение. Известно, что в любой момент ремонта можно проехать из любой точки города в любую другую. Доказать, что в Никитовке можно ввести одностороннее движение так, что из любой точки города удастся проехать в любую другую точку. 28. Пусть К (х) равно числу несократимых дробей -^- таких, что а<Сх и Ь<Сх(а и b — натуральные числа). Например, К\-^-)=3 (дроби 1; 2; —\ . Вычислить сумму К(НЮ) + «(И) +K(f) + ... +«(?} +KQ - 29. См. задачу 34 для 300 прямых и 100 треугольников. IX класс 30. В пятиугольнике длины всех сторон равны между собой, а величины всех углов меньше 120°. Доказать, что все углы пятиугольника тупые. 31. См. задачу 26. 32*. Улицы города «ЛЬ> представляют собой правильную квадратную сетку размером 20X20 кварталов. На некоторых 123
перекрестках имеются станции метро. Известно, что, выйдя на улицу в любом месте, можно добраться до метро, пройдя не более двух кварталов по улице. Какое наименьшее число стан- станций метро могло быть в городе «М»? 33*. Существуют ли рациональные числа а, Ь, с, d, удовлет- удовлетворяющие равенству (а + 6д/2J" + (с + сЬ/2Jп = 5 + 4д/2 (где п— натуральное число)? 34*. На плоскости проведено 3000 прямых, причем никакие две из них не параллельны и никакие три не пересекаются в одной точке. По этим прямым плоскость разрезана на куски. Доказать, что среди кусков найдется не менее: а) 1000 треуголь- треугольников, б) 2000 треугольников. X класс 35*. В пространстве расположены плоскость П и треугольник ABC, не принадлежащий этой плоскости. Треугольник Л1В1С1 является прямоугольной проекцией треугольника ABC на плоскость П. Доказать, что треугольник А\В\С\ может быть накрыт полностью треугольником, равным треугольнику ABC. 36. Даны два набора чисел: х\, лг2, ..., хп и у\, у2, .... у„- Известно, что: а) Х\>х2> >а-„>0, у\ >у2> ¦¦¦ >у«>0; б) X, > уи Х,+Х2 > У1+У2, .- , *1+*2+ ... +ХП > t/l + У2 + Доказать, что для любого натурального k справедливо нера- неравенство Хк1+Х2 + — +Хкп>У1+У2 + — +Уп. г 37. На четырехстах карточках написаны числа 1, 2, 3, ... , 400. Игроки А и В играют в следующую игру: первым ходом А берет себе любые 200 карточек, а остальные отдает В. Затем В берет 100 карточек из каждого набора, а остальные отдает А; таким образом, у обоих игроков оказалось по 200 карто- карточек. Следующим ходом А снова берет по 100 карточек из каждо- каждого набора, а остальные отдает В и т. д. После 200-го хода игрока В оба подсчитывают сумму чисел на своих1 карточках: СА и Св — и Л выплачивает игроку В разность Св — СА. Какую максимальную сумму может получить игрок В при правильной игре обоих? 38. Рассмотрим все рациональные числа между нулем и еди- единицей, знаменатели которых не превосходят п. Расположим их в порядке возрастания. Пусть -^- и ~ какие-то два соседних числа (дроби несократимы). Доказать, что \bc — ad\ = \ 39*. На всех клетках шахматной доски 8X8 расставлены натуральные числа. Разрешается выделить любой квадрат разме- размером 3X3 или 4X4 и увеличить все числа в нем на 1. Мы хотим в результате нескольких таких операций добиться, чтобы числа во всех клетках делились на 10. Всегда ли это удастся сделать? 124
XXXVI ОЛИМПИАДА A973 г.) 1-й тур VIII класс 1. На острове, имеющем форму квадрата, расположено не- несколько стран. Можно ли разбить эти страны на меньшие так, чтобы не появилось новых точек пересечения границ и чтобы можно было раскрасить карту этого острова в два цвета? 2. Может ли число, составленное из 600 шестерок и некоторо- некоторого количества нулей, быть квадратом целого числа? 3. На плоскости расположены пять точек так, что никакие три из них не лежат на одной прямой и никакие четыре — на одной окружности. Доказать, что среди этих точек можно выбрать такие две точки, которые расположены по разные стороны от окружности, проходящей через оставшиеся три точки. 4. Рассматриваются решения (х, у) уравнения 1 =—, где х, уи р — натуральные числа, р>\. Доказать, что при простом р это уравнение имеет ровно 3 решения; если же р — составное, то число решений больше трех. Замечание. Решения (а, Ь) и (Ь, а) при аФЪ считаются различными. 5. Игра «Чехарда». В трех вершинах квадрата находятся три кузнечика. Они играют в чехарду, т. е. прыгают друг через друга, причем если кузнечик А прыгает через кузнечика В, то после прыжка он оказывается от В на том же расстоянии, что и до прыжка, и, естественно, на той же прямой. Может ли после нескольких прыжков один из кузнечиков попасть в четвертую вершину квадрата? IX класс 6. На каждой стороне параллелограмма взято по точке. Пло- Площадь четырехугольника с вершинами в этих точках равна поло- половине площади параллелограмма. Доказать, что хотя бы одна из диагоналей четырехугольника параллельна стороне парал- параллелограмма. 7. Квадрат разбит на выпуклые многоугольники. Доказать, что их можно подразбить на меньшие выпуклые многоугольни- многоугольники так, чтобы в новом разбиении квадрата каждый многоуголь- многоугольник граничил с нечетным числом соседей (соседи — многоуголь- многоугольники с общей стороной). 8. Дан многочлен с целыми коэффициентами. В трех целых точках он принимает значение 2. Доказать, что ни в какой целой точке он не принимает значение 3. 9. В городе X с любой станции метро можно проехать на любую другую. Доказать, что одну из станций можно закрыть на ремонт без права проезда через нее так, чтобы с любой из оставшихся станций можно было проехать на любую другую. 125
10. Грани кубика пронумерованы числами 1, 2, ... , 6, при- причем сумма чисел на противоположных гранях равна 7. Имеется шахматная доска 50X50 клеток; клетки доски равны граням куби- кубика. Кубик перекатывается через ребра из левого нижнего угла доски в правый верхний угол. При перекатывании он каждый раз перемещается только вправо или вверх (влево и вниз переме- перемещаться запрещено), и на каждой клетке по пути его движения отпечатывается то число, которое расположено на грани, соприка- соприкасающейся с этой клеткой. Какое наибольшее значение может при- принимать сумма всех написанных чисел? А какое наименьшее? X класс 11. С натуральным числом k проделывается следующая опера- операция. Оно сначала раскладывается на простые множители: k = =Pi-p2- ... -pn-i-pn, а затем находится сумма Р1+Р2 + ... + +pn-i +Рл+ 1. С полученным числом проделывается та же опера- операция, и т. д. Доказать, что получающаяся последовательность чисел, начиная с некоторого момента, будет периодической. 12. См. задачу 6. 13. Многочлен Р (х) с целыми коэффициентами при некоторых целых х принимает значения 1, 2 и 3. Доказать, что существует не более одного целого числа х, при котором значение этого многочлена равно 5. 14. Доказать, что у всякого выпуклого многогранника най- найдутся две грани с одинаковым числом сторон. 15. На «черном ящике» (с неизвестным внутренним устрой- устройством) находится табло с N лампочками и пульт из N пере- переключателей на два положения — тумблеров. При переборе всех возможных состояний пульта на табло последовательно заго- загораются все возможные комбинации лампочек. Состояние табло однозначно определяется по состоянию пульта. Известно, что всег- всегда при переключении одного тумблера гаснет или загорается ровно одна лампочка. Доказать, что состояние каждой лампочки зависит от положения ровно одного переключателя (для каждой лампочки своего). « 2-й т у р VII класс 16. Из некоторого четырехзначного числа вычитают число, составленное из тех же цифр, но расположенных в обратном порядке. Может ли получиться число 1008? 17. Дан остроугольный треугольник ABC. Построены круги, центры которых расположены в вершинах ААВС, а радиусы равны высотам, проведенным из этих вершин. Доказать, что каждая точка треугольника покрыта хотя бы одним из кругов. 126
18. Лист клетчатой бумаги размером 100x100 раскрасили в 100 цветов. При этом каждую клетку либо закрасили одним из этих цветов, либо вовсе не закрасили. Раскраска называется правильной, если в каждой строке нет двух клеток оди- одинакового цвета. Можно ли закрасить лист правильным спосо- способом так, чтобы оказались закрашенными все клетки, если перво- первоначально были правильно закрашены: а) 1002—1; б) 1002 — 2; в) 100 клеток? 19. См. задачу 22а. VIII класс 20. На бумагу поставили кляксу. Для каждой точки кляксы определили наименьшее и наибольшее расстояние до границы кляксы. Среди всех наименьших расстояний выбрали наибольшее, а среди всех наибольших — наименьшее и сравнили полученные два числа. Какую форму имеет клякса, если эти два числа оказались равными между собой? 21. См. задачу 18, заменив число 100 на произвольное п. 22. В центре площади, имеющей форму квадрата, находится полицейский, а в одной из вершин — гангстер. Полицейский может бегать по всей площади, а гангстер — только по сторо- сторонам квадрата. Известно, что отношение максимальной скорости . I полицейского к максимальной скорости гангстера равно: а) —-; б) 0,49; в) 0,34; г) —. Доказать, что в каждом из этих случаев О полицейский может бегать так, что в какой-то момент окажется на одной стороне с гангстером. 23. Доказать, что в выпуклый равносторонний (не обязательно правильный) пятиугольник можно поместить правильный тре- треугольник с той же длиной стороны так, чтобы одна его сторона совпала со стороной пятиугольника, а весь треугольник оказался внутри пятиугольника. IX класс 24. Имеется 100-значное число, состоящее из единиц и двоек. Разрешается выбрать произвольно 10 идущих подряд цифр и поменять первые пять цифр с пятью следующими. Два таких числа называются похожими, если одно получается из другого несколькими описанными операциями. Какое наибольшее коли- количество попарно непохожих чисел можно выбрать? 25. На бесконечной шахматной доске с клетками размером 1 X 1 проведена замкнутая несамопересекающаяся ломаная, про- проходящая по сторонам клеток. Внутри ломаной оказалось k чер- черных клеток. Какую наибольшую площадь может иметь фигура, ограниченная этой ломаной? 26. См. задачу 29 для т = 5. 127
27*. На концах отрезка стоит по единице. Первым шагом между ними ставится их сумма — число 2. Следующим шагом между каждыми двумя соседними числами ставится их сумма, и т. д. 1 000 000 раз. (На втором шаге получается последователь- последовательность 13 2 3 1.) Сколько раз в результате будет написано число 1973? 28*. В условиях задачи 22 максимальная скорость гангстера в 2,9 раза больше скорости полицейского. Сможет ли полицейский оказаться на одной стороне с гангстером? X класс 29. Пусть тип — натуральные числа, не меньшие 2. Дока- Доказать, что найдется такое натуральное число k, что {2 30. У трехгранного угла проведены биссектрисы всех плос- плоских углов. Доказать, что попарные углы между биссектрисами одновременно либо все острые, либо все тупые, либо все прямые. 31. Двенадцать маляров живут в 12 красных и белых домах, расположенных по кольцевой дороге. Каждый месяц один из маляров, взяв с собой достаточное количество белой и крас- красной краски, выходит из дома и идет вдоль кольцевой дороги по часовой стрелке. Увидев красный дом, он перекрашивает его в белый цвет и идет дальше, а увидев белый дом, он перекрашивает его в красный цвет и идет мыть кисть. В течение года каждый маляр ровно один раз проделывает такое путешест- путешествие. Доказать, что в конце года каждый дом будем покрашен в первоначальный цвет, если в начале года хотя бы один дом был красным. 32. См. задачу 24. 33*. По арене круглого цирка радиуса 10 м бегает лев. Двигаясь по ломаной линии, он пробежал 30 км. Доказать, что сумма всех углов, на которые поворачивал лев, не меньше 2998 радиан. , XXXVII ОЛИМПИАДА A974 г.) 1-й т у р IX к л а с с 1. Доказать, что число 100...001, в котором 21974+ 21000 — 1 нулей, составное. 2. Доказать, что в круг радиуса 1 нельзя поместить без наложений два треугольника, площадь каждого из которых больше 1. 3. Две одинаковые шестеренки имеют по 32 зубца. Их совмес- совместили и спилили одновременно 6 пар зубцов. Доказать, что одну 128
шестеренку можно повернуть относительно другой так, что в местах сломанных зубцов одной шестеренки окажутся целые зубцы второй шестеренки. 4. Из отрезков, имеющих длины а, Ь и с, можно составить треугольник. Доказать, что из отрезков с длинами Ь+с ' а+Ь а + с также можно составить треугольник. 5. Выпуклый многоугольник обладает следующим свойством: если все прямые, на которых лежат его стороны, параллель- параллельно перенести на расстояние 1 во внешнюю сторону, то полу- полученные прямые образуют многоугольник, подобный исходному, причем параллельные стороны окажутся пропорциональными. Доказать, что в данный многоугольник можно вписать окруж- окружность. К класс 6. См. задачу 4. 7. Доказать, что для любого 13-угольника найдется прямая, содержащая ровно одну его сторону, однако при любом п>13 существует n-угольник, для которого это неверно. 8. См. задачу 3, где у шестеренок с 92 зубцами спилено 10 пар зубцов. 9. На кубе отмечены вершины и центры граней, а также проведены диагонали всех граней. Можно ли по отрезкам этих диагоналей обойти все отмеченные точки, побывав в каждой из них ровно по одному разу? 10. См. задачу 5. 2-й т у р VII класс 11. Точка О внутри правильного шестиугольника со стороной 1 соединена со всеми его вершинами. Доказать, что среди треугольников, на которые разбивается шестиугольник, найдутся два таких, что все их стороны будут не меньше 1. 12. На прямой расположено 100 точек. Отметим середины всевозможных отрезков с концами в этих точках. Какое наимень- наименьшее число отмеченных точек может получиться? 13. Сколько сторон может быть у выпуклого многоугольника, все диагонали которого имеют равную длину? 14. Несколько стеклянных шариков разложено в три кучки. Мальчик, располагающий неограниченным запасом шариков, мо- может за один ход взять по одному шарику из каждой кучки или же добавить из своего запаса в одну из кучек столько 5 Заказ 247 129
шариков, сколько в ней уже есть. Доказать, что за несколько ходов мальчик может добиться того, что в каждой кучке не останется ни одного шарика. VI11 класс 15. См. задачу 13. 16. См. задачу 12 для п точек на плоскости. 17. В клетках прямоугольной таблицы, имеющей 8 строк и 5 столбцов, расставлены натуральные числа. За один ход разре- разрешается одновременно удвоить все числа одной строки или же вычесть единицу из всех чисел одного столбца. Доказать, что за несколько ходов можно добиться того, чтобы все числа таблицы стали равными нулю. 18. Дан выпуклый пятиугольник, все углы которого тупые. Доказать, что в нем найдутся две такие диагонали, что круги, построенные на них как на диаметрах, полностью покроют пяти- пятиугольник. 19. Сумма 100 натуральных чисел, каждое из которых не больше 100, равна 200. Доказать, что из них можно выбрать несколько чисел, сумма которых равна 100. IX класс 20. Существует ли такая последовательность натуральных чисел, чтобы любое натуральное число 1, 2, 3, ... можно было представить единственным способом в виде разности двух чисел этой последовательности? 21. Доказать, что в произвольном 2п-угольнике найдется диагональ, не параллельная ни одной из его сторон. 22*. Имеется несколько гирь, масса каждой из которых равна целому числу. Известно, что их- можно разбить на k равных по массе групп. Доказать, что не менее чем k способами можно убрать одну гирю так, чтобы оставшиеся гирн нельзя было разбить на k равных по массе групп. 23. Дан треугольник ABC, AD и BE — его биссектрисы. Известно, что АС>ВС. Доказать, что AE>DE>BD. 24*. Прямоугольный лист бумаги размером aXb см разрезан на прямоугольные полоски, каждая из которых имеет сторону 1 см. Линии разрезов параллельны сторонам исходного листа. Доказать, что хотя бы одно из чисел а или Ъ целое. X класс 25. См. задачу 20. 26. Доказать, что в десятичной записи чисел 2"+ 1974" и 1974" содержится одинаковое количество цифр. 130
27. Шарообразная планета окружена 37-ю точечными астеро- астероидами. Доказать, что в любой момент на поверхности планеты найдется точка, из которой астроном не сможет наблюдать более 17 астероидов. Примечание. Астероид, расположенный на линии горизонта, не виден. 28. На конгресс собрались ученые, среди которых есть друзья. Оказалось, что любые два из них, имеющие на конгрес- конгрессе равное число друзей, не имеют общих друзей. Доказать, что найдется ученый, который имеет ровно одного друга из числа участников конгресса. 29. См. задачу 24. XXXVIII ОЛИМПИАДА A975 г.) 1-й т у р X клас с 1. Найти все действительные решения уравнения с 4 неиз- неизвестными: 2. Точка А расположена на расстоянии 50 см от центра круга радиуса 1 см. Разрешается точку А отразить симметрич- симметрично относительно произвольной прямой, пересекающей круг; по- полученную точку отразить симметрично относительно любой пря- прямой, пересекающей круг, и т. д. Доказать, что: а) за 25 отражений точку А можно переместить внутрь круга; б) за 24 отражения этого сделать нельзя. 3. Натуральные числа а, Ь, с таковы, что числа р = Ьс-\-а, q = ab-\-c, r = c"-\-b простые. Доказать, что два из чисел р, q, r равны между собой. 4. На шахматной доске размером 8X8 отмечены 64 точки — центры всех клеток. Можно ли отделить все точки друг от друга, проведя 13 прямых, не проходящих через эти точки? 5*. Можно ли разместить в пространстве четыре свинцовых шара и точечный источник света так, чтобы каждый исходящий из источника света луч пересекал хотя бы один из шаров? 2-й т у р VII класс 6. См. задачу 10 а) при л=100. 7. В окружность вписан выпуклый 7-угольник. Известно, что какие-то три его угла равны 120°. Доказать, что найдутся две его стороны, имеющие одинаковую длину. 8. Коля и Внтя играют в следующую игру. На столе лежит куча из 31 камня. Мальчики делают ходы поочередно, а начи- начинает Коля. Делая ход, играющий делит каждую кучку, в которой 5* 131
больше одного камня, на две меньшие кучки. Выигрывает тот; кто после своего хода оставляет кучки по одному камню в каж дой. Сможет ли Коля сделать так, чтобы выиграть при любой игре Вити? 9. В последовательности 19752... каждая цифра, начиная с пятой, равна последней цифре суммы предыдущих четырех цифр. Встретится ли в этой последовательности: а) набор цифр 1234; 3269; б) вторично набор 1975? VIII класс 10. Какое из двух чисел больше: а) 2= \ 1 ? п раз > (л— I) | J или З3 J i* ) п paj л ( In \) раз' б) З1 ) или 4" 11. См. задачу 7. 12*. См. задачу 9 с добавлением пункта: в) встретится ли набор 8197? 13. Имеются две страны: Обычная и Зазеркалье. У каждого города в Обычной стране есть «двойник» в Зазеркалье, и наоборот. Однако если в Обычной стране какие-то два города соединены железной дорогой, то d Зазеркалье эти города не соединены, а любые два иесоеднненпых в Обычной стране города обязательно соединены железной дорогой в Зазеркалье. В Обыч- Обычной стране девочка Алиса не может проехать из города А в го- город В, сделав менее двух пересадок. Доказать, что Алиса в Зазеркалье сможет проехать из любого города в любой другой, сделав не более двух пересадок. 14. В футбольном турнире принимают участие п команд. Каждая команда встречается с каждой по одному разу, при этом выигравшей команде присуждается 2 очка, сыгравшей вничью — 1 очко, проигравшей — 0 очков. Какой максимальный разрыв в очках может быть между командами, занявшими соседние места? 15. См. задачу 10. 16. См. задачу 7. 17. См задачу 14. IX класс 132
18*. В государстве Мантисса города соединены дорогами. Длина любой дороги меньше 500 км, и из любого города в любой другой можно попасть, проехав по дорогам меньше 500 км. Когда одна дорога оказалась закрытой на ремонт, выяснилось, что из каждого города можно проехать по оставшимся дорогам в любой другой. Доказать, что при этом можно проехать меньше 1500 км. 19*. Можно ли какой-нибудь выпуклый многоугольник раз- разрезать на конечное число невыпуклых четырехугольников? Л" класс 20. См. задачу 10. 21. См. задачу 8 с заменой числа 31 на 100. 22. См. задачу 18. 23*. Арена цирка освещается п различными прожекторами Каждый прожектор освещает некоторую выпуклую фигуру. Известно, что если выключить один произвольный прожектор, то арена будет по-прежнему полностью освещена, а если выклю- выключить произвольные 2 прожектора, то арена полностью освещена не будет. При каких значениях п это возможно? 24. См. задачу 19. XXXIX ОЛИМПИАДА A976 г.) 1-й тур X к л а с с 1. Найти все положительные решения системы уравнений ( Х\+ Х2 = х\, I Х + Х = Х < I I х5+ Xx=xl. 2. В остроугольном треугольнике ABC проведены медиана AM, биссектриса ВК и высота СИ. Пусть М'К'Н' — тре- треугольник с вершинами в точках пересечения трех проведен- проведенных отрезков. Может ли площадь полученного треугольника быть больше 0,499 площади треугольника ABC? 3. Каковы первые четыре цифры числа 1' + 22 + З3 + ..+ 4-999999+1000'000? 4. Астрономический прожектор освещает октант (трехгран- (трехгранный угол, у которого все плоские углы прямые). Прожектор помещен в центр куба. Можно ли его повернуть таким образом, чтобы он не освещал ни одной вершины куба? 5. На бесконечном листе клетчатой бумаги (размер клетки 1X1) укладываются кости домино размером 1X2 так, что они 133
покрывают все клетки. Можно ли при этом добиться того, чтобы каждая прямая, идущая по линиям сетки, разрезала бы лишь конечное количество костей домино? 2-й т у р VII класс 6. Даны четыре одинаковых по виду шара массой 101 г, 102 г, 103 г и 104 г, а также чашечные весы со стрелкой, на которых можно взвесить произвольный груз. Сделав два взвешивания, определить массу каждого шара. 7. Может ли выпуклый неправильный пятиугольник иметь ровно 4 стороны одинаковой длины и ровно 4 диагонали одина- одинаковой длины? 8. Существует ли такое натуральное число п, что сумма цифр числа п2 равна 100? 9. Можно ли на плоскости расположить конечное число точек таким образом, чтобы у каждой точки было бы ровно три ближайших к ней точки? 10. В клетках таблицы размером 10X20 расставлено 200 различных чисел. В каждой строчке отмечены 2 наибольших числа красным цветом, а в каждом столбце отмечены 2 наиболь- наибольших числа синим цветом. Доказать, что не менее трех чисел от- отмечены в таблице как красным, так и синим цветом. VIII класс 11. См. задачу 8. 12. У квадрата ABCD длина стороны — целое число. Внут- Внутри квадрата размещены параллельно его сторонам отрезки, делящие квадрат на более мелкие квадраты, длины сторон ко- которых тоже целые. Длина каждого отрезка — целое число. До- Доказать, что сумма длин всех отрезков кратна 4. 13. См. задачу 17. 14. См. задачу 10. 15. См. задачу 24. IX к л ас с 16. Может ли число п\ оканчиваться цифрами 1976000...000? 17. На сферическом Солнце обнаружено конечное число круг- круглых пятен, каждое из которых занимает меньше половины поверх- поверхности Солнца. Эти пятна предполагаются замкнутыми (т. е. граница пятна принадлежит ему) и не пересекаются между собой. Доказать, что на Солнце найдутся две диаметрально противоположные точки, не покрытые пятнами. 134
18*. Доказать, что существует такое натуральное число п, большее 1000, что сумма цифр числа 2" больше суммы цифр числа 2" + I. 19. Дано число N,. в записи которого нет нулей. Если в нем стоят рядом две одинаковые цифры или два одинаковых двузнач- двузначных числа, то их разрешается вычеркнуть. Кроме того, разре- разрешается также в любое место вставить две одинаковые цифры или два одинаковых двузначных числа. Доказать, что, комбинируя эти операции, можно из числа N получить число, меньшее 10 . 20*. На столе лежит большой лист клетчатой бумаги со стороной клетки 1 см и неограниченное количество пятикопеечных монет (пятаков); радиус пятака равен 1,3 см. Доказать, что лист можно покрыть пятаками, не налегающими друг на друга, так, чтобы все узлы листа оказались покрытыми. X класс 21. Существует ли такое натуральное число А, что если приписать его к самому себе справа, то полученное число ока- окажется полным квадратом? 22. Существует ли такой выпуклый 1976-гранник, который обладал бы следующим свойством: при произвольной расста- расстановке стрелок на концах всех его ребер сумма полученных век- векторов отлична от 0? 23. В клетках таблицы размером 10X20 расставлено 200 различных чисел. В каждой строчке отмечены 3 наибольших числа красным цветом, а в каждом столбце отмечены 3 наиболь- наибольших числа синим цветом. Доказать, что не менее 9 чисел отмечены в таблице как красным, так и синим цветом. (Ср. с задачей 10.) 24. На плоскости задано конечное множество точек. Дока- Доказать, что в нем найдется точка, у которой имеется не более трех ближайших к ней точек из этого же множества. (Ср. с задачей 9.) 25. Каждая точка пространства окрашена в один из фикси- фиксированных пяти цветов, причем имеется 5 точек, окрашенных в различные цвета. Доказать, что существуют прямая, все точки которой окрашены не менее чем в три цвета, и плоскость, все точки которой окрашены не менее чем в четыре цвета. XL ОЛИМПИАДА A977 г.) 1-й тур X к л а с с 1. Последовательность натуральных чисел (х„) строится сле- следующим образом: х\=2, лг„ +1 =[1,5лгп]. Доказать, что в этой последовательности бесконечно много: а) нечетных чисел; б) чет- четных чисел. 135
2. На столе расположено п картонных квадратов и п пласт- пластмассовых квадратов. Никакие два картонных квадрата не имеют общих точек, в том числе и точек границы. То же самое верно и для пластмассовых квадратов. Оказалось, что множество вер- вершин картонных квадратов совпадает с множеством вершин пласт- пластмассовых квадратов. Обязательно ли каждый картонный квад- квадрат совпадает с некоторым пластмассовым? 3*. а) Двенадцать тонких твердых стержней длины 1 скре- скрепили так, что получился каркас куба. Можно ли сделать в плос- плоскости отверстие площади меньше 0,01, не разбивающее плоскость на куски, чтобы через него можно было бы протащить этот кар- каркас? б) Тот же вопрос для каркаса тетраэдра с ребром 1. 4. Каждая точка числовой оси, координата которой — целое число, покрашена либо в красный, либо в синий цвет. Доказать, что найдется цвет со следующим свойством: для каждого нату- натурального числа k имеется' бесконечное количество точек этого цвета, координаты которых делятся на k. 2-й т у р VII класс 5. В каждой вершине выпуклого л-уголыжка находится охотник, вооруженный лазерным ружьем. Все охотники одно- одновременно стреляют в зайца, сидящего в точке О внутри этого л-угольника. В момент выстрела заяц пригибается и все охотники погибают. Доказать, что, кроме точки О, никакая другая точка не обладает тем же свойством. 6. Куб 3X3X3 составлен из 14 белых и 13 черных кубиков с ребром 1. Столбик — это три кубика, стоящие подряд вдоль од- одного направления: ширины, длины или высоты. Может ли быть так, что в каждом столбике нечетное количество: а) белых ку- биков; б) черных кубиков? 7. Доказать, что найдется более 1000 троек натуральных чисел (а, Ь, с) таких, что для них выполняется равенство 8. В доске торчит 1977 гвоздей. Двое играющих делают ходы по очереди. Ход состоит в том, что играющий соединяет про- проводом два гвоздя. Если в результате хода цепь замкнулась, то сделавший этот ход считается выигравшим. Кто выигрывает при правильной игре: первый игрок или второй? Замечание. Не разрешается соединять проводом два ранее соединенных гвоздя. 9. Найти такое наименьшее п, что любой выпуклый 100- уголышк можно получить в виде пересечения п треугольников. Доказать, что для меньших п это можно сделать не с любым выпуклым 100-угольннком. 136
VIII класс 10. См. задачу 5. 11. См. задачу 6. 12. См. задачу 7. 13. См. задачу 17 а). 14. См. задачу 9. IX класс 15. В пространстве расположено п отрезков, никакие три из которых не параллельны одной плоскости. Для любых двух отрезков прямая, соединяющая их середины, перпендикулярна обоим отрезкам. При каком наибольшем п это возможно? 16. а) Существуют ли такие 6 различных натуральных чисел, что для любых а и b из них сумма а-\-Ь делится на разность а — Ь? б) Тот же вопрос для 1000 чисел. 17. а) После окончания волейбольного турнира оказалось, что для любых двух команд существует третья, которая выигра- выиграла у обеих. Доказать, что число команд в турнире не меньше 7. б) В другом волейбольном турнире для любых трех команд найдется команда, которая выиграла у этих трех. Доказать, что число команд не меньше 15. 18*. В пространстве расположен выпуклый многогранник, все вершины которого находятся в целочисленных точках (т. е. все три координаты каждой вершины — целые числа). Других целочисленных точек внутри, на гранях и на ребрах много- многогранника нет. Доказать, что число вершин многогранника не превосходит 8. 19*. Дан многочлен Р (х) с целыми коэффициентами. Для каждого натурального п значение Р (п) больше п. Рассмотрим последовательность *| —1, x2 = P{xt), .... хп — Р{хп-\), Известно, что для любого натурального /V найдется член после- последовательности, делящийся на N. Доказать, что справедливо ра- равенство Р (х) = х + 1. X класс 20. Можно ли на плоскости расположить бесконечное множе- множество одинаковых кругов так, чтобы любая прямая пересекала не более двух кругов? 21. См. задачу 16 для 15 чисел. 22. См. задачу 17 б). 23. Последовательность натуральных чисел (х„) строится по следующему правилу: *i=2, *2 = 3, .... л:„ + | =[1,5лг„]. Доказать, что последовательность {уп), где уи = (—1)*", непериодична. 24. См. задачу 19. 137
XLI ОЛИМПИАДА A978 г.) VII класс I. Найти все пары целых чисел (х, у), удовлетворяющие уравнению 3-2х-\-1=у2. 2*. На плоскости расположен пластмассовый треугольник. Если начать перекатывать его через стороны и если он в какой- то момент пересечется со своим первоначальным положением, то окажется, что он просто совпал со своим первоначальным положением. Какие треугольники удовлетворяют такому условию? Указать все виды таких треугольников. 3. Доказать, что в прямоугольник размером пХ.2т (п и т — целые) можно уложить в два слоя кости домино размером 1X2 так, чтобы каждый слой полностью покрывал прямо- прямоугольник и чтобы никакие две кости из разных слоев не совпа- совпадали друг с другом. 4. См. задачу 16 а). VIII класс 5. См. задачу 10. 6. См. задачу 2. 7. См. задачу 3. 8. См. задачу 16 а). 9. Дано некоторое 1000-значное число А. Про него известно, что любые его 10 идущих подряд цифр образуют число, крат- кратное 210. Доказать, что число А делится на 21000. IX кла с с л 10. В «-угольнике расположено несколько точек таким обра- образом, что в произвольном треугольнике, образованном любыми тремя вершинами л-угольника, расположена по крайней мере одна точка. Какое наименьшее число точек могло быть? II. Существует ли на плоскости конечный набор различных векторов п\, ач, .... а„ такой, что для любой пары различных векторов из этого набора найдется такая другая пара из этого набора, что суммы каждой из пар равны между собой? 12. См. задачу 16. 13. На плоскости расположено несколько прямых и точек. Доказать, что на плоскости найдется точка А, не совпадающая ни с одной из данных точек, расстояние от которой до любой из данных точек больше расстояния от нее до любой из данных прямых. 14. В поселке живут 100 жительниц. У каждой из них имеют- имеются 3 знакомые жительницы. 1 января одна узнала интересную новость и сообщила ее трем своим знакомым; 2 января те 138
сообщили новость всем своим знакомым и т. д. Может ли случиться так, что 5 марта еще не все жительницы будут знать эту новость, а 19 марта — все? X класс 15. У белой сферы 12% ее площади окрашено в красный цвет. Доказать, что в сферу можно вписать параллелепипед, у которого все вершины белые. 16. Город имеет форму квадрата. В нем 6 улиц: 4 стороны квадрата и 2 его средние линии. Полицейский гоняется по улицам за гангстером. Если в какой-то момент полицейский и гангстер оказываются на одной улице, то гангстер сдается полицейскому. Доказать, что полицейский сможет поймать гангстера, если его скорость: а) в 3 раза; б)* в 2,1 раза больше скорости гангстера. 17. См. задачу 13. 18*. Доказать, что существует: а) одно; б) бесконечно много таких натуральных чисел п, что последние цифры числа 2" образуют число п. 19*. Дано 8 действительных чисел: а, Ь, с, d, e, f, g, h. Доказать, что хотя бы одно из шести чисел ac-\-bd, ae-\-bf, ag-\-bh, ce + df, cg-\-dh, eg+fh неотрицательно. XLII ОЛИМПИАДА A979 r.) VII класс 1. На плоскости отмечена точка О. Можно ли так располо- расположить на плоскости: а) 5 кругов; б) 4 круга, не покрывающих точку О, чтобы любой луч с началом в точке О пересекал не менее двух кругов? 2. Имеется несколько гирь, общая масса которых равна 1 кг. Каждой гире присвоен свой номер: 1, 2, 3, ... . Доказать, что найдется такой номер п, что масса гнри с номером п строго больше Tjr кг. 3. Квадрат разрезан на прямоугольники. Доказать, что сумма площадей кругов, описанных около каждого прямоуголь- прямоугольника, не меньше площади круга, описанного около квадрата. 4. Коля и Витя играют в следующую игру на бесконечной клетчатой бумаге. Начиная с Коли, они по очереди отмечают узлы клетчатой бумаги — точки пересечения вертикальных и го- горизонтальных прямых. При этом каждый из них своим ходом должен отметить такой узел, чтобы после этого все отмеченные узлы лежали в вершинах выпуклого многоугольника (начиная со второго хода Коли). Тот из играющих, кто не сможет сделать очередного хода, считается проигравшим. Кто выигрывает при правильной игре? 139
VIII класс 5. На плоскости отмечена точка О. Можно ли так располо- расположить на плоскости: а) 7 кругов; б) 6 Кругов, не покрывающих точку О, чтобы любой луч с началом в точке О пересекал не менее трех кругов? (Ср. с задачей 1.) 6. См. задачу 2. 7. Четырехугольник ABCD вписан в окружность с центром О. Диагонали АС и BD четырехугольника взаимно перпенди- перпендикулярны. Доказать, что длина перпендикуляра ОН, опущен- опущенного из центра окружности на сторону AD, вдвое меньше длины стороны ВС. 8. См. задачу 3. 9. На химической конференции присутствовало k ученых хи- химиков и алхимиков, причем химиков было больше, чем алхими- алхимиков Известно, что на любой вопрос химики всегда отвечают правду, а алхимики иногда говорят правду, а иногда лгут. Оказавшийся на конференции математик про каждого ученого хочет установить, химик тот или алхимик. Для этого он любому ученому может задать вопрос: «Кем является такой-то: хими- химиком или алхимиком?» (В частности, может спросить, кем являет- является сам этот ученый.) Доказать, что математик может установить это за: а) 4/г вопросов; б) 2k— 2 вопросов. IX класс 10. Имеется несколько камней, масса каждого из которых не превосходит 2 кг, а общая масса равна 100 кг. Из них выби- выбирается несколько камней, суммарная масса которых отличается от 10 кг на наименьшее возможное для данного набора число d. Какое максимальное значение может принимать число d для всевозможных наборов камней? 11*. Можно ли все пространство представить в виде объе- объединения бесконечного числа попарно скрещивающихся пря- прямых? 12. а) Существует ли последовательность натуральных чисел о.\, C2, аз, ..., обладающая следующим свойством: ни один член последовательности не равен сумме нескольких других и а„^п при любом л? б) Тот же вопрос, если ап-^.п^ при любом п. 13. См. задачу 7. 14*. См. задачу 9 при числе вопросов, равном 2k— 3. X класс 15. См. задачу 10. 16. На отрезке длины 1 отмечено несколько интервалов. Из- Известно, что расстояние между любыми двумя точками, принадле- 140
жащими одному или разным отмеченным интервалам, не равно 0,1. Доказать, что сумма длин отмеченных интервалов не пре- превосходит 0,5. 17. Функция у = /(х) определена на отрезке |0; 1] и в каждой точке этого отрезка имеет первую и вторую производные. Извест- Известно, что /@) = /A) = 0 и что \f" (х)\ ^ 1 на всем отрезке. Какое наибольшее значение может принимать максимум функции / для всевозможных функций, удовлетворяющих этим условиям? 18. Объединение нескольких кругов имеет площадь 1. Дока- Доказать, что из них можно выбрать несколько попарно непересе- непересекающихся кругов, сумма площадей которых больше —. 19. См. задачу 14. XLIII ОЛИМПИАДА A980 г.) VII класс 1. Найти наибольшее пятизначное число А, у которого чет- четвертая цифра больше пятой, третья больше суммы четвертой и пятой, вторая больше суммы третьей, четвертой и пятой и пер- первая цифра больше суммы остальных. 2. В каждой клетке прямоугольной таблицы записано одно из двух чисел: +1 или —1. При этом количество +1 не меньше двух и количество — 1 не меньше двух. Доказать, что найдутся две строки и два столбца такие, что сумма четырех чисел, стоя- стоящих на их пересечении, равна 0. 3. Доказать, что максимальное количество сторон выпукло- выпуклого многоугольника, стороны которого лежат на диагоналях дан- данного выпуклого 100-угольника, не больше 100. 4. Три прямолинейных коридора одинаковой длины / образу- образуют фигуру, изображенную на рисунке 8. По ним бегают гангстер и полицейский. Максимальная скорость полицейского в 2 раза больше максимальной скорости гангстера. Полицейский сможет увидеть гангстера, если он окажется от него на расстоянии, не большем г. Доказать, что полицейский всегда может поймать гангстера, если: а) л>—; б) г~>-г- 5. Десять вершин правильного 20- угольника ЛИ2Л3...Л20 покрашены в чер- черный цвет, а десять других — в белый. Рассматривается множество, состоящее из диагонали А\АА и всех диагоналей, равных ей. Доказать, что в этом множе- множестве количество диагоналей с двумя чер- черными концами равно количеству диаго- диагоналей с двумя белыми концами. 141 120° 120' Рис. 8.
VIII класс 6. Доказать, что если а\ ^аг^аз^.-.^аю, то 6 ^* 10 7. См. задачу 2. 8. На хорде А В окружности К с центром в точке О взята точка CD — вторая точка пересечения окружности К с окруж- окружностью, описанной около &АСО. Доказать, что CD = CB. 9. См. задачу 4. 10. См. задачу 5. IX класс 11. oi, иг, а3, ..., ап, ... — возрастающая последовательность натуральных чисел. Известно, что ап + |^10а,, при всех нату- натуральных п. Доказать, что бесконечная десятичная дробь 0, а\п2аз..., полученная приписыванием этих чисел друг к другу, непериодическая. 12. На пульте имеется несколько кнопок, с помощью которых осуществляется управление световым табло. После нажатия лю- любой кнопки некоторые лампочки на табло переключаются (для каждой кнопки есть свой набор лампочек, причем наборы могут пересекаться). Доказать, что число состояний, в которых может находиться табло, равно некоторой степени числа 2. 13. На прямоугольном листе клетчатой бумаги размером /лХ« клеток расположено несколько квадратов, стороны которых идут по вертикальным и горизонтальным линиям бумаги. Извест- Известно, что никакие два квадрата не совпадают и никакой квадрат не содержит внутри себя другой квадрат. Каково Наибольшее число таких квадратов? 14. См. задачу 4 для случаев: а) л> —; б)* л>—. 15. См. задачу 8. X класс 16. См. задачу 11. 17. См. задачу 12. 18. См. задачу 14. 19. В каждой клетке таблицы 1980X1980 записано одно из чисел -J- 1, — 1,0. Сумма всех чисел равна 0. Доказать, что суще- существуют две строки и два столбца такие, что сумма четырех чисел, стоящих на их пересечениях, равна 0. 20*. На сфере радиуса 1 расположено несколько дуг больших окружностей (большая окружность — это пересечение сферы с плоскостью, проходящей через ее центр). Сумма длин всех этих дуг меньше л. Доказать, что найдется плоскость, проходящая через центр сферы, которая не пересекается ни с одной из дуг. 142
XLIV ОЛИМПИАДА A981 г.) VII класс 1. Натуральное число А при делении на 1981 дало в остатке 35, при делении на 1982 оно дало в остатке также 35. Каков остаток от деления числа А на 14? 2. См. задачу 11 для числа с 13 разрядами. 3. На двух равных круглых листах бумаги художник нарисо- нарисовал одинаковых драконов. Оказалось, что на первом листе глаз дракона совпал с центром круга, а на втором — не совпал. Дока- Доказать, что второй лист бумаги можно разрезать на такие две час- части, чтобы из них удалось сложить круг того же радиуса с тем же драконом, но чтобы его глаз уже находился в центре круга. 4. Дано число х, большее 1. Обязательно ли имеет место равенство 5. Имеется 5 гирь. Их массы равны 1000 г, 1001 г, 1002 г, 1004 г и 1007 г, но надписей на гирях нет и внешне они неотли- неотличимы. Имеются весы со стрелкой, которые показывают массу в граммах. Как с помощью трех взвешиваний определить гирю в 1000 г? VIII класс 6. В пятиугольнике проведены все диагонали. Какие 7 углов между двумя диагоналями или между диагоналями и сторонами надо отметить, чтобы из равенства этих углов друг другу следо- следовало, что пятиугольник правильный? 7. См. задачу 2. 8. См. задачу 3. 9. См. задачу 4. 10. Дано 10 натуральных чисел: at <а2<аз<...<аю. До- Доказать, что их наименьшее общее кратное не меньше 10 at. IX класс 11. Дано число, имеющее нечетное число разрядов. Дока- Доказать, что одну из его цифр можно вычеркнуть так, что в полу- полученном числе количество семерок на четных местах будет равно количеству семерок на нечетных местах. 12. Натуральные числа at, a2, .... а„ таковы, что каждое не превышает своего номера (а*^/г) и сумма всех чисел — четное число. Доказать, что одна из сумм at ±а2±аз±...±ап равна нулю. 13. А" и Y — два выпуклых многоугольника, причем много- многоугольник X содержится внутри Y. Пусть S (X) и S (Y) — площади этих многоугольников, а Я (А") и Р (Y) — их периметры. Доказать, S(Y) 143 что S(X 2 что Р(Х) <-z P{Y)
14*. Можно ли разбить множество натуральных чисел на бесконечное число бесконечных подмножеств, каждое из ко- которых получается из любого другого подмножества прибавле- прибавлением одного и того же целого числа к каждому элементу? 15*. У правильного 1981-угольника отмечены 64 вершины. Доказать, что существует трапеция с вершинами в отмеченных точках. X класс 16. Рассматривается функция у=/(х), определенная на всем множестве действительных чисел и удовлетворяющая для не- некоторого числа AgfcO соотношению / (x-\-k)-(\ — f (x))= 1 + f{x). Доказать, что / (х) ¦— периодическая функция. 17. Дан многочлен Р (х) степени п со старшим коэффици- коэффициентом, равным 1. Известно, что если х — целое число, то Р (х) — целое число, кратное р (р— натуральное число). Доказать, что п\ делится на р. 18. Доказать, что последовательность *n = sin(n2) не стре- стремится к нулю при л, стремящемся к бесконечности. 19. В квадрате со стороной длины 1 расположена ломаная без самопересечений, длина которой не меньше 200. Доказать, что найдется прямая, параллельная одной из сторон квадрата, пересекающая ломаную не менее чем в 101-й точке. 20. Радиус вписанной в треугольник окружности равен —, а длины высот треугольника — целые числа, сумма которых рав- равна 13. Вычислить длины сторон треугольника. 21*. За круглым столом сидят л человек. Разрешается лю- любых двух людей, сидящих рядом, поменять местами. Какое наи- наименьшее число таких перестановок необходимо сделать, чтобы в результате любые два соседа остались бы соседями, но сидели бы в обратном порядке? XLV ОЛИМПИАДА A982 г.) VII класс 1. Петя купил в магазине «Машины Тьюринга и другие вы- вычислительные устройства» микрокалькулятор, который может выполнять следующие операции: по любым числам х w у он вы- вычисляет х-\-у, х—у и — (при хфО). Петя утверждает, что он может возвести любое положительное число в квадрат с по- помощью своего микрокалькулятора, сделав не более 6 операций. А вы можете это сделать? Если да, то попробуйте перемножить любые два положительных числа, сделав не более 20 операций 144
(промежуточные результаты можно записывать, неоднократно используя их в вычислениях). 2. В квадрате ABCD находятся 5 точек. Доказать, что расстояние между какими-то двумя из них не превосходит -»-АС. 3. Петя приобрел в магазине вычислительный автомат, ко- который за 5 к. умножает любое введенное в него число на 3, а за 2 к. прибавляет к любому числу 4. Петя хочет, начиная с еди- единицы, которую можно ввести бесплатно, набрать на автома- автомате число 1981 и затратить наименьшую сумму денег. Во сколько обойдутся ему вычисления? А что будет, если он захочет набрать число 1982? 4. Какое наименьшее количество точек на плоскости надо взять, чтобы среди попарных расстояний между ними встретились числа 1, 2, 4, 8, 16, 32, 64? VIII класс 5*. Упростить выражение 6. Прямоугольник разрезан на пять прямоугольников. Дока- Доказать, что среди полученных пяти найдутся два прямоугольника, один из которых полностью может располагаться внутри другого. 7. Числа 1, 2, 3, .... 1982 возводятся в квадрат и запи- записываются подряд в некотором порядке. Может ли полученное многозначное число быть полным квадратом? 8. Каждая диагональ выпуклого пятиугольника параллельна одной из его сторон. Доказать, что отношение каждой диагонали к соответствующей стороне равно — (д/5 + 1). п 9*. Считая известной формулу 13 + 23 + - + =("(+П) . доказать, что для различных натуральных чисел а\, аг, ---, ап справедливо неравенство (а]-\-а1-\-...-\-а7п)-\-(а51+а2-\-...+а„)'^ * 2 + ... +a3nf. Возможно ли равенство для каких-нибудь ? ( f различных натуральных чисел ait a.i, ..., ап? IX к л а с с 10. Найти все натуральные числа п, для которых число п-2"+1 кратно трем. 11. Найти на плоскости точку, сумма расстояний от которой до четырех заданных точек минимальна. 12. На плоскости отмечены точки с целочисленными коорди- координатами. Доказать, что найдется окружность, внутри которой ле- лежат ровно 1982 отмеченные точки. 145
13. Число Л =0,1+0,02+ 0,003 + ... +л 10-" + ... записано в виде бесконечной десятичной дроби. Доказать, что в получен- полученной записи не встретятся подряд идущие цифры 1982. 14. В выпуклом четырехугольнике две стороны равны 1, а другие стороны и обе диагонали не больше 1. Какое максималь- максимальное значение может принимать периметр четырехугольника? X класс 15. а) Доказать, что если из некоторой точки внутри пра- правильного тетраэдра все его ребра видны под одинаковыми углами, то эта точка — центр описанной около тетраэдра сферы. б) Существуют ли вне тетраэдра точки, из которых все его ребра видны под равными углами? Примечание. Если точка лежит на ребре или его про- продолжении, то считается, что из нее это ребро видно под углом л или 0 соответственно. 16. а) а, Ь, с — длины сторон треугольника. Доказать, что a< + b< + c<-2{a2b2 + a2c2 + b2c2)+a2bc+b2ac + c2ab^0. б) Доказать неравенство из пункта а) для любых неотрица- неотрицательных a, b и с. 17*. Петя приобрел в магазине «Машины Тьюринга и другие вычислительные устройства» микрокалькулятор, который может по любым действительным числам х и у вычислить ху+х +у +1 и не имеет других операций. Петя хочет написать «программу» для вычисления многочлена 1 -(-д--|-лг2 +... -f-д:'982. Под «про- «программой» он понимает последовательность многочленов /t (x), ..., fn(x) такую, что /| (х) = х и для любого 1 = 2, ..., л /,-(*) = с,- или fi(x)=fi(x)-fk{x) + fk(x)+fj(x)+l, где /<i, k<i, причем fn(x) = = 1+*+ ..+*1982. а) Помогите Пете написать «программу». б) Сумеете ли Вы написать «программу», еСли калькулятор имеет только одну операцию ху-{-х-\-у? 18. Найти все такие натуральные л, для которых числа — и -^тту выражаются конечными десятичными дробями. 19*. Внутри правильного шестиугольника находится другой правильный шестиугольник с вдвое меньшей стороной. Доказать, что центр большого шестиугольника лежит внутри малого шести- шестиугольника. XLVI ОЛИМПИАДА («983 г.) VII класс 1. Найти все пары целых чисел (х, у), удовлетворяющих урав- уравнению 146
2. Белая плоскость произвольным образом забрызгана чер- черной тушью. Доказать, что для любого положительного / сущест- существует отрезок длины /, у которого оба конца одного цвета. 3. Найти наименьшее натуральное число, начинающееся с цифры 4 и уменьшающееся в четыре раза от перестановки этой цифры в конец числа. 4. Двум друзьям необходимо попасть в соседний город. У них есть один велосипед, на котором может ехать только один человек. Каково минимальное время, за которое оба могут добраться до города (считая по последнему прибывшему), если скорости пешеходов и\ и «2, их скорости на велосипеде v\ и V2, расстояние до города равно S (они могут возвращаться и остав- оставлять велосипед друг другу) ? 5. Существует ли пятиугольник со сторонами 3, 4, 9, 11 и 13 см, в который можно вписать окружность? VIII класс 6. Доказать, что при любых х>л}2 и y>i/2 выполняется неравенство л:4—х3у-\-х2у2— ху3-\-у* > х2 -{-у 7. На сторонах треугольника ABC вне его построены пра- правильные треугольники АВС\, ВСА\ и САВ\. Доказать, что 8. Может ли квадрат какого-либо натурального числа начи- начинаться с 1983-х девяток? 9. В вершинах правильного 1983-угольника расставлены чис- числа 1, 2, ..., 1983. Любая его ось симметрии делит числа, не ле- лежащие на ней, на два множества. Назовем расстановку «хоро- «хорошей» относительно данной оси симметрии, если каждое число одного множества больше симметричного ему числа. Существует ли расстановка, являющаяся «хорошей» относительно любой оси симметрии? 10. На окружности выбрано пять точек Аи Ai, A3, Л4, Н. Обозначим через /г,; расстояние от точки Н до прямой АЛ/. Доказать, что /г|2*/гз4=Л|4-/г2з. IX к л а с с 11. Доказать, что при любой расстановке знаков « + » и « —» у нечетных степеней х выполнено неравенство {х—произвольное действительное число, а л — натуральное). 12. Три окружности радиусов 3, 4, 5 внешне касаются друг друга. Через точку касания окружностей радиусов 3 и 4 прове- проведена их общая касательная. Найти длину отрезка этой касатель- касательной, заключенной внутри окружности радиуса 5. 147
13. Доказать, что 11983 + 2|983 + ...+19831983 делится на Ц И 983 14. Двадцать городов соединены 172-мя авиалиниями. До- Доказать, что, используя эти авиалинии, можно из любого города перелететь в любой другой (быть может, делая пересадки). X класс 15. Пусть Ли В\, С\ — точки, в которых окружность, вписан- вписанная в треугольник ABC, касается сторон ВС, АС и АВ соответст- соответственно. Известно, что АА\ = ВВ\ = СС|. Доказать, что треуголь- треугольник ABC правильный. 16. Доказать, что 4т —4" делится на 3* + | тогда и только тогда, когда т — п делится на 3*. Решить задачу: а) при /г = 1, 2, 3; б) при произвольном k. 17. На доске после занятия осталась запись: «Вычислить = cos 5* + *cos 4* + *cos 3x-f-*cos 2*-f-*cos x-\-*». Увидев ее, сту- студент мехмата сказал товарищу, что он может вычислить эту сум- сумму, даже не зная значений стертых с доски коэффициентов (вместо них в нашей записи *). Не ошибается ли он? 18. В пространстве выбрано 8 точек, никакие 4 из которых не лежат в одной плоскости. Проведено 17 отрезков, у каждого из которых оба конца лежат в упомянутых точках. Доказать, что: а) отрезки образуют хотя бы один треугольник; б)* треуголь- треугольников на самом деле не меньше четырех 19. За круглым столом сидят 13 богатырей из k городов, где 1</г<13 Каждый богатырь держит в руке золотой или серебряный кубок, причем золотых кубков тоже k. Князь повелел каждому богатырю передать свой кубок соседу справа и повто- повторять это до тех пор, пока какие-нибудь два богатыря из одного города оба не получат золотые кубки. Доказать, что желание князя всегда будет исполнено XLVII ОЛИМПИАДА A984 г.) VII класс 1. Назовем автобусный билет счастливым, если сумма цифр его номера делится на 7. Могут ли два билета подряд быть счастливыми? 2. Дорожки в зоопарке образуют равносторонний треуголь- треугольник, в котором проведены средние линии. Из клетки сбежала обезьянка. Ее ловят два сторожа. Смогут ли они поймать обезьян- обезьянку, если все трое будут бегать только по дорожкам, скорость обезьянки и скорости сторожей равны и они видят друг друга? 148
3. Покупатель взял у продавца товара на 10 р. и дал 25 р. У продавца не нашлось сдачи, и он разменял деньги у соседа. Когда они расплатились и покупатель ушел, сосед обнаружил, что 25 р. фальшивые. Продавец вернул соседу 25 р. и задумался. Какой убыток понес продавец? 4. Из бумажного треугольника вырезали параллелограмм. Доказать, что площадь параллелограмма не превосходит поло- половины площади треугольника. 5. На шахматной доске 20X20 стоят 10 ладей и один король. Король не стоит под шахом и идет из левого угла в правый верх- верхний по диагонали. Ходят по очереди: сначала король, потом одна из ладей. Доказать, что при любом начальном расположении ладей и любом способе маневрирования ими король попадет иод шах. VIII класс х3 6. Решить уравнение —==-\-х —4 = 0. V4— ХГ 7. Каждые две из шести ЭВМ соединены своим проводом. Ука- Укажите, как раскрасить каждый из этих проводов в один из пяти цветов, чтобы из каждой ЭВМ выходило пять проводов разного цвета. 8. Доказать, что сумма расстояний от центра правильного семиугольника до всех его вершин меньше, чем сумма расстояний до них от любой другой точки. 9. Сумма пяти неотрицательных чисел равна единице. Дока- Доказать, что эти числа можно расставить по окружности так, что сумма всех пяти попарных произведений соседних чисел будет не больше —. О 10. Разрежьте квадрат на 8 остроугольных треугольников. 11. Является ли четным количество всех 64-значных нату- натуральных чисел, не содержащих в записи нулей и делящихся на 101? IX класс 12. У треугольной пирамиды три боковые ребра равны и пло- площади трех боковых граней равны. Доказать, что в ее основании лежит равнобедренный треугольник. 13. Имеются провода двенадцати цветов. Можно ли 13 прибо- приборов соединить попарно проводами так, чтобы от каждого из них выходило 12 проводов разных цветов? (Ср. с задачей 7.) 14. Какова минимальная ширина бесконечной полосы, из ко- которой можно вырезать любой треугольник площади 1? 149
15. По окружности расставлено л неотрицательных чисел, сумма которых равна 1. Докажите, что сумма л попарных произ- произведений соседних чисел не больше —. (Ср. с задачей 9.) 16*. В прямоугольнике размерами 3X4 расположены четыре точки. Докажите, что среди них найдутся две, расстояние между 25 которыми не превосходит -^-. 17. Существуют ли три отличные от нуля цифры, с помощью которых можно записать квадраты бесконечного числа различ- различных целых чисел? X класс 18. Доказать неравенство (не используя калькуляторов, таб- таблиц и т. п.) sin I <log31/7- 19. Жюри олимпиады решило по ее результатам поставить в соответствие каждому участнику натуральное число таким обра- образом, чтобы по этому числу можно было однозначно восстановить баллы, полученные участником за каждую задачу, и чтобы из каж- каждых двух школьников большее число соответствовало тому, кто набрал большую сумму баллов. Помогите жюри решить эту за- задачу! 20. Решить в целых числах уравнение 19л:3 — 84у2= 1984. 21. В некотором царстве, в некотором государстве было вы- выпущено неограниченное число монет достоинством в п\, л2, лз, «4, ... копеек, где П\ <Сni<С«з<<С... — бесконечная после- последовательность, состоящая из натуральных чисел. Доказать, что эту последовательность чисел можно оборвать: найдется такое число N, что любую сумму, которую можно уплатить без сдачи выпущенными монетами, на самом деле можно уплатить только монетами достоинством в п\, п2, «з nN копеек. 22. Квадрат разрезан на остроугольные треугольники. Дока- Доказать, что их не меньше восьми. (Ср. с задачей 10.) 23*. Треугольное сечение куба касается вписанного в куб шара. Доказать, что площадь этого сечения меньше половины площади грани куба. XLVIII ОЛИМПИАДА A985 г.) VII класс 1. Найти все значения х и у, для которых ху-\-\=х-\-у. 2. Даны 5 различных положительных чисел, которые можно 150
разбить на две группы так, чтобы суммы чисел в этих группах были одинаковыми. Сколькими способами это можно сделать? 3. Длины а, Ь, с, d четырех отрезков удовлетворяют нера- неравенствам 0<a<6<c<d, d<.a-\-b-\-c. Можно ли из этих от- отрезков сложить трапецию? 4. В центре квадрата сидит заяц, а в каждом из четырех углов по одному волку. Может ли заяц выбежать из квадрата, если волки могут бегать только по сторонам квадрата с макси- максимальной скоростью, в 1,4 раза большей максимальной скорости зайца? 5. В магазин привезли цистерну молока. У продавца имеются чашечные весы без гирь (на чашки весов можно ставить фляги), а также три одинаковые фляги, две из которых пусты, а в третьей налит 1 л молока. Как отлить в одну флягу ровно 85 л молока, сделав не более 8 взвешиваний? (Предполагается, что во флягу помещается больше 85 л. Под взвешиванием понимается сле- следующая операция: на одну чашку весов ставится фляга с мо- молоком, на вторую — пустая и в пустую флягу доливается столько молока, чтобы весы пришли в равновесие.) VIII класс 6. Найти все значения х, и, z, удовлетворяющие равенству {y y y 7. Числа щ, аг, ..., ciges представляют собой переставленные в некотором порядке числа 1, 2, ..., 1985. Каждое число Ok умно- умножается на его номер k, а затем среди всех полученных 1985-и произведений выбирается наибольшее. Доказать, что оно не меньше 9932. 8. На клетчатый лист бумаги положили бумажный квадрат, площадь которого равна учетверенной площади клетки. Какое наименьшее число узлов может накрывать этот квадрат? (Узел — это точка пересечения линий бумаги; если узел лежит на границе квадрата, он считается накрытым.) 9. За дядькой Черномором выстроилось чередой бесконечное число богатырей. Доказать, что он может приказать части из них выйти из строя так, чтобы в строю осталось бесконечно много богатырей и все они стояли по росту (не обязательно в порядке убывания). 10*. Доказать, что если длина каждой из биссектрис треуголь- треугольника больше 1, то его площадь больше —. V3 IX класс 11. Найти все значения х, у, z, удовлетворяющие равенству 151
12. В некоторой стране 1985 аэродромов. С каждого из них вылетел самолет и приземлился на самом удаленном от места старта аэродроме. Могло ли случиться так, что в результате все 1985 самолетов оказались на 50 аэродромах? (Землю можно считать плоской, а маршруты — прямыми. Все попарные расстоя- расстояния между аэродромами предполагаются различными.) 13. Сколько узлов клетчатой плоскости накрывает квадрат размером 2X2, если их не менее 7? (См. замечание к задаче 8; сторона клетки равна 1.) 14. Доказать, что в любой группе из 12 человек можно вы- выбрать двоих, а среди оставшихся 10 человек еще пятерых так, чтобы каждый из них удовлетворял следующему условию: либо он знаком с обоими выбранными вначале, либо не знаком ни с одним из них. 15*. (Задача Эйлера.) Доказать, что любое число 2" при «^3 можно представить в виде 2п = 7х2-\-у2, где х и у — нечетные числа. X к л а с с «„ ,-. х — 49 . х — 50 49 . 50 16. Решить уравнение — 1—-_=—_+- 50 ' 49 х — 50 ' х — 49 " 17. См. задачу 3. 18. Назовем «сложностью» данного числа наименьшую длину числовой последовательности (если такая найдется), которая на- начинается с нуля и заканчивается этим числом, причем каждый следующий член последовательности либо равен половине преды- предыдущего, либо в сумме с предыдущим составляет 1. Среди всех чисел вида ^, где т = 1, 3, 5, ..., 250— 1, найти число с наиболь- наибольшей «сложностью». 19. Даны 1985 множеств, каждое из которых состоит из 45 элементов, причем объединение любых двух множеств содержит ровно 89 элементов. Сколько элементов содержит объединение всех этих 1985 множеств? 20. Доказать, что если расстояния между скрещивающимися ребрами тетраэдра равны соответственно h\, /12, h3, то объем тетраэдра не меньше —Л1Л2Л3. О
ЧАСТЬ II РЕШЕНИЯ, УКАЗАНИЯ, ОТВЕТЫ 1. V = -J__ м/с, L=-^- 1г — м '2 —' 1.02. Зафиксируем одну вершину А на первой (произвольной) из данных прямых и повернем вторую прямую относительно А на 90°. Точка В пересечения повернутой прямой с третьей прямой и будет второй вершиной квадрата. 1.03. Указание. Угол а при вершине удовлетворяет ра- равенству sin -^- = — cos a. 1.05. Из свойств вписанных углов вытекает, что точки, из которых диагональ видна под фиксированным углом, лежат на сфере. Для того чтобы этот угол был наименьшим, требуется, чтобы радиус сферы нельзя было увеличить, не «соскользнув» с куба. Отсюда ясно, что искомые точки — вершины куба, не принадлежащие рассматриваемой диагонали. 1.06. Одно, так как если х + у = 2, то ху^.1. 1.07. Если ЬфЪ, то решений нет. При 6 = 0 имеем *—у. 1.08. п2Bп2-\). 1.09. 30 способов. 1.10. Число способов равно (п — 2)-\-{п— 3) + —+ 1- Действи- Действительно, если x + y + z = n, то х может быть равным 1, 2, 3, .... п—2; y + z —« — х и при фиксированном х у нас есть п—х— 1 возмож- возможностей для значений у и z. 2.01. 7744 =882. 2.03. N= — log2 Iog2~\p/T..y2 (N радикалов). 2.04. Указание. Построить точку Р', центрально-сим- центрально-симметричную Р относительно центра круга; на РР' как на хорде построить дугу величиной л — а (а — данный угол); точка пере- пересечения этой дуги с окружностью — один из концов искомого диаметра. 2.06. Указание. Выразить х5 + у5 через х+у и ху. 2.07. Указание. Докажите, что для всех треугольников пе- периметра 2р, построенных согласно условию задачи, будет вне- вписанной одна и та же окружность, вписанная в данный угол и касающаяся его сторон на расстоянии р от вершины. 2.09. 282 = 784 способа. Указание. Если 106 = B°' -5Р')Х -5р2)-Bаз.5Рз), то а1 + а2 + а3 = 153
РНС. д. 2.10. Число способов четно и заключено от 2 до 14. 3.01. @, 0, а), @, а, 0), (с, 0, 0). Указан и е. Воз- Возведя в куб первое уравнение и вычтя из него третье, полу- получим: S(x+y)(x + z){y + z) = 0. Если х-\-у = 0, то z = a, и тог- тогда из второго уравнения полу- получаем: х=у = 0. Аналогично находятся остальные решения. 3.03. Обозначим отрезки че- через АВ и CD соответственно. Построим параллелепипед на век- векторах АВ, АС и AA' = CD. Тогда очевидно, что: а) высота параллелепипеда равна расстоянию между прямыми и потому не зависит от положения отрезков; б) все основания паралле- параллелепипедов при разных положениях отрезков равны; в) объем тетраэдра равен —объема параллелепипеда (см.. рис. 9). 3.05. 30 (число вершин додекаэдра + число плоскостей, пер- перпендикулярных его большим диагоналям). 24 4.01. Указание. Две последовательные центральные сим- симметрии относительно точек X и Y есть параллельный перенос на вектор 2XY. 4.02. Указание. Если m — 1 плоскостей уже проведены, то гп-я плоскость добавляет столько частей, на сколько эта пос- последняя разрезана прямыми ее пересечения с пг — 1 плоскостями. Тем самым наша задача сводится к аналогичной, но более прос- простой: на сколько частей разделяют плоскость k прямых {k = \,2 п — 1)? 4.04. 686 чисел. Вычеркнем из 999 чисел, меньших 1000, [999П —j = 199. Далее вычеркнем числа, крат- кратные 7, их Г9991 оо [з5~]=^о чисел, одновременно кратных 5; они будут вычеркнуты дважды. Итого, нами вычеркнуто 199+142 — 28=313 чисел- осталось 999 — 313=686 чисел. Замечание. Если требуется найти только приближенный ответ, его можно искать по формуле (попробуйте доказать ее сами): Л^,Ш0.(.-!)(,_-1)=|.,000=685.7... Это полезно, если нужно найти число чисел, не делящихся на 3, 5, 7, 11, 13 154 = 142. Но среди чисел, кратных 7, имеется
5.01. При Ь = 0 решение одно: x=y=z = 0. При решений нет. 5.04. Указание. Обозначьте y=^Ja-\-x; тогда -\ja—у=л\ Возведя эти уравнения в квадрат и сложив их, получим: (х+у)(х~у+1)=0. 5.05. Опишем окружность около данного треугольника. Про- Продолжение биссектрисы пересекается с дугой окружности, стя- стягиваемой основанием треугольника, в ее середине. Отсюда легко вытекает требуемое утверждение. 5.06. (a2 + a + \){as-a7 + as—a4 + a3 — a+\). Указание. Разложите сначала двумя способами на множители выражение а'5-1. 5.07. Указание. Замените четные коэффициенты на 0, а не- нечетные — на 1. 5.08. Указание. Предположив, что хорда CD уже прове- проведена, проведите дополнительно хорду СК, параллельную прямой АВ, и найдите дугу KD. 5.09. Остаток 5. Указание. Если п — г кратно 6, то 10"—10' кратно 7. 5.10. Искомая сумма отрезков равна сумме расстояний от точки Р до всех ребер основания, умноженной на тангенс угла наклона грани к основанию. Но сумма длин всех перпендикуля- перпендикуляров, опущенных из точки Р на стороны правильного многоуголь- многоугольника, вдвое превосходит отношение площади многоугольника к его стороне, т. е. постоянна. 5.11. На 30 частей. Указание. Сведите задачу к анало- г, . п(п2 — Зл+8) гичнои для окружностей на плоскости. Для п сфер: — —¦—- частей. 6.01. S(b-c){c—a){a-b). 6.02. 35 суток. 6.03. 24 нуля (столько раз 5 встречается в качестве со- сомножителя в числе 100!). 6.06. Цифра 7. 6.07. 7 решений. Указание. Если данные четыре точки не лежат на одной окружности, то непременно часть из них будет лежать вне искомой окружности, а часть — внутри. Число реше- решений равно числу разбиений данных четырех точек на два непустых подмножества. 6.09. Пусть abc=a\+b\ + c\. Поскольку 7!=5040> 1000, среди цифр а, Ь, с нет цифр больше 6. Среди них нет также цифры 6, так как в противном случае abc>6\=720. Дальнейший разум- разумный перебор показывает, что единственное подходящее число: 145=1! + 4! + 5!. 6.10. N = 121 -64 = 7744. 6.12. В каждой точке паркета должны сходиться 4 четырех- четырехугольника, повернутые разными своими углами. 6.13. -i-A422— 142)+ 142= 10153. 155
6.14. Число точек самопересечения равно максимальному целому п, для которого «а< 180°. 6.15. 300! > 100*". 6.16. Указание. Докажите, что высоты треугольника O1O2O3 — перпендикуляры к серединам сторон треугольника ABC. 6.J7. ^(^-+^1 п V а а а2 а3 а„ 'а, ) ^ V а2 а3 а, 6.18. 2857 чисел. Указание. Остатки от деления 2х— хг на 7 периодически повторяются через 21. 7.02. 523152 и 52365G. Указание. Разделите 524000 на 504 = 7-8-9 с остатком. 7.04. Часть дуги окружности, построенной на ОР, как на диаметре. 7.05. n(n+l) ( )) 7.08. Пусть Р (хо) = 0. Если х0 четно, то разность Р (х0) — Р @) также четна,— противоречие. Если же хо нечетно, то Р (х0) — — РA)= — РA) должно быть четным, а РA) нечетно,—снова противоречие. Следовательно, целых корней многочлен Р (х) не имеет. 7.10. *=—2 или *>2. 7.П. 31-2+1 = 63 корня. Указание. Постройте графики функции y = sinjt и у = ^— . Затем воспользуйтесь приближен- приближенным равенством ?s31,8... 7.13. На три части. Указание. Из центра вписанной окружности опустить перпендикуляры на стороны треугольника. 7.15. а = 8 и а=12. Указание. Сделать подстановки х = а и х = 10. 7.16. Указание. Простое число р>3 имеет вид 6«±1, и его квадрат равен 12« C«2± 1J+ 1. 7.17. Указание. Точки Н\, Нг, #3 лежат на описанной окру- окружности. 7.19. Указание. Круг наименьшего радиуса, содержащий все точки, на своей границе содержит либо две диаметрально противоположные точки, либо три точки в вершинах остроуголь- остроугольного треугольника. Поэтому г^1. 7.20. (а, Ь, с) = C. 1, 2); (-3. - 1, -2); A, 2, - I); (- 1, 1, -2). (См. 7.J5.) 7.21.@:0); A;0); @; 1); B; 1); A;2); B; 2). Указание. Преобразуйте уравнение к виду (х—IJ-\-(у—\J-\-(х — уJ = 2. 7.22. Некоторая окружность, центром которой служит середи- середина общего перпендикуляра к данным прямым. Указание. Исследуйте сначала случай пересекающихся прямых. 7.23. Указание. Сумма квадратов медиан к катетам в 5 раз больше квадрата гипотенузы. 156
8.01 и 8.06. Частное a—b. 8.02. Указание. Заменить каждое число на — 2л 8.03. 29, 38, 47, 56, 65, 74, 83, 92. 8.07. 625 и 376. Указание. {Nk — N)](N2 — N)j 1000. 8.08. Указание. Множество решений первого и второго уравнений есть соответственно пара прямых, пересекающихся в начале координат, и окружность, центр которой скользит (при изменении параметра а) вдоль оси абсцисс. 8.09. Длина этого отрезка равна разности между гипотенузой и меньшим катетом. 8.10. 1 769 580. Указание. Подсчитайте отдельно сумму единиц, десятков, сотен и тысяч. 8.11. 240 сторон. 8.12. Указание. Провести через точку Р прямую, парал лельную АВ. 8.14. (х.и) = {6. -21); D, 15); G, -12); C,6); (8, -9); B,3); 11, —6);. <-1,0); A4,-5); (-4,-1); B3,-4); (—13,-2). Указание. Переписать уравнение в виде (х — 5) (у + 3) = — 18 8.16. Рассмотрите еще дополнительно треугольник АВ'С, центрально-симметричный треугольнику ABC относительно вер- вершины А (окружность катится по стороне ВС). Тогда АВ' и АС высекут на окружности дугу, равную дуге, высекаемой отрезками АВ и АС. Но угол А равен полусумме этих двух дуг, а с другой стороны, он равен 60°. 9.01. 3 угла. Указание. Сумма всех внешних углов рав- равна 360°. 9.02. Указание. Сделать поворот на 60° по часовой стрел ке вокруг точки В. 9.03. Число 1946. Указание. Частное от деления в обоих случаях должно равняться 112 — 98=14. 9.04. х, = 1, х2 = 2, *з = 3, дг4 = 4, х5= — 4, jc6=—3, *?=— 2, jcb= — 1. Указание. Сложите все уравнения, получив утроен- утроенную сумму всех неизвестных. Далее сложите первое, четвертое и седьмое уравнения и получите хi == 1. Остальные неизвестные находятся аналогично. 9.05. Указание. Данное произведение Р (х) удовлетворяет равенству Р {х) = Р (— х). 9.07. Указание. Проведите через А вторую прямую; из конца исходного отрезка проведите прямую, параллельную сто- стороне рассматриваемого угла. 9.08. Указание, п2- х—— sin 2x 9.10. Производная функции у = ¦%-?¦ равна —5 5 и при ?( 0, ~-\ положительна, так как 2*>sin2*. 9.11. 7 или 14. 157
9.12. Указание. Данное выраже- выражение преобразуйте к виду (х — 2у) (х — у) X Х{х + у){х + 2у){х + 3у). Полученные со- сомножители, легко видеть, попарно раз- различны. Но число 33 нельзя разложить более чем на 4 различных сомножителя. 9.13. х=-^-(ОА + ОВ). Рис |Q 9.14 и 9.20. 1°. Обозначим через а„ дорогу, выходящую из пункта Ап. Если машина т задерживается на перекрестке Рпт, то угол а„ ближе к 90°, чем ат (дорога ап идет круче, чем дорога ат\ рис. 10). 2°. Пусть ат — самая крутая из всех дорог, пересекающих ап. Если ат круче, чем а„, то машина m не может быть задержана раньше точки Р„т (доказывается от противного). Самая крутая дорога аи (см. условие), и поэтому машина № 14 нигде не будет задержана. Все машины, путь которых пересекается с ati, бу- будут задержаны (это машины № 1, 2, 3, 4, 6, 7, 8, 10, 12, 13, 18, 19, 22, 27, 28, 29, 30). Дальнейший аналогичный анализ показы- показывает, что нигде не будут задержаны только машины № 14, 23, 24 и этот ответ не зависит от расстояний между соседними пунк- пунктами. 9.15. Если общее число маршрутов равно N, а на маршруте я = 3 остановок, то N = n(n—1)-|-1=7. Пример из 7 маршрутов постройте сами. 9.16. 10 очков. 9.17. Найдется; первым из них будет член с номером 7501. Указание. Рассмотрите последовательность а*, составлен- составленную из последних четырех цифр чисел исходной последователь- последовательности (а*<104), и воспользуйтесь принципом Дирихле. 9.18. Указание. Воспользуйтесь тем, что множество S не изменится, если произвольным образом параллельно пере- перенести отрезки АВ, CD и EF вдоль сторон треугольника PQR. 9.19. 8 остановок. Указание. Докажите сначала, что каж- каждый маршрут имеет одинаковое число п остановок и через каждую остановку проходит ровно п маршрутов. Получите отсюда урав- уравнение п (п — 1) + 1 = 57. Замечание. Эта задача возникла из нужд проективной геометрии. А именно пусть остановки — это точки на плоскости, а маршруты — прямые. Тогда условие 1) означает, что через любые две точки проходит прямая, а условие 2) — что любые две прямые пересекаются ровно в одной точке (что неверно в обычной геометрии, но верно в проективной). Теперь ясно, что основное отличие получающейся «геометрии» от обычной состоит в том, что число точек на плоскости не бесконечно, а равно 57. Оказывается, такая геометрия тоже существует; в ней каждая прямая имеет по восемь точек. 10.01. Остаток 6. Указание. Уменьшив каждое слагаемое на 1, получаем, что каждое новое слагаемое делится на х—1 158
10.03. Член Jt18 будет отсутствовать, так как 18 нельзя представить в виде суммы пятерок и семерок. Коэффициент при хи будет равен 20-^^=3420. 10.05. Эти окружности равны описанной окружности. 10.06. Свободный член полученного многочлена Р (х) равен р @)=((...(D-2J-2J — ...J-2J=:...=D-2J = 4. Используя это и обозначив через Ak и В* коэффициенты при х и х2 соответствен- соответственно, можно доказать, что Ак = 4Ак-\ и Bk = Al-i + 4?*_i. Так как А\ = —4 (поскольку (х — 2J=jt2 — 4х + 4), то отсюда легко сле- следует (по шагам), что Ак=—4*. Подставляя в формулу для В*, выраженную через Ak-i, Ak-2, .... А\, В\, значения Z?i = l, Ai = —4, ..., получаем: Вк = 4 -т-г= * 10.07. Указание. Достаточно найти число, не имеющее с остальными пятнадцатью числами общих множителей 2, 3, 5, 7, 11 и 13. 10.08. 12 + 22 + ... + 82 = 204. 10.09. Заменив х на (—х), получим, что коэффициенты при четных степенях х не изменятся. Значит, коэффициенты при а:20 у данных многочленов те же, что и у многочленов A +*2 + +JC3I и A — х2 — л:3I000. Ясно, что первый из новых много- многочленов имеет больший коэффициент при х20. 10.10. 0,89001. Указание. Можно ограничиться первыми пятью сомножителями. 10.11. Две касательные плоскости к цилиндру радиуса d и осью АВ, проходящие через точку М. 10.12. Указание. Сначала положите на чашку весов по одному кубику; затем, положив на одну чашку оба эти кубика, на вторую кладите поочередно по паре из оставшихся кубиков. 10.13. 31 цифра. Указание. Поскольку 210 = 1024, то 2100 немного больше, чем 100010, т. е. имеет 31 цифру. 10.15. Указание. Если многоугольник можно разрезать на параллелограммы, то для каждой его стороны найдется дру- другая, ей параллельная. Поэтому число сторон должно быть четным. 10.16. Указание. Среди выбранных чисел найдутся два таких, частное которых есть степень двойки. 10.18. Указание. Докажите, что в каждой строке четное число встречается уже на одном из первых четырех мест. 10.20. Указание. Рассмотрите наибольшие нечетные де- делители выбранных чисел. 10.21. Указание. Докажите, что скрещивающиеся ребра равны. 11.01. {2; 4; 4}, {2; 3; 6), C; 3; 3}. 11.03. У к а з а н и е. Применим индукцию. Проведем еще одну прямую к имеющимся. Затем все цвета областей, 159
лежащих по одну сторону от нее, поменяем на противополож- противоположные — условие задачи будет по-прежнему выполнено. Тем самым индукционный шаг завершен. 11.04. Если первое число равно т, то второе равно 2 (ат~1 -f- + ат-2 + - + а+1). где а = 2". 11.05. Указание. Воспользуйтесь тем, что -\[\0^3,16>л. 11.06. Указание. Рассмотрите сначала частные случаи, когда точка А' лежит на ребре и когда она лежит на грани пира- пирамиды ABCD. 11.07. Длина пути из точки А в точку В, идущего по ука- указанным в условии правилам, равна разности длин tA и tD каса- касательных AM и ВМ, проведенных к окружности в точках А и В. Поэтому длина общего пути по заданной ломаной будет равна {1а—1в) + (*в — 1с) + -+{1к—ч) + A1.—*л) = Ъ, что и требовалось доказать. 11.08. B; 4), D; 2). У к а з а н и е. Докажите сначала, что х и у — степени одного и того же числа. П.10. Нет, так как точка, симметричная одному центру сим- симметрии относительно другого, тоже центр симметрии. 11.11. Указание. Соедините середину диагонали с задан- заданными серединами сторон и воспользуйтесь неравенством тре- треугольника. р р+1 11.13. jt=(?±IJ , y=(?±Dj . Указание. Пусть y = kx (k— рациональное число), тогда из данного уравнения следует, что x = kk-1, y = kk~l. Пусть -!_ = -?-, тогда к 1 q = , тогда *=(?±!) , Р+Ч y=(EJ Но так как х и у—рациональные, а р и q — взаимно простые, то из чисел р и p + q должен извлекаться корень q-й степени. Поскольку при q~^2 и р = пч справедливо нера- неравенство пч<р + <7< (я+ 1)'. то 9 = 1- 11.14. Радиус наибольшей окружности ^—. Эта окружность вписана в максимальное по площади сечение куба, являющееся правильным шестиугольником. П. 15. 19 801 решение. 11.16. 4 луна (например, лучи, проведенные из центра тет- тетраэдра в его вершины). 11.17. Указание. Разложить вектор движения луча на три составляющие, перпендикулярные зеркалам. После каждого отражения соответствующая составляющая меняет знак. 12.01. Указание. Разложите 26 460 на множители и до- докажите, что заданное выражение делится на 5-72 и на 2г-33. 12.03. Указание. Правая часть равенства всегда делится на более высокую степень двойки, чем левая. 12.04. Пусть А — произвольная точка ломаной, а В — такая 160
точка ломаной, что обе части ломаной, соединяющие А и В, имеют длину — • Докажем, что середина отрезка АВ — искомый центр О круга радиуса —, покрывающего всю ломаную. Дейст- Действительно, если М — произвольная точка ломаной, а ЛГ — цент- центрально-симметричная ей точка относительно О, то АМВМ' — па- параллелограмм. Сумма длин двух его соседних сторон не больше —, следовательно, ММ' <.— . Отсюда OM<L~, что и требовалось доказать. 12.08. Обозначим левую часть уравнения через у, правую — через у\, тогда x=y2i+2ay[+-r^, т. е. х выражается через у\ точно так же, как у выражается через х. Отсюда следует, что параболы у—у{х) и у\—у\ (х) симметричны относительно биссектрисы первого координатного угла. Точки пересечения этих двух графиков лежат на этой биссектрисе (докажите это!), откуда у=х=у\. Отсюда сразу находим оба действи- действительных корня (o<a<-J-) : *,. 2= 12.11. Фишки неизбежно займут те же места. Указание. Если занумеровать 12 полей через 4 на пятое в порядке 1, 6, 11, 4, 9, 2, 7, 12, 5, 10, 3, 8, то одним ходом фишка сдвигается на соседнее место. 12.12. Точки Z в общем случае заполняют^ шестиугольник, стороны которого равны и параллельны сторонам треугольников ABC и DEF. 12.13. Указание. Докажите сначала, что разность масс двух любых гирь (в граммах) четна, а затем — что она делится на любую степень двойки. 12.14. Если расположить в вершинах шестиугольника гири равной массы, то точка пересечения медиан любого из образовав- образовавшихся треугольников будет центром тяжести системы. 12.15. Какие-то два из чисел щ, а\-\-а2, a\-\-a2-\-az, .... ai + -.-+aioo при делении на 100 дают одинаковые остатки. Раз- Разность этих чисел и является искомой суммой. 12.17. Куб 2X2X2. 12.18. Наибольший центрально-симметричный многоугольник с данным центром О, вписанный в треугольник ABC, представ- представляет собой пересечение треугольника ABC и треугольника А'В'С, симметричного ABC относительно О. Таким образом, остается только выбрать точку О так, чтобы это пересечение было возможно большим по площади; для этого надо, чтобы точка О совпала с точкой пересечения медиан треугольника ABC. 12.19. Указание. Докажите, что хотя бы одна из точек lg 2, 2 lg 2, 3 lg 2, ... попадает в один из промежутков (l 6 Заказ 247 161
k = l, 2, .... a M — заданная комбинация цифр, с которой начинается степень двойки. 13.01. Радиус этой окружности равен —-\flO, а центр лежит в центре черной клетки. 13.02. Указание. Отделив последние 6 гирь, поделите оставшиеся на 61 группу по 9 гирь в каждой. В каждой из 62 групп гири разделить на 3 равные по массе группы теперь легко. 13.04. Против большей стороны треугольника лежит больший угол, поэтому (Л — В) (a — fc)>0, (В — С)(Ь — с)>0, (С— А){с—а)> 35=0. Сложив эти неравенства, получаем искомое. 13.05. 25 мин. Указание. Докажите, что при передвижении на 2 км используется второй способ. 13.06. Всегда. 13.07. Может (см. рис. 11). Угол А—октант (трехгранный угол с прямыми плоскими углами), AC = AD=l, AB — 3, А' выбрана достаточно близко к вершине В, например А'В=\. Тогда AB + AC + AD — 5, A'B + A'C + A'D = 1 + 2V5>5. Воз- Возможно и много других примеров. 13.08. Разложим сначала 9 гирь с массами л2, (п+1J, ¦¦-, (га + 8J на три группы так: 1-я группа: {я2, (п + 5J, (л + 7J}; 2-я группа: {(п + 1J, (л + 3J, (л + 8J}; 3-я группа: {(п + 2J, (л + 4J, (п + 6J}. Масса первых двух групп одинакова, а третья группа легче на 18 г. Затем следующие за ними 9 гирь разложим так, чтобы первая и третья группы весили одинаково; наконец, после- последующие 9 гирь разложим так, чтобы вторая и третья группы име- имели одинаковую массу, а первая была легче на 18 г. Сгруппировав затем все первые, все вторые и все третьи группы, получим разло- разложение любых 27 последовательных гирь на 3 кучи равной массы. Осталось эту процедуру повторить 3 раза, и тогда все гири окажутся разложенными требуемым способом. 13.09. 5^x^10. Указание. Выделить под радикалами полные квадраты. 13.11. 13 сторон. Очевидно, сторонами такого многоуголь- многоугольника могут быть самое большее две диагонали, проведенные из данной вершины. Отсюда легко вывести, что число сторон не боль- больше 13. С другой стороны, если, например, исходный 13-угольник правильный, то среди многоугольников разбиения будет 13-уголь- 13-угольник (в центре). Аналогичен ответ для любого многоугольника с нечетным числом сторон. 13.12. Указание. Восполь- Воспользоваться тем, что -1— -о—-7-Х 98 99 1 / 1 \ 99 100 100 V 10/ ' 13.14. Указание. Обозна- Обозначим через kn число способов, ко- 11 торыми можно соединить попарно 162
какие-либо 2п точки окружности п непересекающимися хордами. Докажите, что kn = kn-i + kikn-2 + fakn-3 + ... + kn-2ki+kn-l. Отсюда k-2 = 2, к3 = 5, /г4=14, /г5 = 42 /г,0=16 796. Общий ответ для произвольного п таков: kn = —!—CSn + i. 13.16. Выберем сначала из данной последовательности а\, ..., aioi убывающую подпоследовательность следующим образом: первый ее член — а\, второй — первое из чисел, меньших а\, третий — ближайший член, меньший ач, и т. д. Затем выбро- выбросим из «большой» последовательности эти члены и построим тем же способом вторую убывающую подпоследовательность, затем третью и т. д., пока не исчерпаем все заданные числа. Если одна из наших последовательностей имеет больше 10 членов, то за- задача решена. В противном случае мы имеем не меньше 11 после- последовательностей; докажите сами, что можно выбрать из каждой из них по одному члену так, чтобы они образовывали возра- возрастающую последовательность. (См. задачу 48.09.) 13.17. Указание. Пусть А, В, С и D — вершины данного четырехугольника; К, L, М, N — точки касания, причем К лежит на АВ, L — на ВС и т. д. Докажите сначала, что KL лежит в одной плоскости с диагональю АС и что MN тоже лежит в одной (но отличной от первой) плоскости с АС, а затем, что либо KL и MN обе параллельны АС и, следовательно, параллельны между собой, либо пересекаются с АС в одной и той же точке. (См. также решение задачи 19.19.) 13.18. Можно. Рассмотрим, например, 10 прямых плоскости, никакие две из которых не параллельны и никакие три не пере- пересекаются в одной точке. Будем считать, что прямые — это авто- автобусные маршруты, а их точки пересечения — остановки. При этом с каждой остановки можно проехать на любую другую: если остановки лежат на одной прямой, то без пересадки, а если нет, то с одной пересадкой. Далее, если даже отбросить в этой схеме одну прямую, то все еще останется возможность проехать с каждой остановки на любую другую, сделав в пути не больше одной пересадки. Однако если отбросить две прямые, то одна остановка (точка пересечения этих прямых) уже вовсе не будет обслуживаться оставшимися маршрутами и с нее будет невоз- невозможно проехать на какую-либо другую. 14.03. Первое число меньше второго. 14.05 и 14.09. 3 звена; 4 звена. 14.06. 0,239. 14.07. Равносторонний треугольник со стороной а. 14.08. R>2r. 14.10. Пусть х, у, г — «нормальные» шкалы на данных прямых. Положим теперь / = 10х (это значит, что в точке с ко- координатой, например, 2 мы ставим метку / = 100), /?=10 2у и G=10~z. Если три точки на шкалах лежат на одной прямой, то x + z = 2y, откуда G = 10-'= 10х- КГ2»=/-/?. 6* ' 163
14.11. Это число больше A050K и меньше A050+1K. 14.13. Указание. Докажите по индукции, что эта сумма равна (l+2 + ... + лJ. 14.16. Коэффициент при xli равен (—1). 14.18. Количество чисел от 1 до т, кратных Ь, равно [-^-1 Из условия следует, что ни одно из чисел 1, 2, ..., 1951 не делится сразу на два из чисел а\, ..., ап. Поэтому количество чисел из последовательности 1, 2, ..., 1951, делящихся на одно из чисел Г 1951 I . . Г 1951 I ,„С1 щ, , а„, равно + ••+ и оно не превосходит 1951 „ /1951 ,\ . ./1951 ,\ ,„-. 1951 . . 1951 ^, Отсюда (— 1) + -+(— 1)<1951; __+...+__< < 1951+п<2-1951, и -—1-...+—<2, что и требовалось. а„ ' ' а„ ' г 14.19. Проекция тетраэдра всегда совпадает с проекцией ка- каких-то двух его граней, например ABC и BCD. Пусть К, ~L, M, N — середины сторон АВ, ВС, CD и AD соответственно, тогда легко видеть, что KLMN — квадрат, площадь проекции которого ровно вдвое меньше, чем площадь проекции тетраэдра. Поэтому пло- плоскость искомой проекции должна быть параллельна плос- плоскости квадрата. 14.20. Кривая, отличная от окружности, которая обладает тре- требуемым свойством, состоит из двух равных дуг окружности, со- содержащих по 120°; радиус дуг равен стороне треугольника. По- Подумайте сами, как можно построить другие кривые с тем же свой- свойством (их существует бесконечно много). 14.21. а) Докажем лемму: если в квадрате из 49 клеток отмечены крестиками 24 клетки, то можно из него вычеркнуть 4 строки а\, «г, аз, а» и 4 столбца b\, b2, Ьъ, Ь* так, что на пере- пересечении строки а, и столбца Ь; стоит крестик при всех /=1,2,3,4 Для доказательства нужно показать сначала, что можно вы- вычеркнуть одну строку и о д и н столбец, на пересечении кото- которых стоит крестик (в дальнейшем о такой операции мы будем говорить коротко: «вычеркнуть крестик»), оставив при этом в таблице не менее 15 крестиков. Если в каждой строке и в каждом столбце стоит не более 5 крестиков, то это очевидно (можно вычеркнуть любой крестик); если же в какой-то строке стоит 6 крестиков, то в одном из соответствующих шести столбцов их не более 4 и можно вычеркнуть этот столбец вместе со строкой. Случай 7 крестиков в строке рассматривается аналогично. Далее следует показать, что из таблицы 6X6 с 15-ю крести- крестиками можно вычеркнуть крестик так, чтобы в квадрате 5X5 осталось не менее 8 крестиков, и затем в третий раз вычеркнуть строку и столбец, оставив хотя бы 1 крестик. Тем самым лемма будет доказана Вернемся теперь к задаче: между 7-й и 8-й остановками в автобусе едет н° более 25 человек. Составим квадрат 7X7, 164
занумеровав его строки от 1 до 7 («первые 7 остановок»), а столбцы — от 8 до 14. Точку, стоящую в i-й строке на /-м месте, отметим нуликом, если в автобусе есть пассажир, едущий от О, до Oj (i^7, /^8); остальные точки отметим крестик а- м и. Поскольку в квадрате не более 25 нуликов, там есть 24 крестика, и по лемме в квадрате есть 4 строки и 4 столбца, пересечения которых отмечены крестиками. Их номера и будут номерами нужных восьми остановок. б) Предположим, что в автобусе ехало ровно по одному пас- пассажиру от i-й до j-й остановки для всех i, j, l^.i^j^.10 (а по- последние 4 остановки автобус ехал пустым). Нетрудно убедиться, что при этом условие задачи выполнено (больше всего народу в автобусе — 25 человек — ехало между 5-й и 6-й остановками) и не существует пяти пар остановок, между которыми бы никто не ехал. 15.01. Указание. Углы LMN и ANL равны половине дуги LN, но AANL=AALN<90°. 15.04 и 15.06. Пешеход С должен выйти из N за — часа до выхода А из М. 15.11. Указание. Из рисунка 57 к решению задачи 32.25 видно, что в рассматриваемых треугольниках ABD и BCD поме- помещаются окружности, сумма радиусов которых равна R. 15.12. Указание. Все числа последовательности, начи- начиная самое позднее с а3, не превосходят 81+81+4 = 166. Для чисел от 1 до 166 нужно разумно организовать перебор. 15.14. Указание. Начните с равенства cos arcsin >: = = sin arccos x=^Jl —x2. 15.15. Доказательство по индукции: при я = 2 неравенство очевидно; если оно верно для п, то A — х)"+| + +A+*)"+'<((l-x)"+(l+x)")((l-*)+(l+*))<2n.2=2'I+1. 15.16. Указание. Если А, В, С — точки касания сферы с плоскостями угла, то треугольники OAS, OBS, OCS равны. 15.17. Если а>0, то при очень большом р решений нет вообще. Если а<0, то при большом р меньший корень будет отрицательным (это видно из графика). Значит, а = 0. 15.20. Указание. Пусть окружности, вписанные в тре- треугольники ABC и ACD, касаются диагонали АС в точках К[ и /Сг- Докажите, что расстояние между К\ и Кз равно y-\(AD + BC)-{AB + CD)\. * -,, 15.21 и 15.23. Указание. Если 0<сс<1, то a<Vcc<l- 15.22. Это множество изображено на рисунке 12. 15.24. Указание. ANEAcoANDB 8 и ANADoo &NBF. 15.25. Если А и В — два таких числа рИс 12. 165
и — — целое, то А —в в тоже целое и, как нетрудно пока- показать, делится на 9. Но по- А-В скольку в заведомо меньше 7, то Л — Б = 0. 15.26. я = 100 и я = 198. 15.27. Заметим, что все х,=^=0. Из первого и второго уравнений вытекает, что xi=x3, из третьего и четвер- четвертого: хз — х5 и т. д.; аналогично x2 = xi = ... . Если п нечетно, то xi=xn и из последнего уравнения следует, что jci = ±1; тогда из первого уравнения вытекает х2=х\. Итак, для нечетного п справедливо Х\ = х2= ...=хп-\=хп= ± 1. Если п четно и Х| = а, 1 Рис. 13. Рис. 14. то х2= и мы имеем: 1 а 15.28. Оси цилиндров должны быть перпендикулярны друг другу (см. рис. 13). 15.30. Указание. Из вершины А проведите прямую AR под углом 60° к основанию АС (рис. 14). Она пересечет QC в точке S, которую соединим с точкой Р. Докажите, что AQPS = = AQRP. Отсюда APQS= APQR, а так как их сумма равна 60°, то Z.PQC = 30°. 15.31. Пусть А — самый низкий из высоких, а В — самый высокий из низких. Рассмотрим С, стоящего на пересечении продольного ряда, где стоит Л, и поперечного ряда, где стоит В. Тогда А^С^В, т. е. А не ниже В. 2л 15.32. Обозначим данную функцию через / (х). Тогда \f(x)dx = = 0. Отсюда следует, что если f (х) принимает положительные значения, то она принимает и отрицательные (из непрерывности /). Осталось показать, что / (х) не есть тождественный нуль. 16.02. Указание. Если число 11...11 (п единиц) кратно 33...33 A00 троек), то п должно делиться на 3 и на 100. 16.03. Восставим перпендикуляр / к АВ в точке А; отметим на нем точки С и D. Восставим далее перпендикуляр /' к АВ в точке В, а затем — перпендикуляры к прямой / в точках С и D до их пересечения с /' соответственно в точках С и D'. Обозначим через X точку пересечения прямых AD' и BD и через Y — точку пересечения прямых АС и ВС. Тогда прямая XY пересекает отрезок АВ в его середине. 16.04. n2 + 8n + 15 = (n + 4J—1. 16.06. Указание. Обозначьте радикал, стоящий в знаме- знаменателе, через а. Тогда радикал в числителе равен ^/2-\-а. Осталось доказать требуемое неравенство при 0<Са-<2. 166
16.10. Указание. OOl=-^-(AAi 16.11. Пусть x2m.y2O° + l=f(x).g(y), f@)=a, g@) = b. Под- Подставляя в написанное равенство сначала х = 0 (у— произ- произвольно), затем у = 0 (х—произвольно) и наконец х=у=0, получим: а-е(у)=1, b'f(x)=l, ab = \. Отсюда f (x)X () = 1 Ф 1 +х200у20 —прот ( противоречие с предположением. 16.12. BF\\CE и AC\\EF, т. е. AFEC — параллелограмм (точ- (точнее, ромб). Следовательно, треугольники АСЕ и AFE равно- равновелики. Отрезая от треугольника АСЕ треугольники АЕ\Е и EE\D\ и добавляя их к треугольнику AFE, мы одновременно к оставшемуся треугольнику ACD\ добавляем равновеликие им треугольники C\D\D и DD\E и получаем многоугольник ABB\CC\DED\A, равновеликий четырехугольнику ADtEF (рис. 15). 16.21 и 16.27. Все четные колеса вращаются в одну сторону, а все нечетные — в противоположную. Следовательно, чтобы система колес вращалась, число колес должно быть четным. 16.22. Сумма углов всех N треугольников, на которые раз- разбивается 1000-угольник, равна сумме всех углов 1000-угольника и всех 500 полных углов, т. е. равна 998-180°+500-360°. Зна- Значит, число треугольников равно 998 + 2-500 = 1998. 16.24. Разобьем четырехугольник диагональю на два тре- ab cd угольника; их площади не превосходят -~- и — соответственно. Таким образом, S^— (ab-\-cd); аналогично доказывается, что S^.—(ad-{-bc). Складывая эти неравенства, получаем искомое. 16.25. Пусть известно, что число А =а\а2—ап-\ап{п= 1953) делится на 27; докажем, что В=апа\...ап-\ тоже делится на 27. Для этого достаточно доказать, что 10Б+а„ при делении на 27 дает остаток а„. Но 10Б+а„=а„-10"+Л = а„-99...9+Л + 1953 + а„, а так как 1953-значные числа 99...999 и Л делятся на 27, то все доказано. Даль- Дальнейшее просто: надо перебрасывать по одной цифре из конца числа в начало. 16.26. В первой группе количество мно- многоугольников больше. Дело в том, что любой многоугольник второй группы мож- можно «дополнить» вершиной А\ и получить многоугольник первой группы (обратная операция возможна не всегда, например для треугольника). 16.28. Xk=\ при k четном; Хк= — 1 при k нечетном. Указание. Вычтем из Рис. 15. 167
каждого уравнения предыдущее. Тогда из последнего уравнения находим: jtioo—1, из предпоследнего: xgg = = 1 — 2xioo= — 1 и т. д. Рис. 16. 16.31. Значения трехчлена 4~*2 + ¦\-bx-\-c в точках х\ и х2 равны ^ —о"*2/ и ~тгх%' следовательно, они име- имеют разные знаки. Поэтому один из корней трехчлена расположен между Х[ и х2. 16.33. Искомыми будут четырехуголь- четырехугольные пирамиды АС'СВВ', AC'D'A'B' и ACCDD' (рис. 16). 16.34. {1; 2; 3; ...; п). 16.36. Указание. Воспользуйтесь „_V2=K*,_,+zt)- равенством хп Рис. 17. 2х„_, X Х(*„-1— т/2J. Получите из него, что хп — тД<— (xn_i— -^2). Вто- Вторым соотношением воспользуйтесь до х3о, а первым — с хз[ до *зб- 17.02. Да, будут равны. 17.03. Возможны два ответа: 1014 (при делении на 3 и на 2) и 1035 (при делении на 9 и на 5). 17.04. Не существуют, так как ш2 — п2 нечетно или кратно 4, а 1954 — нет. 17.05. 100, так как 1OOQ + 'Ofe+c <Ю0. а + Ь+с 17.06. Ответ показан на рисунке 17. 17.07. Если повернуть все построенные векторы на 90° и умножить их на 2, то они станут равны векторам, построенным на сторонах многоугольника, и поэтому их сумма будет равна 0. Но тогда и исходная сумма равна 0. 17.11. Условие задачи означает, что / (х0) = f (x0) = 0. Следо- Следовательно, х=Хо — двукратный корень первого уравнения, и потому наш многочлен делится на (x—xof. 17.12. Искомое число равно 99999 78 596061...99100. Из двух чисел с одинаковым количеством цифр больше то, у которого больше первая (слева) несовпадающая цифра. Поэтому сна- сначала надо вычеркнуть 84 цифры слева, оставляя все встре- встречающиеся девятки: получим 99999 505152...99100. Затем вычер- вычеркиваем 15 цифр 5, 0, 5, 1, ..., 5, 6, 5 — следующая цифра будет 7. Далее можно вычеркнуть только еще одну цифру 5 из числа 58. 17.13. Из данных неравенств вытекают следующие нера- неравенства: 168
г —а3), а2 —а3>2 (аз — а4), ..., Сложив их все, получим 0J&0, откуда легко вывести, что а,\ —0,2—0,0 — аз = --- = 0- 17.15. Не существуют. Указание. Построим в пространстве треугольники ABC и BCD со сторонами 8, 10 и 13 (как указано в условии) и общей стороной ВС—13. Зафиксируем плоскость треугольника ABC и будем вращать треугольник BCD вокруг стороны ВС. Легко видеть, что AD максимальна, когда D лежит в плоскости треугольника ABC. Тогда ВС и AD — диа- диагонали параллелограмма ABCD и легко подсчитать, что /Ш< 13. Поэтому при любом расположении точки D, очевидно, AD< 13 и, следовательно, искомого расположения точек А, В, С, D в пространстве не существует. 17.16. Указание. Дискриминант квадратного уравнения D = 4 sin2 (xy) — 4^0, отсюда |sin (ху)\ = 1 и х = — sin (xy). 17.21. Две. Эти буквы надо расставить в узлах сетки в шах- шахматном порядке. 17.22. Данная система обладает тем свойством, что коэф- коэффициент Ok при Xk в k-м уравнении по модулю больше сум- суммы модулей всех остальных коэффициентов. Воспользуемся этим: пусть хт— наибольшее по модулю из чисел x\,...,xj. Тогда в m-м уравнении член атхт больше по модулю суммы всех осталь- остальных. Но так как правая часть равна 0, то хт = 0. Поскольку он наибольший по модулю, то имеется единственное решение: Х{=^Х2 = — =Х7 = 0. 17.23. Ни одной, одна или 7 осей симметрии. Указание. При отражении одной оси симметрии относительно другой снова получается ось симметрии. 17.24. Указание. Если под делителем q написано (+1). то под делителем 2q будет стоять (—1), и наоборот. 17.25. Не смогут. Указание. Докажите, что на каждой прямой сетки возникает свое направление движения. 17.27. Указание. Принять точку О за начало прямоуголь- прямоугольной системы координат, а прямую гп\— за ось Ох; если k\, &2, &з — угловые коэффициенты остальных прямых, то (V (считаем |ОЛ|| = 1), что и требовалось доказать. 17.31. Окружность с центром на 1\. 17.33. Указание. Заменив строгие неравенства в условии на нестрогие, докажите, что сумма трех максимальных чисел может быть равной 100 только в двух случаях: A) когда максимальное число равно 100, а все остальные 99 чисел равны 1 Г\Г\ нулю и B) когда какие-то 9 чисел равны -г—, а остальные рае- О ны нулю. 169
17.34. Не существует. Указание. Переставить числа в порядке возраста- возрастания. 17.35. Может. Указание. По- Построив очередные 5 отрезков, мы увели- увеличиваем число свободных концов на 4. 17.39. Легко видеть, что если \х\^1, то левые части обоих данных уравнений положительны. Поэтому и их полусум- полусумма положительна, т. е. не обращает- обращается в 0. 17.40. В первом классе должны содержаться все числа с нечетным числом двоек, во втором — счетным. 18.02. АВАС=-60°, Z.ACB = 30°. Указание. Докажите, что AB = BD, далее, BD = DC, ABDA=2 Z.BCA, /LBAC = = 2 Z.ACB. Но Z.ВАС + Z.ACB =90°. 18.03. Объединение высоты ВН и дуги АС величиной 120°, которая построена на стороне АС как на хорде (рис. 18). 18.05. Только прямоугольники размером аХ13а. 18.07. Это множество точек заполняет параллелограмм. 18.09. Плоскость, полуплоскость, а также полоса между двумя параллельными прямыми. 18.11. Указание. Представить таблицу в виде суммы двух таблиц: у первой таблицы в /-й строке стоят числа k-(i—l),...,k-(i—1), а у второй — числа 1, 2, 3 k(i=l, 2, ..,, k). 18.12. Кольцо, заключенное между двумя концентрическими окружностями, центр которых — середина отрезка, соединяющего центры исходных окружностей, а радиусы равны полусумме и полуразности радиусов данных окружностей (при равенстве ра- радиусов кольцо превращается в круг). 18.13. A; 0); @; 1). Указание. Из второго уравнения следует, что х^1 и у<С1. Но если jc^I, то *3^1 и тогда из первого уравнения у>0; аналогично xZ^O. Если 0<х<1, то х3>х4, у3^у* и уравнения системы противоречат друг другу. 18.14. Рассмотрим остатки от деления всех чисел щ на р. Их ровно р, и среди них нет нуля (иначе соответствующее а, не было бы простым числом); поэтому есть два числа а, и щ, дающие одинаковые остатки, так что а,-— а, кратно р. Но \щ — — щ\ —k-d, где d — разность прогрессии, /г<р — целое. Отсюда следует, что d кратно р. 18.15. Указание. Пусть Н — точка пересечения АВ с про- продолжением CD. Поскольку точки А и В равноправны, можно считать, что точка Е находится на АН. Докажите, что разность ЕС — ED уменьшается, когда точка Е движется от Я в направ- направлении А. 170
18.19. Можно. Для этого достаточно провести плоскость через точки А, В, С, равноудаленные от точки О. Тогда треуголь- треугольники ОАВ, О АС и ОВС равнобедренные, поэтому все углы при основаниях АВ, ВС, АС острые. 18.20. jc=y = z=O. Указание. Видно, что х3 четно, поэто- поэтому х3 делится на 8. Сокращая на 2, видим, что у3 четно, поэтому у делится на 8. Снова сокращая на 2, получаем таким же образом, что z3 делится на 8. 18.21. Указание. Если f(x)=a — VT(*)<1 пРи достаточно большом х. 18.23. 28 партий. Сначала по олимпийской системе за 25— 1 = =24 партии определяется чемпион. Второй по силе — тот, с кем играл чемпион; их может быть 5. По олимпийской системе еще за 4 партии выявляем чемпиона среди них. 18.24. Можно последовательно получать нужные разбиения для любого числа п прямоугольников, начиная с /г=5, при- прикладывая новые прямоугольники «по спирали», как показано на рисунке 19. 18.25. Положим f (x)=-\/ax2 + bx + c и устремим х к +оо. Легко видеть, что о>0 и f(x+i)+f(x) fCH-D+fM ^ Если х целое, то f (х) и f(x+l) тоже целые, а потому и d==-\Ja целое. Более того, поскольку разность f (x-{-l) — f (х) целая, то при достаточно больших х она обязана равняться своему пре- пределу: f (x+ \)—f (x)—d. Пусть это соотношение верно, начиная с некоторого х0, и у = хо + п; тогда f (y)f=f (xo) + nd, т. е. ay2 + by + с=(f (аго) + ndf=(dy -dxo + f (xo)f. Отсюда уже легко следует утверждение задачи. 18.26 и 18.33. Указание. Сделать инверсию относительно окружности, проходящей через точку касания В первых двух { —^ —у а) 76 77 12 13 8 9 5 1 3 г 7 6 V 70 15 М- 18 б) Рис. 19. 171
окружностей и имеющей центр в точке А пересечения внутрен- внутренней касательной с третьей окружностью. Докажите, что окруж- окружность инверсии проходит через две точки М и N пересечения внешней касательной и третьей окружности. Отсюда AM=AN. 18.27. Просуммируем все номера на сторонах и на всех от- отрезках OAi. Поскольку каждый отрезок ОД- учитывается дважды, эта сумма равна A + ... + п) + 2 A +-- + ") = 3"(g+1) . Значит, сумма номеров на сторонах любого из п треугольников равна -2~(я + 1). Так как эта сумма — целое число, то и =^10 (пункт б). Пример для я =9 придумайте сами. 18.29. Будем считать, что о>0. Тогда все возникающие в задаче числа неотрицательны. Если а<. ~ , то \Na]<iN—1, и потому среди чисел [а], ..., [Na] есть совпадающие. Итак, а^—-—. Но тогда и —^—-— (второе условие аналогично первому с заменой а на —| , так что —^—^а^—; . Легко r J а) N N—\ видеть, что все числа из этого интервала годятся. Случай а<С0 рассматривается аналогично. 18.31. Можно, например, выбрать систему отрезков: [1; 2], [1 1 ~\ Г 3 3 ~\ 2—; 3— , 3—; 4——I , ..., а в отрицательной части оси Ох симметрично относительно О (промежутки между отрезками об- образуют убывающую геометрическую прогрессию —, —, —, ...). Докажите самостоятельно, что не существует бесконечной арифметической прогрессии, все члены которой попадают в «ще- «щели» между отрезками указанной системы. 18.32. Пусть х\, Х2-—два положительных корня, х\>Х2. Раз- Разделим наше уравнение на хп, затем подставим вместо х оба корня по очереди и вычтем получившиеся равенства друг из друга; легко видеть, что получается противоречие. 18.34. Сыграно 5 партий; существует единственное распре- распределение играющих (с точностью до перенумерации игроков и туров). 18.35. По теореме Безу f (х) делится на х — —, поэтому q-f{x) делится на qx — р, и в частном получается многочлен с целыми коэффициентами. А так как р и q взаимно просты, то f (k) делится на qk — р. 18.38. С?954. Указание. Следует брать все тройки, содер- содержащие произвольно выбранную точку А. 19.02. 18, 45, 90, 99. Указание. При умножении на 9 сумма цифр не меняется, поэтому исходное число кратно 9. Дальше — проверка умножением на оставшиеся цифры. 172
19.03. Через каждую точку самопересечения проходит 2 зве- звена, и на каждом звене лежит ровно одна точка самопересечения. Поэтому число звеньев вдвое больше числа точек самопересече- самопересечения. 19.04. 13. См. решение задачи 19.09. 19.05. 1304 точки. 19.07. Данное число обозначим через а, округленное — через ао и а = а — ао. Если а<0,01, то aQ=0 и — = 0. Если то ^. = 1—-^-- ПрИ этом ао>0,01, а<0,01. Значит, 0< — <4~ а а а 2 и -о-< 1—— <[1. Докажите сами, что величина — может при- принимать любое значение р\ удовлетворяющее неравенству 0 <!(}<; —. После округления числа — могут получиться числа: 0; 0,50; 0,51; ...; 0,98; 0,99; 1,00. 19.09. Если числитель и знаменатель делятся на k, то на k делятся также a(cl-\-d) и c(al-\-b), а следовательно, и их раз- разность ad— be. 19.10. Каждое число, не стоящее с края, не превосходит наибольшее из своих соседей, т. е. не максимально. 19.11. У к а з а н и е. Если масса четырехугольника сосредо- сосредоточена в его вершинах (скажем, если в вершинах размещены гирьки равной массы), то точка пересечения описанных пря- прямых является центром тяжести четырехугольника. 19.14. Таких способов либо 8, либо 4, либо 2. Указание. Множество точек D есть пересечение каждой из двух параллель- параллельных плоскостей, удаленных от плоскости треугольника ABC на h с каждым из двух цилиндров с осями АС и ВС и радиусами ?!_и 22- АС ВС 19.16. Квадрат, описанный около окружности радиуса г так, что одна его сторона лежит на основании треугольника, пере- пересекает обе боковые стороны треугольника; поэтому его сторона 2г больше стороны а рассматриваемого квадрата К. Квадрат, вписанный в указанную окружность, имеет сторону, меньшую сто- стороны квадрата К, т. е. а> г -\[2. 19.19. Пусть Ai, hi, .... Л„—расстояния от точек А\, ..., Ап до данной плоскости. Тогда указанное в задаче произведение равно h Л2 h3 1и "' hn hi Замечание. Тем же способом можно решить заднч\ 13.17, а именно провести плоскость через три точки касания; аналогичным п.. 'очг ш показать, что четвертая точка также лежит на этой же члоскости
19.22 . Л, В, = 2. Указание. JL(JL(....L( 1 + Л2В.) + l) 19.25. Указание. Провести индукцию по числу сорванных листьев. 19.26. Будем распределять груз так. Грузим ящики на первую машину, пока их масса не превысит 1,5 т; последний ящик сни- снимем и положим рядом с машиной. После этого будем грузить ящики на вторую машину, потом — на третью и так до тех пор, пока не загрузим 8 машин. Общая масса ящиков на этих 8 ма- машинах и ящиков рядом с ними больше 1,5-8=12 т. Поэтому масса оставшихся ящиков меньше 1,5 т и их сможет увезти девятая машина. Лежащие на земле 8 ящиков разобьем на две четверки; каждая четверка ящиков имеет массу меньше 4-350=1400 кг, и поэтому их можно увезти на двух машинах. 19.27. Указание. Пусть сумма чисел г-й строки S^518; докажите, что сумма всех чисел в двух строках и двух столбцах с номерами i и 9—/ не меньше 4S—112^ 1960> 1956. 19.30. Предположим, что это не так. Занумеруем наши 9 прямоугольников произвольным образом и начнем закра- закрашивать их: сначала первый, потом второй и т. д. Закрашивая первый, мы закрасим площадь, равную 1. Закрашивая второй, о 1 мы закрасим площадь, большую — (площадь, меньшая —, воз- У У можно, уже закрашена, так как содержится в первом прямоуголь- прямоугольнике). Подобным же образом, закрашивая третий прямоугольник, мы закрасим больше чем —- и т. д. Получится, что в общем 8 7 1 закрашенная площадь больше чем 1 +-q- + -5~ + ---+-5~ = 5. Противоречие. 19.32. Можно. Указание. Плоскостями, параллельными граням куба, разобьем его на 133 = 2197 кубиков с ребром 1. Среди них, очевидно, имеется такой, внутри которого нет ни одной из 1956 точек. 19.33. Основные шаги решения: 1) числа х, у, z неотри- неотрицательны; 2) среди них есть нуль; 3) среди чисел xit y\, z\ также есть нуль, так что среди чисел х, у, z два совпадают. После этого остается проверить две тройки чисел: @, 0, 1) и @, 1, 1). 19.35. Указание. Докажите сначала, что если вершин кле- клеток нет ни на сторонах, ни внутри треугольника, то его площадь равна —. Затем, воспользовавшись формулой Герона для пло- площади треугольника, докажите, что квадрат одной стороны тре- треугольника не меньше суммы квадратов оставшихся. 19.36. Пусть сц, Я2, .... ап — данные числа, а,— одно из под- подчеркнутых и k — наименьшее число такое, что сумма а,+а,+| + 174
положительна (если а,>0, то k = 0). Тогда легко видеть, что все числа a,-+i, а,+2, .... а;+* подчеркнуты. Начнем теперь складывать все подчеркнутые числа, начиная с а.\ (если оно подчеркнуто). Из приведенного рассуждения видно, что сум- сумма нескольких первых подчеркнутых чисел положительна. Теперь можно отбросить начало последовательности и аналогично до- доказать, что сумма нескольких следующих подчеркнутых чисел положительна, и т. д. Таким образом, сумма всех подчеркну- подчеркнутых чисел, если их правильно сгруппировать, состоит из несколь- нескольких положительных слагаемых, т. е. положительна. (Решение задачи 29 аналогично.) 19.39. Указание. Рассмотреть развертку пирамиды. Она — треугольник, разделенный на 4 треугольника средними линиями. 20.01. Искомая трапеция отсекается от правильного 5-уголь- ника его диагональю. 20.02. Положив х = 0, получаем, что d кратно 5. Полагая да- далее х—\, х =— 1 и х = 2, легко показать, что 26, о-\-с и о — с кратны 5. Отсюда вытекает утверждение задачи. Замечание. Здесь существенны два обстоятельства: а) степень много- многочлена меньше 5; б) число 5 простое. Придумайте сами многочлен 5-й степени, который при всех к делится на 5, и многочлен 3-й степени, который лрн всех х делится на 4, причем у каждого многочлена и е все коэффициенты делятся на 5 и 4 соответственно. 20.03. Число горизонтальных звеньев, по которым улитка уда- удалялась от пункта (звено — это отрезок, по которому улитка про- проползает за 15 мин), равно числу горизонтальных звеньев, по которым она приближалась к нему. Значит, число всех гори- горизонтальных звеньев четно. По тем же соображениям четно и число вертикальных звеньев. Но числа вертикальных и гори- горизонтальных звеньев одинаковы. Следовательно, общее число всех звеньев кратно 4. 20.05. 40 столбов. 20.06. Две концентрические окружности с тем же центром и радиусами R и ^J2R2— г2. Указание. Сумма квадратов рас- расстояний от любой точки О до концов одной диагонали равна сумме квадратов до концов другой. 20.08. Квадрат. Докажите сначала, что это прямоугольник. 20.09. Обозначим через Sk сумму всех чисел ft-й строки (всего в таблице тп строк), через 2;— сумму чисел /-го столбца (всего столбцов п) и через А — сумму всех чисел таблицы. Тогда Si+S2 + ... + Sm = ?i-T-22-T- — + %i=A. Так как каждое число в k-й строке равно произведению Sk на сумму чисел Hi того столб- столбца /, в котором оно стоит, то сумма всех чисел в fe-й строке равна S* = S|.St + Z2-S* + ... + SB-St = StBi + ... + Sll) = Sk • А. Поэтому либо А = \, либо каждое S*=0, а значит, каждое число, стоящее в клетке (k, I), равно Sk-2;=0. 20.12. л: = У4. Указание. [х]—х — [х], где 0<{л:}< 1 — дроб- дробная часть; л:3—л: + М=3, откуда 2<л: (х2— 1)<3. 175
20.16. п четное (любое), нечетные п не подходят 20.20. Можно. 20.21. Часть круга радиуса /, заключенная между окружностью радиуса / и границей квадрата, вершинами которого являются точки пересечения прямых ОА и ОВ с этой окружностью. 20.22. Можно. Указание. Достаточно занумеровать кон- контакты лампы от 1 до 7 по часовой стрелке, а отверстия штеп- штепселя — против часовой стрелки. 20.23. Третья сторона должна быть равна большей из двух имеющихся. Указание. Зафиксируйте меньшую сторону b (b^a) и заставьте оставшуюся вершину треугольника пробегать окружность радиуса а с центром в одном из концов отрезка Ь. 20.24. 60°, 60°, 60°. Указание. Обозначив углы исходного треугольника через а, р, у, вычислите углы второго и третьего треугольников. 20.25. Если S = ak+l + ak+2 + ...+ak+&, то ak+9=ak+7 + +аАг+8<5<а*+ю = 5-|-аЛ+2, т. е. S не входит в последова- последовательность. 20.28. Заметим, что каждое из трех слагаемых равно отно- шению -г?- перпендикуляров (Ж, и GHit опущенных из точек и/if О и G чна одну и ту же i-ю сторону треугольника ABC(i = = 1, 2, 3). Но GHl = GH2 = GH3=a, a O OK O GH = 3a, откуда искомая сумма равна — =3. Замечание. Утверждение задачи верно для любого треугольника. 20.29. Указание. Легко видеть, что все три неизвестных обращаются в нуль лишь одновременно. Если же произведение всех неизвестных отлично от нуля, то, рассмотрев систему урав- уравнений, полученную из исходной заменой всех частей уравнений на им обратные, и сложив новые уравнения, получим равенство откуда каждое слагаемое равно нулю и поэтому справедливо равенство х\=Х2 = х$ = 1- 20.30. См. указание к задаче 20.24. 20.31. Указание. Любое поступательное движение раскла- раскладывается в сумму двух: горизонтального и вертикального; до- докажите утверждение для каждого движения отдельно. 20.33. Можно всегда, если число контактов четно. В самом деле, предположим противное. Тогда при каждом повороте на свое место попадает ровно один контакт. Пусть i-й контакт имеет номер а,- на розетке и номер 6, на штепселе. Тогда разности о, — bi должны принимать все значения 0, 1, .... 19 (по модулю 20). Следовательно, сумма (а\ — Ь|) + (а2 — 62) + — + (°2о — 620) сравнима по модулю 20 с суммой 1 +24- — +19 = 190. Но 176
и 6i-)---. + b2o са- сами равны 190, и мы получаем противоречие. Замечание. Если число кон- контактов нечетно, аналогичные рас- рассуждения не приводят к противоре- противоречию (см. задачу 20.22). 20.34. Одно из слагаемых равно 164, а каждое из ос- остальных равно 163. Указа- Указание. Числа должны быть при- примерно равными. 20.36. Указание. Если на трех сторонах ABCD лежат 3 вершины прямоугольника с заданным направлением сто- сторон, то четвертые вершины прямоугольника образуют отрезок. 20.37. При четном п имеется 4 решения, для которых х\ равно 0,1, ' ~ и — "у5Г1 соответственно (очевидно, Х2, .... хп немед- немедленно восстанавливаются по х\). Если п нечетно, первые два решения отсутствуют. Указание. Рассмотрим на плоскости точки с координатами (х\, хг) и (х2, хз). Обе они лежат на пара- параболе у=1—х2 (см. рис. 20). Чтобы попасть из первой точки во вторую, нужно пройти по двум стрелкам рисунка. Система будет удовлетворяться, если, пройдя по 2я стрелкам, мы вер- вернемся в исходную точку. Разберите сами, исходя из рисунка, почему при других х\ это невозможно. Замечание. Точка А на рисунке (видно, что стрелки сходятся к ней) называется притягивающей, а точка В — отталкивающей. Рис. 20. 20.38. Указание. Опустите перпендикуляры OKi и на все грани тетраэдра (i=l, 2, 3, 4). Далее см. решение задачи 20.28. 20.39. Указание. Возьмем произвольную цифру в числе из первой последовательности и справа от нее поставим 0 — по- получим число из второй последовательности. Это позволяет уста- установить взаимно однозначное соответствие между цифрами первой последовательности и нулями второй. 20.40. Можно. Указание. Надо поместить два самых боль- больших числа, т. е. ап и а„-и в разные группы и затем размещать очередное число (ап-г, ап-з и т. д.) в ту группу, сумма чисел ко- которой в данный момент меньше. 21.01. Указание. Начинающий может, например, добиться того, что решением системы будут числа @; 1; —1). (Для этого нужно, чтобы при у и 2 все время были равные коэффициенты.) 177
21.02. Пусть О — центр круга. Тогда точки Р и Q лежат на окружности диаметра ОМ, а длина хорды PQ, опирающейся на угол POQ, постоянна. 21.03. 12 + 22 + ... + 192 = 2470. Указание. Выясните сна- сначала, сколько имеется четырехзначных номеров, у которых сумма первых цифр и сумма последних цифр равна фиксированному k = 0, 1,..., 18. 21.04. Нужно, чтобы точки А и В были противоположными вершинами квадрата. 21.05. Указание. Сумма чисел в каждой строчке, начиная со второй, делится на 1958. 21.08. Указание. Найдите тот наименьший диаметр каж- каждой сосны, при котором два человека, обходя поляну, не видят друг друга. Покажите затем, что и три человека, двигаясь соглас- согласно условию задачи, не увидят друг друга. 21.10. Указание. Используйте тот факт, что четыре поло- полосы, попарно непараллельные, имеют максимальную площадь пере- пересечения в случае, когда их оси пересекаются в одной точке (см. условие задачи 24.17). 21.11. Указание. гп растет как п, a Sn— как и2. 21.12. Две непараллельные прямые Л и h в пространстве имеют 3 оси симметрии. Три непараллельные прямые имеют поэтому не более 9 осей симметрии. 21.13. jci = 1, Jt2=l, Xk = k при k = 3, 4, ..., п. Указание. Заметьте, что правая часть больше -д-, откуда jci = 1. Из уравне- уравнения 1— =- получаем: х=- . Воспользовавшись да- Z-\-a 1+* 1 + с лее тем, что если два числа равны, то равны и их целые части, получаем ответ. 21.14. Указание. Перейдите в троичную систему счисле- счисления. 21.16. Несложно проверить, что A155979J< 11551958 + 341958< <A155979 + 2J. Число A155979+1J явно не годится, так как оно четное. 21.18. 5 = 1 +2 + — +100. Указание. Все места равно- равноправны. 21.19. Докажем более общее утверждение: если по п попарно непараллельным пешеходным дорожкам с постоянными ско- скоростями движутся п пешеходов, причем известно, что 1-й пешеход встречается со всеми остальными и 2-й пешеход встречается со всеми остальными, то: а) каждый пешеход встречается с к а ж д ы м; б) в каждый момент все пешеходы располагаются на одной прямой. Для доказательства к нашей плоскости, в которой располагаются все дорожки, добавим вертикальную пря- прямую — ось времени и рассмотрим графики двжения («мировые линии») всех пешеходов. Эти графики — прямые линии, причем первая и вторая «мировые линии» пересекаются со всеми осталь- 178
ными «мировыми линиями». Но первая и вторая «мировые ли- линии» определяют в пространстве плоскость. Произвольная 1-я «мировая линия» пересекается как с первой, так и со второй «мировой линией»; следовательно, она лежит в той же плоско- плоскости. Итак, все «мировые линии» лежат в одной плоскости. И если теперь предположить, что какие-то два пешехода не встретятся, т. е. их «мировые линии» не пересекутся, то эти «мировые линии», находясь в одной плоскости, обязаны быть параллельными. Но тогда их проекции на плоскость, в которой находятся дорожки, также параллельны, что противоречит усло- условию. Следовательно, любые два пешехода встретятся и пункт а) доказан. Рассмотрим момент < = 0. Этот момент определяет- определяется как момент начального положения всех пешеходов, с одной стороны, и как пересечение исходной плоскости с плоскостью «мировых линий». Следовательно, в начальный момент все пе- пешеходы расположены на прямой пересечения этих плоскостей, а так как движение каждого из них равномерное, то и в каждый момент все пешеходы находятся на одной прямой, параллель- параллельной начальной. Второе решение. Перейдем в подвижную систему коор- координат, связанную с первым пешеходом. В ней 1-й стоит непод- неподвижно, следовательно, остальные проходят через точку, в которой он стоит. Из того, что 2-й встречается с 3-м и 4-м, следует, что все они идут по одной прямой. Поэтому 3-й встретится с 4-м. 21.20. Отметим внутри каждого многоугольника по точке и занумеруем их. Соединим первую точку со всеми остальными точками непересекающимися кривыми, проходящими через общие стороны многоугольников. То же проделаем и со второй точкой. Тогда, если общее число точек больше четырех, то одна из них окажется изолированной и ее нельзя будет соединить непересе- непересекающейся кривой со всеми остальными. 21.21. Указание. Докажите сначала, что можно изменить знаки ровно у двух произвольных чисел, а затем — что ровно у одного. 21.22. Можно. 21.23. (п!J=(Ьп)-B.(п-1))Х... ...X(k-(n-k+\))-...-(n.l). Но (k— \)n>(k— \)k при n>k, откуда k(n — k+\)>n. Итак, (n!) состоит из п сомножите- сомножителей, каждый из которых боль- больше п (кроме двух крайних, рав- равных п), т. е. превосходит п". 21.24. Можно. Длина этой ломаной mn-\-m-\-n. 21.27. Проведем дополни- дополнительное построение, как на ри- рисунке 21. В треугольнике OQP м N Рис. 21 179
высоты PD и QE пересекаются в точке С, откуда OC±PQ. Для решения задачи поэтому нужно доказать, что MN\\PQ. Построим на PQ как на диаметре окружность; очевидно, точки D и Е ле- лежат на ней. Поэтому опирающиеся на дугу ЕР углы EDP и EQP равны. Аналогично, построив окружность на диаметре DE, мы видим, что /LMND=/LMED. Так как EM\\PD, углы EDP и MED также равны; итак, /LMND— Z.EQP Наконец, так как ND\\EQ, то и MN\\PQ. 21.28. В правой части столько же сомножителей, сколько и в левой, но все сомножители справа равны п, а слева есть сомножители, меньшие п. 21.29. а^Ь. Примеры а>Ъ и а = Ъ придумайте сами. Указание. Докажите, что а кругов с теми же центрами, но вдвое большего радиуса, покрывают полностью многоуголь- многоугольник М. 21.31. Указание. Каждая прямая, проведенная через точ- точку О вдоль произвольного луча, должна делить плоскость на две полуплоскости, в каждой из которых лучей поровну (или их число отличается на 1). Например, можно расположить лучи так, чтобы между любыми двумя соседними угол равнялся — 21.33. а^Ь, поскольку в каждом из а кругов находится не более одного центра круга из второго набора. 21.34. 100 A0 цепочек из 10 последовательно соединенных сопротивлений, соединенные параллельно, или цепочка из 10 зве- звеньев по 10 параллельных соединений в каждом). 21.35. х = 3; у— 1. Указание. Если у = 1, то х = 3 (второй корень квадратного уравнения х= — 1 отрицателен). Пусть у>\. Числа jc и х + 2 одной четности, поэтому (дс+1) четно: х+ 1 =2/г. Получаем: Bk- \Jy + Bkfy=Bk + \Jy, откуда неслож- несложно усмотреть (раскрыв скобки по биному Ньютона), что у кратно k при у>\ Разделив теперь обе части уравнения на BkJy, по- получаем: й Отсюда y<Lk, а потому у не может делиться на k. Значит, при у>\ решений нет. 21.36. Если отрезок АВ целиком содержится в многоуголь- многоугольнике, то прямая АВ делит его границу на две части, длина каждой из которых по условию больше 1; поэтому периметр больше 2. Пусть теперь отрезок А В полностью в многоугольнике не содержится. Определим на прямой АВ точки А' и В' ее пересечения с границей многоугольника, лежащие на про- продолжениях отрезка АВ и ближайшие к А и В соответствен- соответственно; точно так же определим точки А" и В" как ближайшие к А и В (соответственно) точки пересечения отрезка АВ с грани- границей. Отрезки А'А" и В'В" разбивают весь многоугольник на 180
Рис. 22. А F \ Л Рис. 23. три области: границей одной является отрезок А'А" и часть границы многоуголь- многоугольника, границей другой — отрезок В'В" и часть границы многоугольника и третья ограничена отрезками А'А", В'В", а также ломаными аир (рис. 22). Ломаные АА"аВ'В и АА'$В"В не выхо- выходят за пределы многоугольника и имеют длины, большие 1 (по условию). Добав- Добавляя отброшенные области, получаем тре- требуемое. 21.38. 4-. о 21.39. Указание. Если на види- видимой стороне последней карточки написа- написано число k, то на ее обороте может стоять либо k—1, либо k-\-l. Если человек за- захочет ответить «k—1», то он до этого должен был либо увидеть карточку с числом k-\-l и быть уверенным, что на ее обороте стоит k-\-2, либо увидеть две карточки с числами k-\-l. To же относится и к числу k-\-l (с естественными изменениями). Для k — 2 или k-\-2 рассуждение аналогично. 22.01. Указанная процедура есть умножение «в столбик» в двоичной системе счисления. 22.02. Разность 22'959 —1 =42'958 —1 кратна 4 — 1=3, что и требовалось доказать. 22.03. Можно. Указание. Отметим 9 точек на плоскости и занумеруем их числами от 1 до 9. Проведем все отрезки, попарно соединяющие эти 9 точек. Каждому трехзначному числу abc, не оканчивающемуся на 0, сопоставим отрезок, ведущий из точки с номером а в точку с номером с. Теперь достаточно доказать, что существует путь, проходящий один раз по каждому отрезку, который начинается у кончается в одной и той же точке. Пройдя этот путь 10 раз (для 6 = 0, 1, ..., 9), мы получим нужную последовательность. 22.04. Наименьшее число поворотов равно 6-2+1 + 1 = 14 (при движении ладьи по спирали из левого нижнего угла в центр доски). 22.05. Ответ показан на рисунке 23. 22.06. Нельзя. В результате всех переливаний будет перелито a = k B— -д/2) + / V2 литров, где k и / — целые числа. Если / — кфО, то а иррационально; если же l=k, то a = 2k — четное число (аф 1). 22.08. Проведем диагональ BD. Так как DK — медиана в треугольнике ADB, то S^AKD=-^-S^ABD, аналогично SABMC = 181
(рис. 24). Проведя вторую диагональ АС, получим: S^BKC-\-S^AMD = ==-2~5/Чвсо. Значит, Sabcd—S&akd-\- _!_<? l<г Рис. 24. ^ ^ (площади тре- треугольников ВСР и AOD в сумме учтены дважды). Значит, — SMPKO + S^BCp+S^AOD=0, что и требова- требовалось доказать. 22.11. Возведем сумму щ +a2 + ...+Gi959 в степень 1000. Раскрыв скобки, получим сумму слагаемых, каждое из которых есть произведение тысячи из наших чисел, взятых в произволь- произвольном порядке. Выбрав среди этих произведений только нужные, получаем, что их сумма меньше 1. 22.13. Указание. Докажите, что общие хорды окружно- окружности О и каждой из окружностей, проходящих через точки А и В, пересекаются в одной точке, лежащей на прямой АВ. Для этого воспользуйтесь теоремой о касательной и секущей, прове- проведенных из точки к окружности. 22.14. Из узла О мы можем выйти по одному из четырех звеньев. При попадании в очередной узел листа у нас остается только три возможности выбрать следующее звено ломаной. Итак, второе, третье, ..., k-e звено ломаной мы можем выбрать тремя способами каждое. Значит, различных непересекающихся ломаных не больше р*<4-3*~' =2-2-3*~'<2-3*. 22.15. Указание. Самое длинное ребро тетраэдра не может прилегать к тупому углу. 22.16. Пусть *<*/. Тогда xft+y* = <y < (y-\-lf. Отсюда у<2<у + 1, т. е. г не целое. 22.19. На N2 — ./V. Указание. Докажите, что сумма чисел любой строки, не содержащей 1, на N больше, чем сумма чисел в столбце с тем же номером. 22.20. Допустим противное. Тогда, выбрав k чисел щ, Ог, ..., G*. получим: Gft<-^-Gi. Выбрав k чисел a*, ak+\, .... G2A-1, получим: G< Oi. И Т. Д.: Gn(fc_i)+l<-2rGl. РаССМОТрИМ Сумму ^r + ...)=2ai. Следовательно, и S2 = G2 + g*+1 + ... + аП(А-0+2+ + + + () + (+) + <i Просуммировав эти неравенства, получим: S=Si+S2 + ...+ *_, <2 (k — 1) oi =2 (k— 1) ^-< 1. Но по условию S =S, + ...+ + ... = 1. Противоречие. 182
22.21. Сгруппируем члены в левой и правой сумме по два с разных концов. Тогда разность этих сумм будет состоять из сумм скобок (akbk + an-k+\bn-k + i — akbn-k+i—an-k+\bk) = (ail — —dn-k+i) (bk — bn-k+\), которые положительны, так как a.k>an-k+\ и bk>bn-k+i по условию. Значит, левая сумма боль- больше правой. 22.22. Такой точкой будет ортоцентр (точка пересечения вы- высот) треугольника. 22.23. На число А = А,00 ... AiOO ... А,00 ... 0А,, где А, = 1II ... 1 222 ... 2 ... 777 ... 7 88 ... 8 9 выписывается произвольное число раз, а число нулей в проме- промежутках равно 9. Указание. 999 999 999 • А = А 000 000 000— —А= 111 ... II, откуда А=А000 000 000—111 ... 11. Вычитая «в столбик», находим последние девять цифр числа А; под- подставляя их в число А00... 0, продолжим вычитание и найдем следующие девять цифр числа А; снова подставим их в А00 ... 0 и так будем продолжать сколь угодно долго. 22.24. Переставим цифры так, чтобы выполнялись неравенст- неравенства п\ 2г «4^02^05 ^G3^G6- Тогда О < (а\ + а2 + аз) — (о4 + а5 + ае)< (oi + а2 + аз) — (а2 + а3 + а6) = 22.25. Половина слагаемых в данной сумме равна (+1), по- половина-— (—1); отсюда n = 2k. Но J№+i =— 1 тогда и только тогда, когда сомножители разного знака. Значит, k есть число перемен знака в последовательности х\, х2, ..., хп, х\, а так как начало и конец ее — одно и то же число, то k = 2l. Итак, л = 2А = 4/. 22.28. Указание. Пусть О — центр вписанной в треуголь- треугольник ABC окружности, А', В', С"— вершины искомого треуголь- треугольника, лежащие против вершин А, В, С. Докажите, что на отрезках АА', В В' и СС лежат центры О,, О2, О3. 22.29. См. задачу 22.32. 22.31. Возьмем для начала числа Х\, х2, ... , хьо- Если их сумма отличается от — борьше чем на т^г, то пусть она, например, 2, IUU меньше -д-; тогда хь\ + ... +дгюо больше -^-. Начнем «поднимать» числа нашего набора, заменив сначала Хбо на хъ\, затем хъ\ на хьъ, затем, например, х4э на xso и т. д. Поскольку мы, действуя так, в конце концов дойдем до набора {*5i, .... Хюо), в какой-то момент сумма впервые станет больше -^-. Легко видеть, что в 183
этот самый момент сумма чисел набора будет отличаться от I — меньше чем на 22.32. Угол между какими-то двумя отрезками не меньше — Возьмем тот из двух вертикальных углов величиной не меньше —, сумма длин а и Ь двух сторон которого (ку- (кусочков данных отрезков длины 1) не меньше I. Нетрудно проверить, что длина третьей стороны треугольника со сторонами а, Ъ и углом а~^ — между ними не меньше третьей стороны треугольника ними. со сторонами —, — и углом — между 22.33. Предположим, что вершины А и В обе обладают указанным свойством. Тогда Z.C/4B+ Z.DAB> 180° и Z. СВА + /-.DBA > 180°, тогда как сумма всех шести углов тре- треугольников CAB и DAB составляет всего лишь 180° + 180°. Проти- Противоречие. 22.34. Куб любого натурального числа имеет вид либо 9k, либо 9fe±l- Отсюда легко вывести, что числа вида 9fe + 4 и 9А: + 5 не представимы в виде суммы трех кубов. 22.35. Занумеруем поля доски (кроме центрального) в порядке обхода их шахматным конем (рис. 25, а); белые кони стоят на полях 1 и 3, черные — на полях 5 и 7. На рисунке 25, б указан- указанные поля расположены по окружности; белый кружок отвечает белому коню, черный — черному. Так как после одного хода коня цвет поля меняется, каждый конь сделает четное число ходов. Покажите сами, используя рисунок 25, б, что никакой конь не может сделать ровно 2 хода. Следовательно, каждый конь сделал не меньше четырех ходов, откуда общее число ходов не меньше 16. 22.37. Указание. Точка пересечения отрезков К1К2 и КзК* будет центром тяжести набора гирек подходящих масс (выясните сами каких), если поместить эти гирьки в вершинах четырехугольника. 22.38. См. решение задачи 42.18. 22.39. Пусть точка z лежит вне многоугольника М= {с\ ... с„>. Тогда многоугольник М виден из точки z под углом, меньшим л, т. е. точки z — с\,... , z — с„ лежат внутри некоторого угла а<.п с вершиной в нуле. Теперь заметим, ; 4 7 6 2 J 8 5 а) Рис. 25. что точка — получается из точки z симметрией относительно оси х 184
умножением I на Iz|2 I Отсюда следует, —— также лежат внут- Си 7 7 0 0 0 0 0 0 0 У/, 7 7 0 0 0 0 0 0 0 у//, 1 г 0 0 0 0 г 0 0 7 7 7 0 0 7 7 0 0 % 7 0 1 0 7 I 7 0 0 ш 0 0 1 7 7 7 0 7 7 % 7/г Уг 1 1 7 7 0 7 '/г й* Уг 1 7 7 7 7 0 Уг Уг 'А Рис. 26. ЧТО ТОЧКИ z — С\ ¦ ¦ z- ри угла р, симметричного углу а. Но тогда сумма этих комплексных чисел не может равняться нулю. 22.40. Предположим, что при любом расположении дисков части первого рода составляют меньше половины окружно- окружности. Тогда общее число случаев, когда цве- цвета секторов не совпадают, меньше п-2п. Но если совершить полный оборот меньшего диска, то произволь- произвольный сектор большего диска будет п раз покрыт секторами того же цвета и п раз секторами другого цвета. Значит, общее число случаев, когда секторы обоих дисков окрашены по-разно- по-разному, равно п-2п. Противоречие и доказывает утверждение задачи. 23.01. Такому условию удовлетворяют все суммы, не меньшие 10 р., и только они. Указание. Действительно, если S^IO, то S = l + 1+ ». +1 (S раз) и S = 10+l+ ... +1 (S-10 еди- единиц). Четную же сумму, меньшую 10 р., нельзя представить не- нечетным числом билетов достоинством 1, 3 и 5 р. 23.02. Указание. Соединить общую точку пересечения ок- окружностей с точками А\, А2, А3. 23.03. 26 человек. 23.04. Точки А\, N, В2 лежат на одной прямой (/LAiNB2 = = /LAiNM-\- /-BiNM = n, так как вписанные углы A\NM и B2NM опираются на диаметры А\М и В2М). /-А\В\М=^^- (опирается на диаметр А\М) и /LBiA-iM^^r- (опирается на диаметр В2М), поэтому А\В\ и В2А2— высоты треугольника A\MB2\ MN также высота этого треугольника. Но высоты треугольника пересекают- пересекаются в одной точке. 23.05. Если d\—делитель п, то с?2=-т- также делитель п, и поэтому меньший из них не больше -л/п. Итак, все дели- делители можно разбить на пары (кроме -Jn, если п — точный квадрат); следовательно, их не более -\[n-\-^jn — 2-\[n. 23.06. Данное число кратно 3 и не кратно 9. 23.07. 9 человек (на рис. 26 приведен пример соответствую- соответствующей турнирной таблицы). 23.08. Проведем через вершину В прямую, параллельную диагонали АС; пусть Р — точка ее пересечения с продолже- продолжением стороны DC и R — середина отрезка DP. Легко видеть, что прямая AR искомая. 23.09. 0, 2 или 4. Указание. Провести прямые О А и О В и рассмотреть различные случаи расположения отрезка CD по 185
отношению к двум вертикальным углам с вершиной О, один из которых содержит отрезок АВ. 23.10. Таких чисел бесконечно много уже среди множества точных квадратов. Действительно, если т2=р + п2к, то р=(т—пк) (т-\-пк). Поэтому, поскольку р простое, т — п* = 1 и р = m-f-/г* = 2я* + 1. Но для бесконечного множества пар (п, k) число 2пк -\-1 не простое. 23.11. Пусть-т правильная дробь. Найдем наименьшее целое число qi, для которого q\-^-~^\. Обозначив ^-= = <7i-r 1, найдем наименьшее целое q% для которого <72*тЦ> 1- Введем дробь p-z=Q2'P— 1 и определим тем же спосо- способом целое число q$, затем дробь jp- и т. д. Заметим, что -г->^->^-> ... ; знаменатели этих дробей не превосходят Ъ. Поэтому описанный процесс прервется через конечное число шагов, т. е. найдется такое п, что qn.^L = \. Легко ви- Оп— I деть, что тогда . qn ' Замечание. Такое представление не единственно. Например, 13 1 1 11 I 50 4 "МОО 5 ~*~ 17+50-17" 23.13. Пусть А и В — точки пересечения прямой /, проходя- проходящей через точку О, с границей многоугольника (А и В лежат на сторонах) и пусть АОфОВ, например АО>ОВ. При повороте прямой / вокруг О на очень маленький угол а точки А и В переходят в Л' и В' (соответственно), причем также выполняется неравенство А'О>ОВ'. Но тогда 2SAOA4'« \OA\ \OA'\ sin a> ~> \ОВ\ \ОВ'\ sin ct«2SA0BBi, хотя из условия вытекает, что эти площади равны. Противоречие. Итак, ОА = ОВ и О — центр симметрии. 23.14. Объединение окружностей радиусов -\J2R2 — О А1 и О А, концентрических с данной окружностью с центром О и радиусом R (А — вершина прямоугольника на этой окружности). См. ука- указание к задаче 20.06. 23.15. Ответом будет векторная сумма двух треугольников, являющихся ортогональными проекциями исходных на середин- серединную плоскость. 23.16. Предположим, что дробь—целое число, т. е. п не- нечетно. Тогда дробь равна ап~х — ап~2Ь-\-ап~3Ь2— ... +&""' (п слагаемых). Докажем, что любые два соседних слагаемых а"~кЬк и — сГ~к~хЬк + 1 при делении на п дают одинаковый 186
остаток. Действительно, их разность, равная а" * lbk(a-\-b), де- делится на п. Следовательно, сумма остатков от деления на п всех слагаемых делится на п. 23.18. Если А—точный квадрат, то его последняя цифра есть х2=1,4 или 9. А тогда ровно одно из чисел л/А+х и ¦yfA—x кратно 5, а значит, и 5* (k — число нулей в А-\-1), и поэтому А— х2>9-10* — противоречие. 23.19. Будем считать, что |gi I ^ IG2I ^ ...^ |о*|- Пусть \щ \ = = ... = |а;| и |g;|>|g;+i|. Разделив обе части данного ра- равенства на о", получим справа 0, а слева сумму: а) / слагаемых, каждое из которых равно ( + 1) или ( — 1), и б) k — l сла- слагаемых!—) , / = /+ 1, ... , k. Но при стремлении п к бесконечности сумма последних k — / слагаемых может быть сделана сколь угодно малой, откуда следует, что первые / слагаемых разбиваются в пары (+1, —1). Отбрасывая их в левой части исходной суммы и проводя это рассуждение дальше, получаем требуемое. 23.20. Если точки А, В, С, D образуют выпуклый четырехугольник, то для любой точки М справедливы неравенства МА-\-МС~^АС, MB -\-MD ~^BD, складывая которые, убеждаемся, что искомой точкой является точка пересечения диагоналей АС и BD. Если же, например, точка D лежит в треугольнике ABC (внутри или на стороне), то можно показать, что для любой точки М выполняется условие AM + BM + CM-\-DM^AD+BD + CD и поэтому искомой является точка D. Наконец, если А, В, С, D лежат на одной прямой (пусть С и D между точками А и В), то искомой точкой будет любая точка М отрезка CD. 23.21. Пусть a^b^c^d — стороны четырехугольника. Тогда можно построить треугольник со сторонами а — d, b, с (выпишите неравенства треугольника), который легко достраивается до тра- трапеции присоединением параллелограмма со сторонами cud. (См. также 48.03.) 23.23. 31-е число, и только оно. 23.24. п = 4. Указание. Все точки лежат на окружности, диаметром которой служит отрезок наибольшей длины с концами в этих точках. 23.27. Пусть Gi — первый наблюдатель; ог — последний из всех наблюдателей, начавших следить за улиткой до того, как за ней кончил следить а.\\ Оз — последний из наблюдателей, начавших следить за улиткой до того, как за ней кончил следить аг, и т. д. Нечетные промежутки наблюдения oi, аз, аъ, ... между собой не пересекаются, так же как четные промежутки G2, Й4, Об,... не пересекаются между собой (иначе один из наблюдателей а, был бы выбран неправильно). Но любой из этих промежутков равен 1 мин, а весь интервал наблюдения со- составляет 6 мин. Поэтому как четных, так и нечетных проме- промежутков не более 5. Отсюда число наблюдателей не больше 10, и, 187
значит, улитка проползла не более 10 м (она проползет ровно 10 м в такой, например, ситуации: ползет улитка только тогда, когда на нее смотрит ровно один наблюдатель; остальное время она стоит). 23.29. /<т и произведение 1т четно. 23.32. Криволинейный треугольник, образованный дугами полу- полуокружностей, построенных на средних линиях треугольника ABC как на диаметрах (рис. 27). 23.33. Рассмотрим фигуру Ф, состоящую из множества точек, удаленных от отрезка длины 10 не более чем на 1 (рис. 28). Разобьем весь квадрат 100X100 на 50 вертикальных полос шириной 2 каждая и во все эти полосы поместим по 8 непере- непересекающихся фигур, равных Ф. Тогда из условия следует, что в каждую из 400 фигур попадает по крайней мере один центр круга и поэтому кругов не меньше 400, что и требовалось доказать. 23.34. Предположим противное. Тогда заметим прежде всего, что среди чисел а.\, .... а* не более 7 единиц. В самом деле, прибавляя остальные числа по одному, мы можем сделать сум- сумму меньше Л, но больше А—8 (до тех пор, пока она меньше, можно прибавить еще одно число). Поэтому, если есть 8 единиц, мы сможем сделать сумму равной А. Далее, среди а.\, .... а* не более 7 двоек. В самом деле, будем рассуждать аналогично, прибавляя за оДин раз по одному четному числу или по два нечет- нечетных (чтобы сумма оставалась четной). В какой-то момент мы получим число, меньшее А, но большее А —18 и притом четное, как и А. Добавив к нему нужное число двоек, мы получим А. Аналогичные рассуждения показывают, что среди наших слагаемых не очень много троек, четверок, ..., девяток. Но тогда сумма заведомо меньше 5040 = 2520-2 B520— первое число, делящееся на 1, 2, .... 9), что противоречит условию. 23.35. Если данное число А равно 10а+ 6, то полученное числоВ равно 106n~'-6 + a. Но тогда 10В — Л=A06" — \)Ь кратно 7 (так как 999 999 кратно 7). А так как А делится на 7, то и 8 тоже. 23.36. Предположим, что некто X имеет т знакомых А\, Л2, ..., Ат. Все эти его знакомые по условию друг с другом незнакомы. Значит, для каждых двух человек (Л,-, Л,) должен найтись еще один общий знакомый, отличный от X и не- знакомый с X (при этом раз- ным парам (Л,-, Л,) отвечают разные люди). Наоборот, каж- дому из незнакомых с X соответ- ствует пара (Ль Л,) их общих с X знакомых. Значит, число Рис. 27. людей, незнакомых с X, равно 188
числу пар С2т. Поэтому п = \ + т + С2т A—это сам X, т — число его знакомых, С2т — число не- незнакомых с X). Отсюда т определяется однозначно, и оно, следовательно, одинаково для всех X (заметим, что п может принимать не все значения, а только 1,2, 4, 7, 11, ...). 23.38. Отметим среди номеров 1, 2, 3, ..., п те, которым отвечают отрезки, идущие вправо или вверх (С3,« способов). Независимо от них можно выбрать п номеров, которым соответствуют отрезки, идущие вправо или вниз (тоже С\п спо- способов). Ясно, что этим выбором маршрут улитки однозначно определяется. Итак, общее число раз- различных маршрутов улитки равно (СЗпJ- 24.02. Легко видеть, что все числа, начиная с а\Ь\С\, делятся на 9 и (если они не равны 0) их средняя цифра равна 9. Поэтому достаточно рас- Рис 2g смотреть десять чисел: 99, 198, 297, .... 990, которые легко проверить непосредственно. 24.04. Эта окружность имеет центр в середине отрезка наи- наибольшей длины с концами в двух из данных точек. 24.06. Указание. Треугольник М\М2М3 подобен треуголь- треугольнику ABC с коэффициентом подобия k=-^-. 24.07. Достаточно одного вопроса, если второй задаст числа а, = 1, О2=100, аз=Ю02, ...,ап=102"-2 (или любой другой набор быстро растущих чисел). 24.10. Если из каждого отрезка натурального ряда вычесть первый член, то после перестановки чисел получатся такие наборы: @,981, 1, 982, 2, ..., 978, 1959, 979, 1960, 980) и (980, 0, 981, 1,..., 978, 1959, 979, 1960). Указание. Выясните сначала, с какого числа с должен начинаться искомый отрезок, если исходные отрезки начинались с чисел а и Ъ. Для этого сложите все числа искомого от- отрезка. 24.14. Пусть K = 4k. Так как у каждого пассажира нет монет мельче 10 к., то каждый из них обязан получить сдачу. Следовательно, 4k из 5k монет должны остаться у пассажиров. Вместе с тем стоимость проезда всех 4k пассажиров составляет 20& копеек, на что нужно не менее k монет. Следова- Следовательно, в кассу было опущено не меньше k монет; общее число монет поэтому не меньше bk. Требуемый пример правильной оплаты при наличии bk монет такой. Разбиваем всех пассажиров на k групп, по 4 человека в каждой; 5 монет в каждой группе распределены так: у 1-го — 15 к., у 2-го — A0+ Ю) к., у 189
3-го—15 к., у 4-го — 20 к. Сообразите сами, как они должны расплатиться друг с другом. 24.15. Указание. Рассмотрите выпуклую оболочку всех то- точек и докажите, что это п-угольник. 24.16. Применим индукцию. Пусть usk делится на 5 а Ы5*+1 дает при делении на 5 произвольный остаток г^о! Тогда числа u5i,+2, и5к+з, «5*+ 4, иъь+ь дают при делении на 5 остатки соответственно г, 2г, Зг, 5г = 0. 24.17. Все оси этих полос должны пересекаться в одной точке. 24.18. См. решение задачи 24.14. 24.19. Проведем из точки О касательные О А и ОБ к окруж- окружности S. Пусть S — центр окружности, D — точка пересечения отрезков OS и АВ. Искомое множество центров — окружность, построенная на отрезке 0D как на диаметре в плоскости, перпендикулярной данной, с выкинутой точкой О. Для дока- доказательства достаточно заметить, что любая окружность касания сферы н конуса проходит через точки Л и В, т. е. плоскость этой окружности вращается вокруг прямой АВ, a 0D — перпен- перпендикуляр к этой плоскости. 24.21. Указание. Применить индукцию по числу проведен- проведенных диагоналей. 24.22. Указание. Рассмотреть отдельно случаи AP = AS и АРФАБ. 24.23. Пусть m — очередное число, сумма цифр которого делится на 11; покажем, что среди следующих 39 чисел есть число с тем же свойством, а) Если последняя цифра m не 0, а предпоследняя цифра не 9, то сумма цифр числа т + 9 равна сумме цифр числа т, т. е. т+9— искомое число. б) Если т кончается нулем, а предыдущая цифра не больше 7, то подходит число m + 29 (его сумма цифр больше на 11) в) Если предпоследняя цифра числа m равна 9, то, прибавив к m не больше 10, мы получим число, оканчивающееся по меньшей мере двумя нулями. Но, прибавляя к такому числу 1, 2, 3, ..., 18, 19, мы получим всевозможные остатки от деления суммы на 11, в частности одно из этих чисел дает остаток 0. Таким образом, подходящее число превышает m не более чем на 10 + 19 = 29. г) Нерассмотренным остался только случай, когда т= ... 80. Но здесь те же рассуждения, что и в случае в), показывают, что следующее подходящее число превосходит m не более чем на 20+19 = 39. Пример т = 999 980 показывает, что эту оценку нельзя улучшить. 24.24. Если звездочек только 6, то всегда можно найти два столбца, в которых не меньше 4 звез- звездочек, вычеркнуть их, а вычеркиванием строк унич- уничтожить две оставшиеся звездочки. Пример для 7 звездочек показан на рисунке 29. Рис.29. 24.25. Так как abcd — a = a (bed— l)= 1961, то 190 *-
а нечетно. Из остальных равенств следует, что Ъ, с, d тоже нечетные. Но тогда число abed нечетное и поэтому разность (abed — а) четна. Противоречие. 24.26. Предположим, что такую ломаную можно провести; ее можно считать незамкнутой. Ломаная входит в каждую область и выходит из нее; при этом для данной области каждому входу отвечает выход, кроме того случая, когда ломаная войдет последний раз в область и больше из нее не выйдет. Так как верхние две области (рис. 3) состоят из пяти (н е- четного числа) отрезков, то один конец ломаной должен лежать в одной из этих областей, а второй — в другой. Но нижняя средняя область (рис. 3) тоже состоит из пяти отрезков, поэтому какой-то конец ломаной должен содержаться и в ней. Однако третьего конца у ломаной нет. 24.27. Опустим из О, точки пересечения диагоналей прямо- прямоугольника, перпендикуляр ОК. на общую внешнюю касательную к окружностям 1 и 3. Проведя радиусы Г\ и гз к этой каса- касательной, получим прямоугольную трапецию, в которой ОК — средняя линия. Значит, О/(=г'^ 3 . Такую же длину имеют и остальные перпендикуляры, опущенные из О на общие касатель- касательные к окружностям 1, 3 и 2, 4 (так как Г\ + г3 = г2 + г4). Зна- Значит, О — центр вписанной в образовавшийся четырехугольник окружности. 24.30. Пусть какая-то четверка совпадает с исходной. Решение распадается на следующие шаги. 1) Доказать, что abcd = \. В противном случае произведение получающихся чисел никогда не совпадает с произведением исходных. 2) Доказать, что ас=1 и, следовательно, bd=l. Это следует из того, что для всех четверок, начиная со второй, такое равенство будет выпол- выполняться. 3) Исследовать, что происходит с четверкой чисел а, Ь, —, -г- в результате наших преобразований. 24.31. Центр этой окружности — вершина равностороннего треугольника с основанием O1O2. 24.32. Будем менять знаки каждой строки и каждого столбца, у которых суммы чисел отрицательны. При этом, возможно, «испортятся» другие строки или столбцы, но сумма всех чисел таблицы S при каждой такой замене возрастает. При этом величина, на которую возрастает S, не стремится к нулю (легко сообразить, что эта величина может принимать лишь конечное число значений) и поэтому в какой-то момент она достигнет максимума. Но если ее нельзя увеличить, то сумма чисел любой строки и любого столбца неотрицательна. 24.33. Указание. Докажите сначала, что существует точка, из которой выходит ровно один отрезок. Для этого пройдитесь по графу из любой точки, пока идти станет невозможным Далее — индукция по числу точек. 191
•'"] Г"^ 24.34. Обозначим наибольший общий / 7/2 \ делитель чисел Ь и р — а через d. Отсю- ' ' 1 да b = kd и р — a = ld, где k и / взаимно просты. Но тогда ak-\-bl = a-——\-b-—-—= а а =p = kp. 24.35. Самый выгодный для Коли — первый способ, самый невыгодный — Рис 30 третий. 24.36. Указание. Докажите снача- сначала, что из любой бесконечной последова- последовательности натуральных чисел можно выбрать неубывающую подпоследовательность. Дальнейшее просто: из последователь- последовательности а.\, а2, ... выбираем неубывающую подпоследовательность af^a2< ...; из отвечающей ей (по индексам) подпоследователь- подпоследовательности Ь\, Ь2, ... выбираем неубывающую Ь"^Ьг'< ...; из отве- отвечающей ей подпоследовательности с", с", ... выбираем неубы- неубывающую cf"<c?"<... . Тогда ai"<O2"<...; bf 24.37. Для того чтобы круг лежал в прямоугольнике, не пересекаясь с квадратами, нужно, чтобы его центр лежал в прямоугольнике размером 19X24 и отстоял от каждого квадрата более чем на -^-, т. е. не попадал в фигуру, изображенную на рисунке 30. Но сумма площадей ста двадцати таких фигур равна 120-C+-^-J < 19-24, так что место для центра круга диаметра 1 найдется. (Ср. с решением задачи 23.33.) 24.39. РС = 5. Проведем луч ВМ так, чтобы /_СВМ= /-АВР и Z. РВМ=60°. Отложим на нем отрезок ВР' = ВР и соединим Р' с точками С и Р. Тогда треугольник РВР' равносторонний, откуда РР' = РВ = 3; АВСР'=аАВР, откуда Р'С=РЛ = 2. Следовательно, PC^IPP' -\-P'C = 5. Равенство достигается, если РР'С — прямая. 24.40. Легко доказать по индукции, что через 2Р шагов получится набор aia^+i, a2a2p+2 ,... , а2*а2р+2*. В частности, через 2* шагов набор будет такой: а?, а\, а\, ... , а^*. т- е- будет состоять из одних единиц. 25.01. Можно ограничиться случаем, когда точки М и N расположены в одной полуплоскости от прямой АВ. Тогда М — центр вписанной в треугольник ABN окружности и \АВ — BN\ равно модулю разности двух отрезков, на которые делится отрезок АВ. 25.02. Указание. Рассмотрите бесконечную треугольную сетку, один из треугольников которой — наш. Сторону нашего треугольника ориентируйте стрелкой. Эта стрелка задает ориента- ориентацию самого треугольника, а также всех треугольников сетки. 192
25.04. Если у двух чисел суммы цифр равны, то разность этих чисел кратна 9. Поэтому 2а — а = а делится на 9. 25.05. Указание. Докажите, что най- найдутся такие числа аи ..., ak, k^n, и та- такие k карточек, что на первой из них на- написана пара (щ, аг), на второй — пара (а2, оз), ..., на (к— 1)-й — пара (afe_i, ак), на k-и — пара (ак, щ). Отбросив их, про- проведите это рассуждение несколько раз для остальных карточек. Рис 31. 25.06. Указание. Отрезки этих пря- прямых разбивают треугольник ABC на четыре равновеликих тре- треугольника. 25.08. Преобразовав наше соотношение так, чтобы выразить п, мы видим, что п = т2— Т. . • Поэтому, положив m=d-\~2, мы найдем целое п, удовлетворяющее условию при йф — 1. Если d= — 1, то следует взять т—1, а п произвольное. 25.12. Задача разрешима только при четных п. 25.14. 1; 1+л/5 ; х~^ь . Указание. Домножьте второе 1 —v/5 1+V5 уравнение на х\, третье на Х\Хъ и т. д. и воспользуйтесь пер- первым уравнением. 25.15. Для этого достаточно, чтобы радиусы были очень ма- маленькими, например в 4000 раз меньше длины меньшей из сто- сторон прямоугольника. 25.16. MN минимально, если точки М и N равно отстоят от точки пересечения данных лучей. 25.17. Указание. Предположив, что разбиение на паралле- параллелограммы произведено, рассмотрите два соседних параллелограм- параллелограмма, имеющих общую вершину квадрата, и докажите, что они обя- обязаны пересекаться. При доказательстве этого воспользуйтесь тем, что никакой треугольник нельзя разбить на параллелограммы. 25.18. Указание. Если п — любое натуральное число и <am+i, то n—am<am_,. 25.21. Нужно пустить шар так, чтобы траектория его движе- движения отсекла от всех углов бильярда равнобедренные треуголь- треугольники, как показано на рисунке 31. Равенство длин всех траекторий шара вытекает из рассмотрения «выпрямленных» траекторий. 25.22. Указание. Пусть О — центр окружности МВС; О\ — центр окружности ABC; N — середина ВМ; L — точка пересече- пересечения ON и ВН. Докажите, что треугольник OO\L равнобедренный. 25.23. а) Площадь уголка равна 3, но 1961 и 1963 на 3 не делятся, б) Прямоугольник 1963 X 1965 разобьем на три прямо- прямоугольника: 1965X1958, 1956X5 и 5X9. У первого одна сторона делится на 3, а вторая — на 2; следовательно, его можно раз- разбить на прямоугольники размером 2X3, каждый из которых разби- 7 Заказ 247 |93
HI _г вается на два уголка. Второй пря моугольник разобьем на прямо- прямоугольники 5x6 и каждый из них — на 10 уголков (рис>132, а) . ri Третий разобьем на 15 уголков а) ' (рис. 32, б) Рис. 32 25.24. Поскольку 1962 делит- делится на 3, данное число есть сумма кубов: N = A0654K+l3. Оно делится на 10654 + 1. 25.25. Возьмем две наиболее далекие друг от друга точки А и В и построим два круга радиуса 1 с центрами в них. Для произвольной точки С возьмем ту из точек А и' В, которая ближе к ней; тогда расстояние до нее меньше 1 (в про- противном случае получили бы противоречие с условием). Итак, все 25 точек лежат в построенных двух кругах, а тогда в одном из них лежит не менее 13 точек. 25.26. Указание. Следует доказать более сильное утвержде- утверждение: найдутся две несоседние вершины, из которых не проведено ни одной диагонали. Это легко доказывается индукцией по числу проведенных диагоналей. 25.27. См. решение задачи 25.32. 25.28. Пусть п — простое число. Рассмотрим вершину А и пусть А' — ближайшая вершина, в которую может перейти вершина А при повороте на угол ф = &а вокруг центра О (k — любое натуральное число). Покажем, что внутри угла АО А' нет других вершин многоугольника. Предположим противное, т. е. допустим, что внутри угла АОА' есть вершины В\, Вг, ... , Вр. Тогда внутри угла А'ОА", где А" — вершина, в которую переходит А' при повороте вокруг О на угол ф, находятся вершины В\, Во, ... , В'р, в которые переходят при этом повороте соответственно вершины В\, Вг, ¦• , Вр. Число вершин многоугольника равняется п = = (р+1)—-, т. е. не равно простому числу п при рфО. Значит, р = 0. Итак, очевидно, что при последовательных поворотах на угол ф вокруг О вершина А может перейти в любую другую вершину многоугольника (при некотором k), откуда следует, что rt-угольник правильный. Но по условию он неправил ь- н ы й. Отсюда вытекает, что наше предположение неверно и п — составное число. 25.29. Заметим, во-первых, что ai+ ... -{-ai0 = 4(xi+ ... +Xs)- Таким образом, мы знаем сумму S=Xi+*2+ ... -\-хь- Пусть щ — наибольшее из чисел а\, ... , am, а ato — наимень- наименьшее. Тогда, очевидно, а\ — сумма двух наибольших из'чисел х\, X2, ..., хъ, а am — сумма двух наименьших. Поэтому 5—а.\ — — аю = хз, где хз — среднее по величине из чисел х\, '..., хь- Теперь уже нетрудно найти все остальные неизвестные: Напри- Например, если а.2 — второе по величине число, то х\=ач—Хз- 25.30. Указание. Опустите перпендикуляры из центров 194
0\ и 02 на хорды AM и ВМ до их пере- / сечения в точке R. Докажите затем, что RPJ_MP. Тогда MR — RH и, поскольку AR=y[Rj и MR — RB (по построению точки -г-/?), получаем, что R — центр искомой описанной около МАНВ окруж- окружности. 25.31. Пусть aix ..., а7, ав — число за- задач, решенных школьником за некото- некоторые идущие подряд 8 дней. По уело- вию fli + a2 + ... + a7 = 25, o2 + a3 + ...+ + Ов = 25, откуда at = aR. Итак, число Р|1С 33- задач, решаемых школьником за день, должно повторяться через каждые 7 дней и школьнику надо разработать план (аи а2, .... сп) только на одну неделю. Если бы школьник решал все 25 задач в понедельник, то он затратил бы на все задачи в течение года время S\. Точно так же опреде- определены числа S2, S3, .... Sl Решая по понедельникам не 25, а щ за- задач, школьник за все понедельники потратит время -j^-Si; анало- гично за все вторники — время ~S2 и т. д. Общее затраченное ZO время равно S=-=-(ciiSi + ... + a7S7). Выбирая из чисел Si, S2, ..., S7 наименьшее S*, полагаем ak = 25, а все остальные о,= 0; тог- тогда S будет минимальным. 25.32. Сумма в условии равна ±(oi—а2)±(а2 — Оз)± —± ±(о25 — fli). Если раскрыть скобки, то в полученной алгебраиче- алгебраической сумме 25 слагаемых будут стоять со знаком плюс и 25 — со знаком минус. Эга сумма, следовательно, не больше 2Ь25 + 2&24 + + ... + 2fru + fci3 — bi3 — 2bi2 —... — 2bu где b\ Ь2ъ — наши чис- числа, расположенные по возрастанию. Для того чтобы получить наибольшую возможную сумму, достаточно расположить числа в следующем порядке: Ь25, Ь\, Ь24, Ь2, .... Ьм, bi2, bi3. 25.33. Из рисунка 33 легко видеть, что площадь многоуголь- многоугольника увеличится больше чем на 12 + л>15. 25.36. Указание. Точка В по точке С строится так: про- проводится СС J_/ (С лежит на окружности) и берется точка пере- пересечения касательной в точке С с прямой /. 25.39. Параллелепипед должен располагаться так, чтобы плоскость, проходящая через вторые концы А, В, С его ребер, й'ыходящих из одной вершины, была горизонтальна. Указа- Указание. Площадь ААВС всегда равна половине площади всей проекции параллелепипеда, а площадь проекции любого тре- треугольника на горизонтальную плоскость равна площади самого треугольника, умноженной на косинус угла между плоскостями. 25.40. Доказательство проведем по индукции. Для k = 2 утверждение очевидно. Если для k = n участники а,\, .... ап распо- 7* 195
ложены в порядке возрастания, то (п+1)-го ста- ставим в этот ряд перед первым из тех, кому он не проиграл. 26.02. См. решение задачи 26.06. 26.05. Нельзя. Прямая, пересекающая „4 50 клеток, должна пересечь не менее 49 разде- разделяющих их линий, а на листе таких линий все- всего 48 A9 вертикальных и 29 горизонтальных). 26.06. 0=(al+a2 + ... + anJ = a2l+a22 + ... + a2n + 2S, откуда ? i g ( ) 26.07. Отрезки, соединяющие середины сторон четырехуголь- четырехугольника ABCD, образуют параллелограмм П, и легко видеть, что Sn=~S. С другой стороны, они являются средними линиями в четырех треугольниках с общей вершиной М, на которые де- делится четырехугольник PQRS, откуда SPQRS — 4Sn. Окончатель- Окончательно SPQRS = 2S. 26.08. Можно вначале найти решения только в натуральных числах, так как если (хо, уо, Zo) — решение, то, изменив знак у любых двух чисел этой тройки, снова получим решение. Данное уравнение умножим на 2xyz и воспользуемся неравенством о2 + Ъ2 > 2ab; 6xyz = 2*У + 2.v V + 2у V=(х2у2 + x2z2) + (х2у2 + +y2z2) + (x2z2+y2z2)^2x2yz + 2y2xz + 2z2xy = 2xyz (x + y + z), от- откуда х + у + 2^3. Но х, у, z — натуральные, поэтому x=y = z = = 1 — единственное решение в натуральных числах. Ос- Остальные решения исходного уравнения таковы: (—1, —1, I), A, -1, -1), (-1, 1, -1). 26.09. Перенесем все прямые параллельно самим себе так, чтобы все они пересекались в одной точке. Углы между ними от этого не изменятся. Но после этого они делят полный угол в 360° на 14 частей, и если бы каждая часть составляла не меньше 26°, то в сумме они дали бы по крайней мере 14-26°= 364°>360°. Получили противоречие. 26.10. При n = 4k. Указание. На рисунке 34 показано, как заполнить карточками прямоугольник размером 4X5, а из таких прямоугольников можно составить прямоугольник 4&Х5. Если п нечетно, то карточками нельзя заполнить прямоугольник, а если «делится на 2, но не на 4, то прямоугольник придется заполнить нечетным числом карто- карточек и потому произведение всех чисел в таблице будет отри- отрицательно, откуда ясно, что не может быть положительным произведение в каждой строке. 26.11. Указание. Если разность прогрессии меньше 10*, то в прогрессии найдется число, состоящее не менее чем из k-\-l цифр, начинающееся с любой цифры, в том числе и с 9. 26.13. Пусть x = a + b, y — b + c, z — a + c; тогда левая часть данного неравенства примет вид: S =-~-(Х + х~^у~г -\- 196
) +>, +>, f+y>. то S>i-F-3)=.? 26.14. См. решение задачи 26.17. 26.15. Нельзя, так как —<4-« 1Ь 7 26.17. Предположим противное. Рассмотрим наименьший вы- выпуклый многоугольник, внутри которого расположены все точ- точки, и возьмем произвольную вершину А этого выпуклого многоугольника. Соединим ее со всеми оставшимися точками. Получим 4 угла, больших 30°, откуда угол А больше 120°. Но углы при всех вершинах выпуклого многоугольника не могут превосходить 120° — противоречие. 26.18. m-f-я— 1. См. решение задачи 5. 26.19. Если все числа а, Ь, с положительны, то неравенство очевидно. Если одно из них (например, а) положительно, а два отрицательны (&<0, с<0), то а>|6| + |с| и потому а"> |&Г +|с|". Это равносильно нужному неравенству. 26.20. Окружность, описанная около треугольника ABC. Дей- Действительно, если М' — точка, в которой пересекаются перпенди- перпендикуляры к прямым AM, ВМ и СМ, то ММ' — диаметр описанной окружности. 26.21. Тангенс суммы 2 трех арктангенсов равен дроби, у кото- которой числитель x + y + z — xyz, а знаменатель 1 —xy — xz — yz. При ?<-^- сама дробь и ее знаменатель положительны, а при ^>~ отрицательны. Поэтому числитель всегда положителен. 26.22. Если бы такая ломаная существовала, то, сориентировав ее звенья против часовой стрелки, получили бы, что сумма 25-и векторов равна 0. Рассмотрим эти векторы с общим началом в точке А и ортогонально спроектируем их на прямую, перпенди- перпендикулярную /: сумма векторов-проекций не равна 0 из-за равенства их длин и нечетности числа 25. Следовательно, наше допуще- допущение неверно. (Ср. с решением 39.22.) 26.24. Указание. Каждая из цифр 1, 2, .... 7 встречается на каждом из мест 720 раз, а 720 делится на 9. 26.25. На 15 частей: 11 тетраэдров A при вершине, 3 в сред- среднем слое, 7 в нижнем) и 4 октаэдра A в среднем слое и 3 в нижнем). 26.26. 8 штук. Указание. Красные шарики разбивают си- синие на три группы, и при выполнении условий задачи легко видеть, что ответ равен числу разбиений числа 7 на три слагаемых (быть может, нулевых). Вот эти разбиения: 7+0 + 0, 6+1+0, 5 + 2 + 0, 5+1 + 1, 4 + 3 + 0, 4 + 2+1, 3 + 3+1, 3+2 + 2 26.28. Указание. Все прямые проходят через точку пересе- пересечения медиан ?\АВС. 197
1 О; 02 Рис. 35. 2 3 шшш I III ill I Рис. 36. 26.29. 75 чисел. Указание. Нужно взять все нечетные числа, делящиеся на 13. 26.31. Составим диаграмму (она на- называется диаграммой Юнга), как показа- показано на рисунке 35, где в i-й строчке распо- расположено а, квадратов. Тогда Ь; — число квадратов в /-м столбце. Отсюда сразу вытекает требуемое. 26.32. Предположим противное. Тогда, если путь из одной клетки в другую тре- требует k шагов, то разность чисел, стоящих в этих клетках, не превосходит 4k. Но раз- разность чисел 1 и 64 равна 63, а число шагов между клетками, в которых стоят эти числа, не больше 14. Так как 4-14<63, получаем противоречие. (Решение зада- задачи 26.39 аналогично.) 26.33. Круг с центром О и радиусом -s-/? (без границы), где О — центр дан- данной окружности радиуса R Указание. Рассмотреть предельный случай прямо- прямоугольных треугольников. 26.34. 655 чисел. Указание. Рассмотрите числа вида 3k -f-1, k = 0, I, 2, ..., 654 (разность любых двух из них кратна 3, а сумма — нет). Большего количества чисел выбрать нельзя, так как иначе нашлись бы два числа, отличающиеся на 1 или на 2, и их сумма делилась бы на их разность. 26.35. Движение джентльменов периодично с периодом 12 мин: через каждые 12 мин любой из них находится в первоначаль- первоначальной точке на аллее и движется по ней в т у ж е сторону. Поэтому ось времени t естественно представлять намотанной на окруж- окружность длины 12 мин, а эту окружность считать трехслойной: первый слой образуют две дуги, по 6 мин каждая, второй — 4 дуги, по 3 мик каждая, третий — 6 дуг, по 2 мин каждая. Дуги каждого слоя покрасим в два цвета, черный и белый (цвет определяется направлением движения данного джентльме- джентльмена по аллее), так, чтобы эти цвета в одном слое чередовались. Рассмотрим пересечение одноцветных дуг первого и второго слоев, получим по крайней мере две общие черные дуги а и р, по 2 мин каждая, и две белые дуги у и б, также по 2 мин. 6 дуг третьего слоя делятся на 3 черные и 3 центрально-симметричные им белые дуги. Поэтому, если пересечение дуг аире тремя черными дугами третьего слоя составляет дугу меньше 1 мин, то пересечение дуг у и б с белыми дугами составляет дугу, большую 1 мин, и утвержде- утверждение задачи доказано. 26.37. 17 точек. Пример такой ломаной на рисунке 36. 198
Указание. Замкнутая 14-звенная ломаная имеет 7 вертикаль- вертикальных и 7 горизонтальных звеньев, так как из каждой ее вершины вы- выходит одно горизонтальное и одно вертикальное звено. Зануме руйте горизонтальные звенья от I до 7 и выясните, сколько то- точек самопересечения может лежать на каждом из них. 26.38. Указание. Описать вокруг 10-угольника окруж- окружность; для каждого образовавшегося треугольника измерять его углы в единицах -?-. После этого задача сведется к комбинаторной задаче по подсчету числа дуг на окружности, суммы которых п о три одинаковы, а сами дуги различны. 26.41. В самом деле, если п нечетно, то zn—xn + y" делится на х-{-у. Но тогда z делится на х + у и, следовательно, z~^x-\-y, т. е. zll>xn-\-yn. 26.42. (я!J ломаных. Указание. Если i\, 12, .... L — произ- произвольная нумерация горизонтальных прямых (таких нумераций «!), а /I. /2, •••, /л — произвольная нумерация вертикальных прямых (их также «!), то.ломаная со звеньями (i\, ji), (/2, /2), .... (U. /л) по этой нумерации строится однозначно. 26.43. Нужно взять любые 12 векторов подряд. Указание. Если OS —сумма максимальной длины (О — центр), а / — прямая, проходящая через О перпендикулярно OS, то в сумму OS входят все векторы, лежащие в той же полуплоскости от /, что и OS . 26.44. Пусть А', В' — середины сторон АВ и ВС соответ- соответственно. Очевидно, SAA.BD. =—-SAABC и аналогичная формула имеет место для других треугольников. Поэтому сумма площадей треугольников ABC, BCD, CDE, DEA, EAB в 4 раза больше суммы площадей малых треугольников, дополняющих A'B'C'D'E' до ABCDE. С другой стороны, эти пять треугольников покры- покрывают пятиугольник ABCDE меньше 2 раз, так что сумма их площадей превосходит площадь ABCDE меньше чем в 2 раза. Отсюда легко следует нужное неравенство. 26.45. Указание. Докажите по индукции. что an = 4an-i— а„_2- 2i5.46. Указание. Если, например, х — наибольшее из этих четырех чисел, то х*>х*-\-x'>zz + t'. 26.47. Указание. Расположите все десятичные дроби друг под другом и рассмотрите бесконечное число столбцов (разря- (разрядов) высоты 11: в каждом столбце не меньше двух одинаковых цифр, откуда и вытекает утверждение. 26.48. Р(х) = ах(х— 1)(х—2)...(х — 25). Указание. Дока- Докажите последовательно, что Р (х) делится на х, на х— 1, на х — 2, .... на х — 25. Отсюда Р {х)=х{х— \)(х—2)...(х — 25)-Q (х). Затем до- докажите, что Q (x—l) = Q (х), откуда Q (x) = const. 199
Рис. 37. Рис. 38. 26.50. Предположим противное. Занумеруем эти дуги римски- римскими цифрами I, II и III и дополним их до окружностей с теми же номерами. Обозначим диаметрально противоположные точки пере- пересечения окружностей I и II через А и А\\ II и Ш — через В и В\; I и III — через С и Си Эти точки пересечения принадлежат хотя бы одной дуге; пусть А\ и С\ лежат на дуге I, В\ — на дуге II, тогда В и С принадлежат дуге III, А — дуге II. Обозна- Обозначим через а, р\ у плоские углы трехгранных углов, как показано на рисунке 37 (О — центр сферы). Поскольку дуга I не содержит точек А и С, должно выполняться неравенство 360° — р>300°. Аналогично дуга II не содержит точек В и А\, откуда 180° + а>300°, и, наконец, для дуги III имеем: 360° — у>300°. Отсюда получаем: Р<60°, а>120°, v<60°, значит, а>р + у — противоречие с теоремой о плоских углах трехгранного угла. Замечание. Оценка 300° для трех дуг на сфере точная 27.01. 90°, 45° и 45°. 27.02. Пусть О — центр окружности, К — точка пересечения касательной в точке С с касательной ВМ. Тогда КС —KB (как отрезки касательных, проведенных из одной точки). И если /LCAB = a, то /LACO = a, а так как Z.OCK = 90°, то АКСМ = =90° — а. Из треугольника АМВ Z. КМС = 90° — а, откуда КС = КМ. Поэтому КВ = КМ=±-ВМ. 27.03. Число с суммой цифр 5 имеет вид 9^ + 5 и, следо- следовательно, при делении на 3 дает остаток 2. Но любой квадрат при делении на 3 дает остаток 0 или 1. 27.04. 41 звено. Звенья следует стирать через одно. 27.05. Указание. Докажите (по индукции), что все члены последовательности, кроме первых двух, дают при делении на 4 остаток 2 или 3. 27.07. Числа вида р и 2р, где р — простое, и числа 8 и 9. 27.08. Если хфО, то число вида х+дбе никогда не является полным квадратом, так как х+л/л? лежит между (~\[хJ и (л/х+ IJ. Поэтому у=1. Отсюда х—п2, z — n, где п — произвольное число. Если же х=0, то 2=0, а у — произвольное число. 27.10. Единиц на одну больше (за счет числа 106). Указа- Указание. Единицы получаются из чисел вида 9&+1, а двойки — из чисел вида 9fc + 2. 200
27.11. x=y = z=l. Указание. Докажите сначала, что либо все три числа больше 1, либо все три меньше 1. Затем дока- докажите, что все они равны между собой и xyz=l. 27.12. Если m(m + l) = aft, то, поскольку т и m-f-1 взаимно просты, каждое из этих чисел также является /г-й степенью, что невозможно. 27.13. Обозначим 27 — к — т, тогда a-k3 а-273 т 272 — З-27/n+ тя. Следовательно, целое при всех целых т, что возможно только при а==273. 27.15. Если в треугольнике провести две высоты, то их основания и оставшаяся вершина треугольника являются верши- вершинами подобного треугольника. Следовательно, каждый из четырех треугольников, на которые разбивается диагоналями четырех- четырехугольник MNPQ, подобен соответствующему треугольнику из четырех, на которые разбивается диагоналями четырехугольник ABCD; значит, и сами четырехугольники подобны, что и требова- требовалось доказать. 27.16. Аг— 1681. Указание. Зачеркнем две последние циф- цифры числа N и заменим их кулями. Очевидно^ получится тоже точный квадрат некоторого числа п. Тогда ^]N^n+i, так что (п + 1J — «^<99. Отсюда п^49. С другой стороны, п, очевидно, кончается нулем. Таким образом, максимальное подходящее я = 40, т. е. Л/ = 412. 27.18. с = 271964. См. решение задачи 27.13. 27.20. На 5 тетраэдров. Указание. К каждой из двух проти- противоположных граней куба примыкают не менее 2 тетраэдров, и все эти 4 тетраэдра различны. Легко видеть, что Сбъем каждого из них не больше — а3. Следовательно, эти 4 тетраэдра не заполнят всего объема куба и нужен пятый тетраэдр. Пример разбиения на 5 тетраэдров изо- изображен на рисунке 38. 27.21. Обозначим радиусы проведенных окружностей через Г\, г», г. Рассмотрим треугольни- треугольники OiRO и O2SO (рис. 39). , У них Oifl = OO2=r,, OiO=O2S = r2 и Z_OO\R= /-OO2S (поскольку O\R параллельно O2S). Следо- Следовательно, эти треугольники рав- равны по двум сторонам и углу меж- между ними, откуда OR —OS. Опу- Опуская перпендикуляр ОН на хорду 201 Рис. 39.
PQ, получаем: RH = SH и PH=QH; значит, PR = PH — RH = QHSHQS 27.22. Если все знакомы, то задача решена. Пусть теперь А и В незнакомы. Осталось 2я— 2 человека, причем как с Л, так и с В знакомы более чем п человек. Значит, более чем (п-\-п) — B« — 2) = 2 человека являются общими знакомыми для А и В. Посадим теперь А и В друг напротив друга, а между ними — их общих знакомых. 27.23. Будем проводить всевозможные отрезки, соединяющие данные точки друг с другом и с вершинами квадрата и попарно непересекающиеся, до тех пор, пока это возможно. В тот момент, когда это станет невозможным, квадрат окажется разбит на треугольники (иначе в одном из оставшихся многоугольников можно было бы провести еще одну диагональ). Треугольников с вершинами в данных точках будет не меньше 100 (докажите это сами индукцией по числу точек), и их суммарная площадь меньше 1. Поэтому площадь хотя бы одного из них мень- меньше 0,01. 27.25. а = 2. Указание. Положим /=—, тогда / — ра- рациональное число, являющееся корнем уравнения /2 — at +1=0. Но тогда / _с±Уд — Число д/а2— 4 при целом а может быть рациональным только при а =±2. 27.26. Только при и = 2*. Указание. Пусть п — произволь- произвольное число и всю воду удалось за т переливаний перелить в один стакан; докажем, что п = 2*. Очевидно, перед последним переливанием было два стакана, в которых было поровну воды. Таким образом, если принять общее количество воды за 1, то на (т — 1)-м шаге распределение воды по непустым стаканам было( —-, —j . Сделаем теперь индукционное предположение, что на (т — /г)-м шаге воды в стаканах было (-^, -jL, ..., -|н Что было на предыдущем шаге? Поскольку нумерация стаканов произволь- произвольна, мы можем считать, что вода на этом шаге переливалась из вто- второго стакана в первый. Есть две возможности: либо второй стакан опустел, тогда на предыдущем шаге воды было Г^тг. тргг. 4г, ¦¦¦, 4-). либо в нем что-то осталось, тогда предыдущее рас- распределение было (фгг, ip—\~ф+\ • ¦•¦¦ -$п • В обоих случаях мы видим, что все знаменатели имеют вид 2*. В частности, именно та- такими были знаменатели перед первым переливанием. Но это и зна- значит, что и = 2*. 27.27. Указание. Докажите, что в четырехугольнике ОхО2ОО3 (пусть С лежит между А и В) Z.О3= 180°— Z.OCB, ZO2=180°— A OCA, откуда АО2+ ZLO3= 180°. 202
27.28. Указание. Первый 00-^07 игрок должен делать ходы, сим- С метричные ходам второго, пока не представится случай занять угловую клетку. 27.29. Указание. Соеди- нить точку О со всеми вер- шинами и опустить из О все перпендикуляры на стороны; се- миугольник разобьется на 14 прямоугольных треугольников. Рис. 40. Для каждой пары треугольников с общей гипотенузой выразить квадрат ее длины через сумму квадратов катетов (длину сторо- стороны 7-угольника положить равной 1). Сложить 7 полученных равенств и получить отсюда, что сумма отрезков, стоящих в ле- левой части указанного в условии равенства, равна —. 27.33. Расположим всевозможные пары целых неотрицатель- неотрицательных чисел в таблицу и будем их нумеровать в порядке «змейки», показанной на рисунке 40. Тогда легко убедиться, что пара (х, у) имеет номер -^-((х-\-уJ-\-Зх + у). Но это означает, что функция / (х, у)= j~ x+y задает взаимно однозначное соответствие между целыми числами, с одной стороны, и парами чисел — с другой. Отсюда сразу вытекает утверждение задачи. 27.35. Введем систему координат с началом в узле (оси на- направлены по вертикальной и горизонтальной линиям бумаги). Пусть xi и iji — проекции отрезков ломаной на оси координат (с учетом знака). Тогда xi+x2 + ...+xn = 0, у\+у2 + —+Уп=0, x?-\-yf=c (l*?Li<^.n). Рассмотрим отдельно три случая: 1) с де- делится на 4; 2) с имеет вид 4/г+2; 3) с нечетно. В случае 1) в каждой паре (х,-, ?/,-) оба числа четные; пусть 2к — наибольшая степень двойки, на которую все Xi и у; одновременно делятся. Выписанные выше равенства для сумм координат сокра- сократим на 2* и попадем в один из случаев 2) или 3). В случае 2) Xi и у, всегда нечетны, и имеем п нечетных слагаемых с ну- нулевой суммой. Следовательно, п четно и все доказано. В случае 3) числа х, и у, р а з н о й четности. Пусть имеется m нечет- нечетных чисел Xi (остальные четные); тогда п — m нечетных чисел у, (остальные четные). Из равенств сумм нулю следует, что m и п — пг одновременно четные. Значит, и п тоже четно. 27.36. Можно. А именно легко видеть, что здесь применима индукция и мы вправе считать, что в (п — 1) мензурках уже составлены равномерные смеси из (п — 1)-й жидкости, одна мен- мензурка пуста и еще одна «стоит в стороне» — в нее налита та же жидкость, что и с самого начала. Теперь, пользуясь делениями, 203
разольем жидкость из первых п— i мензурок поровну по п мен- мензуркам (в которых она была и пустой, количество жидкости в каждой считаем равным 1). Тог- Тогда в каждой из них жидкости станет 1 (включая бывшую пустую мензурку). Долив их Рис 41 доверху (на —J из последней мензурки, мы получим требуемые равномерные смеси. 27.37. Указание. При таком движении остается неподвиж- неподвижной точка О — центр тяжести данных точек. 27.40. Указание. Каждое натуральное число п> 1 встре- встречается на карточках столько раз, сколько существует несократи- несократимых правильных дробей 0< — <1 со знаменателем п, другими словами, сколько имеется натуральных чисел, меньших п и взаим- взаимно простых с ним. 27.41. Сумма длин проекций всех векторов на координатные оси не меньше 4 (поскольку сумма двух длин проекций одного вектора не меньше его длины). Рассмотрим те векторы, сумма длин проекций которых на одну из четырех координатных полу- полуосей (лучей с началом О) наибольшая — она не меньше 1. Если она больше 1, то эти векторы искомые, если же равна 1, то суще- существует вектор (какой?), добавление которого к выбранным векто- векторам дает искомую систему. 27.44. Проведем построение, как указано на рисунке 41. Легко видеть, чго если О— центр пирога, К и L — середины сторон АВ и ВС, то окружность, построенная на ВО как на диаметре, про- проходит через Д' и ?. Пусть Р — точка, в которой пересекаются раз- разрезы (или их продолжения), проведенные из точек К и ?. Если точка Р лежит внутри круга ОКВ?, то KP^iBO = l и ?Р^ 1, так что в точке Р пересекаются не продолжения, а сами разрезы, и от пирога, таким образом, отрезан кусок KPLB. Пусть теперь Р лежит вне круга. Тогда Z. KPL < Z. KOL, откуда Z-OKP<L Z.&LP, т. е. а<Э- Остается заметить, что так как точки К, L, М, ... идут по кругу, то не могут одновременно выполняться неравенства а < 0, Э < у,..., 6 < а. Это и означает, что хотя бы один кусок будет отрезан. Замечание. Из решения видно, что если длины разрезов меньше 1, то при л>3 всегда можно провести их так, чтобы не отрезать от пирога ни одного куска. 27.45. Рассадим для начала рыцарей произвольным образом. Пусть при этом какие-то два врага — рыцари А и В — оказались 204
рядом, причем В сидит справа от А. Найдем теперь среди друзей рыцаря А такого С, что его правый сосед D—друг В (он найдется, поскольку у А есть п друзей, а число врагов у В, не считая А, всего лишь п— 2). Теперь «развернем» весь участок стола BDE ... FGC от В до С в обратную сторону: CGF ... EDB. Легко видеть, что при этом число пар соседей-врагов уменьшится. Действуя так и дальше, мы в конце концов придем к требуемой рассадке. 28.01. Проводим окружность с диаметром ОМ, находим точку Л' ее пересечения с прямой а (если она есть внутри круга О) и соединяем М с N. MN — искомая прямая. Решений может быть 0, 1 или 2. Указание. ON A-MN, так как ON делит пополам хорду круга О, лежащую на MN. 28.02. Указание. Воспользоваться тем, что 999 делится на 37, и представить число abc 000 в виде abc- 999 -\-abc. 28.03. Пусть О — точка пересечения прямых АВ и CD. Отложим от точки О на этих прямых отрезки ОХ—АВ и OY = CD, а затем построим параллелограмм OXYZ. Точка пересечения прямых OZ и а является искомой. Задача имеет два решения. (Почему?) 28.04. Предположим противное. Пусть а, — число матчей, сыгранных /-й командой; очевидно, что 0^а,^29. Поэтому, если все числа а,- р а з н ы е, то одно из них равно 0, одно — 1, .... одно — 29. Но команда, сыгравшая 29 матчей, сыграла со всеми командами, в том числе и с той, которая сыграла 0 матчей. Противоречие. (Решение задачи 28.08 аналогично.) 28.05. См. указание к задаче 28.02. 28.06. Точка О лежит на пересечении прямых, параллельных сторонам треугольника и отстоящих от них на —-, —- и —- соот- Do 2. ветствующей высоты. 28.07. По свойству биссектрисы ^=4?>4|=§?. Отсюда ЛШ DC Ad i\d легко следует, что точка М отстоит от прямой АС дальше, чем К- Поэтому прямые МК к АС пересекаются в точке Р, расположенной правее С (рис. 42). А тогда Z.MCA как внешний угол треуголь- треугольника СМР больше его внутреннего угла СМР, т. е. р > ф, поэтому из Л МКС видно, что МКЖС. ТЬчно так же угол МКА больше угла КАР, т. е. я|)>а, отсюда АМ>МК. Итак, AM > МК > КС, что и требовалось доказать. 28.09. Указание. Докажите, что если нельзя убрать ни одного куска ковра, то ни е одном месте коридора не лежит сразу 3 ковра. 28.10. Проведем к этим окруж- окружностям касательные, параллель- параллельные сторонам треугольника, так, чтобы каждая из окружностей оказалась вписанной в треуголь- 205
ник, отсекаемый соответствующей касательной от исходного. Эти два отсекаемых треугольника подобны исходному с коэффициен- коэффициентами &i<l и А2<1, поэтому r\ = k\r и г2 — к2Г. Но так как они пересекаются, то Aidifci >¦ 1, откуда>./4 + т2>-Х, что и требова- требовалось доказать. , г Замечание. Тем же способом можно решить и задачу 15.11. 28.12. Отрезок АВ считаем неподвижным, а двигать будем угол вокруг него. Вершина угла опишет тогда дугу АВ окружности, в которой отрезок АВ — хорда. Перпендикуляр к середине X хорды АВ пересекается с биссектрисой угла в се- середине Y дуги а, дополняющей дугу АВ до полной окружности, следовательно, длина отрезка XY постоянна при движении вер- вершины угла. 28.13. У к а з а и4и е. Докажите, что наибольшее из всех чисел, находящихся в карманах, строго больше суммы остальных «кар- «карманных» чисел. (Ср. с решением задачи 28.15.) 28.14. 3932 треугольника и 5896 разрезов. Указание. Найдите сумму углов всех треугольников. (Ср. с задачей 16.22.) 28.15. Если |х|^2, то |xn|>Un-1H-U"-2H-... + l. Поэтому f (х) при х? [ — 2; 2] имеет тот же знак, что и х", и, следовательно, не равно 0. 28.16. Указание. Касательные к окружностям в точках А, В, С пересекаются в одной точке D — центре описанной вокруг треугольника ABC окружности. 28.17. Очевидно, наибольшее значение, которое может принимать *=--f-+yi?_ q, равно ±+^j ±+1=, Пусть теперь у — корень уравнения, удовлетворяющего усло- условию задачи, и |а| ^ 1. Легко видеть, что ау — корень уравнения у2-\-рау-\-аа2=0, которое также удовлетворяет условию. По- Поэтому искомое множество значений есть множество чисел вида ,а|<1, т. е. отрезок [-i а 28.18. Окружность, являющаяся пересечением сферы, по- построенной на АВ как на диаметре, с конусом, вершина которого — точка В, а основание — данная окружность. 28.19. Нельзя. Указание. Если между двумя карточками, на которых написано число а, лежит одна карточка с числом Ь, то между карточками с числом Ъ лежит ровно одна из карточек с а. Отсюда легко следует, что при произвольном расположении карточек количество карточек, лежащих между парами, бу- будет четным. Но согласно условию задачи оно должно рав- равняться 1 +2+ ... + 9=45. Таким образом, искомого расположения не существует. 28.20. Указание. Пусть ap=at. Докажите по индукции, что для любых /, /', таких, что х+/=р + А, выполняется равенство j 28.22. Сделаем все 1965 поворотов. Суммарное число совпа- 206
дений красных секторов будет, очевидно, равно 200-200 = 40 000. Предположим теперь, что при всех, кроме 59, положениях совпа- совпадает не менее 21 красных секторов; тогда общее число совпа дений не меньше 59-0 + A965 — 59)-21 =40 026>4О 000. 28.23. Гангстер должен двигаться навстречу полицейским со скоростью 2v или -тг и. Указание. Допустим, что появля- появляется не 1 гангстер, а много — по одному напротив каждого по- полицейского. Поскольку все они находятся в одинаковом положе- положении, то если ни один полицейский не видит первого гангстера, то он не видит ни одного гангстера. Соответственно ни один гангстер не видит ни одного полицейского. Но это значит, что гангстеров и полицейских можно поменять местами, так что если t)i = au — решение задачи, то и у2=— v тоже решение. 28.28. Построим на сторонах данного треугольника внутрь него дуги по 120° каждая и отметим их середины А, В, С. Искомое множество центров описанных равносторонних треугольников вокруг данного — описанная вокруг A ABC окружность. 28.29. За 2 взвешивания. Первое взвешивание. На одну чашу весов кладем по одной монете из 10 мешков, на дру- другую— 10 монет из оставшегося мешка. Второе взвеши- взвешивание. На первую чашу кладем одну монету из первого мешка, 2 монеты из второго, 3 — из третьего, ..., 10 монет из десятого. На другую — 55 монет из последнего (того же, что и в первый раз) мешка. Подумайте сами, как, сравнивая разности масс при взвешиваниях, установить, какой из одиннадцати мешков содер- содержит фальшивые монеты. 28.30. Предположим, что этот процесс бесконечен. Тогда най- найдутся такие i и у, что /-й слой совпадает с у-м, a (i-\- 1)-й совпа- совпадает с (y-fl)-M. Тогда из условия укладки кубиков легко заме- заметить, что (i—1)-й слой совпадает с (у—1)-м, (/ — 2)-й с (у — 2)-м и т. д. В частности, второй слой совпадает с (/ — / + 2)-м, а пер- первый— с (у—«' +1)-м. Но тогда нетрудно видеть, что процесс можно было остановить на (у—/)-м слое. 28.31. Предположим, что шарик попал в одну из угловых луз, пройдя перед этим путь длиной /. Где же он был, пройдя путь — ? Из соображений симметрии по отношению к началу и концу пути эта точка должна находиться на равном расстоянии от начальной и конечной точек и, мало того, прямая его движения должна располагаться симметрично по отношению к этим точкам. Отсюда легко следует, что шарик должен в этот момент находиться либо в центре бильярда, но это невозможно, так как- на биль- бильярде размером pX%q шар вообще никогда не попадает в центр (почему?), либо на середине борта, т. е. свалится в среднюю лузу, что и требовалось доказать. 28.32. Если бы -это было не так, то суммы давали бы остатки 207
1, 2, 3 In. Но тогда сумма всех полученных чисел при делении на 2« давала бы тот же остаток, что и 1+2 + ... + 2я, т. е. п. Это, как легко видеть, неверно. 28.33. Назовем самый большой ящик ящиком ранга п, следующие по величине два ящика — ящиками ранга л — 1 и т. д. до ящиков ранга 1, в которых лежат монеты. Разность числа гербов и решек в каком-нибудь ящике назовем дефектом этого ящика. Дефект самого большого ящика обозначим через d. Докажем, что всегда найдется ящик, при переворачивании которо- которого общий дефект уменьшается по крайней мере вдвое; тогда задача будет решена, так как |rf|^2" и d всегда четно. Предпо- Предположим противное, и пусть d>0. При переворачивании всех монет в ящике с произвольным дефектом d\ общий дефект d меняется на 2rfi. По предположению имеем: |rf — 2d,|>-j- \d\, либо di<-j-, либо di>^- Дефект d' любого ящика ранга 1 меньше -г\-г> 1 = №'1> так как d чет- четно) , т. е. rf'<—. Отсюда следует, что дефект любого ящика ран- ранга 2 не больше —, а значит, меньше — ; отсюда е свою очередь получится, что дефект любого ящика ранга 3 меньше —, и т. д. В конце концов получим, что дефект ящика ранга п меньше —, хотя он просто равен d, и d>0 — противоречие. (Отметим, что для того, чтобы все монеты стали «смотреть» гербом вверх, необходимо, вообще говоря, 2"~' переворачиваний.) 28.34. Ясно, что р<19. Кроме того, если р нечетно, то рр-\-1 делится на 2. Мало того, если р имеет нечетный делитель, например р = 5k, то рр +1 = (pkf + 15 и делится на рк + 1, а потому не являет- является простым. Итак, остается проверить р=1, 2, 4, 8 16. Но 1616 = 264 = B10N-16, а так как 210>1000, то 260> 10", откуда видно, что 16 имеет больше 19 цифр. Если р=8, то 88+1 = = 224+1=B8+1).B16 — 28 + 1) составное. Остаются р = 1, 2, 4. Прямая проверка показывает, что 2, 5, 257 — искомые простые числа. 28.35. Указание. Докажите, что последние цифры чисел п" и (п + 20)"+2° совпадают (проверьте это для различных последних цифр числа п). Периодически повторяется набор цифр: 0, 1, 4, 7, 6, 5, 6, 3, 6, 9, 0, 1, 6, 3, 6, 5, 6, 7, 4, 9. 28.36. Указание. Если М — точка пересечения отрезка АВ с плоскостью Р, АМ = а и ВМ = Ь, то проведенный через IA диаметр высекаемого на плоскости круга делится точкой М на такие отрезки хну, что xy=^ab. Поэтому этот диаметр мини- минимален при х—у—л/аВ. 208
28.38. Рассмотрим все строки и все столбцы и выберем ту строку (или стол- столбец), где нулей больше всего. Пусть в этой строке к нулей и / единиц, k-\-l = m. Тог- Тогда в k столбцах, соответствующих нулям этой строки, по меньшей мере k единиц (это следует из условия), а в остальных / столб- столбцах не менее / единиц (это следует из вы- выбора исходной строки). Таким образом, общее число единиц не меньше k2l2 \-{т — k)~^—. Придумайте сами при- Рис. 43. мер таблицы, для которой это неравенство превращается в равенство. 29.01. Отрезок, соединяющий середину основания треуголь- треугольника ABC с серединой высоты, опущенной на основание. 29.02. Все числа, кроме делящихся на 20. 29.05. Нельзя, потому что в цепи одинаковые цифры идут парами, а осталось нечетное число экземпляров каждой цифры 29.06. Сложив уравнения, после тождественных преобразо- преобразований получим равенство (х—1)(у—l)+(z—1)(/ — 1)=2. По- Поскольку оба слагаемых в левой части неотрицательны, есть только два случая: а) оба они равны 1, тогда, очевидно, х=у = 2 = * = 2; б) одно равно 2, другое 0. В этом случае мы получаем решение х = 3, у = 2, 2 = 5, /=1 и еще 7 решений, получаемых из него перестановкой неизвестных. Всего задача имеет 9 решений. 29.07. k=65. Указание. Очевидно, при переходе от k к ) знаменатель увеличивается в (k-\-l) раз. А числитель? Его слагаемые умножаются на 19 и 66 соответственно. Поэтому он умножается на некоторую громоздкую дробь, которая заведомо меньше 66, но от 66 отличается ненамного (первое слагаемое много меньше второго и потому не играет особой роли). Легко показать, что если k не слишком мало, например если k> 5, то числитель увеличивается более чем в 65 раз. Таким обра- образом, Ak возрастает, пока &<65, и начинает убывать после этого; Абь максимально. 29.08. Рассмотрим концы радиусов, соединяющих центр ок- окружности с вершинами пятиугольника. Каждая сторона пяти- пятиугольника с вершинами в этих концах не меньше соответствующей стороны данного пятиугольника. А в любом вписанном пятиуголь- пятиугольнике найдется сторона, не большая стороны правильного пяти- пятиугольника, так как имеется сторона, стягивающая угол а<^-?-. 29.09. Числа k образуют арифметическую прогрессию 30;? + 29, р = 0, 1, .... Указание. Для каждого k = \, 2, ... рассмотрите отдельно периоды повторения остатков от деления числа kk -f 1 на 2, 3 и на 5 (длины периодов равны соответственно 2, 6-и 20). 209
Рнс. 45. 29.10. 16 дамок. На рисунке 43 показана нужная расстановка. Докажем, что расставить 17 дамок невозможно. Предполо- Предположим, что на доске стоят 17 дамок; очевидно, ставить дамку на край нельзя, так что они заполняют весь средний квадрат 6X6, за исключением одного поля. Но тогда в центральном квадрате 4X4 находится не менее 7 дамок. Для того чтобы их можно было бить, все они должны соседствовать с единственным свободным полем в квадрате 6x6. Но рядом с этим полем есть только 4 места. 29.11. Решение этой задачи приведено на рисунке 44 (ра- (радиус окружности № 7 равен —, а окружности № 8 —) . 29.12. аюоо = 495. Указание. Проверьте, что начальные члены последовательности а\, аг, ..., а*, ... выглядят так: 1, 1, 1, 1, 2, 2,2,3, 3, 4, 4,4, 5, 5, 6,6 Докажите, что в этой последователь- последовательности каждое натуральное число встречается дважды, за исклю- исключением единицы, которая встречается 4 раза, и чисел вида 2", ко- которые встречаются по 3 раза каждое. Отсюда аюоо находим простым подсчетом. 29.14. 10 линий. Указание. Из условия вытекает, что с данной линии метро можно пересесть максимум на три другие ли- линии, с каждой из этих — еще на две. Поэтому число линий не пре- превышает 1 -f-3 + 2-3 = 10. На рисунке 45 показана схема пересадок для десяти линий, удовлетворяющая условию. Замечание. Можно доказать, что если 9 из линий, соединяющих точки на рисунке 45, прямы е. то и десятая будет прямой. Этот чертеж соот- соответствует теореме Дезарга в проективной геометрии (см., например: М а ц у о К о м а ц у. Многообразие геометрии, чертеж 34, с. 78). 29.15. Рассмотрим число Л/ = 10001000... (количество нулей в числе N мы зададим позже). Выясним, как меняются первые 4 цифры числа k\ при k = N, N-\-l, ... . Если k\ = abcde ... , то k\-1000100 ... = abcx ... , где х равно либо d, либо d + 1, в зависи- зависир мости от того, случается ли перенос от суммы а-\-е в следующий 210
разряд при умножении «в столбик». Так как при этом пятая цифра указанного произведения увеличивается на а, то не реже чем один раз в 10 шагов этот перенос будет происходить и число abed будет увеличиваться на 1. Пусть теперь N = 1 000 100 000. На протяжении 100 000 шагов число abed будет увеличиваться не бо- более чем на 1 при каждом умножении, но и не реже чем через 10 умножений действительно будет увеличиваться на 1. Таким об- образом, четыре первые цифры числа k\ последовательно пробегут 10 000 значений, среди которых встретится и 1966. 29.13. и 29.18. Сформулируем вначале общие принципы, при- применяемые при решении подобных задач. 1°. Если из 2" (или меньшего числа) шаров радиоактивен один, то его можно найти за п испытаний: первым шагом испытать половину шаров и таким образом узнать, в какой половине находится активный, вторым шагом взять половину от этой половины и т. д. 2°. Если шаров больше чем 2", то за п шагов нельзя обеспечить отыскание одного активного шара. Предположим противное. Сделаем п испытаний, отмечая наличие радиоактивности плюсом, а ее отсутствие — минусом. По предположению, зная возникшую последователь- последовательность знаков, мы сможем указать, какой из шаров активен. Но разных последовательностей существует всего лишь 2". Указав по каждой из них активный шар, мы придем к нелепому выводу, что те шары, которые не соответствуют никакой последователь- последовательности, вообще не могут быть активными. 3°. Если из m шаров активны 2, то имеется С*т = ~ вариантов, смотря по тому, какая пара шаров активна. Аналогично, если ~ >2П, то за п испытаний не удастся наверняка найти активную пару. 4°. Если из m шаров мы первым шагом испытываем k, то ре- результат «—» соответствует (?m-k вариантам (оба активных шара находятся среди m — k оставшихся), а результат « + » — осталь- остальным Cm — Cm-k вариантам. Если в нашем распоряжении осталось / испытаний, то ни одно из этих чисел не должно превышать 2'. Естественно, наилучшая стратегия состоит в том, чтобы эти числа были по возможности одинаковыми. Решение задачи 18. Положим, что на первую проверку взято 2 шара и получен отрицательный результат («минус»). Теперь нужно из 9 шаров выделить 2 за 5 проверок. Но так как Сэ = 36>25, сделать этого нельзя (п. 4°). Пусть теперь на первую проверку взято 4 шара. Если был получен « + », то число остав- оставшихся вариантов равно Сп — С? = 34;>25, и задача вновь нераз- неразрешима. Итак, для первой проверки необходимо брать 3 шара. Предположим, что получен « — ». Нам остается из 8 шаров выделить 2 за 5 проверок. Аналогичный подсчет показывает, что на вторую проверку надо брать больше одного, но меньше трех шаров. Итак, берем на II проверку 2 шара. Предположим, что опять получен « — » и остается из 6 шаров выделить 2 за 4 провер- 211
х х X X X X X X X X X X X X V X X X X V х Рис. 46. ки. Но теперь если мы возьмем 1 шар, то в случае результата « —» останется С§—10>23 ва- вариантов; если же на III проверку взято 2 шара, то в случае « + » вариантов останется С§—С? = = 9>23. Итак, выделить 2 шара из 11 за 6 проверок можно лишь при удачном стечении обстоя- обстоятельств. Сделать это за 7 про- проверок нетрудно. Решение задачи 13. Возьмем на I проверку 5 шаров. Предположим, что получен « + ». Тогда на II проверку берем 8 шаров из 14 оставшихся. Если вновь получен « + », то один активный шар — в первой пятерке, а другой — во второй восьмерке, и каждый из них можно най- найти за 3 испытания (п. 1°). Если же вторая проверка дала «—», то на третью проверку берем 4 шара из оставшихся. В случае « + » за 3 испытания находим один шар из пяти и за 2 — один из четырех; случай « —» разберите сами. Пусть теперь первая проверка дала « — ». Остается за 7 проверок найти 2 шара из 14. Теперь следует взять на проверку 4 шара. Дальнейшие рассуж- рассуждения, аналогичные предыдущим, проведите сами. 29.19. Набор гирь: 26, 25, 24, 22, 19 и 11 удовлетворяет усло- условию. Пусть теперь выбраны какие-то 7 гирь. Заметим сразу, что нельзя брать одновременно гири 26, 25, 24, 23, а потому сумма масс любых четырех гирь из этих семи будет меньше 98. Но нетрудно подсчитать, что существует 98 разных наборов из 7 гирь по 1, 2, 3 и 4 гири. Поэтому массы каких-то двух наборов будут совпадать. 29.20. 14. Указание. Переставляя строки и столбцы, всегда можно перейти к расположению, которое аналогично по- показанному на рисунке 46. Далее остается найти число спосо- способов разбиения числа 11 на слагаемые, каждое из которых не меньше 2. 30.01. Существуют. Наименьшая такая пара состоит из чисел N и N—1, где yV = 8999...99OO...O A3 девяток, 14 нулей). 30.02. Искомая точка М — основание высоты СМ треугольника ABC. Действительно, по теореме синусов 1 ^ 1 30.03. Такого способа не существует, потому что слова, у которых первые 8 знаков (обозначим их звездочкой) совпадают, должны по условию попадать в разные группы, а таких слов имеется четыре: (*••), (*•—), (*—•). (* )• 212
30.04. При АВфВС искомых точек М —14; при АВ = ВС искомое множество состоит из окружности радиуса ВА с центром В и еще 6 точек. 30.05. 37.28-2 = 2 072. 30.06. Указание. Пешеходы неизбежно встретятся на самой длинной окружности, поскольку ее длина больше суммы длин всех остальных окружностей. 30.07. Нельзя, потому что суммарная площадь пяти частей, заштрихованных на рисунке 47, равна — (т. е. отлична от —А 30.09. Максимальная разность равна 1006 — 993=13. 30.10. Пусть первые 12 цифр полученных 120-и чисел образуют числа А\, А2, .... ;4|2о, а оставшиеся 108 цифр каждого числа — число В. Тогда надо доказать, что сумма (y?i +... +Л120)-10108 + + 120-В кратна 120. Но А\ + ---+-<4i2o делится на 3 (так как остатки при делении на 3 каждого Д одинаковы, а их 120), а 10|03делится на 40. 30.11. На k-\-\ треугольников при любом k. Пример, для которого k треугольников не хватает, такой: все k точек распо- располагаются на одной диагонали с одной стороны от центра квадрата. 30.12. Решение аналогично решению задачи 29.08. 30.13. Указание. Покажите, что левая часть никогда не дает остаток 5 при делении на 9. 30.14. Указание. Рассмотрите куб, содержащий N3 изю- изюминок, и докажите, что любая плоскость разрезает в этом кубе порядка N2 изюминок. 30.15. Указание. S + 1 =B+ 1)-C+ >)• - •(/»*+ 0- 30.16. Указание. Докажите, что треугольник ABC равно- равносторонний. 30.17. Наименьшая высота равна половине длины диагонали квадрата. Действительно, 4 одинаковых круга рациуса г< — не могут вместе покрыть 4 вершин квадрата со стороной 1 и его центр. 30.18. Сложим несколько чисел вида 21000-10*', где kt=0, а остальные ft,- подбираются так, чтобы сумма (она, очевидно, делится на 21000) не имела нулей на последних m местах. Легко видеть, что это можно сделать для любого т. Таким образом, существует число, делящееся на 21000 и не имеющее нулей на последних 1000 местах спра- справа. Теперь остается заметить, что все осталь- остальные цифры можно просто отбросить (дели- (делимость на 21000 от них не зависит). Второе решение: докажите, что если число из k цифр делится на 2*, то можно к нему слева приписать цифру 1 или 2 так, чтобы полученное число делилось на 2fc+l. По- Поэтому существует далее число, делящееся на 2" Рис 47. 213
и состоящее из единиц и двоек (такая задача предлагалась на Всесоюзной олимпиаде 1972 г.). Замечание. Наша задача в действительности является леммой, помо- помогающей доказать более общее утверждение: если Число m не оканчивается нулем, то существует N,. кратное m и не имеющее в своей записи ни одного нуля. Решите ее сами. 30.19. Указание. Докажите сначала, что если сумма двух целых чисел равна 999 ... 99, то При сложении ни в одном раз- разряде не произошло переноса в следующий разряд. 30.20. Возьмем два «наиболее северных» прожектора (если несколько прожекторов находятся на одной параллели, то вы- выберем «наиболее северные» произвольно). Легко видеть, что, на- направив их на юго-запад и юго-восток, мы осветим всю полу- полуплоскость к югу от них. Теперь легко направить остальные два прожектора так, чтобы вся плоскость была освещена. Замечание. Основная идея этого решения состоит в том, чтобы, разделив множество прожекторов пополам, на «северные» и «южные», свести задачу на плоскости к аналогичной задаче на прямой (осветить всю прямую двумя прожекторами, каждый из которых может осветить полупрямую), кото- которая тривиальна. Аналогично можно свести простраиствеиную задачу 30.34 к ре- решению данной задачи. 30.22. Пусть q — делитель N. Очевидно, если q не делится на р, то q также делитель d (N), в противном случае — является делителем d (N). Отсюда легко вывести, что d (N)^.2d (d (N)). Обозначим d(N)=b. Поскольку Ь не может делиться на числа с>—, за единственным исключением с = Ь, то из неравенства d(b)~^ — следует, что b делится на все числа, меньшие —, кроме, быть может, одного. Отсюда легко вывести, что b?{\2, 8, 6, 4, 3, 2}, a N—pb. Остается перебрать числа вида pb, что мы предо- предоставляем проделать читателю. 30.24. См. решение задачи 47.05. 30.25. Для того чтобы выполнялось утверждение задачи, школьникам необходимо посетить 8 сеансов в одном из кинотеат- кинотеатров и, как минимум, по 2 сеанса в каждом из остальных шести ки- кинотеатров — всего 20 «сеансопосещений». Но за один раз они посещают только 2 сеанса (шестеро — один, седьмой — другой), т. е. за один день они посетят лишь 16 сеансов. 30.26. Указание. Докажите сначала, что при сложении не могло случиться так, чтобы перенос единицы произошел во всех 200 разрядах. 30.27. Указание. Самая длинная последовательность по- получается При АГ| = 1, Х2 = М. 30.28. Указание. Выразить радиус через катеты и гипо- гипотенузу. 214
30.29. Заметим, что нули в конце числа N можно отбрасывать (если N — A-10', то N = A кратно К, а поэтому и А—А крат- кратно К). Пусть N =?р[а2 ... а„-\ап кратно К и ап^\. Вычи- Вычитая из yV'10n+2 число N «в столбик», получим разность Af = a,a2... an_,(an-l)-99(9-an)(9-an_,)... (9 - a2)(!0 - a,). Сложив теперь «в столбик» М с М, получим число п— 1 п—\ Y= 10 999 99 98 ЭОоТГоОО, кратное К. Если бы мы проделали те же действия, отправляясь от чисел ЛМ0п+3 и N, то получили бы число Х = 10 999 ... 99 998 900 ... 000, п — I л — 1 также кратное К. По тогда (А"—У) = 99 кратно К, что и требо- требовалось доказать. 30.30. Занумеруем портреты по порядку: 1, 2, ..., п. Оче- Очевидно, достаточно доказать, что из любого исходного рас- расположения портретов можно получить порядок 1, 2, ..., п. Чтобы его получить, подгоним сначала 1-й портрет ко 2-му (очевидно, это можно сделать без помех, так как 1-й портрет можно пере- переставить с любым другим), затем поведем портреты 1 и 2 вместе до тех пор, пока не подгоним их к 3-му, и т. д. 30.31. Очевидно, достаточно доказать, что ни одно из чисел не вернется на свое место во 2-й, 3-й, ..., п-й строках. Замечание. Легко проверить на примерах, что (п + 1)-я стро- строка, если написать ее по тому же правилу, совпадает с 1-й. Просле- Проследим за судьбой какого-нибудь числа /. При переходе в сле- следующую строку оно сдвигается вправо на одно из (положи- (положительных или отрицательных) чисел п — k, п — m—k, —m. Пусть после нескольких шагов оно х раз побывало в I группе (на одном из мест от 1 до k), у раз во II и z раз в III; тогда оно сдвинулось вправо на N=x(n — k)+y (n — m — fe) + z ( — m) мест. Пусть оно, ни разу не побывав дважды на одном месте, нако- наконец вернулось на исходное место. Тогда Л^=0, причем О^аг^й, 0^y^m — k, 0<z<n — m. Уравнение N=0 имеет два очевидных решения: л; = 0, y = m, z = n — m — k и х=\, y=—J, z=\. Представим эти тройки чисел как координаты векторов ui =@, m, n — т — k) и Ы2=A, —1, 1); каждый из векторов перпендикулярен вектору v = (л — k, п — т — k, — m), откуда легко следует, что любой вектор и — (х, у, z), являющийся решением^ уравнения УУ = О, будет их линейной комбинацией: u = a\U\-\-a2U2. Остается показать, что вектор а\щ +аг«2 не может иметь целых координат х, у, z, удовлетворяющих неравенствам 0^k k O^ — m. Сделайте это сами. 215
0 -а а 0 а 0 0 -а -а 0 0 а 0 а -а 0 30.33. Из условия вытекает, что числа 1, 2, 3, 10, 11, 12 не могут стоять рядом. Следовательно, они стоят через одно по окружности. Но тогда располо- расположить число 4 согласно условию уже не удастся. 30.35. Указание. Докажите сначала, что Рис 48 есть ^ва числа> различающиеся только одной циф- цифрой, к которым приписаны разные цифры. Пусть, например, к числу 11... 1 приписана 1, а к числу 211 ... 1 — цифра 2; тогда неизбежно к числу 33 ... 3 при- приписана цифра 3, а к числу 22 ... 2 — цифра 2, Тогда нетрудно доказать, что к каждому числу приписывается его первая цифра (докажите это сначала для чисел, начинающихся с 3 и состоящих из двоек и троек). В общем случае аналогично доказы- доказывается, что ко всем числам, у которых i-я цифра равна 1, припи- приписывается одна и та же цифра а (а равна 1, 2 или 3); к тем, у которых i-я цифра равна 2, приписывается цифра Ь, а к осталь- остальным — цифра с. 31.01. 4 и 1. 31.02. Единственно возможный вариант показан на рисунке 48. 31.03. Следует выбрать простое число, на которое первые три не делятся. 31.04. Ровно один город должен быть пересадочным; а по- поскольку в связном графе с 50-ю вершинами число ребер не мень- меньше 49, то 49 — наименьшее число авиалиний. 31.05. 66— 12 = 54 ничейных партий. Указание. Каждый нз участников турнира выиграл только по одной партии. 31.06. После четвертого вычеркиваьия должно остаться толь- только одно число. Очевидно, необходимо оставить число 44. Но с другой стороны, поскольку сумма данных чисел делится на 11, легко видеть, что 44 надо было вычеркнуть на первом этапе. Противоречие. 31.07. Нельзя, потому что сумма дуг А\А2А3, АзА^Аь и A$AfAi равна 120° +120° +140° > 360°. 31.08. Запишем данную алгебраическую сумму в вьде S=ATi+( — *2 + *з) + ( — Х* + Хь)+...+{ — ЛГэз+*99) — *100- Чтобы S было максимальным, неположительный член (—лгюо) должен быть равен нулю: агюо=О. Далее, поскольку Xk+i—Xk^Xk, имеем: •S^A'i +*2 + лг4 + —+ *9б + *9в- Равенство будет, если положить jt3 = 2jt2, x5 = 2xt, ..., a:99 = 2jc93. Очевидно, что для получения максимального S надо положить также хъ = 1х\, х* = 2хз, .... Поэтому ответ такой: х\ = 1, ЛГ2 = 2, л:з = 4, ..., лг99 = 298, хюо:=О- 31.09. Выберем систему координат так, чтобы координатные оси не были параллельны ни одному из отрезков. Тогда конец одного из отрезков, имеющий наибольшую абсциссу (или орди- ординату), не может упираться в другой отрезок. 31.10. Существует. Например, условию удовлетворяет произ- произвольная трапеция с основаниями 1 и л/2. Докажем это. Пред- Предположим, что площади Есех четырех треугольников рацио- 216
нальны. Высота указанной трапеции площади 1 равна ——. Высоты треугольников, примыкающих к основаниям трапеции, равны а и ~, где аир рациональны. Итак, а+— =—-—. V2 V2 1+V2 Отсюда следует, что число V2 является корнем квадратного уравнения out +(р + сс —2) х + Р = 0, что невозможно. 31.12. Поскольку длина дорожек в 10 раз больше длины коридора, то либо весь коридор застелен десятикратно (и тогда незастеленных кусков нет вообще), либо есть какая-то точка коридора, застеленная не менее чем 11 кусками до- дорожки. Но тогда эти 11 кусков застилают некоторый сплошной промежуток, так что имеется не более 10 застеленных и соот- соответственно не более 11 незастеленных кусков. Пример, в котором незастеленных кусков действительно 11, постройте сами. 31.13. Можно. Простейшее решение — выбрать те шестизнач- шестизначные номера, у которых сумма цифр делится на 10. 31.14. Указа н и е. рч+ qp = {pp + qp) + (pp+2 — рр); докажи- докажите, что оба слагаемых делятся на (p-{-q). 31.15. Указание. Перелет надо совершать так, чтобы в каждый момент времени в воздухе находилось как можно меньше самолетов, т. е. сажать самолет, как только в баках остальных освободилось достаточно места, чтобы принять все его горючее. 31.16. Первым ходом начинающий должен съесть кучку в 33 конфеты, а другую разделить на кучки в 17 и 18 конфет. В даль- . нейшем он должен играть так, чтобы оставлять партнеру в обеих кучках 5ft+ 2 или 5fe + 3 конфет (проверьте, что он сможет этого добиться). Таким образом, его партнер в конце концов вынужден будет делить кучку в 2 или в 3 конфеты и проиграет. 31.17. Нельзя. Максимальное возможное число классов в данной задаче равно 210= 1024< 1968. Указание. Докажите сначала, что два числа, отличающиеся друг от друга лишь по- порядком цифр, принадлежат одному классу. 31.18. Указание. Докажите по индукции, что если |jc| > 1, то |fn + ,(*)|>IM*)|. 31.19. Множеством центров искомых параллелограммов яв- является объединение трех прямых, каждая из которых есть пере- пересечение двух «серединных» плоскостей для соответствующих пар скрещивающихся прямых. По центру О точки {А} уже строятся однозиачно центральной симметрией прямых {/,}. 31.20. Начинающий должен первым ходом провести диаго- диагональ через центр 1968-угольника, а в дальнейшем поддерживать своими ходами симметрию относительно этой диагонали. 31.21. Указание. Все получающиеся точки будут лежать на одной окружности. 31.22. 49 переходов (дальтоник должен чередовать свои цве- цвета, а красить подряд, начиная с первого участка). 217
r a fi: i p r—Г~л^- : I I I 31.23. x = 4, y = 6. 31.24. Указание. Рас- Рассмотрим систему координат (л:, (), где х — координата точки на траектории пули, / — время. Траектория пули на этой плоскости есть пря- прямая, и лопасть вертилятора попадает на нее,- если отре- отрезок x = о,-, рис. 49. J (это ОДИН ИЗ ИЗО- С браженных на рисунке 49 вертикальных отрезков) пе- пересекается с ней. Итак, тре- требуется доказать, что, како- каковы бы ни были /i, t2, h. In (они соответствуют углам поворота дисков друг отно- относительно друга), существует прямая, пересекающая 4 из нарисованных отрезков. До- Докажите это сами. 31.25. Можно. Для это- этого нужно в каждой груп- группе брать все числа, кроме последнего, по порядку, на- начиная с 1 и исключая уже взятые, а последнее — вида (ш7 — а\ — а2—... — ак). 31.26. Пусть кузнечик сделал 2 прыжка вдоль ло- ломаной ABC, как показано на рисунке 50. Построим точ- точку С", симметричную С относительно ОВ. Легко ви- видеть, что /LABC = 180° — 2а. Отсюда легко получить, что если мы будем отражать путь кузнечика сначала относительно ОВ, затем относительно ОС и т. д., то точки А, В, С, D', Е' и т. д. попадут на одну окружность и будут отсекать от нее равные дуги. Отсюда уже легко вывести нужное утверждение. 31.27. На рисунке 51 показан путь некоторой точки пирога после нескольких операций. Из него видно, что через некоторое время эта точка займет место, симметричное относительно OL тому, которое она занимала вначале. Отсюда уже нетрудно получить нужный результат. 31.28. 990. Указание. Такова разность между числами 218 Рис. 51.
909 909 и 908 919. Чтобы дока- доказать, что искомая разность не может быть больше 990, вос- воспользуйтесь тем, что среди пер- первого миллиона чисел с интерва- интервалом 1001 встречаются числа abcabc, удовлетворяющие усло- условию. 31.30. Искомое множество по- показано на рисунке 52. 31.31. Не более трех. Ука- Указание. Пусть последняя точка А1968 находится между Ak и At. Тогда А1967 находится между рис. 52. Ak-i и Ai-i (за исключением случаев fe=l или /=1, которые нужно рассмотреть отдельно), и потому дуги, возникшие в результате 1968-го разбиения, имеют такие же длины, что и некоторые из возникших раньше. Замечание. Эта задача имеет отношение к одной из теорий современной математики — эргодической теории. В этой теории, в частности, имеется раздел под названием «Перекладывания», в котором изучаются преобразования Т (пе- (перекладывания), переставляющие между собой конечное число отрезков (за под- подробностями отсылаем к книге: Корнфельд И. П., С и и а й Я. Г., Ф о м и н С. В. Эргодическая теория.— М.: Наука, 1980). Любая степень Т" перекладывания Т заданных г отрезков также является перекладыванием, но более мелких отрез- отрезков и в большем количестве. Сколько же различных длин может встретиться среди всех длин этих мелких отрезков? Оказывается, их число не зависит от показателя степени п, а зависит только от исходного числа г пере- перекладываемых отрезков: оно не превосходит 3(г—1). Это утверждение получено в 1983 г. при доказательстве следующего очень важного и интересного математического результата: почти все перекладывания эргодичны. Конструкция, рассматриваемая в задаче 31.31, отвечает перекладыванию отрезков (делая циркулем первую засечку, мы разбиваем окружность иа 2 «отрезка»); здесь л=1968, а число различных длин не превосходит 3(г—1) = = 3B—1)=3. Таким образом, в последние годы получено очень далекое обобщение ут- утверждения задачи 31.31, имеющее важные применения во многих разделах современной математики. 31.32. Выигрывают белые. Для выигрыша они должны вна- вначале двигать короля по маршруту а\—Ы—с\—... до тех пор, пока черный король не сойдет с восьмой горизонтали (если он не сойдет с нее, то белые выиграют на восьмом ходу). После того как черный король сойдет с восьмой горизонтали, белый ко- король должен немедленно занять такое положение, чтобы между королями было четное число горизонталей и вертикалей (напри- (например: 1. Кр а\ — Ь\ Кр Л8 — Л7. 2. Кр Ы — с2) и затем держаться такой симметрии, по возможности приближаясь к черному ко- королю. Тогда легко видеть, что рано или поздно короли займут 219
щ т 1 у//, УУА d т. 1 У//. Ш ш щ У/л уУл ш V//, I 7. у, i ш ш ш У//, ш % '/Л У/Л ш У// У// i Ш \ \л, б) Рис. 53. Y/Л...У'Л ш 1 1 I хоу. в) одно из положений, показанных на рисунках 53, а; 53, б; 53, в при ходе черных. После этого черным придется ходить вверх или вправо — белый король «потащит черного на буксире» и выиграет. 31.34. Предположим сначала, что в числе N поров- поровну единиц и восьмерок, двоек и семерок и т. д. Тогда первое разбиение можно произвести так: 55 ... 5 66 ... 6 77 ... 7 88 ... 8 99 ... 9 li_-:4 33 ... 3 22 ... 2 11 ... 1 00...0 99 ... 9 99 ... 9 99 ... 9 99 ... 9 99 ... 9 сложением можно получить число и третьим шагом задача будет ре- реТогда вторым 99 ... 9 + 9= 100 ... 08 шена. Таким образом, дело сводится к тому, чтобы получить из исходного числа N число с равным (или почти равным) коли- количеством цифр, дополняющих друг друга до 9. Покажем, как это можно сделать, на примере двоек и семерок. Пусть, например, семерок много больше, чем двоек; сделаем следующие три сло- сложения: 1) 2... 2 7 ... 7 7 ... 7 7 ...7 0... О 2..23 5..54 7 ... 7 3) ,334 2 342 ... 2 5 ... 5 7 ... 7 ' 5...50- О 342..23 1.. 10 7 ... 7 2 1 ... 1 7 ... 7 1 ... 1 0... 0 334 2... 2 2... 2 7... 7 Если количество складываемых семерок вначале подобрано пра- правильно, то в итоге число двоек и семерок будет отличать- отличаться не более чем на 7. Таким образом, четвертым сло- сложением мы уничтожим все цифры, кроме нескольких (не бо- более 10), превратив их в девятки. Дальнейшее несложно. 31.35. В общем случае окружность радиуса R касается трех сторон М, продолжения которых образуют треугольник, все вы- высоты которого не меньше 1. А тогда площадь S этого треуголь- 220
1 ника не меньше -^— его полупериметра, а с другой стороны, О S = Rp. Отсюда R~^— (равенство достигается для равносто- о 'роннего треугольника с высотой 1). 31.37. Решим сначала следующую систему уравнений с тре- м я неизвестными: Тройка A, —2, 1) — решение первого уравнения; подстав- подставляя во второе, получаем слева ( — 6). Следовательно, уменьшив эти числа в Х=У—6 раз, получаем решение (х\, Х2, х3) выписан- выписанной системы (при этом х\ = хз, Х2 =—2.vi). Учитывая вышеизложенное, решим следующую систему трех уравнений с девятью неизвестными: ( Первая и последняя тройки неизвестных совпадают с найден- найденным решением (х\, Х2, хз), вторая тройка получается из этого решения умножением на (— \/2). Тогда первые два уравнения выполняются автоматически, а левая часть последнего равна не- некоторому числу X. Уменьшая все 9 неизвестных в У^ раз, полу- получаем значения неизвестных (jti, Х2, ..., хе, хд) — решение этой системы. Затем поступаем аналогично: увеличиваем на 1 число уравнений и утраиваем число неизвестных; при этом все 27 не- неизвестных делятся на три группы по 9; первая и третья группы совпадают с найденным решением, а средняя полу- получается из них умножением на —у2. Тогда сумма этих 27 чисел, сумма их кубов и сумма их пятых степеней равны 0, а сумма седьмых степеней равна некоторому числу Я; деля все числа на Цк, получаем решение этой системы. Аналогичными добав- добавлениями уравнений по одному и утраиванием числа неизвестных получим систему из 11 уравнений с З10 не- неизвестными, которая имеет решение (попробуйте подсчитать, сколько времени понадобится, чтобы выписать полностью от- ответ). 31.38. Может. Сначала отсекаем у треугольника 3 уголка, потом у шестиугольника — 6 уголков, у 12-угольника — 12 угол- уголков и т. д., всего 19 раз. В результате получится 1 +3 + 3-2 + + 3-22 + ... + 3-2|8> 106 многоугольников. Любая прямая пере- пересекает не более двух треугольников каждого ранга, а всего не более 2-19 + 1 <40 многоугольников. 31.39. Предположим противное. Легко видеть прежде всего, что мелом отмечены либо все 8 вершин, либо ни одной. Таким образом, помимо вершин, отмечены 100 или 92 точки. Но легко 221
a) V Рис. 54. видеть, что вместе с каждой точкой не в вершине придется добавлять на других гранях 5 (для точки в центре грани), 11 (для точки на середине ребра) или 23 точки (в остальных случаях) так, что их общее число будет делиться на 6. Но ни 100, ни 92 на 6 не делятся. 32.01. Ладья должна двигаться по средней линии доски, отставая от слона на ход коня. 32.02. Указание. Провести в АЕКА1 медианы MF, EL и NK. 32.03. Еще 2 команды, набравшие по 4 и 3 очка. Указа- н и е. Среднее число очков, приходящихся на одну команду, на 1 меньше числа команд. 32.04. Никакое число, оканчивающееся четырьмя одинаковы- одинаковыми цифрами, не делится на 16. 32.05 и 32.07. См. решение задачи 32.15. 32.08. Ладья должна двигаться по средней линии доски, каждый раз становясь по диагонали (на ход слона) от коня. 32.09. Если ВМ и АН — равные медиана и высота, то, опустив из точки М перпендикуляр МК на сторону ВС, полу- получаем: МК=-^-АН=-^-ВМ. И тогда из треугольника МКВ сле- следует, что /LMBK — 30° (рис. 54, а). Отразив симметрично вер- вершину В относительно точки М, получим точку В', причем Z.АВ'В = 30°. Геометрическое место (множество) точек В' есть дуга окружности в 30°, построенная на АВ как на хорде. Теперь геометрическое место (множество) точек С находится просто: оно получается перемещением множества точек В' на вектор АВ = ВЧ2 (рис. 54,6). 32.10. Можно. Например: —о, Ь, Ь, —а, Ъ, Ь —о, Ь, Ь, —а, Ь, где а и Ь таковы, что 2Ь—а>0 и 136— 7а<0. Одно из решений: а—25, 6 = 13. 222
32.13. Нельзя. В самом деле, пусть а\, ..., aso — такие числа. Тогда а\-\-а2-\-... + аз4> 0, a ai+а2 + ... + азо<СО. Следова- Следовательно, аз\ + aj2 + азз + аз4 > 0. Аналогично докажем, что Язе + азб + азт + азе>0, 039 + 040 + 041 + G42>0 и т. д. Складывая эти неравенства, получим, что 031+032 + ... + aso>0, что, как лег- легко видеть, противоречит второму условию. 32.15. Пусть стена образует некий многоугольник ABCD ...E. Заметьте, что каждый из углов ABC, BCD и т. д. содержит внутри себя угол 37-угольника, описанного вокруг одного из городов, и потому не меньше, чем угол правильного 37-угольника, т. е. не меньше л—-^ . Поскольку сумма внешних углов в любом мно- многоугольнике равна 2л, а каждый внешний угол не превосходит -^- , то число углов не меньше 37. 32.17. Докажем, что последовательность, указанная в задаче, может быть бесконечной только при ai+a2 + ... + aioo—0. Дейст- Действительно, если а\ +...+аюо= — е<0 и Ne>a\ для некоторого натурального /V, то O^ai + ... + aloow+1 = ai — /Ve<0 — противо- противоречие. Отсюда — аюо — аээ^О и laiool = —0100^099= |аээ1- 32.18. Достаточно показать, что последовательность из пяти перфокарт всегда можно укоротить. Из четырех лежащих спра- справа перфокарт могут совпадать только первая и четвертая, иначе сразу выбрасываются две. Пусть они действительно совпадают по цвету, т. е. цвета идут в порядке ...abca. Тогда достаточно подложить справа карту цвета с, чтобы можно было выбросить две карты (с и а). Остается рассмотреть случай, когда 4 край- крайние различны, а пятая совпадает с одной из них. Тогда есть две возможности: ... dabcd и ... cabcd. В первом случае нужно подложить справа цвета cbc, а во втором — цвета bd. 19693 32.19. Существует. Например, h=-r^s- Указание. При любом п дробная часть числа 1969" 1968 равна 1968 32.20. Ответ показан на рисунке 55. 32.21. Указание. щ(—+—+ \ а Ь 32.23. Сможет. Один из способов сделать это показан на рисунке 56. См. 32.37. 32.24. Первым ходом первый игрок вычеркивает 9 чисел от 47 до 55. Остальные числа разбиваются на две группы: от 1 до 46 и от 56 до 101. Для любого k, вычеркнутого вторым игро- игроком, первый вычеркивает |55 — k\\ раз- разность оставшихся двух чисел равна 55. 223 Рис. 55.
Л вг' • 1С А1 Рис. 56. 32.25. Чтобы найти жемчужину за 33 разреза, достаточ- н о сделать их параллельно друг другу на равных расстояниях. Доказательство неразрешимости задачи за 32 разреза основано на следующем утверждении: как бы ни бы- были проведены k разрезов, в получившиеся k-\-l частей можно вписать не более k +1 кругов, суммарный радиус которых ра- равен 10 см. Применим индукцию. При & = 0 это очевидно. Пусть это верно для k разрезов и еде чан еще один разрез. Если он не затрагивает вписанных кругов, то они удовлетворяют усло- условию; если же один из кругов оказался разрезанным, то заменим его двумя, как показано на рисунке 57. При этом суммарный радиус кругов не меняется, и утверждение доказано. Таким обра- образом, после 32-х разрезов можно будет построить 33 круга сум- суммарного радиуса 10 см. Но тогда радиус хотя бы одного из них больше 3 мм, и если жемчужина находится в нем, она останется необнаруженной. Замечание. Если бы требовалось сделать сразу все разрезы, то для обнаружения жемчужины понадобились бы те же 33 разреза. Однако для этого случая решение коренным образом меняется (при до- доказательстве специально рассмотрите поверхность шара). 32.27. Указание. Как бы ни играли партнеры, тот, за кем очередь, всегда смо- сможет сделать свой ход, если есть еще хоть одна незамазанная клетка. Поскольку все- всего клеток на доске 63 (не считая клетки а\), выигрывает независимо от хода игры начинающий. 32.28. Указание. Все четыре указан- указанные прямые пересекаются во второй точке пересечения окружностей, описанных около этих пятиугольников. 224 Рис. 57.
32.29. Предположим (это допустимо, если применить индук- индукцию), что для любого /< 1969 число последовательностей, кон- кончающихся на I, меньше I. Рассмотрим все последовательности, кон- кончающиеся на 1969, и отбросим в них последний член. Тогда все эти последовательности кончаются на одно из чисел 1, 2, 3, .... [УГ96Э]. Для каждого из этих чисел количество последовательно- последовательностей, кончающихся на него, меньше У1969. Поэтому общее их число меньше чем [У 1969]- y'l 969 < 1969. 32.30. Начнем с правого края. Самый правый кубик черный. Действительно, если бы он был белым, то по условию в любом на- наборе из одиого кубика было бы не менее одного белого, т. е. все кубики были бы белыми. Аналогичное рассуждение показыва- показывает, что первые 4 кубика справа черные (иначе среди любых че- четырех кубиков был бы белый и число белых кубиков было бы не меньше 25). Пятый кубик белый: в противном случае в любой пятерке было бы не более одного белого кубика, т. е. число белых кубиков не превосходило бы 20. Рассуждая так и далее, нетрудно доказать, что среди первых k кубиков справа белых ровно —-- . Но отсюда следует, что цвет каждого кубика определяется L 'не J однозначно. (См. задачу 32.40.) 32.32. Докажите самостоятельно лемму: если через точку О провести три различные окружности одинакового радиуса г, то остальные три точки их попарного пересечения — вершины тре- треугольника, радиус описанной окружности вокруг которого ра- равен г, а точка О является точкой пересечения его высот. (Для доказательства леммы воспользуйтесь тем, что этот треуголь- треугольник равен треугольнику с вершинами в центрах трех проведен- проведенных окружностей.) Теперь для построения по заданным точ- точкам А, В, С четвертой вершины параллелограмма ABCD доста- достаточно провести пять окружностей. Сначала проведем две окруж- окружности через точки Л и В и третью — через точки В и С; пусть ее центр О\ и она пересекает первые две окружности в точках Е и F. Проведем теперь еще по одной окружности через точки С и ? и точки С и F — они и пересекаются в искомой точке D. ABCD — параллелограмм, так как точки А, Е, F, D лежат на одной окружности (с центром О2) и ABA.EF, CDA.EF, AB CD OO 32.33. Если дана последовательность х длины п, то обратной к ней х назовем такую последовательность длины п, в которой нули стоят на местах единиц у л, а единицы — на местах нулей. Произведением ху двух последовательностей а: и у на- назовем последовательность г, полученную поразрядным умноже- умножением х к у. Доказательство утверждения задачи вытекает из следующих лемм. Лемма 1. Среди наших 2"~' последовательностей содер- содержится ровно одна из двух последовательностей: х или х (х — про- произвольная последовательность длины п). Действительно, пред- 8 Заказ 247 225
положив противное и добавив к х и х произвольную последова- последовательность у, получаем: х-х-у = 0 — противоречие с условием. В частности, последовательность 0..000..0 не принадлежит данной системе. Лемма 2. Если х и у — две последовательности из нашей системы, то последовательность ху также принадлежит нашей системе. Доказательство аналогично доказательству леммы 1 и проводится с ее использованием. Лемма 3. Пусть х\, х2, .... хг-1 — данные в условии после- последовательности. Тогда их произведение — последовательность Х=х\ ... лег1-1 состоит из одной единицы и (п — \)-го нуля. Дейст- Действительно, ХфО по предыдущим леммам. Если X содержит не меньше двух единиц, то в этих двух разрядах каждая последо- последовательность Xi содержит единицу и тогда, расставляя в остав- оставшихся (п — 2)-х разрядах нули и единицы произвольным обра- образом, получим только 2"~2 различных последовательностей, а не 2"~\ как требует условие.- Лемма 3 тем самым доказывает, что все последовательности xt имеют единицу в том единственном разряде, в котором стоит 1 у X. 32.34. Сможет. В самом деле, сторонников президента име- имеется 200 000. Пусть самые мелкие группы составляют по 5 че- человек, тогда для победы президента нужно, чтобы в группе было 3 его сторонника. Таким образом, 200 000 сторонников дадут (если их правильно распределить) 66 666 «выборщиков первого порядка» из общего числа 4 миллиона. Вновь разобьем эти 4 миллиона на группы по 4 человека. В них для большинства нужно также 3 человека, и Мирафлорес сможет получить 22 222-х «выборщиков второго порядка» из общего числа 1 мил- миллион. Далее, деля на группы то по 5, то по 4 человека (порядок безразличен), он получит: 7407 выборщиков из 200 000; 2469 вы- выборщиков из 50 000; 823 выборщика из 10 000; 274 выборщика из 2000; 91-го выборщика из 500; 30 выборщиков из 100; 10 вы- борщиков из 25; 3 выборщика из 4, обеспечив, таким образом, себе победу. (Подумайте, мог бы победить Мирафлорес, если бы число его сторонников было равно 38-52= 164 025.) 32.35. Опишем вокруг 1000-угольника окружность; его вер- вершины разбивают окружность на 1000 одинаковых дуг, в кото- которых будем измерять длины хорд, опирающихся на эти дуги. Рас- Рассмотрим треугольник, содержащий центр окружности. Одна из [I ООП ~1 —-—[ + 1 =334. К ней при- J J мыкает треугольник, у которого из трех сторон средняя по величине сторона имеет длину, не меньшую -^-; к ней при- примыкает треугольник, из трех сторон которого длина средней 334 стороны не меньше ——, и т. д.; у восьмого треугольника сред- 226
а) б) Рис. 58. няя по величине сторона не меньше >1. Легко по- 2е казать, что указанные 8 диа- диагоналей различны по длине. 32.36. Пусть сначала первый мудрец вычеркнет все нечетные числа, за- затем второй — все числа, большие 512 (их оста- осталось как раз 256), затем пер- первый — все числа, не де- делящиеся на 4, а вто- второй — все числа, большие 256, и т. д. Тогда в итоге второй уплатит первому, как нетрудно видеть, 32. Придумайте сами обобщение этой стратегии для первого мудреца на случай, ког- когда второй играет произвольным образом, чтобы доказать, что первый всегда может добиться разности не меньше 32, и об- обобщение стратегии второго мудреца на случай, когда произ- произвольным образом играет первый, чтобы доказать, что он может добиться разности не больше 32. 32.37. Да. На рисунке 58 показано, как перегнуть треуголь- треугольник так, чтобы получить из него замкнутый отрезок, пройден- пройденный дважды. Аналогичным путем легко из отрезка получить шестиугольник. (Ср. с задачей 32.23.) 32.38. См. решение задачи 32.25. 32.39. Можно. Указание. Начните с крайнего ободка до- доски. Те клетки, которые неясно, как заполнить, проще всего за- заполнять нулями (при этом в дальнейшем не возникнет никаких трудностей). 32.40. Возьмем k белых кубиков и 1969 — k черных и расставим их в ряд, располагая белые на местах (считая справа) с номерами —-—, 2—т—, 3—т—, •-• (нецелые числа следует округлять в сторону увеличения). Нетрудно показать, что такое расположение удовлетворяет условию. Полагая k=0, 1, 2, ..., 1969, мы получим 1970 расстановок. Можно показать, что других расстановок, удовлетворяющих условию, нет (см. задачу 32.30). 33.01. Нельзя, так как белая шашка при взятии черной сдви- сдвигается всегда на четное число вертикалей. Поэтому если она взяла одну шашку, то она всегда будет отстоять от второй на четное число вертикалей и не сможет ее взять. 33.02. Воспользуйтесь тем, что среди чисел 1, 2, .... 99 нечет- нечетных больше, чем четных. 33.03. 53°, 63°, 64° (на 60° меньше углов из условия). 33.04. Указание. Класть гири парами (по одной на чаш- чашку) , причем класть более легкую гирю на перевешивающую чаш- чашку весов. 8* 227
33.05. Указание. Занумеруем кот- коттеджи так, чтобы человек из первого переехал во второй, человек из второ- второго — в третий и т. д. Найдется такое k, что человек из Л-го коттеджа переезжает в первый, и на этом цепь замкнется. Те- Теперь докажите утверждение задачи для первых k коттеджей, а затем рассмотрите остальные. 33.07. Пусть L и М — точки касания Рис. 59. АЕ и DE с окружностью (см. рис. 59). Легко видеть, сравнивая отрезки каса- касательных, что BK + A-L = KC + DM = l. Отсюда EL — BK= = АЕ— 1—число целое и ЕМ —КС—тоже целое. Поскольку E-L = EM, отсюда вытекает, что В К — КС — 0,5. 33.09. Только при k=\ или /г = 2. Действительно, если иа левой чашке лежат гири а\, ..., at общей массой 2, а на пра- правой — гири Ь\, ..., bk общей массой S, то из условия следует, что О<2 — S^2(a,- — Ы) при каждом. t=l, ..., k. Складывая к таких неравенств, получаем: k B—S)^2B— S), откуда k^.2. 33.10. Возьмем в качестве А того, кто проиграл больше всего партий, а в качестве В — того, у которого А выиграл. Докажите сами, что тогда удастся найти С, удовлетворяющего условию. 33.11. Зафиксируем произвольного короля и предположим для определенности, что он чаще ездил влево, чем вправо. Тогда нетрудно видеть, что любой король, живущий правее его, проехал за год больший путь. Отсюда сразу следует, что король, живущий не на краю, проделал не самый большой путь. 33.13. Докажем, что все цифры числа, кроме, быть может, первых 50-ти,— нули. Для этого возьмем 51-значное число А и заменим его последнюю цифру а нулем — получим число А'. Числа А и А', а значит, и их разность а кратны 250, откуда а=0. Следовательно, исходное число кратно 2999. 33.15. Указание. Не более чем за 7 шагов таракан вы- выясняет, в каком из четырех квадрантов находится Истина, а за- затем начинает двигаться параллельно сторонам этого квадранта и делает не более D -\J2<C-k-D шагов. 33.16. Указание. Воспользуйтесь тем, что суммарная мас- масса гирь во всяком случае не больше 19-70=1330 г, и докажите, что существует не менее 100 различных по массе наборов, ко- которые составить не удастся. 33.17. Пусть МК и PL — эти хорды (см. рис. 60). Тогда PK-PM = (PM'f=(MP'f=M-L-MP, т. е. PK=ML. Поэтому PL=MK. 33.18. Если бы все цифры этого числа были различны, то их сумма равнялась бы 0+1 +2 +••• + 9 = 45, т. е. это число де- делилось бы на 9. Однако ни одно число в цепочке на 9 не делится. 228
А а) 1 I 5) Рис. 1 т 61. Рис. 60. 33.19. Проведем ориентированную прямую /, по разные сто- стороны которой расположено по 100 точек (считаем эту прямую вертикальной). Все точки слева от нее покрасим в красный цвет, все точки справа — в синий. Проведем прямую т, пер- перпендикулярную /, и спроектируем на нее наши точки. Начнем теперь непрерывно поворачивать прямую / так, чтобы в каждый момент слева и справа от нее (по отношению к ориентации на /) было по 100 точек, и перекрашивать точки так, чтобы всегда слева от / они были красными, а справа — синими; вместе с / движется и перпендикулярная ей прямая m со спроектированны- спроектированными на нее точками. В первый момент, когда в проекции на пг сольются синяя и красная точки, дадим соответствующим точкам номера 1 и 101 и отбросим их. Затем отметим номерами 2 и 102 следующие две разноцветные точки, проекции которых на m сольются раньше всех, и т. д. Докажите, используя тот факт, что после поворота прямой / на 180° исходные красные точки станут синими, а синие — красными, что прямые A, 101), B, 102), .... A00, 200) попарно пересекаются. 33.20. Указание. Отметьте для начала одну строку (или столбец), а именно ту, в которой число крестиков минимально среди всех строк и столбцов. 33.21. Существует признак делимости числа на 999...99, аналогичный признаку делимости на 9: число делится на 999...99, в котором k девяток, тогда и только тогда, когда сумма его граней по k цифр (грани идут справа налево) делит- делится на 999...99. Например, 24 354 делится на 99, потому что 2 + 43 + 54 делится на 99. (Докажите этот признак сами.) Но легко видеть, что если у числа не более 8 цифр отличны от нуля, то сумма граней не может делиться на 999 999 999. 33.22. Возьмем две произвольные диаметрально противо- противоположные точки А к В. Очевидно, сумма расстояний от них до любой из отмеченных точек больше 2. Поэтому сумма расстоя- расстояний от одной из них до всех отмеченных точек больше 100. 33.23. Смогут. Для этого папа должен контролировать до- дорожку АВ так, чтобы Коля не смог проскочить ни через узел А, ни через В (см. рис. 61, а). Тогда мама будет бежать за Колей 229
Рис. 63. фактически по фигуре, изображенной на рисунке 61, б, и, ко- конечно же, поймает его. 33.24. 1970 разрезов. Указание. Сумма углов квадрата равна 2л, и легко сообразить, что каждый разрез увеличивает общую сумму углов всех частей еще на 2л. Таким образом, пос- после k разрезов получится (ft + 1) многоугольников, а сумма углов будет равна 2л(?+1). У всех 30-угольииков сумма углов сос- составляет 73 -28л. Остальные многоугольники, число которых равно (fe+ 1)—73, имеют сумму углов, не меньшую чем (k — 72)л. Отсюда получаем: 73 • 28л+(ft — 72) я<2л (k + 1), откуда k ^73 -27 —1 = 1970. Легко видеть, что эта оценка точна. 33.25. Расположив придворных по окружности в том порядке, в каком они следят друг за другом, и предположив, что их коли- количество п четно, получаем, что придворный с номером -—- сле- следит за тем, кто следит за первым придворным, хотя им должен быть придворный с номером п. (На рисунке 62 изображен слу- случай л = 8.) 33.26. См. задачу 33.22. 33.27. Сторожа поймают обезьяну, если примут следующий план. Вначале они должны занять вершины А к В (см. рис. 63); очевидно, можно считать, что обезьяна находится в нижней ча- части рисунка. Теперь один сторож должен спускаться вдоль АС, а второй контролировать отрезок АВ, чтобы обезьяна не проско- проскочила через А или В (убедитесь сами, что это возможно). Даль- Дальнейшее просто. 33.28. 2500 деревьев. 33.29. Будем рассматривать только те способы, при которых получается 4 трехзначных числа и (80—4*3):2=34 двузначных. Все такие суммы меньше чем 4000 + 3400 = 7400. Докажите, что существует более 10 Q00 разрезаний указанным способом. 33.30. Указание. Предположив, что ломаную протащить можно, рассмотрите тот момент, когда отрезок, соединяющий концы ломаной, окажется под углом 45° к сторонам угла. 33.31. Пусть из деталей составлен некоторый замкнутый или незамкнутый путь. Очевидно, плоскость можно разбить на квад- 230
А А1 У В1 / / /о 4' с 7 Рис. 64. о) б) Рис. 65. раты так, чтобы детали были вписаны в них, как показа ко на рисунке 64. Закрасим все квадраты в белый и черный цвет в шахматном порядке. Заметим теперь, что поезд, едущий по этой дороге, будет обходить все белые квадраты по часовой стрелке, а все черные — против часовой (или наобо- наоборот). Отсюда сразу следует, что требуемого условием пути не существует. 33.32. B + 2-^/2) м (см. задачу 33.30). Подходящим куском проволоки будет, например, сегмент окружности ширины 1, стя- стягивающий хорду длины B+2-^/2) м. 33.33. Заметьте, что порядок, в котором мы меняем знаки, безразличен (проверьте это на примере одной строки и одного столбца). Поэтому можно считать, что мы вначале переменили знак в k строках, а затем в / столбцах. Но после этого число минусов будет равно (/г + 0'ЮО — kl. Приравнивая его к числу 1970, мы видим, что нужно решить в целых числах, не превы- превышающих 100, уравнение kl —100 (k + l)+1970=0 или A00 — k) X Х(Ю0 — /)=8030, которое, легко видеть, неразрешимо. 33.34. 1699 разрезов (см. решение задачи 33.24). 33.35. Смогут. Для этого один паук должен контролировать ребро АВ, а другой — ребро СС так, чтобы муха не могла пройти ни через одну из вершин А, В, С, С (рис. 65, а). Тогда на оставшейся части куба нет ни одного замкнутого пути (рис. 65,6), так чго муха не сможет уйти от третьего паука. 33.36. Площадь данной сферы равна 400л. Поэтому площаць описанного 19-гранника больше 400л, а площадь некоторой его грани больше Пусть А — точка касания грани со сферой и В — такая точка грани, что AB>-\[2\ (такая точка существу- существует, так как иначе вся грань лежала бы внутри круга радиу- радиуса и имела бы площадь меньше 21л<—-л). Если О ~ центр шара, то ВО>-\[Г21 = И. Отсюда легко следует, что дли- длина отрезка, проведенного из В через О до пересечения с мно- гограгшиком, больше 21. (Можно доказать, что эта длина боль- больше чем 22.) . 231
л 0 0 0 0 0 0 0 0 *г -*г 0 0 0 0 0 0 0 0 *J -*з 0 0 0 0 0 0 0 0 0 0 0 0 0 0 0 0 *s 0 0 0 0 0 0 0 0 0 •\ 0 0 0 0 0 0 0 0 ¦*» 0 0 0 0 0 0 0 0 х» 0 0 0 0 0 0 0 0 х„ 0 0 0 0 0 0 0 0 33.37. Пусть 10 101 010 101 = то Рис. 66. =А, k=A'-B. Если fe<1012, В ^99, и утверждение очевидно. Пусть теперь fe>1012; перенесем первую его значащую цифру на 12 разрядов вправо. Это означа- означает, что из k вычтено число, про- пропорциональное 999...99 A2 девя- девяток), так что оно не перестало делиться на А. С другой сторо- стороны, легко видеть, что число зна- значащих цифр при этом не увели- увеличивается: Отсюда сразу следует утверждение задачи. 33.38. Смогут. Разберитесь в этой задаче сами (ср. с зада- задачами 23, 27, 35). 33.39. Воспользуйтесь тем, что остаток от деления на число вида Bfe + l) всегда не меньше 1, от деления на Dfe + 2) — не меньше 2, от деления на (8fe+4) — не меньше 4 и т. д. 33.40. Достаточно 100 шиллингов (можно просто обводить квадратики на диагонали таблицы). Указание. Докажем, что 99 шиллингов не хватит. Предположим, что волшебная таб- таблица заполнена какчми-то числами х\, х2, ..., хюо так, как пока- показано на рисунке 66. Тогда, какой бы квадрат мы ни обвели, задав один вопрос, можно узнать только одно из чисел xi (то, которое стоит в последнем столбце обведенного квадрата). Поэтому после 99 вопросов нам будет известно только 99 чисел и сумма чисел по диагонали будет неизвестна. 33.41. Число 0. Указание. Если умножить любое число п на однозначное а так, что a-n^itf1, а (а — 1)п<10*, то, вычерк- вычеркнув первую единицу, мы получим число, меньшее исходного. Таким образом, описанной операцией можно уменьшить любое наперед заданное число. 33.42. На олимпиаде выше оценивались те работы, в кото- которых предлагалось размещение большего количества ковров. Наибольший известный ответ: 680 ковров. 33.43. Для того чтобы расположить кости согласно условию, нужно всегда располагать четное число рядом с нечетным. Но на костях домино 32 четных числа и только 24 нечетных. По- Поэтому задача неразрешима. 33.44. Нельзя, так как сумма 1+2 + 3+... + 33 нечетна, а сумма чисел в каждой группе должна быть четной. 33.45. Легко поставить не менее трех переключателей в положение «верх» (сначала поставить вверх пару соседних, за- затем ¦— пару диагональных). Если дверь в пещеру не открылась, значит, 4-й переключатель стоит в положении «низ». Тогда Али- Баба должен вставить руки по диагонали; если нащупан «низ», 232
13 74 24 ;¦•; 2-7 1-1 V1 ы 1:1 v-г 1:1 1:1 1-0 0:4 его следует переставить в поло- | | i \2 ...i2\ jj \n ... гь\ 25 жение «верх» и войти в пещеру, если же оба переключателя стоят в положении «верх», один из них ставится в положение «низ». '¦ vz После этого, очевидно, два со- 72 седних переключателя стоят в по- положении «верх» и два переключа- переключателя — «низ». Затем Али-Баба вставляет ру- руки по стороне квадрата: если оба переключателя в одном и том же положении, он щел- 25 0-'4 0:1- hi 4-0 кает ими и открывает вход. Если же они в разных положе- положениях, то он щелкает ими, а затем Рис. 67. вставляет руки по диагонали и щелкает обоими переключателями в последний раз. 33.46. Соединив точки, в которых расположены фотоаппара- фотоаппараты, получим 1000-угольник (иначе сразу найдется фотоаппарат, из которого видны не все остальные). Но сумма всех углов 1000-угольника равна 180° • 999 > 179° • 1000, т. е. каждый аппа- аппарат не может «обозревать» сразу все остальные. 34.01. Пусть А я В — концы диаметра города. Соединим А, В ломаной, проходящей внутри города; город разобьется на две части. Никакой часовой не видит ни одного часового, находя- находящегося в другой части города. Остается взять часового, нахо- находящегося в той части города, в которой не более 500 часовых. 34.02. См. решение задачи 34.11. 34.03. Последнее место. На рисунке 67 указана соответствую- соответствующая таблица турнира. 34.04. Проведем из каждой точки по 4 отрезка в 4 соседние точки C и 2 отрезка, если точка стоит на краю или в углу). Легко видеть, что число отрезков, проведенных из красных точек, равно числу отрезков из с и н и х точек. Теперь мы долж- должны выбросить отрезки, соединяющие красную точку с синей. Очевидно, при этом равенство не нарушится. 34.05. У к а за н и е. Докажите сначала, что feEI09070l)+ + k B1090701) четно. 34.06. После каждой операции сумма всех чисел 25-уголь- ника удваивается. Сумма всех исходных чисел равна единице. За 100 операций, следовательно, эта сумма станет равной 2100. Поэтому одно из 25 чисел будет не меньше -^-> ioo"^ Ю28 > Ю20. 34.07. Стороны многоугольника М являются средними ли- линиями треугольников, образованных соседними сторонами и ма- малыми диагоналями многоугольника Р. Поэтому периметр М ра- равен полусумме длин всех малых диагоналей и больше полупе- 233
Рис. 68. риметра «звезды» (рис. 68), со- составленной из кусочков малых диагоналей. Периметр «звезды» в свою очередь превосходит пе- периметр Р; следовательно, пери- периметр многоугольника М больше 34.08. Возможны все значе- значения п, отличные от 2* (k^2). 34.09. Справедливо утверж- утверждение: разность двух чисел, составленных из одних и тех же цифр, кратна 9. Пусть из числа 2* переста- перестановкой цифр получилось число 2" и n>k. Тогда разность 2" —2* кратна 9, откуда и 2"~*—1 делится на 9. Но 2""*=¦»?-<;9, поскольку при перестановке цифр число не может увеличиться в 9 или более раз. Получили проти- противоречие. 34.10. Первое решение. 2" •V2=[2n--v/2]+ ее, 0<сс<1. Если а<-^-, то [2"+l.V2]—чет- „ 1 ное число. Если же а> —, Рис. 69. то z T --^z = z |z »-^2]+2а и [2" + 1 •-$.] — нечетное число, причем {2а}<а. Умножая 2"—\{2 на последовательные степени двойки, мы будем каждый раз уменьшать дробную часть результата, пока эта дробная часть не станет меньшей —. Второе решение. Докажем, что в по- последовательности а* бесконечно много четных чисел. Дейст- Действительно, гели в этой последовательности с какого то места идут только нечетные числа, то их двоичная запись оканчивается на 1, а тогда в двоичной записи а* с какого-то места идут одни единицы. Это противоречит иррациональности -ф.. 34.11. Пусть отрезок А\Ач пересекает сферу в точках а\. а-/', отрезок А2Аз — в точках а'г, а"; отрезок АъА* — в точках аз, а"\ ...; отрезок Ап^хАп — в точках а'п-\, а", и, наконец, отрезок А:,А\— в точках а'п, a" (dhc. 69). Нам надо доказать, что из равенств А\а[~А\а", А<иЛ = Ага", .... An.-ia'n-i=Aa-ia'n-i следует, что PaccMJTpiiM плоскости треугольников О А \Ач, ОА2А3, — и ОА„А\ 2.44
и в них — равнобедренные треугольники Оа[а", ... , Oa'n-ia", Оа'па'{. Все они равны между собой, откуда, в частности, следует равенство а'па'п-\=а'па". Применим теперь теорему о том, что если из точки X вне сферы проведен луч, пересекающий ее в двух точках Y и Z (считаем XY <.XZ), то произведение XY-XZ есть постоянная величина, не зависящая от того, какой луч проведен (а зависящая только от выбора точки X). Следовательно, Апа'пУ. XАпа" =Апа" • А„а'п-1, откуда Апа'п (Апа'„ + а'па")=Апа" (Апа" + а"ап-1). Если теперь предположить, например, что Апа'п <. Апа", то левая часть последнего равенства будет меньше правой и получаем противоречие. Точно так же не может выполняться противопо- противоположное неравенство. Тем самым утверждение задачи доказано. 34.12. Утверокдать, что Петя лжет, нельзя. Простейший при- пример приведен на рисунке 70. Основная идея построения подобных примеров состоит в том, чтобы оказалось нечетное число уголков. Тогда при утере одного уголка оставшиеся уголки можно разбить на пары и каждую пару поместить в прямоугольник 2X3. 34.13. Положим yk=—k—i-; очевидно, Зуп— уп-\ =п. Легко видеть, что \xk — yk\—-z-\Xk-\—yk-\\ и потому при больших k О условие XkZ&yk выполняется с достаточно высокой точностью. В частности, *i97i~f/i97i =985,250000. 34.14. k=\. Указание. Отрезая несколько раз малыми диагоналями неостроугольные треугольники, получите остро- остроугольный треугольник со всеми k точками внутри него. 34.15. При правильной игре выигрывает первый игрок. Его стратегия: поскольку своим ходом он может взять 1, 2, 3, 4 или 5 спичек, то каждым своим ходом независимо от хода второго игрока он оставляет ему число спичек, кратное 6. Поэтому через конечное число ходов он оставит второму игроку ровно 6 спичек и затем, после хода второго, возьмет ос- оставшиеся спички, чем закончит игру. 34.16. Существует. Нетрудно сообразить, а потом проверить умножением «в столбик», что таким числом является 111 111 111. 34.17. Повернем квадрат ABCD вокруг его центра на 90°. Тогда повернутый квадрат совме- совместится с исходным, а прямые АН{, ВН2, СН3 и DH4 перейдут в прямые АО, ВО, СО, DO, которые пересекаются по условию. 34.18. Число бактерий через k минут равно (и — k)-2k. Поэто- Поэтому через п минут бактерий не останется. 34.19. Пусть на горизонталь- Рис. 70. 235 'г1 1 7
Рис. 71. Ш ной прямой / три идущих подряд узла закрашены в разные цвета а, Ь, с (если таких трех узлов нет, то прямая I — искомая). Тогда легко видеть, что три узла над ними должны быть закрашены в цве- 1— та с, d, а соответственно, а следующие три — опять а, Ъ, с (см. рис. 71) и т. д. Теперь ясно, что любая из прямых I, II, III удовлетворяет условию за- задачи. 34.20. Решение приведено на рисунке 72. Сначала размещаем прожекторы 1, 2, 3 и 4 требуемым в условии способом, а затем добавляем к ним нужное количество других прожекторов E, 6, 7, ...). 34.21. Забудем на время, что наше число 29-значное, и оп- определим, какое наибольшее число цифр оно могло бы иметь. Пусть k (А) — число цифр, равных А. Из условия следует, что если цифры В и С симметричны относительно середины числа, то k(B)=C и k (С)=В. Но тогда эти цифры всюду должны стоять на симметричных местах; поэтому k(B)=k(C) и В = С. Отсюда сразу следует, что всегда k(A)=A, причем цифра, стоя- стоящая на 15-м (среднем) месте, встречается нечетное число раз, а все остальные цифры — четное число раз. Но тогда число цифр числа X не превосходит k B)-\-k D)-\-k F) +ft (8)-f- -f-ft(9)=2 + 4+6 + 8+9=29. По условию их ровно 29. Значит, в нашем числе ровно 2 двойки, 4 четверки, 6 шестерок, 8 вось- восьмерок и 9 девяток. Сумма его цифр равна 22 + 42+6 +82 + 92= = 201. 34.22. 501 (см. рис. 73). 34.23 и 34.35. Обозначим сумму цифр числа X через S (X). Несложно доказать, что для любых натуральных А к В выпол- выполняется неравенство S (i4+B)^S (Л)+5 (В). Выведем из него не- неравенство 5 (AB)^S (A) '5 (В). Пусть Л —aia2... ап. Умножая Л на В «в столбик», мы должны сложить п «сдвинутых» на один разряд друг относительно друга чисел а„-В, а„-\-В, .... а\-В. Поэтому • j 2 1 1  Рис. 72. 236
S(AB)^S B a,B)<B a,)-S (B)= S (A)-S (B). 1=1 1=1 Из этого неравенства следует, что: 2) S (N) = S (Ю5N)=S B5-55yV)<S C2).S E5N)=5- S E5W). Улучшить эти оценки нельзя, так как S A25) = 8-S A000), SC2)=5-SA05). 34.24. Моокно. Будем производить противоположные опера- операции. Берем любое п. Если п оканчивается на 1, 2, 3, 5, 6, 7, 8 или 9, умножим его соответственно на 4, 5, 8, 2, 4, 2, 5, 6. Получим число Л1<10л, оканчивающееся нулем или четверкой, и тогда последнюю цифру можно будет откинуть. После этого число станет меньше п (проведите детальные рассуждения). В результате нескольких таких операций мы придем к однозначно- однозначному числу — 0 или 4. С него-то и следует начать наш прямой про- процесс. 34.26. Указание. Рассмотрите произвольный треугольник, отсекаемый малой диагональю, и докажите, что углы при его основании равны между собой. 34.28. Можно. Указание. Разделим каждую сторону квад- квадрата произвольно (например, на равные части) и будем изменять длины частей следующим образом. Возьмем какие-то 2 отрезка на одной стороне и один удлиним, а второй укоротим с сохране- сохранением их суммарной длины. Это можно сделать таким образом, что в любом прямоугольнике, который составим из имеющихся 400 частей, эти 2 должны будут попасть на одну сторону (до- (докажите, что есть лишь конечное значение длин первого отрезка, при котором наши 2 отрезка «имеют право» попасть на разные стороны). Затем берем другую пару отрезков и делаем то же самое. В конце концов окажется, что любая пара отрезков, принадлежащая одной стороне квадра- квадрата, должна будет попадать на одну сто- сторону прямоугольника. 34.29 и 34.31. Число п равно квадра- квадрату суммы всех заданных чисел. Действи- Действительно, так как по условию 2j jt,-jt.- = O и «¦</ 2 х?=п, то п=1 xf+2-T. лус|=B Xif. il l il Можно доказать, что п кратно 4 (см. по этому поводу задачу 22.25) и что п ф\&. Неизвестно, может ли п быть отличным 50° от 4. Рис. 73. 237
34.30. Проведем через центр n-угольника прямую, не парал- параллельную ни одной стороне и не проходящую ни через одну его вершину. Будем считать эту прямую числовой осью Ох; в каждой вершине поставим число х„ равное по модулю расстоянию от вершины до оси Ох. Ни одно из х, не равно 0, все х-, различны, а сумма чисел в вершинах любого правильного /г-угольника рав- равна нулю, как вторые координаты векторов, сумма которых равна 0. 34.32. Можно заранее считать набор чисел b\, bz, ..., Ьп упорядоченным по убыванию; надо доказать, что макси- максимальная сумма bk-\-aik принимает наименьшее значение, когда числа {aik} расположены в возрастающем порядке. Если для какого-то расположения чисел {a,ft} этот максимум достигается на сумме bs + ais и при этом а^^щ для некоторого l>s, то переставим ais и а,, местами. Тогда обе суммы 65 + а(/ и bi-\-ais не будут превосходить bs + ais, т. е. максимум сумм {fe* + a,J после перестановки не увеличится. Выбирая этот новый максимум, проводим для него те же рассуждения и в конце концов получаем указанное расположение чисел (а,). 34.33. Нули на конце числа всегда можно не принимать во внимание. Игрок должен называть следующие числа. Сначала он должен число А\ удвоить и ответ 2At = ai ... a* ... аа (a* — средняя цифра) представить в виде суммы jci + i/i, где х\ = = ai ... a*0 ... 0, у\ =a,h+\ ... ап, а затем назвать число В\ = =-!-(*,— у,). Тогда А\+Вх=хх, А\—В\=ух, и, таким образом, оба раза получается число с примерно вдвое меньшим числом цифр. Аналогично получаем числа (*2, f/г), (*з, Уз) и т. д., до тех пор пока не получится однозначное число. А однозначное число не более чем за 5 вопросов можно превратить в 1 или 10 (проделайте это сами). Итак, нам понадобится не более (log2 1000 + 3) + 5 < 18 вопросов. 34.34. Существует. Указание. В случае двух прямых проек- проекции исходных точек стремятся к концам кратчайшего отрезка, соединяющего пары точек на прямых. 35.01. Предположим, что не все числа равны между собой — пусть ai><32 (случай а\<.а2 рассматривается аналогично). Воспользуемся тем, что если степени равны, а основания раз- различны, то больший показатель имеет меньшее основание; тогда а\ >а2=*~ а2<.а&=?~ ез>Я4 =*¦ — =*¦ ai6<ai7 =*¦ ам>а\ => а\ <.а2- Первое неравенство в этой цепочке противоречит последнему, стало быть, исходное предположение неправомерно, откуда ai = = ... =ai7- Замечание. При четном количестве чисел утверждение задачи неверно. Придумайте контрпример. 35.02. Из любых трех делегатов всегда найдутся два, кото- 238
'"-**; Рис. 74. рые могут поговорить друг с дру- другом. Отбросим их, тогда для оставшихся делегатов справед- справедливо это же утверждение. После нескольких таких операций мы придем к случаю четырех де- делегатов, для которого утвержде- утверждение легко проверяется. 35.03. Так как 13 — нечет- нечетное число, найдутся две соседние вершины одного цвета. Зануме- Занумеруем вершины по часовой стрел- стрелке от 1 до 13. Пусть выделенные вершины одного цвета имеют при такой нумерации номера 2 и 3. Тогда какие-то три из вершин 1, 2, 3, 4 и 9 лежат в вершинах равнобедренного треугольника и имеют одинаковый цвет. 35.06. Очевидно, что точки Х\, Х2, Х3, ..., Xn-i расположены на одной окружности (АА' — хорда этой окружности), поскольку углы между соответственными прямыми равны между собой (рис. 74). 35.07. Разобьем монеты на две группы по 500 монет и срав- сравним их массы (I взвешивание). Возможны 2 случая: А. Одна из чашек весов (скажем, правая) перевесила. Следо- Следовательно, фальшивые монеты есть (их одна или две) и все они лежат на одной чашке (если бы они лежали по одной на каждой, то весы остались бы в равновесии). Разобьем теперь более тяжелую группу на две по 250 монет и сравним их (II в з в е- шивание). Если одна из чашек перевесила, то фальшивые монеты находятся среди этих пятисот и, следовательно, т я- ж е л е е — в этом случае III взвешивание уже не нужно. Если же чашки в равновесии, то либо фальшивые монеты среди остав- оставшихся пятисот (и легче настоящих), либо они разделились: по одной среди групп по 250. Чтобы выяснить, какой из этих случаев имеет место, одну из групп в 250 монет делим пополам и сраьниваем их (III взвешивание). Если весы в равнове- равновесии, имеет место первый случай, если нет — второй. Б. При I взвешивании чашки в равновесии. Значит, фаль- фальшивых монет имеется 0 или 2 (по одной на кгждой чашке). Опять разбиваем одну из групп на две по 250 монет (II в з в е- шивание). Если чашки опять в равновесии, то на чашках четное число фальшивых монет, а так как их не более од- одной чначит, vx чет вовсе. Если же одна из чашек перевесила, то всего фальшивых монет 2 и ровно одна из них лежит на одной из чашек. Разделив более тяжелую группу пополам, мы третьим ьзвешиванием легко узнаем, тяжелее ли фальшивая монета или легче. Замечания. 1. Существенно для решения, что число монет делится на 8; в противном с/1 vмае трех взвешиваний не хватило бы. 2. Если бы потребовалось 239
в условиях задачи узнать, сколько имеется фальшивых монет, то понадобилось бы ие менее 8 взвешиваний. 3. Легко заметить, что три взвешивания, описан- описанные выше, можно было бы осуществлять, не обращая по ходу дела внимание на то, какая чашка перевешивает: разложить монеты по 500, потом половину их — по 250 и, наконец, разложить половину половины. Мы раскладывали более тя- тяжелую группу только для определенности. 35.08. Ясно, что 50 чисел B, 2, ..., 2) удовлетворяют условию задачи. С другой стороны, 49 чисел (или меньшее количество) можно было бы разбить на 7 групп по 7 чисел; сумма каждой группы была бы не больше 14, а общая сумма S^7-14 = 98. 35.09. Треугольники ADC и ВЕС подобны по двум углам. Если К—коэффициент подобия, то АС = К-ВС и —BC = DC = 1 =Х-ЕС=—-к-АС, откуда Я,= 1 и АС —ВС. Тогда в треуголь- треугольнике ВЕС сторона ЕС, лежащая против угла в 30°, равна половине основания ВС. Поэтому /LBEC = 90° и ZC=60°. Этого уже достаточно. 35.10. Надо доказывать более сильное утвержде- утверждение: пять точек К, Е, Н, М, С леокат на одной окружности. Так как КСНЕ и СМНЕ — равнобочные трапеции, то обе они вписаны в свои окружности. Точки С, Е, и Н, лежащие на ка- какой-то третьей (заранее неизвестной) окружности, лежат и на первых двух. Значит, все три окружности совпадают. 35.11. Указание. Докажите, что fi*m = a*i +(aim —an). 35.12. Расположим высоты деревьев в порядке убывания и в этом же порядке соединим основания деревьев. Длина получен- полученной (быть может, самопересекающейся) ломаной не превосходит суммы разностей высот соседних деревьев, а значит, и высоты максимального дерева. Поэтому достаточно обнести с двух сто- сторон полученную ломаную забором, длина которого не будет пре- превосходить 100+100=200 м. 35.13. Первая сумма больше второй на пг — п. Действительно, докажем, что после отбрасывания одного крайнего слагаемого в каждой сумме (т. е. чисел тип) оставшиеся суммы St и S2 уже будут равны. Рассмотрим для этого все целые точки в пря- прямоугольнике пгХп, левый нижний угол которого расположен в начале координат О'Л^х^пг—1, l^i/^n—1. Всего их (т — \)(п—1), причем р о в н о половина —над диагональю, выходящей из О, и ровно половина — под ней (на диаго- диагонали нет целых точек в силу взаимной простоты тип). Но количество целых точек с абсциссой k равно!/г•—J (то же верно и для ординаты у). Следовательно, Si = S2=—{m—1)(п—1). 35.14. Занумеруем проспекты от 1 до 10 сверху вниз. Пусть первый полицейский находится на первом перекрестке, второй — на втором, ипредположим, что гангстер находится в верхней полосе. Тогда первый полицейский, пройдя по своему проспекту не более 100 кварталов, обнаружит гангстера. Если же этого не 240
произошло, то, следовательно, гангстер находится не в первой по- полосе и полицейские могут спуститься на одну полосу вниз. Контро- Контролируя теперь вторую и третью полосы, они гарантируют, что ганг- гангстер не попадет на первую полосу. Действуя, как и раньше, и опускаясь каждый раз (если это нужно) на одну полосу вниз, полицейские в какой-то момент обнаружат гангстера. 35.15. Все жители разбиваются на два множества, причем любые двое жителей из одного множества дружат между собой, а из разных — не дружат. Затем, ссорясь в одном множестве, каждый житель будет переходить в другое множество. Это будет происходить до тех пор, пока все жители не станут друзьями. 35.16. Предположим, что в последовательности не более од- одного составного числа. Следовательно, все числа, начиная с не- некоторого а, простые. Что же приписывается к числу а? При- Приписывать четные цифры или цифру 5 нельзя, а приписать 1 или 7 можно не более одного раза, так как приписывание этих цифр увеличивает на 1 остаток от деления на 3. Значит, с какого-то места будет приписываться только цифра 3. Если до этого места было получено простое число р, то, приписав к нему не более р троек, получим число, кратное р, так как из чисел 3, 33, ..., 33... 3 (р троек) хотя бы одно кратно р. 35.17. Очевидно, все грани тетраэдра равны; обозначим че- через S площадь грани. Примем одну из них за основание и спроецируем на нее три остальные — они покроют ее. Отсюда S =S cos a-\-S cos P + S cos y; cos a + cos p + cos y = 1, где a, p, у — двугранные углы при основании. Отсюда легко следует, что сумма косинусов всех двугранных углов равна 2. 35.18. Указание. Множество Т есть пересечение п полу- полуплоскостей, ограниченных сторонами /г-угольника. 35.20. Докажем сначала, что в точке О диагонали делятся пополам. Если а, Ъ, с, d — длины четырех отрезков ОА, ОВ, ОС, OD и cZ^a, d~^b, то, отложив на отрезке ОС отрезок ОМ длины а, а на отрезке OD отрезок ON длины Ь, получим параллелограмм ABMN, разделенный на 4 треугольника того же самого пери- периметра — противоречие. Итак, ABCD — параллелограмм, а из сравнения периметров двух смежных треугольников следует, что он ромб. 35.21. Указание. Проведите через вершину треугольника, образованного прямыми Ь, с, d, прямую, параллельную прямой а. Воспользуйтесь тем, что в треугольнике средняя линия парал- параллельна основанию. 35.22. Одно из 12 последовательных натуральных чисел крат- кратно 12. Сумма делителей этого числа превосходит это число. 35.23. Предположим, что рыцарь странствует настолько дол- долго, что по какой-то дороге он проехал не менее 6 раз. Тогда по ней он проехал не менее 3 раз в одном направлении, следо- следовательно, по одной из двух дорог, на которые можно затем свернуть, он двигался дважды в одну сторону. Но тогда весь 9 Заказ 247 241
его дальнейший путь будет совпадать с уже пройденным (до вступления на эту дорогу) и будет проходить через тот замок, из которого рыцарь выехал в первый раз. 35.24. Обозначим МВ = а, BK = b, KC = c, CA=d, AM = e. Так как площадь треугольника ВМК больше площади треуголь- треугольника МКС, а площадь треугольника ВМК больше площади треуголь- треугольника АМК, то Ь>с, а>е. Допустим, что -^~ <;—, т. е. с-\-а-\-е 3 + d. Тогда 2a+2fc<d, откуда а + е + и АВ + ВС<АС, противоречие с нера- неравенством треугольника. 35.26. d = d(af — be) = adf — bed = (adf-bcf)+(bcf — bed) = =f(ad — bc) + b(cf — ed)^f-l-['b-\—f4-b, что и требовалось доказать. 35.27. Фиксируем первую часть дорог и заданное односто- одностороннее движение на них (т. е. данную в условии первую раз- разметку); дороги покрасим в красный цвет и сориентируем их соответствующими стрелками. Задача решается в два этапа. На первом будем увели- увеличивать число красных дорог в разметке, добавляя (указанным ниже способом) новые красные дороги со стрелками так, что- чтобы при каждом новом добавлении выполнялось условие задачи. Мы прекратим добавлять новые красные дороги, когда достиг- достигнем следующего: для любых двух перекрестков А и В, непосред- непосредственно соединенных красной стрелкой, идущей из А в В, су- существует также и путь из В в А, идущий только по красным стрелкам (цикличность красной разметки). На втором эта- п е мы укажем искомую ориентацию дорог, удовлетворяющую утверждению задачи, чем и завершим доказательство. Первый этап. Пусть перекрестки А и В соединяются красной стрелкой А-уВ, но из В в Л нет красного пути, т. е. на любом пути, ведущем из В в Л, не все стрелки красные, имеются и «пустые» дороги. Возьмем любой такой путь из В в Л. Все красные дороги, содержащиеся в этом пути, имеют согласованную ориентацию со стрелкой А->В (так как из В в Л есть проезд по выбранному пути); на «пустых» дорогах введем такую же (согла- (согласованную с А-+В) ориентацию и покрасим их в красный цвет. Тогда из В в Л можно проехать по красным стрелкам. В силу конечности числа дорог в городе мы за несколько шагов добьем- добьемся цикличности красной разметки, т. е. получим «островки» из красных циклов. Второй этап. Дороги, не попавшие в красные циклы, закрасим зеленым цветом; на них пока что двойная ориентация. Оставим на всех зеленых дорогах по одной стрелке, отвечающей второй разметке, о которой сказано в условии задачи. Таким образом, те- теперь уже во всем городе введено одностороннее движение. Докажем, что, двигаясь по дорогам согласно введенной ориен- ориентации, можно из любого перекрестка Л попасть в любой другой 242
перекресток В. Действительно, возьмем тот путь из А в В, ко- который отвечает второй разметке из условия задачи. Теперь он состоит из красных и зеленых стрелок. По зеленым стрелкам мы всегда будем двигаться в нужном направлении, а по красным — не всегда. Если на дороге XY, входящей в рассматриваемый путь А ... XY ... В, отмечена красная стрелка из Y в X (т. е. дви- двигаться по дороге XY нельзя), то пойдем по пути из красных стрелок, ведущем из X в Y и дополняющем стрелку Y-+X до цикла согласно построениям первого этапа. Полученный путь из Л в В — искомый. 35.28. Рассмотрим все дроби, сократимые и несократимые, у которых числитель и знаменатель не превосходят 100, их 1002. Если HOD (a, b) — n, то дробь -2- сократима на п, при этом I С\С\ полученная несократимая дробь —^- такова, что а\ ^ и 01 П I ^- ЮО ., / 100\ „ „ о 1^; , т. е. в количестве л( J несократимых дробей мы уч- учтем и эту дробь. Следовательно, искомая сумма равна количеству всех рассмотренных вначале дробей, т. е. равна 10 000. 35.30. Пусть в пятиугольнике ABCDE угол С не тупой, а дли- длины всех сторон равны 1. Проведем диагонали АС и СЕ, которые разобьют пятиугольник на три треугольника, причем два из них, А АВС и ACDE, равнобедренные с углами при вер- вершине, меньшими 120°. Значит, третий оставшийся треуголь- треугольник АСЕ остроугольный, причем угол при его вершине С меньше 90° — 2-30° =30°, а боковые стороны (они одновремен- одновременно являются основаниями треугольников АВС и CDE) имеют длины, меньшие -\/3. Но тогда из теоремы косинусов легко пока- показать, что его основание АЕ имеет длину меньше 1, что противоре- противоречит равенству АЕ — \. Значит, угол С пятиугольника тупой. 35.32. 60 станций. Указание. Расставить по спирали непе- непересекающиеся фигуры, изобра- изображенные на рисунке 75, а, так, что- чтобы они заполнили весь квадрат 20X20. Таких фигур не меньше 60, и в каждой из них находится по крайней мере одна станция. Значит, станций не меньше 60 (ровно 60 можно расставить с по- помощью той же конструкции; рис. 75,6). а) 35.33. Если выполнено равен- равенство, данное в условии, а числа а, Ь, с, d рациональные, то выпол- выполнено и равенство: (а — b^j2Jn-\- 6) ' Рис. 75. 243 тает
Но 5 — 4 У2<0, а левая часть положительная. Противоречие. Следовательно, исходного равенства быть не может. 35.34. а) Возьмем прямую а и ближайшую к ней точку А пересечения двух других прямых Ъ и с. Треугольник, образован- образованный прямыми а, Ь, с, не пересекает никакая другая прямая, ина- иначе нашлась бы более близкая к прямой о точка пересечения пря- прямых, чем точка А. Следовательно, к любой прямой примыкает треугольник, а так как каждый из них при подсчете общего числа учитывается трижды, то таких треугольников не менее 3000X Х-г~=Ю00. б) Имеется не более двух таких прямых, что все точки пересечения остальных прямых лежат с одной стороны от каждой из них. Действительно, если таких прямых больше, выберем три из них. Они разобьют плоскость на 7 частей, и все точки пересечения остальных прямых должны лежать только в одной из этих частей, чего не может быть (четвертая прямая пересекает две области). Итак, у 3000 — 2 = 2998 прямых мы дополнительно к пункту а) насчитаем еще по треуголь- треугольнику, т. е. насчитаем 2998 + 3000 треугольников. Однако каж- 3000 дый из них будет просчитан трижды, откуда N~^2-—- 2 ——. Поскольку N целое, то ^^2000. 35.35. Длины сторон и высот спроектированного треугольника А\В\С] не превосходят длин сторон и высот соответственно проеци- проецируемого на плоскость П треугольника ABC. Теперь следует рас- рассмотреть (сделайте это сами) два случая: а) какие-ro два угла аАВС не меньше соответствующих углов АА\В\С\\ б) наоборот, два угла /\АВС не больше соответствующих углов 35.36. Докажем, что если имеется набор невозраст а- ющих чисел d ^аг^а.ч^... ^ап^0 и выполнены все условия задачи, то а\Х\ -\-a2X2-\-¦¦¦-\~о.пХп^а\у\ -\-a-2U2-\-...-{-апуп- Действительно, существуют такие неотрицательные числа Ь\, ..., Ьп, что ап — Ь\, а„_i = 61 +• 62, •--, п2=-^ bt, a\ = /= i л =2 Ь,. Имеем: i= i „уп. Доказанное утверждение применим k раз. 1) Полагаем а, — дг*, 1</<п. Тогда ¦214
2) Полагаем а, = х? 2у„ получаем: л^ 'i/i+...+л^ + ...+хкп~2у1 3) Полагаем а, = л-?~3у?, получаем: я* 2у2 + . *3i k) Наконец, положив а, = у?~', l^t^/i, получим Объединяя все й неравенств в одну длинную цепочку, по- п п лучасм утверждение задачи: 2j**^2jy*. i = I i = I 35.37. 20 000. Если jci ^...^л'200—числа первого набора, а yi^... ^1/2оо— второго, и при своем ходе игрок б берет карточки Xi, ..., л'юо и у{, ..., 1/юо, то он обеспечит себе (*, —*ioi) + --- + + (л-,оо — лг2оо)+ («/¦ — t/ioi) + -+0/ЮО — г/2оо)>2ОООО. Но игрок А, положив первым ходом числа 1, 2, .... 100, 201, ..., 300 в первый набор, а остальные — во второй, после каждого хода В будет возвращаться к этой же ситуации; при этом Св — Си = 20 000. 35.38. Индукция по п. Для п = 3 все такие числа легко выписать: —, —, —, —, —, а затем проверить справедливость о 3 2 3 3 утверждения. Пусть утверждение верно для п — 1. Новый набор из п чисел получится из предыдущего набора добавлением не- некоторых чисел. Если соседние числа в новом наборе являются соседними и в старом, то для них все доказано. Если же дробь — оказалась соседней с дробями — и — старого набора, то до- докажем, что A=kb— ар = \ и В = ср — kd=\ (-^-<— <" Предположим противное: пусть максимальное из чисел А и В больше 1. Тогда =p(bc — ad)=p\ b + d<p. Следовательно, во всяком случае, =^ —. Но а. - а-\-с с Тb+d d ' и поэтому не соседняя дробь с -^- и — противоречие. Замечание. Эту задачу можно решить с помощью числовых решеток на плоскости; такое доказательство имеется в книге Г. С. Кокстера «Введение в геометрию» (М., Наука, 1966 г., с. 303—304). 35.39. Не всегда. Будем рассматривать числа по моду- модулю 10 (т. е. рассматривать последние цифры этих чисел). Возьмем набор «все нули» и подсчитаем, сколько наборов мы смо- сможем получить, исходя из него с помощью указанных в условии операций. Клеток размером 3X3 и 4x4 имеется (8 — 3+lJ-f- 245
Рис. 76. Рис. 77. + (8 — 4+1)-' = 61, т. е. искомых наборов не больше 106'. Однако всевозможных различных наборов существует 1064. Сле- Следовательно, существует набор, который с помощью указанных операций из нулевого набора получить не удастся. Именно этот набор и следует принять за исходный. 36.01. Достаточно «раздвоить» границы всех стран, как это показано на рисунке 76. 36.02. Не может. Если число является точным квадратом, то оно оканчивается четным числом нулей и их можно не рас- рассматривать. Оставшееся число имеет вид 2А, где А — число из 600 троек и некоторого числа нулей, оканчивающееся на 3. Итак, А нечетно, значит, 2А — не точный квадрат. 36.03. Среди пяти точек существуют такие две (обозначим их Л и В), что оставшиеся три точки С\, С2, С3 лежат по одну сторону от прямой АВ. Без ограничения общности будем считать, что /.АС,В<С /LAC2B< /LAC3B. Тогда точка С, лежит вне окружности, проходящей через точки А, В и С2, а точка С3— внутри нее. 36.04. Умножив данное уравнение на хур, получим: рх-\-ру = =ху или (х — р) (у — р) = р2. При простом р отсюда следует, что: 1) х — р = \; у~р = р2, или 2) х—р = р; у — р = р, или 3) х — р = р2; у — р=\. Если же р составное, то р2 раскла- раскладывается на множители и другими способами. 36.05. Не может. 36.07. Сначала все многоугольники разобьем на треуголь- треугольники так, чтобы никакая вершина треугольника не лежала внут- внутри стороны другого треугольника. При этом все внутренние треугольники будут граничить с тремя треугольниками (т. е. с не- нечетным числом). Остался случай, когда одна из сторон тре- треугольника лежит на стороне квадрата. В этом случае произво- производим разбиение, указанное на рисунке 77. 36.08. Если афЪ — целые числа, то ak — bk делится на а — Ь; поэтому Р (а) — Р (Ь) делится на а — Ь. Пусть Р {а\) = Р (аг) = = Р(а3) = 2, Р(Ь) = 3. Тогда P{b) — P{ai)=\ делится на Ь — а, (t=l, 2, 3), откуда |а, — b\ = l. Но это равенство не может вы- выполняться для трех различных чисел а,-. 36.09. Пусть А — произвольная станция метро. Тогда следует закрыть такую станцию метро, до которой от А ехать дальше всего (по числу остановок). 246
36.10. Средняя сумма отпечатанных на доске чисел равна 3,5-99 = 346,5. Если на пути движения кубика два раза встре- встречается число а, то между ними обязательно встречается и число 7 — а. Чисел вида а и 7—а может быть либо поровну, либо число их может отличаться на 1. Следовательно, сумма отпеча- отпечатанных чисел может отличаться от средней суммы этих чисел на 4,5 в ту или другую сторону. Поэтому максимальное значение суммы равно 351, а минимальное — 342. 36.11. Воспользуемся тем, что ab~^a-\-b при а ^2, 6^2. Обозначим через f (k) число, получающееся из числа k одной операцией. Тогда \ (k)^.k-\-\ при любом k. При этом если k — четное и больше 7, то f (k) = 2-{-p2 + ...-{-pn-\-\ 3 = 3+—<Zk. Следовательно, f(/(ft))<fe для любого k^7, т. е. рассматриваемая последовательность принимает значения толь- только из отрезка [1, Л+1]. Значит, она принимает какое-то значение дважды, а потому является периодической. 36.14. Пусть существует выпуклый многогранник, все грани которого — многоугольники с р а з н ы м числом сторон. К грани с максимальным числом сторон п примыкает п граней с числом сторон от 3 до л—1. Противоречие. 36.15. а) В любом положении тумблеров состояние любой фиксированной лампочки можно изменить переключением ка- какого-либо одного тумблера, поскольку переключениям разных тумблеров соответствует изменение состояний у разных лампочек. б) Приведем тумблеры в такое состояние, чтобы все лампоч- лампочки погасли. Это состояние каждого тумблера назовем «выклю- «выключенным», а противоположное ему — «включенным». Если теперь включено k тублеров, то горят r^k лампочек. И если r<Zk, то можно переключить так г тумблеров, чтобы все лампочки по- погасли (см. а), хотя при этом не все тумблеры выключены. Противоречие приводит нас к равенству г = &. в) Занумеруем тумблеры числами 1, 2, ..., п, а затем зану- занумеруем лампочки так, что если включен /-й тумблер, а осталь- остальные выключены, то горит /-я лампочка. Докажем, что если вклю- включены тумблеры /i, h, ..., ik, то горят лампочки с теми же но- номе р а м и. Из б) следует, что горят k лампочек. Пусть одна из них имеет номер /, отличный от i\, ..., /*. С помощью k — 1 переключений можно погасить остальные k — 1 лампочек (см. а). В результате включен один из тумблеров 1\, ..., ik, а горит лампоч- лампочка /. Это противоречит выбору /, и утверждение доказано. 36.16. Не может. Пусть ауа^а^а^— искомое четырехзначное число и 01020304—О4Озо2О| = 1008. Начиная вычитать с конца, мы видим, что О|=о4 + 2, о3^О2+1. Но тогда разность должна иметь цифру сотен, равную 9, а не 0. 36.17. Пусть О — точка пересечения высот треугольника ABC Тогда треугольник ABC разбивается на 3 четырехугольника, 247
каждый из которых покрыт соответствую- щим кругом (даже кругом меньшего ра- диуса). 36.18. а) Можно; б), в) вообще гово- Рнс- 78. рЯ> нельзя. 36.20. Пусть наименьшее расстояние от произвольной точки кляксы до ее границы принимает наиболь- наибольшее значение т\ в точке А, а наибольшее расстояние принимает наименьшее значение г2 в точке В. Тогда круг радиуса г\ с цент- центром А лежит внутри кляксы, а круг радиуса г2 с центром В содержит кляксу. По условию rt = r2 = r, значит, точки А и В совпадают и клякса имеет форму круга радиуса г. 36.25. 4/2+1 (см. рис. 78). Указание. К каждой черной клетке примыкает не более четырех белых. Докажите, что не менее k — 1 белых клеток примыкает сразу к двум черным и, сле- следовательно, были только что учтены дважды. 36.27. 1972 раза. Указание. Любое число п будет выписа- выписано столько раз, сколько имеется чисел, меньших п и взаимно простых с п (так называемая «функция Эйлера» ф(л))- Но 1973 — простое число. 36.29. Число х= "~*v" — удовлетворяет уравнению х-\ = — п. Положим ^т+(~) =km, тогда km + { = km(x-\-~\—km-i — = n-km — km-l. Поэтому раз х-\ целое, то и k = xm-\—^-це- xm-\—^-целое при любом т. Но тогда хт = „ ~ . 36.30. Пусть дан трехгранный угол. Отложим на его ребрах векторы а, Б, с единичной длины. Тогда векторы а-\-Ъ, Б-\-с, с-\-а направлены по биссектрисам плоских углов. Скалярные произведения i BB tf ), (c+a)-(a равны между собой и равны 1 -\-a-b-{-b-c-\-c-a. Это и означает, что углы между биссектрисами будут одновременно острыми, прямыми или же тупыми. 36.31. Указание. Проверьте сначала, что окончательная раскраска не зависит от порядка выхода маляров (достаточно проверить это для двух маляров). 36.33. Мысленно «выпрямим» траекторию льва, вращая арену цирка последовательно относительно точек поворота льва. При таком выпрямлении центр О арены каждый раз перемещается на отрезок, не больший произведения соответствующего угла по- поворота льва (выраженного в радианах) на 10 м (так как центр отстоит от льва не далее 10 м). Всего точка О переместится на величину, не превосходящую сумму всех углов поворота льва, умноженную на 10 м. Но нетрудно видеть, что длина отрезка, 248
соединяющего начальное положение точки О с конечным, не меньше 29 980 м (так как 30 000 — 20 = 29 980), откуда и следует утверждение задачи. 37.01. Обозначим Ю2'000 через а, а 2974-f-l — через п. Тогда наше число равно ап-\-\ и оно кратно a-f-1- 37.02. Докажем, что если площадь треугольника больше 1 и он расположен в круге радиуса 1, то центр круга находится строго внутри треугольника. Действительно, все высоты треуголь- треугольника больше 1, поскольку каждая сторона не больше 2 (диаметра круга), а площадь треугольника больше 1. Следовательно, дан- данный треугольник является пересечением трех полос, ширина каж- каждой из которых строго больше 1, а поэтому содержит внутри себя центр круга. Если же в круг радиуса 1 помещены 2 тре- треугольника, площади которых строго больше 1, то они оба содер- содержат внутри себя центр круга, а стало быть, пересекаются друг с другом. 37.03 и 37.08. Обозначим через п число спиленных пар зу- зубцов (п = 6 или п = 10). Тогда у каждой шестеренки п2 — п + 2 зубцов. Всего существует п2 — п-{-\ таких поворотов верхней шестеренки относительно нижней, при которых все зубцы обеих шестеренок оказываются совмещенными. Назовем дыркой то место шестеренки, где отсутствует зубец. Рассмотрим произволь- произвольную дырку нижней шестеренки. При п — 1 положениях верхней шестеренки (кроме исходного) над этой дыркой оказывается дыр- дырка верхней шестеренки. Но дырок на нижней шестеренке п, поэто- поэтому из п2 — п-\-\ поворотов верхней шестеренки только при п (п—1) из них наблюдаются совпадения дырок обеих шестере- шестеренок. Поскольку (п2 — п+ 1) — п (п— 1)== 1, найдется такой поворот верхней шестеренки, когда совпадения дырок не будет. Этот поворот искомый. 37.04. По условию числа а, Ъ, с удовлетворяют неравенствам >c, + a>b. Очевидно, а + с ¦>- > Ti> , . .ч . / , ,ч . отсюда следует, что —; \--т\—>——г. b+c (a + b)+(a + b) J а + с Ь + с а + Ь Аналогично доказываются остальные два неравенства треуголь- треугольника. 37.05. Разобьем полоску ши- шириной 1 вокруг многоугольника на прямоугольники, построенные на его сторонах, и «уголки» около вершин (рис. 79). Из «уголков» составим многоуголь- многоугольник, подобный данному: его сто- стороны равны разностям сторон первоначального и полученного многоугольников, а углы равны 249 Рис. 79.
б) Пятнадцати- Семнадцати- угольник угольник 6) Рис. 80. соответственным углам исходного многоугольника. Но в послед- последний многоугольник, очевидно, можно вписать окружность (ра- (радиуса 1). Из этого следует утверждение задачи. 37.07. Предположим, что на каждой прямой, проходящей че- через стороны 13-угольника, лежит не менее двух сторон 13- угольника. Тогда количество этих прямых не превосходит 6. Каждая из них пересекается поэтому не более чем с 5-ю другими, и, следовательно, на каждой прямой лежит не более 5-и вершин 13-угольника. Но стороны, лежащие на произвольной прямой, не пересекаются и не имеют общих вершин. Значит, их не более двух на этой прямой и тогда сторон должно быть не более 12. Однако их 13 — противоречие. Отсюда следует, что на одной из прямых расположена только одна сторона 13-угольника. Построение п-угольников при четном п с требуемым свойством видно на рисунке 80, а, а построение п-угольников при нечетном п можно начать с 15-угольника (рис. 80,6), а затем отрезать четное число углов так, как это показано на рисунке 80, в. 37.09. Нельзя. Двигаясь по отрезкам диагоналей, мы прохо- проходим поочередно через вершины и центры граней. Но у куба 8 вершин и только 6 граней. 37.11. Центр шестиугольника принадлежит одному из 6 имею- имеющихся треугольников, и у этого треугольника все стороны не меньше 1, поскольку каждый его угол при единичном основании не меньше 60°. Несложно показать, что один из двух треуголь- треугольников, имеющих общую сторону с рассмотренным треугольником, также имеет стороны, не меньшие 1. 37.12. 197 точек. См. решение задачи 37.16. 37.13. 4 или 5. Указание. Известно, что в квадрате и пра- правильном пятиугольнике все диагонали равны. Предположив, что в п-угольнике, где п^б, все диагонали равны, возьмем его сторону АВ и пересекающиеся диагонали AD и ВС (вершины С и D несоседние с А и В). Тогда AD+BC>AC + BD, что противоречит равенству AC = CB=BD — DA. Итак, случай п^6 невозможен. 37.16. Обозначим через А и В точки, расстояние между ко- которыми максимально. Соединим точку А со всеми точками, 250
кроме В. Середины полученных п — 2 отрезков не совпада- ю т (иначе совпадали бы вторые концы отрезков) и лежат в круге радиуса ^-АВ с центром в точке А. Аналогичные рассуждения, проведенные для точки В, дают еще п — 2 середины, лежащие в круге радиуса —АВ с центром в В. Построенные два круга с центрами А и В имеют ровно одну общую точку — середину отрезка АВ. Итак, мы явно указали (п — 2)-\-{п—2) + + 1=2« —3 середины отрезков. Пример Bп— 3)-х середин: все точки располагаются на одной прямой на равных расстояниях друг от друга. Значит, наименьшее число середин всевозможных отрезков равно 2п — 3. 37.17. Рассмотрим сначала один столбец. Несколькими вы- вычитаниями единицы из всех чисел этого столбца добьемся того, чтобы наименьшее число в нем стало равным 1. Теперь строки, содержащие 1, удвоим, а затем из всех чисел нашего столбца вычтем по единице. Тем самым мы смогли единицы столбца не изменить, а все числа, большие 1, уменьшить на 1. Действуя таким образом и далее, мы вскоре сумеем сделать все числа столб- столбца равными 1, а потом вычитанием 1 получить 0 в каждой его клетке. Аналогичную процедуру проделаем и со всеми осталь- остальными столбцами. 37.18. В пятиугольнике ABCDE зафиксируем вершину А, про- проведем диагонали АС и AD и построим на них как на диа- диаметрах круги с центрами в точках О\ и Оо. Эти круги покрывают треугольники ABC и AED, поскольку углы ABC и AED тупые. Если один из углов ACD или ADC тупой, то один из кругов с центром О\ или Оо покрывает треугольник ACD. Если же углы ACD и ADC не тупые, то круг с центром О\ покры- покрывает треугольник АН С, а круг с центром Ог— треугольник AHD, где Н — основание высоты АН в треугольнике ACD (точка Я ле- лежит на отрезке CD). Тем самым круги с центрами О\ и Оо полностью покрывают весь пятиугольник. 37.19. Для равных чисел все очевидно. Пусть теперь среди данных 100 чисел а.\, .... аюо имеются хотя бы два различных, например а\ и а2- Рассмотрим 100 следующих чисел: а.\. Go, ot + oo, ai+G2 + O3, ¦¦¦, oi + ... +O99- Докажем, что одно из них или же разность двух из них делится на 100; поскольку это число больше 0 и меньше 200, оно в точности равно 100. Для доказательства рассмотрим две последние цифры чисел исходной последовательности. Среди получившихся двузначных чисел либо встретится 00, и тогда делимость на 100 доказана, либо найдет- найдется пара одинаковых чисел. Во втором случае это не пара а\, аг- Взяв разность чисел из этой пары, получим кратную 100 сумму 37.20. Существует. Числа в последовательности будем выпи- выписывать парами Первая пара: а\ = \, аг = 2 Пусть мы уже смогли 251
построить k пар; построим (к-\-\)-ю пару.Рассмотрим всевозмож- всевозможные разности, реализуемые среди имеющихся k пар чисел, и обозначим через d наименьшую разность, которая еще не реали- реализована. Положив теперь O2*+i = 2аг* + I, 024+2 = 02*+1 +^. по- получаем требуемую последовательность. 37.21. У выпуклого 2л-угольника пBп — 3) диагоналей. Легко видеть, что диагоналей, параллельных данной стороне, не более п—2 (любые 2 такие диагонали соединяют разные пары вершин). Сторон имеется 2п, так что число диагоналей, параллельных од- одной из них, не больше 2п(п — 2), т. е. меньше общего числа диагоналей. 37.22. Назовем гирю существенной, если, убрав ее, мы не сможем разбить оставшиеся гири на k равных по массе групп. Все прочие гири будем называть несущественными. Предположим, что утверждение задачи неверно, т. е. что сущест- существенных гирь меньше k. Мы тогда докажем, что при любом на- натуральном k масса любой несущественной гири кратна k", т. е. делится на сколь угодно большое число, а значит, равна н у л ю,— противоречие. Доказательство этого проведем по индук- индукции. Для п = \ оно верно. В самом деле, масса всех гирь делится на k и, убрав произвольную несущественную гирю, можно ос- оставшиеся гири разбить на k равных по массе групп (по опреде- определению несущественной гири). Следовательно, масса убранной гири кратна /г1. Предположим теперь, что масса любой несущест- несущественной гири кратна k", и докажем, что она кратна kn + l. По пред- предположению существенных гирь меньше k, значит, существует группа, все гири в которой несущественные (в противном случае несущественных гирь было бы не меньше, чем групп). Масса всей группы из несущественных гирь кратна k", всего групп k, а масса любых двух таких групп одинакова. Отсюда следует, что масса всех гирь кратна k"+l как до того, как убрана несущественная гиря, так и после. Следовательно, масса несущественной гири кратна k"+l. 37.23. См. решение задачи 28.07. 37.24. Указание. Утверждение задачи можно доказать ин- индукцией по сумме целых частей о и Ь. Для [o]+[fc]=l все оче- очевидно. Предположение индукции: если [o]-|-[fc] = n, то одно из чисел — о или b — целое; надо доказать это же утверждение для [o] + [fc] = n+l. Для доказательства следует рассмотреть два слу- случая. Первый: к нижней стороне листа бумаги все прямо- прямоугольники примыкают своими сторонами, длины которых отличны от 1. Второй, более сложный случай, состоит в том, что предлагается существование прямоугольника, примыкающего к нижней стороне листа своей единичной стороной. (Подробное решение в ж. «Квант», № 4, 1975 г.) 37.26. Поскольку 1974">103", то количество цифр в числе 1974" не меньше Зп. Обозначим это количество через ^предпо- ^предположим, что у 1974"+ 2" больше цифр, чем у 1974". Тогда 252
" + 2п>10*. Легко видеть, что 987"<2*-"-5*<987п + 1. по- поскольку 10* делится на 2". Следовательно, 2*~п • 5" = 987" + 1, fe —л>2п. Если я>2, то 2к-"-5к делится на 8, но 987"+ 1 на 8 не делится (дает в остатке 2, 4 или 6). Поэтому получен- полученное равенство не имеет места. Утверждение доказано. 37.27. Проведем через центр планеты и произвольную пару астероидов плоскость П (будем считать ее экваториальной). Проведем ось планеты, перпендикулярную плоскости П. Она пе- пересекает поверхность планеты в «полюсах» А и В. Тогда легко видеть, что наблюдатели в точках А и В видят вместе лишь 37 — 2 = 35 астероидов, а потому один из них видит менее 18 астероидов. 37.28. Рассмотрим ученого с максимальным количе- количеством друзей, равным п. По условию все его друзья имеют различное число друзей, отличное от нуля, но не большее п. Таких возможностей ровно п: один друг, два друга, .... п друзей; следовательно, все случаи реализуются. Поэтому существует, в частности, ученый, имеющий ровно одного друга. 38.01. x = y = z = t = 0. Указание. Замените уравнение равносильным ±х2+(±-х-у) +(-1_*_2) +(-Ljc-f) =0. 38.02. Обозначим буквой О центр круга. Проведем произ- произвольную прямую /, пересекающую круг, и отразим относительно нее точки Л и О, получив точки А' и О' Из треугольника ОАО' следует что длина стороны 00' не меньше 0/1 — — О'А\, а так как О'А = ОА', то \ОА'— ОА\ не превосходит удвоенного расстояния от точки О до прямой /, т. е 2 см. Следовательно, за 24 отражения точка А сможет приблизиться к центру круга не более чем на 48 см и поэтому еще не попадет в круг; за 25 же раз загнать ее внутрь круга можно. 38.03. Два из чисел а, Ь, с одной четности; пусть ими будут числа а и Ь. Так как Ьс той же четности, что и /;, тор = /)г + а чет- четное. Но р — простое число, стало быть, р = 2, откуда а = Ь—\ А тогда q=\bjtc = ca-\-\=r, что и требовалось. 38.04. Нельзя. Для доказательства соединим центры 28 край- крайних клеток доски. Получим замкнутый контур квадрата Г. Каж- Каждая из 13-и прямых пересекает контур Г не более чем в двух точках, откуда общее число точек пересечения с Г не больше 26, и они не могут отделить друг or друга даже эти 28 центров, а тем более все центры. 38.05. Можно. Пусть источник света располагается в точке О. Построим правильный тетраэдр ABCD с центром О Рассмотрим далее 4 круговых конуса, содержащих строго внутри себя пира- пирамиды ОВСП. OACD, OABC, UABL) и имеющие общую вершину О (конусы бесконечные). Э[и конусы частично пересекаются, так чти л ю б о й луч, исходящий из источника света, содержится внигри одного из конусов В нмигм теперь в кажяый конус по одчиму шару так, чтобы -»г» -'эры не пересекались (для этого
радиусы шаров должны сильно отличаться друг от друга, напри- например они могут равняться 10, 104, 107 и 1010). Очевидно, что любой луч ОХ, исходящий из источника, пересекает хотя бы один из шаров. 38.07. Указание. Докажите сначала, что два из трех углов по 120° примыкают к одной стороне семиугольника (ср. с реше- решением задачи 31.07). Затем докажите, что если многоугольник вписанный, то из равенства углов ABC и BCD следует АВ = CD 38.08 и 38.21. Указание. Коля проигрывает, если число камней равно 2* — 1; в остальных случаях он выигрывает, каждый раз загоняя Витю в положение, где число камней в наибольшей куче имеет вид 2'—1 (например, если « = 100, первым ходом Коля должен разделить кучу на 63 и 37 камней). При я = 31 выигрывает Витя. 38.09 и 38.12. а) Заменим все четные цифры в последова- последовательности на 0, а все нечетные — на 1. Получим последователь- последовательность, в которой периодически повторяются цифры НПО. Набо- Наборам 1234 и 3269 отвечают четверки цифр 1010 и 1001, которые в последовательности не встречаются, б) Имеется лишь конеч- конечное число четырехзначных чисел (в них включаются и числа, начинающиеся с нулей). Поскольку вся последовательность бес- бесконечна, найдутся два одинаковых четырехзначных числа: 1975 ... А ... А ... . Докажем, что в промежутке между этими равными числами расположено число 1975. Для этого заметим, что нашу последовательность можно однозначно продолжить влево с сохранением условий задачи (т. е. зная 4 идущие подряд цифры, можно найти не только цифру, идущую за ними, но и цифру, стоящую перед ними). Следовательно, зная лишь один кусок последовательности (А ... А), м ы можем восста- восстановить бесконечную в обе стороны последо- последовательность. И если бы в промежутке между Л и Л не встречалось число 1975, то оно не встречалось бы и во всей последовательности, что противоречит условию. Следовательно, набор 1975 встретится вторично, в) Заметим, что перед набором 1975 стоит цифра 8. Следовательно, мы нашли набор 8197 в бесконечной в обе стороны последовательности. Стало быть, он имеется ив исходной последовательности. 38.10. а) При всех л!>3 правое число больше (доказательство по индукции), хотя при п=2 верно обратное, б) Обозначим че- через Ап левое число (степени из п троек), через Вп-\—правое (степени из п — 1 четверок). Докажем по индукции, что АП + \> >2ВП. Предполагая, что An >2Bn~i, имеем: An+1 = 3А"> 32B"~' = =9В""' =B,25)в"' • 4В— > 2 ¦ 4В"-' = 2В„. 38.13. В Обычной стране города А и В не соединены доро- дорогой (иначе из А в В можно было бы попасть без пересадок), поэтому они соединены дорогой в Зазеркалье. Теперь пусть С и D — любые два города. В Обычной стране С не мог соединяться 254
с А и В одновременно (иначе из Л в В можно было бы проехать с одной пересадкой), так что в Зазеркалье из С ведет дорога в А или В. То же верно для D. Поэтому Алиса в Зазеркалье может проехать из С в D (через города А и В), сделав не более двух пересадок. 38.14. п очков. Например, первая команда выигрывает у всех остальных, набирая In— 2 очка, а остальные команды играют друг с другом вничью, набирая по л — 2 очка каждая. 38.18. Пусть X и У — два города таких, что после закры- закрытия дороги АВ кратчайший путь /изАвУне меньше 1500 км. Легко видеть, что на пути / должен быть город М, от- отстоящий не менее чем на 500 км как от X, так и от Y. Отсюда сле- следует, что до закрытия дороги АВ кратчайший путь из X в Y проходил через дорогу АВ (имел вид ХАВМ) и кратчайший путь из Y в М тоже имел вид либо YABM, либо YBAM. В пер- первом случае после закрытия дороги АВ остались открытыми пу- пути ХА и YA и путь XAY не превосходит 1000 км, что противоре- противоречит предположению, что кратчайший путь из X в Y не меньше 1500 км. Во втором случае пути ХА, ВМ, YB и AM по-прежнему открыты, ХА +ВМ<500 и УВ+ЛМ<500. Поэтому путь XAMBY также короче 1000 км. Замечание. Улучшить оценку 1500 км нельзя, что видно из следующего примера: пусть в Мантиссе только 4 города А, В, С и D, причем АВ=\ км, AC=CD — DB = 498 км. Тогда после закрытия дороги АВ кратчайший путь из А в В — почти 1500 км. 38.19. Предположим, что выпуклый многоугольник М разре- разрезан на п невыпуклых четырехугольников; сумма их углов 5 = = 360°п. Вершину четырехугольника, угол при которой больше 180°, будем называть «невыпуклой». Невыпуклая вершина X не может лежать на границе многоугольника М, и никакие два четырехугольника не могут иметь общую невыпуклую вершину (иначе они пересекались бы). Отсюда число невыпуклых вершин равно числу четырехугольников, причем все невыпуклые вершины расположены строго внутри М. Определим сумму углов с верши- вершинами в точках Xi, ..., Хп, являющихся невыпуклыми вершинами четырехугольников. Она равна 360°п. Но тогда S не меньше, чем эта сумма плюс сумма всех углов многоугольника М. Про- Противоречие. Замечание. Сферу можно разбить на конечное число невыпуклых четырех- четырехугольников (разбиение придумайте сами). 38.23. Искомое освещение возможно для произвольно- г о числа прожекторов. Занумеруем их числами 1, 2,..., п. Заметив, что число пар (i, j) равно — п (п— 1), построим следующую кон- конструкцию. Расположим над ареной п круглых дисков того же радиуса, что и арена. В каждый диск впишем правильный 255
'л-—*¦ л"^ ~х-п(п—1)-угольник одинаковым образом. Отрежем теперь от всех дисков, кроме выбранных двух, один и тот же маленький сегмент, образованный стороной вписан- вписанного многоугольника и опирающейся на нее дугой окружности (рис. 81). Потом от- откинем другую пару дисков и у остальных отрежем один и тот же сегмент и т. д., всего —п{п—1) раз. В результате у каж- дого диска будет отрезано п — 1 сегментов. Очевидно, что полученная система про- прожекторов удовлетворяет заданным усло- условиям. 39.01. Обозначим через X и У соответ- соответственно наибольшее и наименьшее числа среди Х\, ..., х$. Тогда из соответствующих уравнений имеем: Х2^2Х и У2^2У. По- Поскольку Х>0, У>0, получаем: 2<!У<; ^Х^2. Следовательно, система имеет единственное решение: Х\ = Х2 = ...^=Х5 = 2. 39.02. Может. Для этого угол ABC дол- должен быть почти прямым, сторона АВ (к которой проведена высота СН) — очень маленькой, а сторона ВС (к которой про- проведена медиана AM) — очень большой. Б предельном положении точка Н совпа- совпадает с В и площадь треугольника М'Н'К' (в пересечении) станет равной —. Oie- довательно, в близком к предельному по- положению площадь этого треугольника мо- может быть больше 0,499. 39.03. Первые четыре цифры числа бу- будут 1000. Указание. Оценив сумму первых 999-ти членов геометрической про- прогрессии 999 + 9992+... + 999999, докажите, что при сложении с 10001000 она не влияет на четвертую цифру слева. 39.04. Можно. Поместим в центр куба 8 прожекторов, осве- освещающих все пространство, так, чтобы один из них освещал ровно две вершины куба (это можно сделать, поскольку угол между большими диагоналями куба острый). Тогда один из прожекторов, по принципу Дирихле, не освещает ни одной вершины данного куба. 39.05. Можно. Для этого следует, например, выложить костя- костями домино указанные на рисунке 82 бесконечные уголки. 256 Рис. 82.
39.07. Может. Для этого следует сделать общей вершиной точку С у равных тупоугольных треугольников ABC и CDE (ту- (тупой угол отличен от 108°; см. рис. 83), а затем расположить их так, чтобы длины диагоналей АС, СЕ, ЕВ,-AD были равны друг другу и отличны от длины BD; длина стороны АЕ не равна 1. 39.08. Существует. Например, число 10 111 111 Ш. 39.09. Можно. Указанное расположение изображено на ри- рисунке 84 (каждая точка соединена с тремя ближайшими к ней). 39.12. Указание. Вставляя перегородки длины 1, разбейте все квадраты на единичные. 39.16. Не может. Сократив п\ на максимальную степень ш, получим число, кратное большой степени 2. Однако 1976 не делится даже на 16. 39.17. Рассмотрим пятно наибольшего радиуса и про- проведем окружность чуть большего радиуса с тем же центром (она содержит рассматриваемое пятно), не пересекающуюся с пятнами. Отразим центрально-симметрично (относительно центра сферы) все пятна. Легко видеть, что отраженные пятна не будут покрывать всю окружность. Любая непокрытая точка окружности и ей диаметрально противоположная искомые. 39.18. Указание. Предположив противное, докажите, что для любых натуральных А и k справедливо неравенство S(A-26k)^S(A) + 27k, где S (А) — сумма цифр числа А. Но S(/4-26fc)<9lg А + Ш, откуда легко получается противоречие. 39.19. Дока ж ем, что в числе N любые две соседние цифры можно переставить местами. Обозначим эти цифры че- через а и b и добавим слева к ab число ЪЬ, справа число аа, т. е. получим ....bbabaa... . Вычеркнув теперь два стоящих под- подряд числа ~Ьа, получим искомую перестановку ab++Ьа. Отсюда вытекает, что всегда можно одновременно вычеркивать две оди- одинаковые цифры, даже не обязательно стоящие рядом. Сделав все такие вычеркивания, получим число с попарно различными цифрами; оно меньше 109. Замечание В математике это задача о структуре группы, в которой квадрат любого элемента равен единице (соответствующий «сомножитель» можно вычеркнуть или вписать). Рис. 84. 257
\Z /\ \/ \Z Рис. 85. 39.20. На клетчатой плоско- плоскости можно так расположить оди- одинаковые пятиугольники с верши- вершинами в узлах («домики»), чтобы они покрывали все узлы (рис. 85). Можно покрыть каждый «домик» пятаком так, чтобы пятаки не соприкасались. 39.21. Существует. Наимень- Наименьшее число, обладающее этим свойством, равно Л = (Ю" X-y|f= 13 223 140 496. Указа- Указание. Если число АА — точный квадрат, где А — л-значное чис- число, то 10"+1 должно делиться на квадрат простого числа р. Следует положить р=11, а п выбирать нечетными. 39.22. Существует. Например, произвольная пирамида, в ос- основании которой расположен выпуклый 1975-угольник. Указа- Указание. Расставим произвольно стрелки на ее ребрах и спроеци- спроецируем полученные векторы на прямую, содержащую высоту пира- пирамиды. Сумма проекций отлична от нуля. 39.23. Указание. Заменив 10, 20 и 200 на k, r и kr, решите задачу в общем виде индукцией по сумме k-\-r. 39.24. Введем на плоскости такую прямоугольную систему координат, чтобы все точки оказались в первом квадранте. Среди всех рассматриваемых точек выберем множество таких, расстоя- расстояния от которых до ближайших к ним минимально возможные (все эти расстояния одинаковы). Рассмотрим точку А из этого мно- множества, являющуюся в выбранной системе координат самой правой точкой из всех самых нижних точек по отноше- отношению к остальным точкам этого множества. Соединим А со все- всеми ближайшими к ней точками В\, Во, В3, ... . Тогда все углы Z.B\AB2, AB2AB3, ... не меньше 60°. Следовательно, у точки А не более трех ближайших соседей. 39.25. Будем считать, что имеющиеся по условию 5 разноцвет- разноцветных точек таковы, что никакие 4 из них не лежат в одной плос- плоскости (противоположный случай почти очевиден). Соединим по- попарно эти 5 точек 10-ю прямыми и предположим, что каждая из них окрашена только в два цвета. Возьмем плоскость, не параллельную ни одной из проведенных прямых. Тогда среди всех точек пересечения этой плоскости с указанными прямыми найдутся по крайней мере 4 точки разного цвета. Далее, взяв эти 4 разноцветные точки, проведем аналогичное рассуждение для плоскости, получаем прямую, окрашенную не менее чем в 3 цвета. 40.01. Заметим, что если хп ч е т н о и равно произведению некоторой степени двойки на нечетное число, хп=2кт, то xn+k н е- 258
четно. Если же хп н е ч е т н о, xn = 2k Bm-}- 1)+ 1, то таким же рассуждением получаем, что xn+k ч е т н о. Следовательно, нечет- нечетных чисел, так же как и четных, в данной последовательности бесконечно много. 40.02. Обязательно. Указание. Рассмотрите выпуклую оболочку всех 2п квадратов. 40.03. Можно в обоих случаях. Для протаскивания прово- проволочного куба годится щель в форме буквы «П» с единичной длиной перекладины и достаточно длинными вертикальными «ножками». Для протаскивания тетраэдра годится щель в виде буквы «Т» с длинными прямолинейными частями. 40.04. Пусть А к В — множества соответственно синих и красных точек. Если бы множество А содержало лишь конеч- конечное число точек с координатами, кратными а, а множество В содержало лишь конечное число точек с координатами, крат- кратными Ь, то оказалось бы, что в ряду натуральных чисел имеется лишь конечное количество чисел, кратных ab. 40.06. а) Не может быть. Иначе в вертикальных столбиках находилось бы нечетное число белых кубиков, а их 14. б) Мо- Может. 12 черных кубиков можно расположить вдоль шести ре- ребер АВ, ВС, СС, CD', D'A', А'А большого куба ABCDA'B'C'D' и еще один черный кубик надо поместить в центр. Тогда в каждом столбце будет или 1, или 3 черных кубика. 40.07. Исходное равенство равносильно такому: . .15 . . . 15 (т) +(f) «¦ Выбрав произвольные числа п и k, положим — = п, — = k. Тогда c = nl5-{-k15, после чего а и b определятся однозначно: 40.08. Второй. 40.09. п = 50. Любой выпуклый 100-угольник можно получить в виде пересечения 50-и треугольников, беря вершины 100-уголь- ника через одну (рис. 86, а). 100-угольник с одной очень длинной стороной и очень маленькими остальными не представляется в виде пересечения меньшего числа треугольников (рис. 86,6). Л \ 5) Рис. 86. 259
40.16. Существуют (во всех рассматриваемых случаях). Ес- Если п искомых чисел а\, G2, •-., a«-i, an уже построены, то новые п-\-1 чисел строим так: полагаем Ь0 = НОК (oi, a2, .--, ап), Ь\ = а\-\- -\-Ь0, bo = ao-\-bo, ..., bn = an-\-bo. 40.18. Предположим, что вершин у многогранника боль- больше 8, и рассмотрим произвольные 9 его вершин. Из них по крайней мере 5 имеют первую координату одной четности (или все 5 четные, или все 5 нечетные). Вторые координаты по крайней мере трех из этих 5 точек также имеют одинаковую четность. Наконец, третьи координаты каких-то двух из этих трех точек также имеют одинаковую четность. Следовательно, мы нашли такие две вершины многогранника, у которых совпа- совпадают четности каждой из трех координат. Но тогда середина отрезка, соединяющего эти две вершины, имеет целые коорди- координаты, т. е. является узлом решетки. А в силу условия выпуклости многогранника она принадлежит многограннику. Получили про- противоречие с условием. 40.20. Можно. Например, круги можно расположить далеко друг от друга так, чтобы их центры лежали на параболе у = х2. 40.23. Пусть последовательность уп периодична с периодом Т начиная с некоторого п0. Тогда хп + т—хп четно при всех п>п0, с другой стороны, оно равно (-^-)" "°ш(хПо+т—хпо). При боль- большом п последнее число нечетно. Противоречие. 41.01. @, 2); @, -2); C, 5); C, -5); D, 7); D, -7). 41.02. Треугольники с углами: .Л Л Л . <-ч 2л Л Л.чЛ Л Л . » Л Л Л а' Т' Т' Т' ' Т' "б"' Т' ' ~2~'Т'Т' ' Т'Т'Т- 41.09. См. решение задачи 33.13. 41.10. п — 2 точки. Соединим произвольную вершину п-уголь- ника с остальными, получим п — 2 треугольника. Значит, мини- минимальное число точек не меньше п — 2. Докажем, что п — 2 точки можно расставить требуемым способом. Для этого выделим произвольную сторону А В и рассмотрим все треуголь- треугольники с основанием ЛВ (их п — 2). В каж- каждом полученном треугольнике очень близ- близко от его вершины, противолежащей сто- стороне А В, поместим по точке так, чтобы в каждом треугольнике, образованном малой диагональю и двумя сторонами многоугольника, содержалась точка. До- Докажем теперь, что в треугольнике с про- извольно выбранными вершинами XYZ содержится хотя бы одна точка. Дейст- Действительно, один из углов AXYZ содержит некоторый ААХкВ (где Xk совпадает с X, Y или Z), а потому содержит точку, распо- расположенную вблизи Xk (рис. 87). 260
Рис. 88. 41.11. Не существует. Указание. Рассмотреть все век- векторы с одной и той же максимальной лг-координатой и вы- выбрать из них два вектора, имеющие наибольшие у-координаты. 41.13. Выберем в качестве оси произвольную прямую /, не перпендикулярную ни одной из данных прямых и достаточно удаленную от множества S точек пересечений данных прямых друг с другом. Тогда от точки А на оси /, имеющей достаточно большую координату R, расстояния до всех точек множества S превосходят все расстояния до данных прямых (если первые расстояния «/?, то расстояние от А до произвольной прямой л;R sin ф, где ф — угол между этой прямой и осью /). 41.14. Может. На рисунке 88 указан граф сообщений, со- содержащий 22 ромба и 100 вершин; в каждой вершине стоит число, равное количеству дней, необходимых для попадания в нее новости из вершины 0. Вершине 64 отвечает 5 марта; еще через 8 дней A3 марта) новость становится известной всем. 41.15. Заметим сразу, что 12% в условии задачи можно заме- заменить на E0 — е)%, где е>0 — произвольное положительное, но сколь угодно малое число. Действительно, отразив центрально- симметрично сферу, получим, что не вся сфера красная: на ней имеется полностью белый участок. Центрально-симметрич- . ный ему участок на сфере также белый. Выбрав по 4 вершины квадрата в каждом из этих белых участков (квадраты центрально- симметричны относительно центра сферы), получим искомые 8 вершин параллелепипеда. (Другое решение состоит в том, что при трех отражениях сферы относительно трех взаимно перпендикулярных плоскостей, проходящих через центр сферы, у полученной сферы будет закрашено красным не более 8-12% =96% площади; следовательно, найдется белая точка, ко- которая в совокупности с точками, попавшими в нее при отраже- отражениях, дают искомые белые вершины параллелепипеда. На этом пути можно доказать, что существует куб с белыми верши- вершинами.) 261
41.18. Наименьшее п, при котором 2" = ...п, равно 36: Далее нужно применить индукцию: пусть 2" = ... \а\п где а — цифра, стоящая слева от п; докажите, что 20"= Нл 41.19. Рассмотрим четыре вектора и, =(а, b), v2 = (c,d), v3 = = (е, f), Vi = (g, h). Указанные шесть чисел равны попарным скаляр- скалярным произведениям этих векторов: ac-\-bd = vi • v2, ae-\-bf = v\ -vz, ..., eg-\-fh = V3-V4. Но один из углов между введенными векто- векторами не превосходит -^-, поэтому одно из шести скалярных произведений неотрицательно. 42.01. а) 5 кругов искомым образом расположить можно. Например, можно разбить плоскость на 5 одинаковых углов ои, СС2, аз, cd, as (а, = 72°) и вписать по одной окружности в углы aiL)ct2, CC2UCC3, о.з[]а4, cciLl^s, asU^i- б) Нельзя. Рас- Рассмотрим углы с вершиной в точке А, в которые вписаны окружно- окружности. Величина каждого из них меньше 180°, и, следовательно, сумма всех углов меньше 4-180° = 720°. Однако, если бы произ- произвольный луч с вершиной А пересекал не менее двух из этих четырех кругов, эта сумма была бы не меньше 2-360° = 720°. Противоречие. 42.02. Пусть гирь п штук и рк — масса гири с номером k. Пусть pi^—, рг^-т-, ..., Рл^трг-; тогда общая масса всех гирь не превосходит 1—™-. Противоречие с условием. 42.03. Среди всех прямоугольников, вписанных в данный круг, наибольшую площадь имеет квадрат (докажите сами). Значит, площадь S (П) прямоугольника П, вписанного в круг К, и площадь S (К) круга К удовлетворяют условию 4pS (П) <S (К). Пусть квадрат К' площадью S {К') разбит на прямоуголь- прямоугольники П1, П2, П3, ..., П„. Тогда, учитывая, что площадь круга, описанного около квадрата, равна —-S (К'), и используя выпи- выписанное выше неравенство, получаем: -^-•S {K')^ — (S (П|)+ ...+ + S (nn))<S (/A) + ... +S (/(„), где /С, — круг, описанный вокруг прямоугольника П,-. Требуемое неравенство доказано. 42.04. Витя всегда выигрывает. Для этого он может посту- поступать так: выбрать центр какой-то клетки и отмечать точки сим- симметрично точкам Коли относительно этого центра. Легко видеть, что Вите всегда найдется ход (выпуклый многоугольник после хода Вити остается выпуклым). После 6 ходов Коля может отме- отмечать только узлы бумаги, расположенные в шести треугольных областях, образованных каждой стороной шестиугольника и про- продолжениями соседних с ней сторон, т. е. у Коли остается лишь 262
конечное число возможностей. Следова- Следовательно, игра кончится в пользу Вити через конечное число ходов. (Ср. с решением задачи 31.20.) 42.05. См. решение задачи 42.01. 42.07. Проведем хорду АЕ, перпенди- перпендикулярную DA. Дуга АЕ равна дуге СВ и величина каждой из них равна 180°—\^DA. Из прямоугольного треугольника ADE ¦ г ИС. oJ. (рис. 89) следует, что ОН=—АЕ. 42.10. Пусть D — максимальное значение числа d. Рассмотрев набор из 55 камней одинаковой массы ур, мы видим, что ?>>-jy ¦ Докажем, что D=~. Предположим, что D > yj-, т. е. существует набор камней с общей массой 100 кг, масса любого поднабора которого удовлетворяет неравенству \М — 10|>ур. Рассмотрим тогда такой поднабор, для которого Л1>10, но после выкидывания любого камня масса уже становится меньше 10 (очевидно, такой поднабор существует). Пусть xi, ..., Xk — массы камней этого поднабора, M=xi + ... + %. По предположению М>10+ но М—х,<10—— при любом L Это означает, что все массы камней xi, 1=1, 2, ..., k, больше —. По условию j откуда следует, что число камней k>5 Но теперь ясно, что 20 М'=х\ + #2 + лгз + #4 +-*Г5 удовлетворяет неравенствам 5-ур< <М'<Ю, т. е. масса первых пяти камней отличается от 10 кг меньше чем на 10 —-=—- . Противоречие. 42.11. Можно. Например, поступим так. Возьмем прямую а и через две точки на ней проведем перпендикулярные к ней непа- непараллельные между собой прямые Ь и с. Рассмотрите далее мно- множество плоскостей, параллельных друг другу и прямой а; проведем прямые через точки пересечения прямых Ь и с с каждой из этих плоскостей. И наконец, совершим всевозможные повороты пространства вокруг оси а. Образы построенных прямых и обла- обладают нужным свойством. Замечание. Другое (неэлементарное) решение состоит в том, что все пространство есть объединение софокусных однополостных гиперболоидов, каждый из которых «соткан» из попарно скрещивающихся прямых (так называемая линейчатая структура однополостного гиперболоида или подвиж 263
ная стержневая модель гиперболоида). Соответствующие конструкции см в книге: Гильберт Д., К о н-Ф о с с е н С. Наглядная геометрия.— М ¦ Наука. 1981. 42.12. а) Существует. Строим последовательность кусками. Нулевой кусок состоит из одного члена: по — 1, п-й кусок состоит из 4~^п+1 членов, равных -j-bl, -\-bl-\-bn, ~bl ..., b2n, где Ь„ = 23" . Сумма членов n-го куска равна Значит, сумма членов предыдущих кусков не больше Ьп — bo, т. е. меньше Ь„, откуда следует, что никакой член «-го куска не равен сумме нескольких других. Оценим теперь п-й член последователь- последовательности. В (п—1)-м куске число членов равно -~-Ьп-\-\-\, следова- следовательно, номер любого члена в n-м куске больше 23"~ ~1, а его величина не превосходит Итак, aN<c64N6 для любого N, откуда aN<CN7 при Для всех N,. меньших 64, неравенство aN < ./V7 проверяется непосредственно, б) Не существует. Убедитесь сами, что уже число аъ не удастся выбрать так, чтобы выполнялось условие задачи. 42.09, 42.14, 42.19. Поставим всех ученых в одну шеренгу и занумеруем их слева направо. Затем начнем спрашивать каж- каждого в них, начиная с 1-го, о его правом соседе. Пусть п — минимальное число, при котором п-й сказал о своем соседе (т. е. об (п + 1)-м), что тот алхимик; до этого все говорили, что их соседи — химики. Тогда пару (п, л+1) выводим из цепочки и спрашиваем (п— 1)-го о его новом правом соседе и т. д. В кон- конце концов, выбросив несколько пар, мы дойдем до конца цепочки. Теперь в цепочке каждый сказал о своем правом соседе, что тот химик, а в каждой паре один сказал о другом, что тот ал- алхимик. Поэтому: 1°. В каждой из пар не меньше одного алхимика; поэтому 2°. В оставшейся цепочке химиков больше, чем алхимиков, в частности в ней есть хоть один химик. 3°. Если i-й человек в цепочке — химик, то и все последующие — химики. В частности, последний человек в цепочке всегда хи- химик, а если в цепочке больше одного человека, то двое послед-- них — химики. До сих пор было задано k — 1 вопросов. Поэтому если в це- цепочке больше одного человека, то достаточно теперь задать последнему в цепочке k — 2 вопроса о всех остальных, кроме предпоследнего в цепочке, общее число вопросов равно (k—1)+ -\-(k — 2)=2k — 3. Если же оставшаяся цепочка состоит из одного 264
человека (и он химик), то в каждой из отброшенных пар ровно по одному алхимику (иначе алхимиков было бы больше, чем химиков). Теперь достаточно задать по вопросу про одного из ученых выкинутой пары; число вопросов не превосходит k—1 + Замечание. Московский математик П. М. Блехер (победитель 26—28-й Московских олимпиад) доказал, что точная оценка в этой задаче равна -^-(&—1) - (Доказательство опубликовано в его статье «On a Logical Problem», Discrete Math. 43 A983), с. 107—110.) 42.16. Разрежем отрезок [0, 1] на 10 равных частей и про- пронумеруем их. Затем все части с нечетными номерами сдвинем вправо на 0,1, а все с четными —-влево на 0,1. Множество отмеченных интервалов перейдет в множество интервалов, не пере- пересекающихся с исходными (это следует из условия), а общая длина отмеченных и полученных интервалов будет не больше 1. Значит, сумма длин исходных интервалов не больше 0,5, что и требовалось. 42.17. Наибольшее значение равно -^-, оно принимается функцией g(x)—-2~л:A — х) в точке х=-~-. Действительно, предположим, что нашлась такая функция /, что /@)=f(l)=0, |/"(дг)|<1 для всех х? [0, 1] и в некоторой точке а6@, 1) f{a)>±-. Положим h (х) = f (х) — ^ • g (х). Так =-I, f(a)>-i->g(a)>0, то Л(О) = ЛA)= 0, h" (х)= ^ Кроме того, Л(а)=0. Из условия h" (x)>0 как g(x)=I, f(a)> следует, что h' (x) монотонно возрастает. Значит, на одном из отрезков [0; а] или [а; 1] она не меняет знака. Но тогда либо а 1 Л@)= — \ h'{x)dx=?0, либо h{\)=\h'{x)dx=?Q, что приводит 0 а оба раза к противоречию. 42.18. Возьмем круг наибольшего радиуса и рассмотрим но- новый круг с тем же центром втрое большего радиуса. Затем выбросим все круги, оказавшиеся полностью внутри нового круга: оставшиеся круги уже не пересекаются с исходным. Возьмем среди них максимальный круг и проделаем с ним ту же процедуру. Так делаем до тех пор, пока не получим несколь- несколько «раздутых» кругов, площадь объединения которых больше 1. Отвечающие им исходные (втрое меньшего радиуса) круги не пересекаются, и их общая площадь превосходит —. 43.01. 95 210. 265
43.03. Ориентируем все стороны отме- отмеченного внутреннего многоугольника про- против часовой стрелки. Каждой стороне по- поставим в соответствие вершину исходного многоугольника, в которую направлен ко- конец стрелки на этой стороне. В силу выпуклости многоугольников разным сто- сторонам отвечают разные вершины. Поэто- Поэтому число сторон внутреннего многоуголь- Рис. 90. ника не превосходит числа сторон исход- исходного, т. е. не больше 100. 43.05. Повернем 20-угольник вокруг его центра на угол, рав- равный по величине дуге окружности, концы которой совпадают с концами черной диагонали. При этом одна черная точка перей- перейдет в другую черную точку (в другой конец дуги). Следова- Следовательно, какая-то белая точка перешла опять в белую. Эти две белые точки соединяются диагональю той же длины, что и чер- черная диагональ. 43.08. Достаточно доказать, что р = 2ос (рис. 90), отсюда сразу следует, что треугольник BCD равнобедренный. Для доказательства соединим центр О окружности с точками А и D. Углы AOD и ACD опираются на одну и ту же дугу AnD, поэтому /LAOD = $. Но угол AOD центральный в окружности О, а угол ABD вписанный в нее, поэтому /LABD=—/LAOD, т. е. р = 2а. 43.11. Указание. Используйте тот факт, что в числе о* либо столько же цифр, сколько в a*_i, либо на одну больше. Поэтому если наша дробь периодична, то найдется такое k, что число аи в точности совпадает с периодом. После этого рассмотрение числа a*+i быстро приводит к противоречию. 43.12. Д о к а з а т е л ь ст в о проведем по индук- индукции. Пусть число состояний табло равно 2*, когда на пульте п кнопок (для п = 1 имеется ровно два состояния, отличающиеся нажатием кнопки). Добавим еще одну кнопку. Если она «ба- «базисная», т. е. при ее нажатии появляется новое состояние табло, отличное от всех 2* предыдущих, то каждому из состояний отвечает противоположное (когда зажженной лампочке отвечает незажженная и наоборот). Всего получаем 2-2* = 2*+1 состояний табло для пульта из (п + 1)-й кнопки. Если же кнопка не «ба- «базисная», то при ее нажатии загорается одно из уже встречав- встречавшихся 2* состояний табло. Итак, в любом случае получаем степень двойки. 43.13. тп квадратов. У каждого квадрата отметим его пра- правую верхнюю вершину. Никакие две отмеченные вершины у раз- разных квадратов не совпадают, что вытекает из условия задачи. Следовательно, количество квадратов на листе не больше числа узлов бумаги, не лежащих на левой и нижней сторонах листа, 266
Рис. 91 которых в точности тп. (тп квадратов размещаются требуемым способом однозначно. Докажите!) 43.20. Пусть cci, а2, ... — указанные дуги, причем по условию oti +а2 + ...<л. Рассмотрим произвольную дугу а, и выберем точку S на ней, а также полярную к S окружность Os (поляр- (полярной окружностью к точке S называется большая окружность, все точки которой равноудалены от точки S — полюса). Очевидно, что полярная окружность к произвольно выбранной точке на окружности Os проходит через точку 5. Заставим теперь 5 пробегать всю дугу а,-; при этом полярная окружность Os «за- «заметет» заштрихованную на рисунке 91 область. Площадь этой области составляет -^-=—-ю долю площади всей сферы. 2л л Окружности, полярные к любой точке этой области, и только они, пересекают дугу щ. Построим аналогичным образом такие же области для всех дуг а,-, аг, ... . Они не покрывают полностью сферу в силу условия 2j — <1. Следовательно, найдется не- I покрытая этими областями точка. Плоскость полярной к ней окружности искомая. 44.01. Остаток равен 7 (остатку от деления 35 на 14). 44.03. Наложим второй круглый лист на первый так, чтобы драконы совпали; тогда глаз нарисованного на втором листе дра- дракона совпадает с глазом первого дракона, т. е. находится в его центре О. Разрежем второй лист по дуге АпВ (см. рис. 92) и наложим отрезанную часть на соответствующую часть первого круга; глаз дракона окажется в центре. 44.04. Обязательно. Указание. Если \^/~фс]=п, то п4^ ^.v<(«+lL. Теперь легко доказать, что [y[^*j] = «. 44.05. Легко проверить, что, зная сумму масс любых двух гирь, можно определить массу каждой гири в отдельности. Поэ- Поэтому поступаем так. Выбираем произвольные две пары гирь и взвешиваем каждую пару. Тем самым мы определяем, есть ли хотя бы в одной из них гиря массой 1000 г. Если есть, то третьим взвешиванием одной из гирь соответствующей пары опреде- 267
ляем ее, если нет, то оставшаяся пятая гиря имеет массу 1000 г. 44.10. Обозначим наименьшее общее кратное всех чисел через А. Тогда A = k,a1 = ... = Аюаю, где k\ ki0— натуральные, и из условия получаем: k\ >/:2>...>k$>k\D. Значит, минимальное значение ki равно 10, т. е. A = ktai^ 10а,. 44.12. Нетрудно видеть, что если а и b — натуральные числа, причем a^.k и b^.k, то справедливо хотя бы одно из неравенств: 1, |а —6 Поэтому найдутся наборы знаков « + » и « —» такие, что одно- одновременно будут выполнены неравенства \an±cin-i\ <«— 1, lan±an-i d=a,,-2l <« —2 |an±an-i±...±a,|<l. Но число ап±ап-\±-.-±а\ четно, поскольку сумма ai + ...+an четна. Значит, последняя сумма равна нулю. 44.13. Указание. Докажите сначала следующее утвержде- утверждение: если М — выпуклый многоугольник, S — его площадь, Р — периметр, г = г(М)—радиус наибольшего расположенного внутри М круга, то i-r<-|-<r. Отсюда §g*<r (*)<r 44.14. Можно. Сначала рассмотрим два множества натураль- натуральных чисел Л и В: в Л входят все натуральные числа, у которых на четных местах в их десятичной записи, считая с правой последней цифры, стоят нули, а в В входят все натуральные числа, у которых нули стоят на нечетных местах. Каждое натуральное число п единственным способом представляется в виде n = a-\-b, a?A, b?B. Искомое разбиение натурального ряда на подмножества А\, Л2, Л3, ... таково: Л1=Л, а каждое Л* (k = 2, 3, ...) получается из множества Л прибавлением ко всем его элементам числа bk?B, т. е. Л2, Аз, ... — сдвиги множества А на соответствующие элементы множества В. 44.15. Справедливо общее утверждение: если у правильного п-угольника (п^4) отмечено k = \ " -f-1 вершин, то существует трапеция с вершинами в отмеченных точках. Для п=1981 получаем k = 64. 44.16. Из условия следует равенство f (x-\-k)= ,,? - Тогда + 2*)=—1_; f(x + Ak)^-n-^=f(x). Значит, f (х) - периодическая функция с периодом 4k. 44.17. Рассмотрим многочлены Pi (х) = Р (х) — Р (х — 1), Pi (x) = =PI(x)-Pl(x-l), ..., Pi{x) = P,-l(x)-Pi-X(x-l), .... Рп(х) = = Pn-i (х) — Рп-\ (х— 1) (многочлен Pi (x) называется i-й раз- разностью многочлена Р (х)). Нетрудно проверить, что степень много- многочлена Р, (х) равна п —i и что Л- (х) при любом целом х делится на , р. Но Рп(х) = п\, поэтому п\ кратно р. 268
44.18. Воспользуемся следующим тригонометрическим нера- неравенством: |sin (а — р)| < |sin а| + |sin р|. Пусть sin (k2) ->- 0. Выберем е< <—|sin2| и такое N, что |sin(n2)|<e при любом n>N. Ис- 8 пользуя тригонометрическое неравенство дважды, получаем: Isin ((n + \f-n2)\ = |sin B/2 + 1I < | sin (n + 1J| + I sin («2)l <2e, I sin (Bn + 3)-Bn+ 1))| = Isin 2| < |sin Bn + 3)| + Isin Bn+ 1I < <2e + 2e = 4e, откуда следует противоречивое неравенство: Isin 2|<4e<4-!5!!pi=-i-Isin 2|. 44.20. a,=f-,a2=-f-,a3=-^. л/То Vl5 V15 44.21. Если n = 2k, то наименьшее число перестановок равно 2-Cl = k(k—\)\ если n = 2k-\-\, то оно равно Cl+Cl+i=k2. Указание. Наименьшее число перестановок получается, если всех людей разделить на две одинаковые (в четном случае) или отличающиеся на единицу (в нечетном случае) группы, разделив круглый стол по диаметру, а затем в каждой поло- половине сделать еначала перестановку одного человека со всеми людьми этой половины, затем перестановку второго со всеми (кроме первого), затем — третьего и т. д. 45.02. Проведя через центр квадрата прямые, параллельные его сторонам, разрежем квадрат на 4 одинаковых квадрата. Какие-то 2 из 5 точек лежа'т в одном из этих квадратов, и рас- расстояние между ними не превосходит длины диагонали этого квадрата. 45.03. Любое вычисление числа а состоит из вычислений цепочки чисел: ao=l, at, аг, ..., an-i, а„, где либо а«, = 3а*_1, либо а/; = а*_1+4. Очевидно, вычислимы только нечетные числа, так как если а* нечетно, то числа За* и а*+ 4 тоже не- нечетны. В частности, 1982 невычислимо, а 1981 вычисли- вычислимо. Стоимость набора числа 1981 равна 42 к. 45.04. 8 точек. Очевидно, что 8 точек Аи А2, ..., As, распо- расположенных на расстояниях ЛИ2 = 1, А2А3 = 2, ..., А7Ае = 64, удовлетворяют условию. Покажем, что меньшего числа точек на плоскости расположить нельзя. Для каждого k=0, 1, ..., 6 выберем пару точек, между которыми расстояние равно 2*, и соединим их отрезками. Из неравенства треугольника следует, что полученные 7 отрезков (и никакая часть из них) не обра- образуют замкнутого многоугольника. Следовательно, число точек должно по крайней мере на 1 превосходить число этих отрезков, т. е. быть не меньшим 7 + 1=8. 45.05. 1+V5. 45.07. Не может. Указание. Полученное число дает при делении на 3 остаток 2 и потому не является полным квадратом. 269
45.08. Из условия следует, что EDCD' и DCBC — параллелограммы (рис. 93) Поэтому EC' = EB — C'B = EB — DC = = EB-ED' = D'B. дЕС'В'со AAC'B, aED'Cco aAD'B откуда M-=SI?L==^§_=_AB_= JA AB C'B ED' EC _ AB AB Рис. 93. ~ EB' + B'C - Ei Обозначив л:=——, получаем уравнение х=——, корень r\ts i —|— x которого х=^ ~ (отрицательный корень отбрасываем). Значит, СИЛУ симметрии остальные отношения равны тому же числу. Замечание. Любой такой пятиугольник можно получить из правиль- правильного пятиугольника некоторым аффинным преобразованием (например, сжа- сжатием вдоль произвольно выбранного направления). 45.09. Доказательство проведем по индукции. Для п = 1 вер- верно а\ + а? > 2а?оа? (а, — 1 J > 0. Предположив, что при n=k неравенство справедливо, дока- докажем его для n = k-\-\. Пусть ai<C...<Oft+i — произвольные нату- натуральные числа. По предположению индукции (al + ...+al) + (? ?J(? +iJ Но 2a? + 4? (? + + f) i+4ai+i--|-(a* + i —lJa*+i=a*+i+a|+i (здесь мы суще- существенно использовали, что все а, различны). Сложив эти нера- нераI ? ( венства, получим: (aI + ...+a*+i)+(a? + ... + aft+i)>2(ai + ...+ -4-a*+iJ, что и требовалось доказать. Равенство справедливо при щ = \, а2 — 2, .... ап = п. 45.10. Все числа вида 6/г + 1 и 6/г + 2, /г = 0, 1, 2 45.11. См. решение задачи 23.20. 45.12. Легко видеть, что точка Л (-у/2;—) удалена от всех узлов квадратной решетки на различные расстоя- расстояния. Измерим расстояния от А до всех узлов решетки и рас- расположим их по порядку: R\ <.R.2<. — <.R\sb2<.Risw<.... . Тог- Тогда окружность с центром в точке А и произвольным радиусом R, где /?1982</?<^198з, содержит внутри себя ровно 1982 узла решетки. 45.13. Указание. Наше число является бесконечной перио- периодической дробью -^-=0,A23456790) и поэтому не содержит даже цифры 8. (Подробнее см. в книге [24], задача 92.) 270
45.14. Р = 2 + 4 sin 15°. Указание. Сумма диагоналей четырехугольника ABCD больше суммы противоположных сто- сторон, и поэтому противоположные стороны не могут равнять- равняться 1. Следовательно, можно считать, что АВ = ВС=\, причем ААВС^.60° (поскольку АС^. 1) и точка D лежит в секторе ABC круга радиуса 1 с центром В. Теперь нетрудно показать, что пери- периметр будет наибольшим, если D лежит на окружности, притом на равных расстояниях от Л и С, и Z.ABC — 60°. 45.16. Обозначив левую часть неравенства через f (a, b, с), заметим, что f (a, b, c)=(a-\-b-\-c) (abc — (b-\-c — a) (a-\-c — b)X Х(^ + а — с)). Среди чисел Ь-\-с — а, а-\-с — b, Ь-\-а— с не более одного отрицательного (если а-\-Ь — с<СО, Ь-\-с — а<С0, то26<С0). Если отрицательно ровно одно из этих чисел, то их произведение неположительно и поэтому f (a, b, с)^0. Если же они все неотрицательны, то a2b2c2^(a2 — [b — сJ) (b2 — (acJ)(c2(abJ) = (b + caf-(a + cbJ(b + acY=> )( Y ( )( )( ) откуда f(a,b,c)^O при любых неотрицательных а, Ь, с. 45.17. а) Обозначим ф (х, у)=ху-\-х-\-у-\-1. Многочлен х-\-а при любом аф\ вычисляется по такой программе Р(х-\-а): ( ) *• !5f; «-2:ч>(*. |Ет): «р(ф(*. fer) - °-2)- пр» а=1 программа Р(х-{-1) такова: х; 0, <р (х, 0) = х-{-\. Для любого п программа Р (/„) вычисления многочлена [„ (х) = = 1 -\-х-{-...-\-х" строится индукционно с помощью «схемы Горнера»: P(fn (/) ( Р((/1 ) б) Пусть g\, g2, .., gn — произвольная программа. Индук- Индукцией по п докажем, что если ?„# const, то^„( — 1) = = — 1. Для п=1 это очевидно: gi(x) = x. Далее, gn=gigj + gi + + gj(i. /<«). и. например, g,-^const. Тогда gi{— 1)= — 1 (пред- (предположение индукции) и gn (— 1)= — gj (— 1)— 1 -\-gj (— 1)= — 1, что и требовалось доказать. Но gn=fn по условию; однако f„ (—1) = 0 или fn (—1)=1, a gn (—1)= — 1. Полученное противоречие доказывает, что для /„ (х) искомой программы (с указанной операцией калькулятора) не существует. Замечание. Все многочлены, вычислимые иа калькуляторе с операцией фО*. у) — хУЛ~х-\-у< имеют вид Fn(x)=A (х-\-\)"—1, и любой такой многочлен вычислим. 45.18. п = 1 илип = 4. Указание. Покажите, что п = 2" • 5* и п-\-1 = 2е¦ 5^ (а, Ь, с, d — целые числа). 45.19. Пусть центр О большого шестиугольника Q не лежит в малом 6-угольнике q. Обозначим через d радиус вписанного в Q круга (одновременно это диаметр круга, вписанного в q, т. е. расстояние между его противоположными сторонами). Опустим 271
из О перпендикуляр ОМ на самую дальнюю сторону q; легко видеть, что \OM\>d. Поэтому точка Н на пересечении ОМ и вписанной в Q окружности лежит ближе к О, чем М. Проведем через Н касательную к этой окружности до пересечения с Q в точках А и В; очевидно, отрезок АВ параллелен стороне малого 6-угольника и длиннее ее. Но это немедленно ведет к противоре- противоречию: легко показать, что отрезок касательной АВ к окружности, вписанной в правильный многоугольник со стороной а, имеет длину меньше -|-. Замечание. Приведенное рассуждение годится для произвольного пра- правильного л-угольника, где п — четное число. 46.01. D, 1); D, -3); (-4, 1); (-4, -3). Указание. Пред- Представим правую часть уравнения в виде (у-\- \f-\-12; перенеся (у-f-lJ влево, получим, что (х—у—1) {х-\-у-\-1)= 12. Заметив, что каждая скобка — четное число, получаем 4 возможности: одна из скобок равна ±2, вторая ±6, откуда легко следует ответ. 46.02. Две вершины равностороннего треугольника со сто- стороной /, имеющие одинаковый цвет, искомые. 46.03. Обозначим искомое число через Х = \аЪ...с — \-10" + Л, где A=ab...c {А — n-значное). После перестановки первой цифры в конец получаем число Y=ab...c4 = ЮА-\-4. По условию 4-10"+А=4(\0А+4), откуда 39Л =4-99...96 или 13.4 =4-33...332. Производя деление л-1 л—1 ОООП на 13 «в столбик», находим наименьшее значение А: А = 4 13 = 10 256. Следовательно, наименьшее значение X равно 410 256. 46.05. Нет. Указание. Докажите сначала, что стороны длин 11 и 13 должны лежать рядом. Воспользуйтесь тем, что отрезки сторон от любой вершины до точек касания равны между собой. 46.06. Левая часть равна ^—tJL Поскольку x>^J2 и 46.07. Заметим, что AA\=AC + CAU BBt=BA+ABu CCt = = СВ-\-ВСх (таким образом, стороны треугольника ABC ориен- ориентированы против часовой стрелки). Сложив эти равенства, получим: Сумма векторов в первой скобке равна 0, а каждый вектор во второй скобке получается из соответствующего вектора первой 272
скобки поворотом его на 60° по часовой стрелке; отсюда следует, что и вторая сумма равна 0. 46.08. Может. Одним из таких чисел является N = 999...995 A983 девятки). Действительно, /V2 = A01984-5J=102-198^-10-101984 + 25 = 99...9900...025 A983 девятки и 1983 нуля). Вообще, годится любое число, у ко- которого первые 1983 цифры — девятки, а 1984-я цифра не меньше 5. (Ср. с задачей 15.23.) 46.09. Существует, и притом единственная. Найдем эту рас- расстановку. Проведем ось симметрии через вершину с номером 1983 и будем считать, что она вертикальна, а вершина 1983 лежит на верхней полуокружности. Ось симметрии разбивает окруж- окружность на левую и правую половины. Номера вершин на левой половине обозначим (отсчитывая от вершины 1983 против ча- часовой стрелки) через а.\, аг, ..., Оээо, oggi, а номера симметричных им вершин — буквами bt с теми же индексами. Пусть п\>Ь\, п2>Ь2, .... О99о>^99о, Оэ91>^991. Проведем теперь ось симмет- симметрии через вершину 6991 (она пройдет между вершинами 1983 и а\). Так как расстановка «хорошая» и относительно этой оси, а 1983>Gi, то Ь\>ао, Ь2>а3, ..., ЬдвэХЗээо, йзэоХЗээ!- Объ- Объединяя эти и предыдущие неравенства в одну цепочку, получаем: 1983>а, >Ь\ >а-2>Ь2> ...>a99o>^9oo>099i- Отсюда вся рас- расстановка полностью определяется: а\ = 1982, ft, = 1981, .... а99о = 3, 6990 = 2, G99i = l. Простая проверка показывает, что эта расста- расстановка является «хорошей» относительно любой оси симметрии. 46.10. Указание. Докажите, что треугольники НН\оНц и HH2.iHn подобны, где //,, — основание перпендикуляра, опущен- опущенного из точки Н на -прямую AiAj. 46.11. Достаточно доказать, что f (х) = х2п— х2"~х -\-х2п~2— —... — х3+х2— х-{-1>— для любого положительного х. Но f(*)= ' если х^ 1, то, очевидно, f (х)^ I; если же х<С\, l+x нат( 40v'3 то знаменатель не превосходит 2 и опять дробь больше — • 46.12. 46.13. Указание. Докажите общее утверждение: S= 1" + t-...+я" при нечетном п делится на 1 + 24-¦-¦ + »=" (" + * (т. е. 2S делится на п и на п-|-1)- Для этого просуммируйте по- попарно крайние члены в сумме S, а затем отбросьте член п" и просуммируйте таким же образом оставшуюся сумму. 46.14. Рассмотрим произвольный город А. Предположим, что из него можно добраться только в «<20 городов. Множество из п городов, в которые можно добраться из А, обозначим че- через X; множество оставшихся 20 — п городов обозначим через 10 Заказ -Ш 273
Y. Тогда ни из одного города множества X нельзя попасть ни в один город множества Y, поскольку в противном случае из го- города А можно было бы попасть в какой-то город множества Y. Итак, среди С2о=19О возможных прямых авиалиний отсутствуют пB0 — п) авиалиний. Но п B0 — п)^ 19 при любом 0<п<20, и тогда 20 городов оказываются соединенными менее чем 172-мя авиалиниями, что противоречит условию. Стало быть, наще предположение неверно и утверждение задачи доказано. 46.16. Достаточно доказать, что из делимости 4т~" — 1 на 3*+| следует, что (т — п) делится на 3*. Это следует из того, что наименьшее а такое, что 4° — 1 делится на 3*+| равно 3*. 46.17. Студент прав. Сумма равна 10. Указание, cos 0-{- ¦ 2я ¦ i 8л л. i Зл. ¦ ¦ 9л г\ + COS -—I---+COS — = COS ——[-COS — +... +COS — = 0. О О О О О 46.18. а) Выберем точку А, из которой выходит максималь- максимальное число отрезков; обозначим это число через п. Если бы отрезки не образовывали ни одного треугольника, то вторые кон- концы отрезков, выходящих из А, не были бы соединены между собой. Из каждой из оставшихся 7 — п точек выходит не более п отрезков, поэтому общее количество отрезков не превосходит п-\-{7 — п)п = п (8 — п)<116, а их по условию 17. Противоречие. б) По доказанному в пункте а) существует треугольник ABC, составленный из наших отрезков. Сколько к нему может примыкать (общей стороной) треугольников? Если пд, пв и пс — количества отрезков, выходящих из вершин А, В, С (не считая сторон треугольника ABC), то таких треугольников не мень- меньше пд-\-пв-\-пс — 5. Если пд + пв + пс^8, то вместе с треуголь- треугольником ABC уже имеются 4 треугольника, составленных из наших отрезков. Пусть теперь пд + пв -f- пс <[ 7. Без ограничения общ- общности можно считать, что пд+пв^4. Выкинем точки А и В и все выходящие из них отрезки (всего пд-\-пв-\-3 отрезков). Останется 6 точек и не менее 17 — 7 = 10 отрезков. Остается проверить, что они образуют 3 треугольника. Один треугольник есть в силу рассуждений пункта а). Далее рассуждениями, ана- аналогичными приведенным, сводим задачу к проверке того, что 5 отрезков между 4-мя точками образуют 2 треугольника. Но это уже очевидно. 46.19. Пусть В\, ..., Bi—богатыри из некоторого города М. Предположим, что кубки обошли полный круг. Тогда у каждого из богатырей В, побывал каждый из золотых кубков, следова- следовательно, они держали золотые кубки в руках kl раз. Поскольку l<;fe<13, к1ф\Ъ. Если fc/>13, то в какой-то момент у богаты- богатырей из М было 2 золотых кубка одновременно, и все доказано. Остается случай ?/<с13. Но тогда в некоторый момент у богатырей из М не было ни одного золотого кубка. Поскольку число золотых кубков равно числу городов, в этот момент 2 зо- золотых кубка были у богатырей какого-то другого города N, что и требуется. 274
47.01. Могут. Например, годятся би- билеты 159 999 и 160 000. Вообще, годится любая пара номеров, первый из которых имеет вид ab 9 999, если а-{-Ь равно б или 13. Можно доказать (сделайте это сами), что других подходящих пар нет. (См. 30.01 и 08). 47.02. См. решение задачи 33.27. 47.03. 25 р. (одолженные и возвра- возвращенные соседу деньги можно не прини- принимать во внимание) 47.04. Пусть сначала все стороны па- параллелограмма попарно параллельны сто- сторонам треугольника. Очевидно, наиболь- наибольший из таких параллелограммов распо- расположен так, как на рисунке 94,а. Если S — площадь AAKL, Si и S2— площа- площади А В КС и ADCL, to St = K2S, S2=(l— к) S, где К=— , 1—К=— — D а) Рис. 94. коэффициенты подобия, и =[-^—\- 2 —U ] S не меньше чем 4-S ( равенство достигается при Если же одна из сторон параллелограмма не параллельна стороне треугольника, то можно через одну из вершин треуголь- треугольника провести прямую так, как на рисунке 94, б. Она разрезает треугольник и параллелограмм на два меньших, и в каждом из малых треугольников одна из сторон параллелограмма уже па- параллельна стороне треугольника. Проделав такую операцию один или два раза, мы сведем общую задачу к уже рассмотрен- рассмотренному случаю. 47.05. Если предположить, что король ни разу не встал под шах, то каждая ладья должна была по ходу игры сменить как свою первоначальную вертикаль, так и свою первоначальную горизонталь, т. е. сделать не менее двух ходов. Отсюда общее число ходов всех ладей не меньше 20. Но это число долж- должно совпадать с числом ходов короля, т. е. быть равным 19. Противоречие. 47.06. Из данного уравнения следует, что Возводя обе части в квадрат и извлекая кубический корень, получаем: 4 — л;2 = дс2, откуда л'2 = 2 и х=±-^2. Проверка показывает, что отрицательный корень посторонний. Поэтому х=д/2 — корень данного уравнения. 10* 275
Рис. 95. Рис. 96. 47.07. Расположим б точек (ЭВМ) в вершинах правильного шестиугольника. Его стороны закрасим через одну цвета- цветами 1 и 2, а диагонали—цветами 3, 4 и 5 (одним цветом закрашиваются 2 па- параллельные малые диагонали и перпенди- перпендикулярная к ним большая). , 47.08. Пусть Л, — вершины данного 7-утольника, О — его центр, Bt — произ- произвольная точка, отличная от О. Повернем Ц2 б б~, вектор OBt на углы Ц-, 2~ получим 7 точек В,, В2 .... В7 в вершинах правильного 7-угольника. Легко видеть, что сумма длин векторов AiBit ..., А\В7 равна сумме 2 длин векторов B\At, ..., ВИ7 (рис. 95). Но ¦±-(a7 ~ = AiO, откуда по неравенству треуголь- треугольника 7-|/i0|< |л7в.1+...+ |Л7В71=2. 47.09. Обозначим данные числа через х , х5. Из очевидного неравенства 2 —J 4--- следует, что Так как 1 ={х\ +...+x5f -\-х4х5-{-хъх1)-[-\х\Хз-[-хзх5-[-х5х2-{-Х2Х4-[-Х4Х1)), то, обозначив две суммы в круглых скобках через 2i и Иг, получаем: 1 4 2 ^1—— =-—, откуда Si4-2^ О О 2 -e-. Следовательно, меньшая из О сумм Si и не больше —. о Замечание. Можно доказать общее утверждение: п неотри- неотрицательных чисел с суммой 1 можно расставить по окружности так, чтобы сумма всех п попарных произведений соседних чисел была не больше —• Для этого надо рассмотреть суммы Si, -2,... попарных произведении соседних чисел по всевозможным расстановкам их на окружности. Каждые два слагаемых хя и Xkv,- {1ф}, кф1) будут во всех этих суммах встречаться равное число раз» Из того, что xi-\-.. -{-xl^— 11 [xl-\-... + x,,)-= 1 следует, что среднее арифме- арифметическое всех сумм Si, ?2, --• не больше — , значит, одна из этих сумм не 1 превосходит —• 47.10. Решение показано на рисунке 96. 276
47.11. He является. Указание. Введем новую «цифру» *, обозначающую число 10, и обозначим А = ***•-.** = I 11...110.. С4 64 Число А, как нетрудно видеть, кратно 101. Для каждого 64-знач- ного числа а = а.\С12...а.бзС1ы, удовлетворяющего условию задачи, рассмотрим «симметричное» ему число р = (*— щ) (* — аг).--(* — ) Оно тоже удовлетворяет условию, поскольку кратно 101 и каждая его цифра (* — а,) отлична от нуля. Могут ли совпасть два сим- симметричных числа? Если а = р, то а, = * — а, при всех i и тогда все а, = 5: такое число единственное и равно 555...55. Остальные ы числа разбиваются на пары (а, Р) неравных между собой чисел. 47.12. Обозначим через а, р, у плоские углы при вершине пирамиды. Выражая площадь каждой грани через длины ребер и синус угла между ними, получаем равенство sin a = sin p = sin у. Отсюда следует, что по крайней мере два из трех углов а, р, у равны между собой, а тогда две из боковых граней — равные треугольники и поэтому их основания равны между собой. 47.13. Нельзя, поскольку число проводов одного (произволь- 13 ного) цвета в таком соединении должно быть равным -^-, т. е. не целому числу. 47.14. Равносторонний треугольник площади 1 имеет сторону а—— и высоту /г=УЗ; легко видеть, что он не поместится V3 в полосу, уже, чем h. Докажем теперь, что любой другой тре- треугольник площади 1 поместится в полосу ширины h. Предпо- Предположим противное; тогда любая из его высот больше h и потому любая его сторона меньше а. Но треугольник, у которого все три стороны меньше а, имеет площадь меньше —a2 sin a. где а — любой из его углов, и так как можно взять угол его площадь меньше 1, что противоречит выбору треугольника. 47.15. Пусть а, Ь, с, d — четыре идущие подряд на окруж ности числа, причем а^Ь: тогда, очевидно, ab -f-be -f- cd^ <!a (b-*rC)-\-{b-\-c) d. Поэтому, выбросив числа Ъ и с и заменив их одним числом Ь-[-с, мы не нарушим условия задачи и заменим сумму Sn на большую Sn_i. Проведя такую процедуру несколько раз, мы сведем дело к четырем числам на окружности; если х, у, z, t — эти числа, то 47.16. Указание. Докажите, что если наименьшее рас- расстояние между четырьмя точками принимает максимально воз- возможное значение, то они расположены в вершинах ромба со 277
м 25 стороной —.две противоположные вершины которого совпадают с противоположными вершинами заданного прямоугольника, а остальные две лежат на больших сторонах прямоугольника. 47.17. Существуют. Примеры: 333 ... 3342= 11 ... 155 ... 56- 66б...бб72 = 44... 488... 89. 47.18. Данное неравенство вытекает из цепочки: sin l<sin-^=^<-Z-< log3 л/7 (последнее неравенство следует из того, что 37<С74). 47.19. Можно, например, каждую задачу оценивать по б-балльной системе (число баллов меняется от 0 до 5). Считая, что количество решаемых участником олимпиады задач равно б, поставим в соответствие ему такое 8-значное число, две первые цифры которого выражают сумму набранных учеником баллов, а каждая из остальных шести цифр равна числу баллов за соот- соответствующую задачу. 47.20. Уравнение не имеет решений в целых числах. Ука- Указание. Перепишем его так: 19 (х3—100) = 84 A -f- у2); правая часть кратна 7, поэтому х3 — 2 кратно 7. Но кубы чисел при деле- делении на 7 не дают в остатке 2. 47.21. Обозначим через d наибольший общий делитель всех данных чисел (т. е. d — наибольшее натуральное число, на которое делятся одновременно все числа пи «2, «з, -¦¦) и через ds — наибольший общий делитель чисел п\, п2, .... ns. Посколь- Поскольку di^d2^d3^ ..., то dk = dk+]= ... =d для некоторого &>2. Возьмем тогда числа п\, .... nk и рассмотрим все суммы, которые можно уплатить этими монетами. Эти суммы, расположенные в порядке возрастания, с какого-то места образуют арифмети- арифметическую прогрессию с разностью dk = d\ эту же арифметическую прогрессию образуют и суммы, полученные из первоначального бесконечного набора монет {«,}, l^i<Coo. Поэтому, добавив к монетам niy ..., tik необходимое количество монет tik+j, ..., nN, не попадающих в указанную арифметическую прогрессию, получаем искомый конечный набор монет ti\, n2 h,v, которыми можно уплатить все возможные суммы, образованные из исходного набора монет nit п2, п3, ... . 47.22. Пусть А и В — две вершины треугольников, лежащие внутри квадрата. Очевидно, к каждой из них прилегает не менее 5 треугольников. Итак, мы уже насчитали 10 треугольников, из которых только 2, быть может, сосчитаны дважды (прилегающие к стороне АВ, если такая сторона есть). Следовательно, число треугольников не меньше 8. Остается разобрать случай, когда внутри квадрата лежит не более одной вершины, а осталь- остальные лежат на сторонах квадрата или на сторонах других треуголь- треугольников. Этот случай разберите сами. 47.23. Пусть ABC — треугольное сечение куба с ребром 2 (площадь грани куба равна 4), касающееся вписанной в куб 278
сферы в точке Н. Обозначим через С", В', А' точки касания сферы с теми граня- гранями куба, на которых лежат отрезки АВ, АС и ВС соответственно (см. рис. 97, а). Легко видеть, что аАС'В = ААНВ, АВА'С=АВНС, АСВ'А-= АСНА. Сле- Следовательно, площадь треугольника ABC равна сумме площадей треугольников АС'В, ВА'С, СВ'А. Выразим площадь каждого из трех последних треугольников через величины х, у, z — расстояния меж- между каждой из точек А, В, С а серединой того ребра куба, на котором она лежит (расстояния берутся со знаком; см. рис. 97,6). Эти три площади равны —A—ху), —A—yz), —-A—zx), и нетрудно дока- доказать, что если |х|<1, \у\<1, |z|<l, то а) АН< 48.01. Одно из чисел равно 1, а дру- другое произвольно. 48.02. Роено одним способом. 48.03. Можно. Указание. См. ре- решение задачи 23.27. 48.04. Может. Для этого заяц должен придерживаться такой стратегии. Сначала он выбирает произвольную вершину А ква- квадрата и бежит к ней по диагонали с мак- максимальной скоростью до тех пор, пока не окажется от А на расстоянии, меньшем — (л/2—1,4) (например, на расстоянии 0,005; сторону квадрата полагаем рав- Рис. 98. ной 1). Затем он, не меняя скорости, сво- сворачивает на 90° и движется перпендику- перпендикулярно диагонали к той стороне квадрата, на которой находится только один волк (если в рассмат- рассматриваемый момент в А находится волк, то заяц сворачивает на 90° в произвольную сторону; рис. 98). Нетрудно видеть, что в момент, когда заяц пересечет сторону квадрата, ни один волк не сможет оказаться в той же точке этой стороны. 3 а- м е ч а н и е. Если скорость волка в -\J2 раз больше скорости зайца, то волки уже ловят зайца: они в каждый момент ока- оказываются в концах «креста» с центром «заяц», отрезки которого параллельны диагоналям квадрата. 279
f б) Рис. 99. 48.05. Указание. Сначала за 5 взвешиваний следует получить в одной фляге 17 л молока, а две другие' сделать пустыми. Затем за 2 взвешивания наполнить 17-ю литрами оставшиеся пустые фляги, получить сливанием 34 л (без взвешивания), восьмым взвешиванием еще 34 л и, наконец, слить вместе 34 л, 34 л и 17 л. Замечание. Аналогичным способом можно отлить BЛ' +1)B + 1)... B"*+1) л молока за # = (п, + 1)-}-. .. + (nfc+l) взвешиваний. У нас 85 = B2+1) B4+1) и N=8. 48.06. у=х или y=z. 48.07. Чисел ak, не меньших 993, в точности 993. Поэтому хотя бы одно из них имеет номер, не меньший 993, и для него 2 48.08. Два узла (рис. 99, а). Действительно, уже впи- вписанный в квадрат круг накрывает не менее двух узлов. Для доказательства этого возьмем клетку, в которой находится центр круга, разобьем ее диагоналями на 4 треугольника и возьмем тот из них, в котором лежит центр. Тогда круг целиком накры- накрывает выбранный треугольник (рис. 99,6), что и требовалось. 48.09. Пусть в череде богатырей не существует богатыря наименьшего роста. Это означает, что для каждого богатыря найдется богатырь меньшего роста и тогда искомая цепочка легко строится последовательным выбором все меньших и меньших по росту богатырей. Если же имеется богатырь А] наименьшего роста, то отбрасываем его и среди оставшихся выбираем бога- богатыря А2 с наименьшим ростом (если же такого А? нет, то для оставшихся проходит описанное выше рассуждение и утвержде- утверждение задачи доказано). Далее, отбрасывая А\ и А2, из остальных 280
богатырей выбираем Аз с наименьшим ростом (если такого Аз нет, то все доказано), потом Ац, Аъ и т. д. В результате получаем цепочку стоящих по росту богатырей А\, А2, Аз, ... . 48.10. Пусть Z.v4^60° — наибольший в треугольнике ABC угол, AD > 1 — биссектриса угла А. Проведем через точку D все- всевозможные прямые, пересекающие стороны угла А, и выберем треугольник ABC минимальной площади. Этот треугольник равнобедренный (сравните площади заштрихованных на рисунке 100 треугольников BMD и CND), и его площадь больше — л/3 48.11. Заменами х = х2, у=у2, z = z2 и возведением обеих частей уравнения в квадрат приходим к задаче 48.06. 48.12. Могло случиться. Расположим сначала 50 аэродромов в вершинах правильного 50-угольника, а остальные аэродромы — в его центре. Тогда все самолеты из центра прилетят в вершины, а из вершин полетят в диаметрально противоположные вершины. Таким образом, все самолеты соберутся во всех 50 вершинах 50-угольника. Учитывая, что попарные расстояния между аэро- аэродромами должны быть различными, немного пошевелим описан- описанную конструкцию, добившись выполнения и этого условия. 48.13. Девять узлов. Указание. Если квадрат 2X2 со- содержит два узла на расстоянии 2^2, то он, очевидно, накрывает все 9 узлов. Остается случай, когда квадрат содержит «домик»: 6 узлов по 3 в два ряда и один узел посредине в третьем ряду (рис. 99, в). Докажите, что проекция «домика» на одно из двух произвольно выбранных перпендикулярных направлений со- содержит точки, удаленные друг от друга не менее чем на 2. (Мож- (Можно также доказывать, что равнобедренный треугольник с основа- основанием 2 и боковой стороной -\/5 единственным способом помещается в квадрат 2X2.) 48.14. Доказательство проведем от противного. Выб- Выбрав любых двух человек А и В, среди оставшихся десятерых выберем таких людей С\, Сг, . .., Ck, каждый из которых знает ровно одного в этой паре. В силу предположения имеем ft ^6, так как при ft ^5 в рассмотренной группе 10 — ft^5 че- человек удовлетворяют условию задачи. Подсчитаем число N троек {А, В, С,} двумя способами. Все- Всего имеется —•12-11=66 пар {А, В), и каждой паре отвечает не менее 6 человек С,-, поэтому Л/^6-66. С другой стороны, С, можно фиксировать и искать для него такие пары {А, В}, в кото- которых он знает ровно одного чело- человека. Если у С, есть п знакомых, то число искомых пар {А, В} равно Рис. 100. 281
л A1—п)^30. Выбрать же С* можно 12 способами, откуда N ^E -6) -12. Итак, 6 -66^ N ^360 — противоречие, доказы- доказывающее утверждение. 48.15. Доказательство по индукции. Для л = 3 утверждение верно; пусть оно верно для n = k: 2к = 7х2-\-у2, х и у нечетны. Рассмотрим две пары чисел: I——(лг—у); —— Gд:-f-f/)| и !-^-(jt+//); -~-Gje—у)\ . Для каждой пары усемеренный квадрат первого числа плюс квадрат второго дает 2*+|. Остается заме- заметить, что в каждой паре стоят числа одной четности, а в разных — разной четности, поэтому числа одной из пар нечетны. 48.16. {О; 99; 49 +Щ . 48.18. Максимальную «сложность», равную 101, имеет число — +-п—(—9~) • Указание. Если ап=^- имеет максималь- максимальную «сложность» среди чисел со знаменателем 2", то среди чисел со знаменателем 2n+I наибольшую «сложность» имеет число ап + 1=тррт, где m2=2n+l — mx. 48.19. По условию любые два множества пересекаются по одному элементу. Докажем, что все множества пересекают- пересекаются по одному элементу. Предположим противное. Возьмем мно- множество № 1. В нем найдется элемент А, который принадлежит по крайней мере еще 45-ти множествам — № 2, 3, 4 46, так как в противном случае общее число множеств не превосходило бы 44-45+1 = 1981, что не так. По нашему предположению, имеется множество, не содержащее элемента А. Оно пересекается по одному элементу с множествами № 1, 2, ..., 46 и поэтому содержит 46, а не 45 элементов. Противоречие. Ответ: 44-1985+1. 48.20. Проведем через три пары скрещивающихся ребер тетраэдра Т три пары параллельных плоскостей, получим парал- параллелепипед П, диагоналями граней которого служат ребра Т. Объем П втрое больше объема Т, а расстояния между парал- параллельными гранями П равны h\, h2, Лз. Докажите сами, что объем П не меньше Л1Л2Л3. (См. 3.03).
ПРИЛОЖЕНИЕ 1 XLIX олимпиада A986 год) VII класс 1. На листе прозрачной бумаги нарисован четырехугольник. Объяснить, как сложить этот лист (возможно, в несколько раз), чтобы определить, является ли исходный четырехугольник ром- ромбом. 2. Доказать, что ни для каких чисел х, у, z не могут одно- одновременно выполняться три неравенства |x|<|0-z|, |y|<|z-x|, \z\<\x — y\ 3. Три гнома живут в разных домах на плоскости и ходят со скоростями 1, 2 и 3 км/ч соответственно. Какое место для ежедневных встреч нужно им выбрать, чтобы сумма промежут- промежутков времени, необходимых на путь от своего дома до этого ме- места (по прямой), была наименьшей? 4. Произведение некоторых 1986 натуральных чисел имеет ровно 1985 различных простых делителей. Доказать, что либо одно из этих чисел, либо произведение нескольких из них явля- является квадратом натурального числа. 5. В кодовом замке имеются ровно три кнопки с номерами 1, 2., 3. Про код, открывающий замок, известно, что он трех- трехзначен. Какое наименьшее число раз следует нажать на кнопки замка, чтобы он наверняка открылся? (Замок открывается, как только подряд и в правильном порядке нажаты все три цифры его кода.) VIII класс 6. См. задачу 1 с заменой ромба квадратом. 7. Найти все натуральные числа, непредставимые в виде разности квадратов каких-либо натуральных чисел. 8. Доказать, что если а{ = 1, ап =^f1 + -— при п = 2, 3, ..., Ю, то О<а,о-У2<1(Г370. 9. Квадратное поле разбито на 100 одинаковых квадратных участков, девять из которых поросли бурьяном. Известно, что бурьян за год распространяется на те и только те участки, у каждого из которых не менее двух соседних участков уже пора- поражены бурьяном (участки называются соседними, если они имеют общую сторону). Доказать, что полностью все поле бурьяном никогда не зарастет. 283
10. Доказать, что система неравенств {\х\ >\y — z + t\, \y\>\x — z + t\, \z\>\x-y + t\, \t\>\x-i не имеет решении. IX класс 11. На листе бумаги отмечены точки А, В, С, D. Распознаю- Распознающее устройство может абсолютно точно выполнять два типа операций: а) измерять в сантиметрах расстояние между двумя заданными точками; б) сравнивать два заданных числа. Какое наименьшее число операций нужно выполнить этому устройству, чтобы можно было наверняка определить, является ли четырех- четырехугольник A BCD прямоугольником? 12. Из точки М по плоскости с постоянной скоростью пол- ползет муравей. Его путь представляет собой спираль, которая на- наматывается на точку О и гомотетична некоторой своей части относительно этой точки. Сможет ли муравей пройти весь свой путь за конечное время? 13. Решить систему неравенств \x\<\y-z+t\, \y\<\x—z + t\, \z\<\x—y+t\, \t\<\x — (Ср. с задачей 10.) 14. Произведение некоторых 48 натуральных чисел имеет ров- ровно 10 различных простых делителей. Доказать, что из этих чисел можно выбрать такие четыре, произведение которых является квадратом натурального числа. (Ср. с задачей 4.) 15. На координатной плоскости нарисованы круги радиу- са — с центрами в каждой точке, у которой обе координаты — целые числа. Доказать, что любая окружность радиуса 100 пере- пересечет хотя бы один из нарисованных кругов. X класс 16. См. задачу II с заменой прямоугольника ABCD квад- квадратом. 17. Биссектриса угла А треугольника ABC продолжена до пересечения в точке D с описанной вокруг него окружностью. Доказать, что AD>-^-(AB + AC). 18. Решить уравнение Xх = 4(jc>0). 284
19. Доказать, что ни для каких векторов а, Ь, с не могут одновременно выполняться три неравенства (Ср. с задачей 2.) 20. Найти минимум по всем аир максимума функции у (х)=\ cos х -\- a cos 2x -\- р cos Зл: |. РЕШЕНИЯ, УКАЗАНИЯ, ОТВЕТЫ 49.01. Достаточно два раза перегнуть четырехугольник по его диагоналям. Если оба раза произошло совпадение наклады- накладываемых треугольников, то четырехугольник — ромб. 49.02. Предположим, что указанные неравенства имеют ме- место. Возведем почленно в квадрат каждое неравенство, пере- перенесем влево все правые части и разложим на множители полу- полученные разности квадратов. Получим: ) z-x)<0, -y)<0. Перемножив почленно все эти неравенства, получаем проти- противоречие: f 49.03. Этим местом встречи является дом первого гнома (который ходит со скоростью 1 км/ч). Для доказательства этого обозначим искомое место встречи гномов буквой А, а дома зану- занумеруем цифрами 1, 2, 3 в соответствии с величинами скоростей гномов. Расстояния между домом 1 и домами 2 и 3 обозначим через а и Ь, а расстояния от точки А до домов 1, 2, 3 — через х, у и z соответственно. Тогда откуда причем равенство достигается при х = 0, т. е. когда точка А совпадает с домом 1. 49.04. Рассмотрим наборы исходных чисел, взятых во всех возможных количествах: по одному, по два, по три и т. д. Таких наборов, как нетрудно подсчитать, 21986— 1. Числа в каж- каждом таком наборе перемножим и полученное произведение пред- представим в виде произведения наибольшего полного квадрата и не- нескольких простых сомножителей (например, Л' = 2|6-3 -5|3-179 представляется в виде B8-37-56-174J-3-5-17, а М = 216-1310 = = B8- 135J). Сопоставим каждому набору исходных чисел тот на- набор простых чисел, который получается после выделения наиболь- 285
шего точного квадрата из их произведения (в рассмотренном примере числу N сопоставляется набор 3, 5, 17, а числу М — пус- пустой набор). Нетрудно подсчитать, что число различных наборов из 1985 простых делителей (по одному, по два, по три и т. д., включая и пустой набор) равно 219 , что меньше количества 2i986—[ — наборов из исходных чисел. Поэтому каким-то двум наборам А и В из исходных чисел отвечает один и тот же набор pi, ..., pk простых делителей, т. е. Л = a2-/?i-/?2- ... •/?*, B = ?r-pi- ... -ph. Следовательно, произведение А-В есть точный квадрат. С другой стороны, А • В равно произведению чисел в наборе А и чисел в наборе В. Выбросив из наборов А и В их общую часть (произведение выбрасываемых чисел есть точный квадрат), получим, что произведение остальных чисел является точным квадратом, что и требовалось доказать. 49.05. Всего существует 33 = 27 различных трехзначных чи- чисел, в записи которых участвуют цифры 1, 2, 3. Кроме первых двух цифр в последовательности нажатий кнопок, каждая из остальных цифр служит последней цифрой какого-то трехзначно- трехзначного числа. Следовательно, в искомой последовательности должно быть не менее 27 + 2 = 29 цифр. Двадцати девяти цифр уже доста- достаточно для открытия замка; например, годится последователь- последовательность 11123222133313121223113233211. 49.06. Если ABCD — ромб (см. решение задачи 49.1), то достаточно перегнуть бумагу так, чтобы вершина С совпала с вершиной В. Если при этом вершина D совпадет с А, то ABCD — квадрат. 49.07. Решим уравнение х2— у2 = п, где п — заданное нату- натуральное число, х и у — неизвестные натуральные числа. Посколь- Поскольку х2 — у2 = {х — У)(х-\-у), а числа х — у и х-\-у имеют одинако- одинаковую четность, то п либо делится на 4, либо имеет вид 4ft ±1. Верно и обратное: все числа п вида 4ft и 4ft ± 1 представимы в виде п = х2 — у2 (кроме чисел 1 и 4). Действительно, если (х — у) (x-\-y) = 4k, то достаточно взять x = k-\-l, y = k—\ (ис- (исключение составляет /г = 4, так как при этом у = 0 — не нату- натуральное число); если же (х — у) (х+у) = 4/г± 1 =2/+ 1, то доста- достаточно взять х = 1-\-1, у = / (исключение составляет п=1, так как тогда у = 0). Следовательно, в требуемом виде непредставимы только числа п=\, п = 4 и все числа вида и = 4ft+ 2. 49.08. Обозначим К=———: ¦ Докажем, что +у2 Действительно, _а„ + ,-У2 Отсюда имеем: 286
т. е. а,о — V2 V2+I Поскольку из неравенства, связывающего среднее арифметиче- арифметическое и среднее геометрическое, следует, что ( неравенство строгое, так как ад^=—) , а.о-л/2 > о.о-л/2 то а,0+л/2 2/2 Следовательно, Заметим, что (д/2+ 1)8 = C + 2 д/2L=A7+ 12 V2J>B4 л/2J = = 1152>103, поэтому (V2+lI024>A03I28=10384>2V2.10383 и Таким образом, нами доказано даже более сильное, чем требу- требуется в условии задачи, неравенство. Замечания. 1) Основная идея доказательства в этой за- задаче та же, что и в задаче 16.37: после каждой итерации количе- количество знаков после запятой примерно удваивается, так что точ- точность вычислений экспоненциальная. 2) Однако из неравенства аю — -\/2<С10~370 нельзя формаль- формально заключить, что первые 370 знаков у чисел а10 и -\/2 совпадают: для этого достаточно было бы удостовериться (лучше всего на ЭВМ), что в десятичной записи -^2 цифра, стоящая после запя- запятой на 371-м месте, отлична от 9 (если она равна 9, то нельзя утверждать даже, что совпадает хотя бы одна цифра; вот контр- контрпример: 1—0,99999999999 =10~10). Как же в этом удостоверить- удостовериться? Ведь, вычисляя по приведенному алгоритму, мы добиваемся только хорошей точности, но не точных цифр числа У§. Выход на самом деле очень прост: достаточно сделать еще одну, 11-ю, итерацию и убедиться, что 371-я цифра числа аи не равна 9. 49.09. Рассмотрим длину всей границы пораженной бурьяном области. Нетрудно проверить, что после возникновения новых «бурьянных» участков общая длина пораженной бурьяном обла- области не увеличится. Поэтому если бы все поле когда-либо поросло бурьяном, то общая длина границы «бурьянной» области стала бы равной 40, хотя в начальный момент (а значит, и во все 287
последующие тоже) она не могла превосходить 4-9 — 36. Полу- Получили противоречие. 49.10 и 13. Решение этих задач аналогично решению зада- задачи 49.2. Проделаем следующие действия: 1) возведем все не- неравенства в квадрат (знаки неравенства при этом не изменят- изменятся); 2) перенесем все правые части влево; 3) представим каж- каждую получившуюся разность квадратов в виде произведения разности на сумму; 4) перемножим все полученные скобки в ле- левых частях неравенств. Все эти левые части отличны от нуля и имеют одинаковые знаки; значит, их произведение положитель- положительно. С другой стороны, оно равно — ((x — y-\-z — t)(x-\-y — z-f-/)X X( — x + y + z — t)(x—y + z + t)J, т. е. отрицательно. Противо- Противоречие. 49.11 и 16. Чтобы определить, является ли ABCD прямо- прямоугольником, достаточно проверить равенства AB = CD, BC = AD и AC=BD — итого 9 операций (по 3 операции на каждое ра- равенство: два измерения и одно сравнение). Прямоугольник ABCD будет квадратом, если АВ = ВС — для этого нужна еще одна, 10-я, операция сравнения длин отрезков АВ и ВС. Докажем, что меньшим числом операций в обоих случаях не обойтись, т. е. что все указанные операции обязательно нужно выполнить. Действительно, если мы не знаем, что какие-то две противо-- положные стороны четырехугольника равны, то нельзя даже утверждать, что ABCD — параллелограмм, ведь годится и равно- равнобедренная трапеция (ее диагонали равны). Поэтому проверка равенств AB = CD и BC = AD необходима. Если же мы не знаем, что AC = BD, то ABCD может быть произвольным параллело- параллелограммом. Значит, необходима и проверка равенства диагоналей. Наконец, в случае квадрата нужна также проверка равенства двух соседних сторон. 49.12. Муравей проходит весь путь по спирали от М до О за конечное время. Действительно, если коэффициент гомотетии /г<1, а первый виток муравей проходит за время t, то на остальные витки он потратит время kt, k2t, k t, ...; поэтому время на весь путь равно t (I -{-k-\-k2 -\-...) = -—-. 1 — К 49.14. Решение аналогично решению задачи 49.04. Предста- Представим произведение произвольной пары чисел (а, Ь) из данного набора в виде произведения квадрата натурального числа на произведение простых делителей в первых степенях (например, если а = 213.34- 193, Ь = 56-77-19, то ab = K2-2-7, где /(=26-32Х Х53-73-192). Сопоставим паре (а, Ь) получившийся набор про- простых делителей. Всевозможных различных пар (а, Ь) в наборе из 48 чисел С48 = 48-—-, а количество наборов из 10 простых делителей (по одному, по два, по три и т. д., включая пустой на- набор) 210. Так как С48>210, то найдутся две различные пары (а, Ь) и (с, d) из набора, которым отвечает один и тот же набор 288
(P\, Pa, -.., Pk) простых делителей (O^fe^lO). Следовательно, abed — точный квадрат. Если при этом пары (а, Ь) и (с, d) не имеют общего элемента, то числа а, Ь, с, d искомые. Если же общий элемент есть, на- например b = d, то тогда ас— точный квадрат. Выкинем на время пару (а, с) из рассмотрения. Тогда мы приходим к набору из 46 чи- чисел, произведение которых имеет не более 10 различных простых делителей. Проведя те же рассуждения, что и выше, и учитывая, что верно также и неравенство С46>210, приходим к выводу о сущест- существовании двух различных пар чисел (х, у) и (z, t) из набора, для которых xyzt — точный квадрат. Если общего элемента у этих пар нет, то х, у, z, t — искомые 4 числа; если же общий элемент есть, например x = t, то yz — точный квадрат. В этом случае искомой четверкой чисел является (а, с, у, г). 49.15. На клетчатой плоскости со стороной клетки 1 выберем самую северную горизонтальную прямую y = k, пересекающую фиксированную окружность радиуса 100 с центром О (прямая у = k + 1 окружности не пересекает). Если все узлы этой прямой лежат вне окружности, то легко подсчитать, что ближайший к окружности узел находится от нее на расстоянии, меньшем 1 „ 1 —, поэтому она пересекает нарисованный круг радиуса — с центром в этом узле. Считаем поэтому в дальнейшем, что на прямой y = k некоторые узлы лежат внутри окружности. Выберем из них узел В, лежащий ближе всего к окружности. Через А обозначим ближайший к нему внешний узел на прямой y = k, т. е. АВ = 1. Предположим, что окружность не пересекает нарисованных кругов радиуса -— с центрами А и В. Имеем тогда: ОЛ> 100+^, 99< ОВ< 100 —-^ , откуда ОА — ОВ>^-, ОА2-ОВ2 = (ОА-ОВ)(ОА + ОВ)> 199~. Если О' — проекция центра О на прямую y = k и О'В = х, то О'А = х+\, и {х+\J-х2 = ОА2-ОВ2->^~, откуда О'В = — х>^. А тогда 2 2 ОО'2 = ОВ2-О'В2<( 100—jL) —(т) <99"' откуда ОО'< 99. Отсюда вытекает, что расстояние от центра О до прямой у = ?+1, равное ОО' +1, меньше 99+1 = 100, т. е. наша окруж- окружность радиуса 100 пересекает также и прямую у = /г+1. Это противоречит сделанному вначале предположению и доказывает утверждение задачи. 289
49.17. Введем обозначения: АВ = а, AC = b, AD = l, Z.C = a, = p, d— диаметр описанной вокруг треугольника ABC окружности. По теореме синусов длина хорды равна произведе- произведению диаметра окружности на синус половины дуги, на кото- которую эта хорда опирается. Поскольку <;АВ = 2а, ^АС = 2$, то a = dsina, fc = dsinp, l = d sin л~° + ^ = ^ Из неравенства d cos ^~a > — (d sin a-\-d sin p) и следует требуемое утверждение. 49-418. Очевидно, что х=^/2— решение уравнения. Функция у=хх монотонно возрастает на [1; +оо), поэтому на этом про- промежутке других решений нет. На интервале @; 1) решений тоже нет, так как на нем значения функции меньше 1. Итак, х = -^2— единственное решение. 49.19. Возведем данные неравенства в квадрат и почленно сложим их все (учитывая, что для произвольного вектора х, \х\2=х-х = х2): откуда (a + b + сJ <0 — противоречие. 49.20. При всех аир справедливо неравенство ^1Г\ l2^2l +t) -(~^+"f")l =^' ГЮЭТОМУ minmaxy(x)>f. Поло- Положим теперь f{x)=cosx—г-соэЗл: и найдем точки экстремума этой функции из уравнения f (x) = 0: f (х)= —sin jc + -n-sin Злг = 0, — 2 sin x-\-C sin x — 4sin3x)=0, откуда либо sinx = 0, x = kn, либо sin2;c=—-, х= ±-jr-+kn. Для функции у (x)=\f (х)\ имеем: у (kn)=-^-<Z~- , y( ±-^-+knj =y-. Отсюда вытекает, /о /п что max у (x)^Z.~- и поэтому min max у (x)^L^~ . х ^ а. р х Л Окончательный ответ: min max y(x) = ~ и достигается при а. р х ^ а = 0, р=—1Г.
ПРИЛОЖЕНИЕ 2 Избранные задачи математических турниров В этом приложении приведены наиболее красивые задачи, предлагавшиеся в разные годы участникам математических кружков, победителям Московских олимпиад при отборе на Все- Всесоюзную олимпиаду, а также некоторые задачи из архива жюри Московских олимпиад, не попавшие ни на один из турниров и поэтому малоизвестные. После условия задачи стоит номер класса, школьникам которого она предназначалась. 1G—9). Найти сумму цифр числа 123456789101112... ... 999998999999. Сколько в этом числе семерок? 2(8—10). Найти наименьшее натуральное число ab ... с, не содержащее в своей десятичной записи нулей, которое в сумме со своим обращенным (т. е. с числом с ... Ьа) дает число, цифры в котором получены перестановкой цифр исходного числа. 3(9—10). Существуют ли такие иррациональные числа а и f5, что ар — число рациональное? 4(8—10). В вершинах куба поставлены числа +1 и —1. В центре каждой грани поставлено число, равное произведению чисел в вершинах этой грани. Может ли сумма полученных 14 чи- чисел оказаться равной нулю? а семерке? 5(8—10). В пространстве расположено конечное (но неизве- неизвестное) количество звезд. Находясь в произвольной точке про- пространства (пункте наблюдения), можно определить количество видимых из нее звезд и направления на них. Однако это наблю- наблюдение может не дать достоверного числа всех звезд, так как звезды, вообще говоря, будут загораживать друг друга. Какое наименьшее число пунктов наблюдения следует выбрать, чтобы определить точное число звезд на небе? Решите ту же задачу и на плоскости. 6G—9). На кольцевой автомобильной дороге стоят п одина- одинаковых автомашин. Общее количество бензина у всех этих автома- автомашин достаточно для того, чтобы одна из них смогла проехать по всей кольцевой дороге. При любом ли расположении автома- автомашин на дороге и распределении горючего по ним найдется такая автомашина, которая может, забирая по пути бензин у осталь- остальных, проехать по всей кольцевой дороге? 7(8—10). Решить в целых неотрицательных числах уравнение 8(8—10). Отметим на числовой прямой красным цветом все точки, отвечающие числам вида 81л:+100у, где х и у — 291
целые числа, и синим карандашом остальные целые точки. Найти на прямой такую точку, что любые симметричные относи- относительно нее целые точки закрашены в разные цвета. 9(9—10). Доказать, что если целые числа х\, хч, ..., хп, уи ..._ ут удовлетворяют неравенствам 1 <.х\ <Сх2<С-.'<хп<су\ <;...< <СУт И Х,+... + Хп>У1+...+Ут, ТО Х1Х2...Хп>У\У2 -¦• Ут- 10A0). Доказать, что любые два равновеликих многогран-1 ника можно разрезать на попарно равновеликие тетраэдры. 11(8—9). Даны квадрат ABCD и точка О внутри его. Дока- Доказать, что 135°< /L ОАВ+А ОВС+ A OCD + A ODA < 225°. 12A0). В пространстве через точку О проведены п прямых так, что для любых двух из них найдется третья из того же на- набора, перпендикулярная первым двум прямым. Доказать, что: а) все прямые, кроме одной, лежат в одной плоскости, а эта последняя прямая перпендикулярна ей и б) п нечетно. 13G—10). Квадратный пирог разделен двумя взаимно пер- перпендикулярными прямыми на четыре части. Три из этих частей весят по 200 г. Сколько весит пирог? 14* A0). В п точках плоскости стоят прожекторы, освеща- освещающие на плоскости углы ос,, ссг, .-., а„, причем а\ -\-а2 + ¦•- + «« = = 360°. Доказать, что прожекторы можно так поменять местами, не вращая, что они осветят всю плоскость. 15(8—9). Про выпуклый четырехугольник ABCD известно, что AC = BD, AABD = 2 AACD, AACD+AADB = 90°. Найти углы В и D этого четырехугольника. 16(9—10). Восемь точек лежат на поверхности куба с реб- ребром 1. Расстояние между любыми двумя из них не меньше 1. Доказать, что эти точки — вершины куба. 17(9—10). а) Проекции тела на две плоскости в простран- пространстве — круги. Доказать, что эти круги одинаковы. б) Проекции выпуклого м-угольника на две непараллельные плоскости в пространстве — правильные и-угольники. Доказать, что эти проекции — равные п-угольники. 18(8—9). Конечно или бесконечно множество натуральных чисел п, таких, что сумма цифр числа 5" меньше 10'00? 19G—9). Доказать, что среди первых 10 миллионов цифр разложения -^2 ни одна цифра не встретится 5 000 001 раз подряд. 20G—9). В Швамбрании 1000 аэродромов, все попарные рас- расстояния между которыми различны. С каждого аэродрома под- поднимаются по самолету и каждый самолет летит на ближайший аэродром. Какое наибольшее число самолетов может оказаться на произвольном аэродроме, если Швамбрания — а) плоскость? б) сфера? 292
21(9—10). В кубическом сыре проделано несколько сфериче- сферических дыр. Доказать, что можно разрезать сыр на такие выпук- выпуклые многогранники, чтобы внутри каждого из них находилась ровно одна дыра. 22A0). Пусть sina=—. Доказать, что sin 25a =^5, где п — целое число, не кратное 5. 23G—10). Три разноцветные лампочки — синяя, зеленая и красная — подсоединены как-то проводами к п переключателям. Каждый переключатель может находиться в одном из трех по- положений. При любом положении переключателей горит ровно одна лампочка, но если переключить сразу все переключатели, загорится другая лампочка. Доказать, что цвет горящей лампоч- лампочки определяется одним фиксированным переключателем и не за- зависит от состояния остальных. 24(8—10). По бесконечной шахматной доске с полями в ви- виде квадратов со стороной 1 прыгает кузнечик, перемещаясь за каждый прыжок на а вправо и на р вверх. Доказать, что если числа a, P и у иррациональны, то кузнечик обязательно по- попадет на черное поле. 25(9—10). Доказать, что 26* (8—10). Найти все решения в натуральных числах урав- уравнения 520 {xyzt + xy + xz + zt+l) = Ъ77 {yzt + y + z). 27G—8). Доказать, что если в каждый момент времени хотя бы один из десяти идущих равномерно будильников показывает правильное время, то среди них хотя бы один будильник идет правильно всегда. 28(9—10). Пространство разбито на одинаковые и одинако- одинаково расположенные параллелепипеды. Доказать, что с каждым параллелепипедом имеют общую точку не менее 14 параллеле- параллелепипедов. 29(8—10). Треугольная пластинка площади 1 двумя прямо- прямолинейными разрезами, идущими от двух вершин к противопо- противоположным сторонам, поделена на четыре части (три треугольника и четырехугольник), три из которых имеют равную площадь. Найти площадь каждой части. 30(8—9). Доказать, что если среднее арифметическое пер- первых 1010 цифр числа 2 — д/2 заключено между 4— и 4—, то О О это же верно и для числа -^2—1. 31(8—10). Доказать, что в любой момент на поверхности Солнца (которая считается сферой) найдется точка, из которой видно не более трех планет (из девяти известных). 293
32G—9). Имеется два сосуда. В первом находится I л воды а второй пустой. Из первого переливают половину имеющейся в нем воды во второй, затем из второго переливают треть имею- имеющейся в нем воды в первый, затем из первого переливают чет- четверть имеющейся в нем воды во второй и т. д. Найти количество воды в первом сосуде после 12 345 переливаний. 33* (9—10). Доказать, что квадраты со сторонами ~t JLf 111 - —, —, ..., —, ... можно без перекрытии разместить в квадрате со „ 5 стороной —, но нельзя разместить ни в каком меньшем квадрате. 34(8—10). Даны деревянный шар, циркуль и лист бумаги. Начертить на листе бумаги циркулем окружность, радиус ко- которой равен радиусу шара. (На шаре циркулем можно чер- чертить произвольные окружности.) 35* (9—10). Квадрат двумя способами разбит на 100 равно- равновеликих частей. Доказать, что можно выбрать 100 точек так, чтобы в каждой части каждого разбиения оказалось ровно по одной точке. 36(8). Доказать, что число КЖЖЖЖЖ) целое, и найти его. 37(8—10). Стол полностью покрыт 100 квадратными скатер- скатертями, в каждой из которых прожжена круглая дырка. Доказать, что уже какие-то три из этих скатертей полностью покрывают стол. 38(8—10). Дано 40 правильных игральных коетей (сумма чисел на противоположных гранях кости равна 7). Они постав- поставлены друг на друга так, что получается параллелепипед. Верно ли, что их можно повернуть так вокруг вертикальной оси, что суммы чисел, стоящих на четырех боковых гранях параллеле- параллелепипеда, будут равны между собой? 39(8—10). Даны две концентрические окружности и две па- параллельные хорды / и т, касающиеся внутренней окружности. На внешней окружности между / и т взята точка А и из нее про- проведены касательные к внутренней окружности. Точки пересече- пересечения касательных с хордами С и D. Доказать, что произведение АС-AD не зависит от положения точки А. 40(8—10). На бесконечной клетчатой бумаге со стороной клетки, равной 1, нарисована система параллелограммов с вер- вершинами в узлах бумаги такая, что начало координат не явля- является вершиной параллелограмма. Доказать, что существует такое целое k, что если подвергнуть клетчатую бумагу растяжению 294
в k раз от начала координат, оставив параллелограммы на месте, то ни одна вершина параллелограмма не совпадет с узлами растянутой клетчатой сети. 41(8—9). Точки А, В к С двигаются равномерно по трем окружностям в одну и ту же сторону с одинаковой угловой ско- скоростью. Как двигается центр тяжести треугольника ABC? - 42(8—9). На доске выписаны числа 1, 2, 3, .... 1974. За один ход разрешается произвольные два числа из набора стереть, а вместо них написать в набор модуль их разности. После 1973 хо- ходов останется одно число. Чему может быть равно это число? 43(8—9). Прямые а и b касаются окружности в точках А и В. На прямой а выбирается точка С, а на прямой Ь — точка D так, что отрезок АВ пересекается с отрезком CD в точке М. г, СА DB Доказать, что— =—. 44(8—10). Число р простое. Дано р+1 различных натураль- натуральных чисел. Доказать, что среди них найдется пара таких чисел х и у, что большее из этих чисел, деленное на их наибольший общий делитель, не меньше р+1- 45(9—10). Числа щ, аг, ¦--, Gfe таковы, что при всех х вы- выполняется неравенство k Zj о.п cos пх ^ — 1. Доказать, что al-\-. 46(9—10). Игра «Задумка» состоит в том, что нужно угадать за три вопроса задуманное пятизначное число, в записи которого имеются только нули и единицы. Угадывающий называет пяти- пятизначное число, составленное из нулей и единиц, а загадавший говорит, в каком количестве разрядов названное число совпа- совпадает с задуманным. Сможет ли угадывающий определить заду- задуманное число? 47(9—10). На поверхности куба с ребром 1 расположена замкнутая ломаная линия. На каждой грани куба находится по крайней мере одно звено ломаной. Доказать, что длина ломаной не меньше чем 3 -^2. 48(9—10). Доказать, что для любого натурального числа выполняются неравенства 49(8—10). Какое наименьшее число полей можно вырезать из шахматной доски размером 9X9 так, чтобы по оставшимся полям шахматная ладья не смогла описать замкнутого пути, не проходя дважды по одному отрезку? 50(8—10). В круге радиуса 16 расположено 650 точек. Дока- Доказать, что найдется кольцо с внутренним радиусом 2 и внешним 295
радиусом 3, в котором лежит не менее 10 из данных точек 51(8—10). Существует ли такое натуральное число п, что любое рациональное число между 0 и 1 представляется в виде суммы п чисел, обратных к натуральным? 52(8—10). Правильный 2п-угольник вписан в правильный 2?-угольник, т. е. каждая вершина 2п-угольника принадлежит контуру 2?-угольника. Доказать, что 2k делится на п. 53(9—10). Внутри куба расположен такой многогранник М, что его проекция на каждую грань куба заполняет эту грань! Доказать, что объем многогранника М не меньше одной трети объема куба. 54(9—10). Город, имеющий форму квадрата со стороной 10 км, разделен на п2 одинаковых квадратных кварталов. Квар- Кварталы занумерованы числами от 1 до п так, что два квартала, имеющие соседний номер, имеют общую сторону. Доказать, что велосипедист может найти нужный ему квартал, проехав не бо- более 100 км. 55(8—10). В выпуклом пятиугольнике провели все диагонали, в результате чего он оказался разбитым на 10 треугольников и один пятиугольник. Из суммы площадей треугольников, приле- прилегающих к сторонам исходного пятиугольника, вычли площадь внутреннего пятиугольника; получилось число S. Потом анало- аналогичную разность вычислили для внутреннего пятиугольника; получилось число s. Доказать, что 56(8). Найти максимальное число вершин невыпуклого неса- мопересекающегося п-угольника, из которых нельзя провести ни одной внутренней диагонали. 57(8). В целых точках числовой прямой расставлелы какие-то натуральные числа. На первом шаге между каждыми двумя со- соседними числами записывается их среднее арифметическое, а исходные числа стираются. На втором шаге с написанными числа- числами проделывается та же операция и т. д. Оказалось, что все числа, которые получаются на каждом шаге, натуральные. Мож- Можно ли утверждать, что на некотором шаге все числа были равны между собой? 58(9—10). Из 27 кубиков со стороной 1 составили куб раз- размером 3X3X3. Каждый кубик покрасили в белый или черный цвет. Через час пришел маляр и все кубики, имеющие четное число черных соседей, покрасил в белый цвет, а все осталь- остальные — в черный. Эту процедуру маляр проделывал каждый час. Доказать, что через некоторое время все кубики станут белыми. 59(9—10). На плоскости расположено п материальных точек равной массы. Возьмем произвольную точку плоскости Ао и рас- рассмотрим те из наших точек, расстояние от которых до Ао мень- меньше 1. Пусть А\ — центр масс этих точек. Теперь рассмотрим те из наших точек, расстояние от которых до А\ меньше 1; пусть 296
Л2 — их центр масс. Аналогично поступим с Аг и найдем центр масс Л3 точек, удаленных от Л2 меньше чем на 1, и т. д. Возникает последовательность точек Ло, Ль Л2, ... . Доказать, что в этой по- последовательности все точки, начиная с некоторой, совпадают. Примечание. Если для некоторой точки Ak внутри круга радиуса 1 с центром Ak нет ни одной материальной точки, то полагаем, что Л&=Ль+1 = ... . 60(8). С упорядоченными наборами из п нулей и единиц разрешается проделывать две операции: изменять первую слева цифру, а также изменять цифру, следующую после первой (сле- (слева) единицы. Доказать, что такими операциями из любого на- набора можно получить любой другой. 61(8) Четыре равные окружности Ои О2, О3, О4 лежат внут- внутри треугольника, причем окружность О\ касается двух сторон треугольника, окружность О2 — другой пары сторон этого тре- треугольника, окружность Оз — третьей пары сторон, а окружность О4 касается первых трех окружностей. Доказать, что центр О4 ле- лежит на одной прямой с центрами вписанной и описанной окруж- окружностей треугольника. 62(8—10). На бесконечной клетчатой бумаге нарисованы две непересекающиеся бесконечные в обе стороны ломаные, звенья которых идут по линиям бумаги и каждая из которых проходит через все узлы бумаги. Обязательно ли эти ломаные имеют общие звенья? 63(8—10). Обозначим через ?„ сумму п первых простых чи- чисел. Доказать, что между числами ?л и 2n + i расположен полный квадрат. 64(8—10). На плоскости нарисованы квадрат и правильный треугольник. Доказать, что какое-то из расстояний от вершины квадрата до вершины треугольника иррационально. 65(9—10). Город, имеющий форму квадрата, разделен на п2 квадратных кварталов. По улицам между кварталами введено двустороннее движение, а вокруг города — одностороннее. Ве- Велосипедист едет по городу, соблюдая правила уличного движе- движения, т. е. едет по правой стороне улицы и на перекрестках не поворачивает налево (на внешней односторонней улице он обязан ехать так, чтобы дома находились все время справа от него). При каких п можно утверждать, что велосипедист может объехать весь город, побывав на каждой стороне каждой улицы по одно- одному разу (на внешней улице — на ее единственной стороне)? Постарайтесь найти возможно более широкий набор искомых значений п. 66(8). Восьмиугольник Л1Л2Л3Л4Л5Л6Л7Л8 вписан в окруж- окружность. Оказалось, что Л|Л2||Л5Лб, Л2Лз||ЛбЛ7, Л3Л4||Л7Л8. Дока- Доказать, что Л4Л5 = Л1Л8. 67(8—9). Дан квадрат со стороной п. Его разбили на п2 квадратных клеток со стороной 1 каждая. Можно ли расставить в клетки п2 различных чисел так, чтобы в любом квадрате, сто- 297
роны которого идут по сторонам данного квадрата или по линиям разбиения, произведение чисел, стоящих на одной диагонали равнялось произведению чисел, стоящих на второй диагонали? 68(8—10). Могут ли три человека пройти из точек, находя- находящихся на расстоянии 0,1 и 2 от начала дороги, до точек, находя- находящихся на расстоянии 1000, 1001, 1002 от начала дороги, не обгоняя друг друга, так, чтобы последний человек всю дорогу видел первого, но ни в какой момент не видел второго человека? Дорога прямая, но не ровная. Ростом людей принебречь. 69(8). Обозначим через S (N) сумму цифр числа N. Доказать, что существует бесконечно много таких N, что в десятичной записи N отсутствуют нули и: а) N делится на S (N); б) N де- делится на S(N)-\-1. 70(8—10). Доказать, что из отрезков, имеющих длины 1, 2, 3, ..., 1980, можно составить выпуклый 1980-угольник, у которого все углы равны между собой. Верно ли аналогичное утвержде- утверждение для 1981-угольника? 71 (9—10). Функции f и g определены на прямой, и при всех х и у выполняется равенство Доказать, что если функция / не равна тождественно нулю, то значения функции g (у) при всех у не меньше — 1. 72(9—10). Имеется полный граф: п точек, каждые две из которых соединены отрезком. Каждый отрезок покрашен либо красным, либо синим цветом, причем из любой точки в любую другую можно пройти, пользуясь как только синими путями, так и только красными. Доказать, что среди п точек найдутся такие четыре точки, что в полном графе, составленном из этих четырех точек и соединяющих их отрезков, будет выполнено то же свойство: из любой точки в любую другую можно пройти как только синими, так и только красными путями. 73(9—10). Многочлен Р (х) при всех неотрицательных х при- принимает неотрицательные значения. Обязательно ли найдутся такие два многочлена, Q (х) и R (х), что Р (x) = (Q (x)f + (R (x)f? 74G—10). Решить в целых числах систему 75A0). Самолет совершил беспересадочный полет из города П в город Гг. В течение всего полета за самолетом велось наблюдение со станций слежения А и В, расположенных на от- отрезке Г,Г2. а) Доказать, что найдется промежуток временя величиной в 1 с, в течение которого самолет сместился из некоторой точ- точки X траектории в точку Y на одинаковые углы относительно каждой из станций слежения (т. е. /LXAY= /LXBY). б) Доказать, что это утверждение перестает быть верным, 298
если хотя бы одна из станций слежения не лежит на отрезке Г|Г_>, как бы близко точки Л и В ни находились от точек Г,, Г2 соответст- соответственно. Примечание. Самолет движется в пространстве; его ско- скорость, вообще говоря, переменная; время всего полета превос- превосходит 1 с; города Pi и Г2, станции слежения Л и В и самолет — точки. 76(9—10). Бассейн имеет форму выпуклого четырехугольни- четырехугольника, в вершинах которого растут деревья. Каждое дерево отбрасы- отбрасывает тень, имеющую форму круга с центром в соответствующей вершине. Известно, что весь бассейн находится в тени. Дока- Доказать, что тень каких-то трех деревьев полностью покрывает треугольник, в вершинах которого эти деревья растут. 77(8—10). На прямой расположена колония из конечного числа бактерий. В моменты t=l, 2, 3, ... некоторые из бактерий могут погибать; новых бактерий не возникает ни в один момент. Погибают те и только те бактерии, от которых ни слева на рас- расстоянии 1, ни справа на расстоянии -\J2 нет бактерий. Существует ли такая колония бактерий, которая будет существовать вечно? 78(9—10). Существует ли конечное множество точек на пло- плоскости, для каждой из которых среди оставшихся найдется не менее 1000 точек, удаленных от нее на расстояние 1? 79* A0). Додекаэдр, вершины которого покрашены красной краской, начинают перекатывать по плоскости через его ребра; при этом вершины додекаэдра дают на плоскости красные точеч- точечные отпечатки. Доказать, что для любого кружка произвольного радиуса г на плоскости додекаэдр можно перекатывать так, чтобы какой-то красный отпечаток одной из вершин попал в этот кружок. 80(9—10). Из трех стержней одинаковой длины изготовлена жесткая пространственная конструкция, в которой стержни не соприкасаются друг с другом, а только связаны нерастяжимыми нитями, прикрепленными к их концам. а) Какое наименьшее число нитей необходимо для этого? б) При каком соотношении между длинами стержней и дли- длинами нитей можно изготовить такую конструкцию?
Литература 1. Балк М. Б. Геометрические приложения понятия о центре тяжести.— М.: Физматгиз, 1956. 2. Барр Ст. Россыпи головоломок.— М.: Мир, 1984. 3. Б и з а м Д., Герце г Я. Игра и логика.— М.: Мир, 1975. 4. Визам Д., Г е р ц е г Я. Многоцветная логика.— М.: Мир 1978. 5. Гарднер М. Математические головоломки и развлече- развлечения.— М.: Мир, 1971. 6. Гарднер М. Математические досуги.— М.: Мир, 1972. 7. Гарднер М. Математические новеллы.— М.: Мир, 1974. 8. Гарднер М. Есть идея! — М.: Мир, 1982. 9. Дьюдени Г. Э. Кентерберийские головоломки.—М.: Мир 1979. 10. Дьюдени Г. Э. 520 головоломок.— М.: Мир, 1975. 11. ЗубелевичГ. И. Сборник задач Московских мате- математических олимпиад (V—VIII классы).— М.: Просвещение, 1971. 12. Избранные задачи (из журнала American Mathematical Monthly).—M-: Мир, 1977. 13. К о л м о г о р о в А. Н. О профессии математика.— М.: Изд-во МГУ, 1959. 14. Кэррол Л. История с узелками.— М.: Мир, 1973. 15. КюршакЙ, Нейкомм Д., Хайоги Д., Шура- ни Я. Венгерские математические олимпиады.— М.: Мир, 1976. 16. Линдгрен Г. Занимательные задачи на разрезание.— М.: Мир, 1977. 17. Лойд С. Математическая мозаика.— М.: Мир, 1980. 18. Математический цветник (Юбилейная книга к 65-летию М. Га рд нер а).—М.: Мир, 1983. 19. Морозова Е. А., Петраков И. С, Скворцов В. А. Международные математические олимпиады.— М.: Просвеще- Просвещение, 1976. 20. Р а д е м а х е р Г. Р., Теплиц О. Числа и фигуры.— М.: Физматгиз, 1962. 21. Сборник задач Московских математических олимпиад/ Сост. А. А. Леман.— М.: Просвещение, 1965. 22. С м а л л и а н Р. М. Как же называется эта книга? — М.: Мир, 1981. 23. С т р а ш е в и ч С, Бровкин Е. Польские математиче- математические олимпиады.— М.: Мир, 1978. 24. Тригг Ч. Задачи с изюминкой.— М.: Мир, 1975. 25. Ш к л яр ски й Д. О., Чепцов Н. Н., Яглом И. М. Избранные задачи и теоремы элементарной математики, ч. I, арифметика и алгебра.—М.: Наука, 1965. 300
26. Шклярски и Д. О., Ч е н ц о в Н. Н., Я г л о м И. М. Избранные задачи и теоремы элементарной математики, ч. II, геометрия (планиметрия).— М.: Гостехиздат, 1952. 27. Ш к л я р с к и й Д. О., Ч е н ц о в Н. Н., Я г л о м И. М. Избранные задачи и теоремы элементарной математики, ч. III, геометрия (стереометрия).— М.: Гостехиздат, 1954. 28. Штейн га уз Г. Задачи и размышления.— М.: Мир, 1974. 29. Э б б о т Э. Флатландия. Бюргер Д. Сферландия.— М.: Мир, 1976. 30. Яглом А. М., Яглом И. М. Неэлементарные задачи в элементарном изложении.— М.: Гостехиздат, 1954. 31. Яглом И. М., Болтянский В. Г. Выпуклые фигу- фигуры.— М.: Гостехиздат, 1951. 32. Яглом И. М. Геометрические преобразования, т. I.— М.: Гостехиздат, 1955; т. II.— М.: Гостехиздат, 1959. Статьи о Московских олимпиадах в журнале «У спех и математических наук» 33. Бончковский Р. Н. Вторая Московская математиче- математическая олимпиада, 1936, т. II, с. 275—278. 34. Г а л ь п е р н С. А. IX Московская математическая олим- олимпиада школьников, 1946, т. I, вып. 3—4, с. 206—211. 35. Петровский И. Я., Фетисов А. И. X Московская математическая олимпиада школьников, 1947, т. II, вып. 5, с. 243—247. 36. К р е й н е с М. А., М о р о з о в а Е. А., Ч е н ц о в Н. Н. XIII Московская математическая олимпиада школьников, 1950, т. V, вып. 5, с. 204. 37. Делоне Б. Н., Введенская Н. Д. XIV Московская школьная математическая олимпиада, 1951, т. VII, вып. 4, с. 180—184. 38. Рашевский П. К.., Введенская Н. Д., Коро- Королев Б. М. XV Московская школьная математическая олим- олимпиада, 1953, т. VIII, вып. 4. 39. М е н ь ш о в Д. Е., Ш а ф а р е в и ч И. Р., Морозо- Морозова Е. А., Золотарев В. И. XVI Московская школьная ма- математическая олимпиада, 1954, т. IX, вып. 3. с. 257—262. 40. Б а х в а л о в С. В., Ж и д к о в Н. П., Сафронов И. Д., Л у п а н о в О. Б. XVII Московская школьная математическая олимпиада, 1955, т. X, вып. 1, с. 213—219. Статьи о Московских олимпиадах в журнале «М атематика в школе» 41. Фетисов А. И. IX Математическая олимпиада школь- школьников в Москве, 1947, № 1, с. 66—67. 301
42. Фетисов А. И. IX Математическая олимпиада уча- учащихся московских школ, 1947, № 3, с. 54—60. 43. Задачи для математических кружков (задачи VI Москов- Московской математической олимпиады школьников), 1948, № 6, с. 62. 44. Т а н а т а р. И. Я- XII Математическая олимпиада уча- учащихся средних школ г. Москвы A-й тур), 1949, № 5, с. 42—47. 45. Танатар И. Я- XII Математическая олимпиада уча- учащихся средних школ г. Москвы B-й тур), 1949, № 6, с. 36—42. 46. Танатар И. Я. XIII Московская математическая олим- олимпиада, 1950, № 6, с. 54. 47. Решения задач, предлагавшихся на Всероссийской олим- олимпиаде по математике учащимся 7—10 классов, 1962, № 1, с. 86—90. 48. Васильев Н. Б. Задачи XXVII Московской матема- математической олимпиады, 1965, № 3, с. 70—75. 49. Гальперин Г. А. XXXV Московская математическая олимпиада, 1972, № 6, с. 58—64. 50. Кириллов А. А., Гальперин Г. А. XXXVI Мо- Московская математическая олимпиада, 1973, № 5, с. 79—82. 51. Г а л ь п е р и н Г. А. Задачи XXXVII Московской мате- математической олимпиады, 1974, № 6, с. 65—69. 52. Колмогоров А. Н., Гальперин Г. А. XXXVIII Московская математическая олимпиада, 1976, № 4, с. 68—72. 53. Тихомиров В. М., Гальперин Г. А. 42-я Мо- Московская математическая олимпиада, 1980, № 3, с. 63—66. Статьи о Московских олимпиадах в журнале «М атематическое просвещение» 54. Г и р с а н о в И. В. XIX школьная математическая олим- олимпиада в Москве, 1957, т. 1, с. 187—194. 55. О л е й н и к О. А., К и р и л л о в А. А. XX школьная математическая олимпиада в Москве, 1958, т. 3, с. 221—227. 56. Болтянский В. Г., Розендорн Э. Р. XXI школь- школьная математическая олимпиада в Москве, 1961, т. 6, с. 301—309.
СОДЕРЖАНИЕ Предисловие редактора Предисловие авторов Из истории Московских олимпиад Указания к работе с книгой Часть I Задачи Московских I олимпиада A935 г.) 20 II олимпиада A936 г.) 21 III олимпиада A937 г.) 22 IV олимпиада A938 г.) — V олимпиада A939 г.) 23 VI олимпиада A940 г.) — VII олимпиада A941 г.) 25 VIII олимпиада A945 г.) 27 IX олимпиада A946 г.) 28 X олимпиада A947 г.) 31 XI олимпиада A948 г.) 33 XII олимпиада A949 г.) 34 XIII олимпиада A950 г.) 36 XIV олимпиада A951 г.) 38 XV олимпиада A952 г.) 40 XVI олимпиада A953 г.) 43 XVII олимпиада A954 г.) 46 XVIII олимпиада A955 г.) 50 XIX олимпиада A956 г.) 54 XX олимпиада A957 г.) 58 XXI олимпиада A958 г.) 61 XXII о-шмпиада A959 г.) 66 XXIII олимпиада A960 г.) 70 XXIV олимпиада A961 г.) 73 математических олимпиад 3 5 18 XXV XXVI XXVII XXVIII XXIX XXX XXXI XXXII XXXIII XXXIV XXXV XXXVI XXXVII XXXVIII XXXIX XL XLI XLII XLIII XLIV XLV XLVI XLVII XLVIII олимпиада олимпиада олимпиада олимпиада олимпиада олимпиада олимпиада олимпиада олимпиада олимпиада олимпиада олимпиада олимпиада олимпиада олимпиада олимпиада олимпиада олимпиада олимпиада олимпиада олимпиада олимпиада олимпиада олимпиада A962 A963 A964 A965 A966 A967 A968 A969 A970 A971 A972 A973 A974 A975 A976 A977 A978 A979 A980 A981 A982 A983 A984 A985 г.) г.) г.) г) г.) г.) г.) г.) г.) г.) г.) г.) г.) г.) г.) г.) г.) г.) г.) г.) г.) г.) г.) ' г.) 78 81 85 89 94 95 99 105 ПО 116 120 125 128 131 133 135 138 139 141 143 144 146 148 150 Решения, указания, ответы Приложение 1 Приложение 2 Литература .... Часть II 153 283 291 300
Гриюрий Александрович Гальперин Алексей Кириллович Толпыго МОСКОВСКИЕ МАТЕМАТИЧЕСКИЕ ОЛИМПИАДЫ Зав. редакцией Р. А. Хабиб Редактор Л. II. Белоновская Младшие редакторы Л. Е. Козырева, Е. А. Сафронова Художник Е. С. Шабельник Художественный редактор Е. II. Карасик Технический редактор Г. В. Субочева Корректоры И. Б. Гитлсвич, И. И. Пешкова И Б № 9670 Сдано в набор 10.11.85. Подписано к печати 03.10.8G. Формат 60 X 9О'/|6- Бум. кн.-журн. отеч. Гарнит. Литературная. Печать высокая. Усл. печ. л. 19 + 0,25 форз. Усл. кр.-отт. 20,31. Уч.-изд. л. 18,83 + 0,46 форз. Тираж 680 000 экз. Заказ 247. Ueiid 75 коп. Ордена Трудового Красного Знамени издательство «Просвещение» Государст- Государственного комитета РСФСР по делам издательств, полиграфии и книжной торговли. 129846, Москва, 3-й проезд Марьиной рощи, 41. Саратовский ордена Трудового Красного Знамени полиграфический комбинат Росглавполиграфпрома Государственного комитета РСФСР по делам изда- издательств, полиграфии и книжной торговли. 410004, Саратов, ул. Чернышевского, 59.